{"correct_option": 4, "explanations": {"1": {"exist": false, "char_ranges": [], "word_ranges": [], "text": ""}, "2": {"exist": false, "char_ranges": [], "word_ranges": [], "text": ""}, "3": {"exist": false, "char_ranges": [], "word_ranges": [], "text": ""}, "4": {"exist": true, "char_ranges": [[0, 193]], "word_ranges": [[0, 30]], "text": "Although other hemorrhagic diseases can have a prolonged thromboplastin time, due to the intensity of the lesion and the child's sex and family history, the most likely diagnosis is hemophilia."}, "5": {"exist": false, "char_ranges": [], "word_ranges": [], "text": ""}}, "full_answer": "Although other hemorrhagic diseases can have a prolonged thromboplastin time, due to the intensity of the lesion and the child's sex and family history, the most likely diagnosis is hemophilia.", "full_answer_no_ref": "Although other hemorrhagic diseases can have a prolonged thromboplastin time, due to the intensity of the lesion and the child's sex and family history, the most likely diagnosis is hemophilia.", "full_question": "An 18-month-old boy, with complete immunization schedule to date, who consults the Emergency Department for right knee swelling after playing in the park, without obvious trauma. In the directed anamnesis, the mother refers that an uncle of hers had similar problems. The ultrasound examination is compatible with hemarthrosis and in the analytical analysis only an APTT lengthening of 52'' (normal 25-35'') stands out. What is the most probable diagnostic hypothesis?", "id": 190, "lang": "en", "options": {"1": "Marfan syndrome.", "2": "Von-Willebrand's disease.", "3": "Ehlers-Danlos disease.", "4": "Hemophilia A.", "5": "Bernard-Soulier disease."}, "question_id_specific": 94, "type": "PEDIATRICS", "year": 2013, "rag": {"clinical_case_options": {"MedCorp": {"RRF-2": [{"id": "pubmed23n0052_9437", "title": "Further evidence for recessive inheritance of von Willebrand disease with abnormal binding of von Willebrand factor to factor VIII.", "score": 0.01852010631912887, "content": "A new family with a bleeding diathesis and FVIII deficiency secondary to abnormal binding of von Willebrand factor (vWF) to factor VIII (FVIII) is described. Two propositi of this family, an 18-year-old male and a 33-year-old female, both with a history of epistaxis, bruising, bleeding from the gums, epistaxis, hemarthrosis, and hematoma, were analyzed. Also additional members of the same family with no bleeding history were also studied. The propositi showed normal vWF activities, low FVIII activity; one of them had been diagnosed as having hemophilia A and the other was a hemophilia A carrier. Both showed a very poor response to treatment with FVIII concentrates and desmopressin (DDAVP) but a good clinical response to cryoprecipitate. APTT was prolonged and no inhibitory activity was noticeable in their plasmas. Thirty-five units per kilogram body weight of Hemofil M was infused to both propositi and FVIII reached basal level within 60 minutes of the infusion. No FVIII response at all was observed in the female after intravenous DDAVP administration. However, the male who received the infusion of 35 U/kg body weight of Humate-P achieved a normal FVIII level that was maintained for 12 hours. Multimeric analysis of vWF was normal in all the members studied. Von Willebrand factor domain for FVIII binding was assayed in the two propositi and in six other members of the same family by using a non-isotopic and sensitive method, a modification of the one previously described, using the Hemofil M concentrate as exogenous FVIII. The data obtained showed that both propositi had similar binding to that observed by using plasma of a patient with severe von Willebrand disease. Furthermore, five siblings had a decreased binding of vWF to FVIII, when compared with plasma from normal individuals or patients with hemophilia A. We also observed that, for screening purpose, the ratio of bound FVIII/immobilized vWF (at saturation of the anti-vWF and offering of 1 U/ml of exogenous FVIII) distinguished two levels of abnormality (normal range 0.70-1.15, propositi 0.004-0.007, and remaining members affected 0.25-0.42). The most probable explanation is that the propositi are homozygous or double heterozygous, the other five siblings affected being heterozygous for a recessive vWF defect. This more accessible assay presented here may be of help in routine analysis for diagnosing this type of von Willebrand disease, which has important implications for therapy and genetic counseling."}, {"id": "pubmed23n1089_3373", "title": "Diagnostic Challenges in Children With Congenital Bleeding Disorders: A Developing Country Perspective.", "score": 0.017704661182922053, "content": "To assess the frequency and characteristics of children with inherited bleeding disorders that were initially misdiagnosed, leading to inappropriate disease management. This study was conducted at the Haematology/Pathology Department of Fauji Foundation Hospital, Rawalpindi, Pakistan, from August 2014 to August 2018. Children who were diagnosed with an inherited bleeding disorder but did not respond to initial therapy were reevaluated. In total, 62 children were diagnosed with a bleeding disorder. Of these, 27 were diagnosed with an inherited bleeding disorder and 35 with an acquired bleeding disorder. Of the 27 children with inherited bleeding disorders, 18% (n = 5) were misdiagnosed and treated inappropriately. The median age of the misdiagnosed patients was 9 years (range, 5-13 years). Three patients with Bernard-Soulier syndrome had been misdiagnosed as having immune thrombocytopenic purpura, 1 patient with von Willebrand disease had been misdiagnosed as having hemophilia A, and 1 patient with haemophilia B had been misdiagnosed as having hemophilia A. There are chances of misdiagnosis and improper or invasive management if comprehensive laboratory evaluation and a thorough clinical evaluation are not performed in children with congenital bleeding disorders."}, {"id": "pubmed23n0596_19559", "title": "The dental patient with a congenital bleeding disorder.", "score": 0.017392619479733817, "content": "Congenital bleeding disorders account for approximately one in 10,000 births. Dentists are often anxious about delivering treatment to this special group of patients. In the Irish Republic, patients with inherited bleeding disorders have their dental care co-ordinated centrally at the National Centre for Hereditary Coagulation Disorders (NCHCD), St James's Hospital, Dublin. Dental care is normally integrated with routine outpatient haematological appointments. This ensures regular monitoring of oral health and the early treatment of any hard/soft tissue pathology. This article describes, in simple diagrammatic form, the normal coagulation mechanism (Figures 1 and 2), explains common coagulation terms (Appendix 1), and examines the three most common congenital bleeding disorders: haemophilia A, haemophilia B, and von Willebrand disease. General recommendations based on the current literature are provided with respect to procedures that are appropriate to perform in a general dental practice setting. Although not discussed in this article, it is important to note that non-coagulation bleeding disorders also exist. These include: hereditary haemorrhagic telangiectasia; blood vessel wall defects resulting from connective tissue disorders such as Marfan syndrome and Ehlers-Danlos syndrome; and, platelet disorders such as Bernard-Soulier syndrome, resulting in defective platelet adhesion."}, {"id": "pubmed23n0780_13321", "title": "Screening bleeding disorders in adolescents and young women with menorrhagia.", "score": 0.015617875056999544, "content": "Chronic menorrhagia causes anemia and impairment of life quality. In this study the aim was the screening of bleeding disorders in adolescents and young women with menorrhagia. The study was performed prospectively by pediatric hematologists. A form including demographic characteristics of the patients, bleedings other than menorrhagia, familial bleeding history, characteristics of the menorrhagia, and impairment of life quality due to menorrhagia was filled out by the researcher during a face-to-face interview with the patient. A pictorial blood assessment chart was also used for evaluation of blood loss. All patients underwent pelvic ultrasound sonography testing and women also received pelvic examination by gynecologists. Whole blood count, peripheral blood smear, blood group, serum transaminases, urea, creatinine, ferritin, PFA-100, PT, aPTT, INR, TT, fibrinogen, VWF:Ag, VWF:RCo, FVIII, and platelet aggregation assays were performed. Platelet aggregations were studied by lumiaggregometer. Out of 75 patients enrolled, 60 patients completed the study. The mean age was 20.68±10.34 (range: 10-48) years and 65% (n=39) of the patients were younger than 18 years. In 18 (46%) of the adolescents, menorrhagia subsided spontaneously. In 20% (n=12) of the patients, a bleeding disorder was detected (1 case of type 3 von Willebrand disease, 2 patients with low VWF:Ag, 1 case of probable von Willebrand disease, 3 cases of Bernard-Soulier syndrome, 2 cases of Glanzmann thrombasthenia, 2 cases of immune thrombocytopenic purpura, 1 case of congenital factor VII deficiency). In patients with menorrhagia, at least complete blood count, peripheral smear, aPTT, PT, VWF:Ag, VWF:RCo, FVIII, and fibrinogen assays must be performed. When there is history of nose and gum bleeding, platelet function assay by lumiaggregometer must also be performed. In nearly 50% of adolescents, menorrhagia is dysfunctional and transient. Detailed coagulation assays can be postponed in adolescents if bleeding history other than menorrhagia and/or family history of bleeding and/or parental consanguinity is absent. All subjects with menorrhagia must consult with gynecologists and hematologists. None declared."}, {"id": "pubmed23n0083_20904", "title": "Ehlers-Danlos syndrome, clotting disorders and muscular dystrophy.", "score": 0.015548567435359888, "content": "Ehlers-Danlos syndrome includes 11 distinct entities. The diversity of this collagen dysplasia and its combination with other abnormalities make it difficult to understand physiopathologically. A case of Ehlers-Danlos syndrome is reported, which is novel owing to its combination with clotting abnormalities and especially with muscular dystrophy. To our knowledge this has not previously been reported. The patient was a young man aged 16 years who presented with Ehlers-Danlos syndrome satisfying Perelman's diagnostic criteria. His father and two brothers had comparable clinical symptoms, but his mother and sister were healthy. The four male subjects had an increased cephalin-kaolin time, reduced levels of factor VIII and Willebrand's factor (but without haemophilia A or Willebrand's disease), and, especially, an abnormal platelet ATP secretion. The proband alone had muscular disease with bilateral quadriceps fatigability and amyotrophy. The muscle enzyme levels were greatly increased, the electromyographic trace was myogenic, and the biopsy showed severe muscular dystrophy. This new observation poses the problem of the relation between clotting abnormalities and collagen abnormalities in the Ehlers-Danlos syndrome. It is difficult to classify this case within any of the 11 known types because of its muscular manifestations. It may perhaps be a fortuitous combination or an extension of the nosological framework of this syndrome."}, {"id": "wiki20220301en081_52264", "title": "Bernard–Soulier syndrome", "score": 0.015040515040515042, "content": "Differential diagnosis The differential diagnosis for Bernard–Soulier syndrome includes both Glanzmann thrombasthenia and pediatric Von Willebrand disease. BSS platelets do not aggregate to ristocetin, and this defect is not corrected by the addition of normal plasma, distinguishing it from von Willebrand disease. Following is a table comparing its result with other platelet aggregation disorders: Treatment Bleeding events can be controlled by platelet transfusion. Most heterozygotes, with few exceptions, do not have a bleeding diathesis. BSS presents as a bleeding disorder due to the inability of platelets to bind and aggregate at sites of vascular endothelial injury. In the event of an individual with mucosal bleeding tranexamic acid can be given. The affected individual may need to avoid contact sports and medications such as aspirin, which can increase the possibility of bleeding. A potential complication is the possibility of the individual producing anti-platelet antibodies."}, {"id": "wiki20220301en071_25146", "title": "Erik Adolf von Willebrand", "score": 0.013859020310633213, "content": "He published a Swedish-language article in 1926 about the disease, titled Hereditär pseudohemofili (\"Hereditary pseudohemophilia\"). He referenced six previous publications from the years of 1876 to 1922, totalling 19 cases on families with bleeding diatheses. The earlier authors attributed the condition to hemophilia (even in the cases of females) or to thrombopathy, which was discovered shortly before as the cause of what had previously been known as purpura hemorrhagica or Werlhof's disease. Von Willebrand also conducted hematological examinations on Hjördis and some of her family members. He recorded a normal or slightly reduced number of platelets and an undisturbed clot retraction, unlike Glanzmann's thrombasthenia. The bleeding time (Duke) was greatly prolonged, extending to more than 2 hours in some cases, while the clotting time was within the normal range. He concluded that the disease was either a new form of thrombopathy or a condition of the capillary endothelium."}, {"id": "wiki20220301en100_5603", "title": "List of MeSH codes (C15)", "score": 0.013703271467831193, "content": "– hemorrhagic disorders – afibrinogenemia – bernard-soulier syndrome – disseminated intravascular coagulation – factor v deficiency – factor vii deficiency – factor x deficiency – factor xi deficiency – factor xii deficiency – factor xiii deficiency – hemophilia a – hemophilia b – hypoprothrombinemias – platelet storage pool deficiency – hermanski-pudlak syndrome – purpura, thrombocytopenic, idiopathic – thrombasthenia – thrombocythemia, hemorrhagic – vascular hemostatic disorders – cryoglobulinemia – ehlers-danlos syndrome – hemangioma, cavernous – hemangioma, cavernous, central nervous system – multiple myeloma – pseudoxanthoma elasticum – purpura, hyperglobulinemic – purpura, schoenlein-henoch – scurvy – shwartzman phenomenon – telangiectasia, hereditary hemorrhagic – waldenstrom macroglobulinemia – vitamin k deficiency – hemorrhagic disease of newborn – von willebrand disease – waterhouse-friderichsen syndrome"}, {"id": "pubmed23n0703_24426", "title": "Rare and unusual bleeding manifestations in congenital bleeding disorders: an annotated review.", "score": 0.012926108374384238, "content": "Epistaxis, superficial and deep hematomas, hemarthrosis, gastrointestinal bleeding, hematuria represent the most frequent hemorrhagic events in congenital coagulation disorders. Occasionally, bleeding manifestations occur in unusual sites or are peculiar. A clotting defect may alter the clinical aspect of skin conditions or infections (hemorrhagic scabies or varicella). Hemobilia may occur as a complication of transjugular liver biopsy in hemophilia or Bernard-Soulier syndrome. Hemarthrosis of small joints of feet and hands occur in patients with hemophilia treated with protease inhibitors. Intramedullary hematomas of long bones have been described in α2-plasmin inhibitor or fibrinogen deficiencies. Spleen fracture with consequent hemoperitoneum has been reported in patients with fibrinogen deficiency. Rectus muscle sheath hematoma may occur in patients with factor VII (FVII)or FX deficiency. Acute or subacute intestinal obstruction may be caused by intramural wall hematomas in hemophilia and von Willebrand (vW)-disease. Physicians should always keep in mind that a congenital hemorrhagic disorder may cause bleeding in any tissue of the body and therefore alter the normal clinical features of a given disease."}, {"id": "article-31270_27", "title": "Von Willebrand Disease -- Differential Diagnosis", "score": 0.012883358471593767, "content": "Factor X deficiency Factor XI deficiency Hemophilia A Hemophilia B Bernard-Soulier syndrome Platelet function defects Antiplatelet drug ingestion Fibrinolytic defects"}, {"id": "pubmed23n0637_1860", "title": "Spectrum of inherited bleeding disorders in southern Iran, before and after the establishment of comprehensive coagulation laboratory.", "score": 0.012847884102173085, "content": "The objective of the present study was to determine the pattern of inherited bleeding disorders in southern Iran and evaluate the effect of a comprehensive coagulation laboratory and related efforts. A total of 545 patients with inherited bleeding disorders were evaluated during 1992-2007 by a cross-sectional study. Data were collected by a data-gathering form. Statistical analysis was done using Statistical Package for the Social Sciences version 15. A P value less than 0.05 was considered statistically significant. Overall 411 patients had common bleeding disorders including 326 hemophilia A, 46 hemophilia B, and 39 von Willebrand disease. Seventy-nine patients had rare coagulation disorders including deficiency of factor VII (n = 26), factor X (n = 18), factor XIII (n = 9), factor I (n = 9), factor XI (n = 7), factor V (n = 4), combined factor VIII and factor V (n = 4), and combined factor X and factor VII (n = 2). Fifty-five patients had platelet disorders including 23 with Glanzmann's thrombasthenia, 15 with Bernard-Soulier syndrome, and 17 with other platelet disorders, most of which (45) were diagnosed after the establishment of the comprehensive coagulation laboratory. Annual mean number of new diagnosed patients with common and rare bleeding disorders increased from 29 +/- 4 to 38 +/- 17. The ratio of the patients diagnosed with rare bleeding disorders to common bleeding disorders significantly increased after the establishment of the comprehensive diagnosis laboratory (P < 0.001).It seems that implementation of collaborative projects by the Shiraz Hemophilia Society and the establishment of the comprehensive coagulation laboratory and treatment centers have been successful in increasing diagnosis of the inherited bleeding disorders and consequently better management of the patients."}, {"id": "wiki20220301en100_5598", "title": "List of MeSH codes (C15)", "score": 0.012756221400363567, "content": "– blood coagulation disorders – coagulation protein disorders – activated protein c resistance – afibrinogenemia – factor v deficiency – factor vii deficiency – factor x deficiency – factor xi deficiency – factor xii deficiency – factor xiii deficiency – hemophilia a – hemophilia b – hypoprothrombinemias – von willebrand disease – disseminated intravascular coagulation – blood coagulation disorders, inherited – activated protein c resistance – afibrinogenemia – antithrombin iii deficiency – bernard-soulier syndrome – factor v deficiency – factor vii deficiency – factor x deficiency – factor xi deficiency – factor xii deficiency – factor xiii deficiency – hemophilia a – hemophilia b – hermanski-pudlak syndrome – hypoprothrombinemias – protein c deficiency – thrombasthenia – von willebrand disease – wiskott-aldrich syndrome – platelet storage pool deficiency – hermanski-pudlak syndrome – protein s deficiency"}, {"id": "wiki20220301en014_34439", "title": "Von Willebrand disease", "score": 0.012316715542521993, "content": "Diagnosis Basic tests performed in any patient with bleeding problems are a complete blood count-CBC (especially platelet counts), activated partial thromboplastin time-APTT, prothrombin time with International Normalized Ratio-PTINR, thrombin time-TT, and fibrinogen level. Patients with abnormal tests typically undergo further testing for hemophilias. Other coagulation factor assays may be performed depending on the results of a coagulation screen. Patients with von Willebrand disease typically display a normal prothrombin time and a variable prolongation of partial thromboplastin time."}, {"id": "pubmed23n0684_10587", "title": "Congenital bleeding disorders in Karachi, Pakistan.", "score": 0.011854657687991021, "content": "To determine the frequency of inherited bleeding disorders, its complications, and treatment modalities available for its treatment. Cross-sectional study. Patients with a history of bleeding tendency were tested for confirmation of the diagnosis. History and clinical findings were recorded. Laboratory analysis included prothrombin time (PT), activated partial thromboplastin time (APTT), bleeding time (BT), and fibrinogen assay. Patients with prolonged APTT were tested for factors VIII (FVIII) and IX (FIX). If FVIII was low, von Willebrand factor: antigen (vWF:Ag) and von Willebrand factor:ristocetin cofactor (vWF:RCo) were performed. When PT and APTT both were prolonged, FV, FX, and FII were tested. Platelet aggregation studies were done when there was isolated prolonged BT. Urea clot solubility test was done when all coagulation tests were normal. All patients with hemophilia A and B were evaluated for inhibitors. Of the 376 patients, inherited bleeding disorder was diagnosed in 318 (85%) cases. Median age of patients was 16.4 years. Hemophilia A was the commonest inherited bleeding disorder that was observed in 140 (37.2%) followed by vWD 68 (18.0%), platelet function disorders 48 (12.8%), and hemophilia B in 33 (8.8%) cases. We also found rare congenital factor deficiencies in 13 (3.4%), low VWF in 11 (3.0%) participants and 5 (1.3%) in female hemophilia carriers. Hemarthrosis was the most frequent symptom in hemophilia A and B (79.7%) involving knee joint. Inhibitor was detected in 21 (15%) cases. Fresh frozen plasma/cryoprecipitate were the most common modality of treatment. In 58 patients, no abnormality was detected in coagulation profile. Hemophilia A and vWD are the most common congenital bleeding disorders in this study. Hemarthrosis involving knee joint was the most common complication. Inhibitor was detected in a significant number of patients. Plasma is still the most common modality of treatment."}, {"id": "pubmed23n0862_14626", "title": "Clinical profile of hemophilia patients in Jodhpur Region.", "score": 0.011667345000678335, "content": "Hemophilia is widely distributed all over the world, but little is known about its clinical profile in resource-limited regions. An insight into its clinical spectrum will help in the formulation of policies to improve the situation in these areas. To study the clinical profile of hemophiliacs (age <18 years) in Jodhpur region and screen them for transfusion-transmitted infections. A cross-sectional study conducted in the Department of Pediatrics, Umaid Hospital, Dr. S. N. Medical College, Jodhpur, over a period of 12 months. Out of a total of 56 cases enrolled, 51 (91%) cases were diagnosed as hemophilia A while 5 (9%) were diagnosed as hemophilia B. Positive family history was found in 26 (46%) cases. According to their factor levels, 25 (44%) cases had severe disease, 20 (36%) had moderate disease, and 11 (20%) had mild disease. The mean age of onset of symptoms and diagnosis was 1.73 ± 1.43 and 3.87 ± 3.84 years, respectively. First clinical presentation was posttraumatic bleed in 20 (36%), gum bleeds in 17 (30%), epistaxis in 4 (7%), joint bleeds in 4 (7%), skin bleeds in 4 (7%), and circumcision bleed in 3 (5%) cases. Knee joint was the predominant joint affected by hemarthrosis in 38 (68%), followed by ankle in 29 (52%), elbow in 20 (36%), and hip joint in 7 (13%) cases. All patients had a negative screening test for transfusion-transmitted infections. Occurrence of posttraumatic bleeds and gum bleeds in an otherwise normal child should warn the clinician for evaluation of hemophilia."}, {"id": "article-22743_12", "title": "Hemophilia A -- Differential Diagnosis", "score": 0.011639607299230371, "content": "Acquired hemophilia Ehlers-Danlos syndrome Factor XI deficiency Glanzmann thrombasthenia Haemophilia C Haemophilia type B Physical child abuse Platelet disorders Von Willebrand disease"}, {"id": "wiki20220301en011_69738", "title": "Bleeding time", "score": 0.011267006802721087, "content": "Normal values fall between 3 – 10 minutes depending on the method used. A disadvantage of Ivy's method is closure of puncture wound before stoppage of bleeding. Duke's method With the Duke;s method, the patient is pricked with a special needle or lancet, preferably on the earlobe or fingertip, after having been swabbed with alcohol. The prick is about 3–4 mm deep. The patient then wipes the blood every 30 seconds with a filter paper. The test ceases when bleeding ceases. The usual time is about 2–5 minutes. This method is not recommended and cannot be standardized because it can cause a large local hematoma. Interpretation Bleeding time is affected by platelet function, certain vascular disorders and von Willebrand Disease—not by other coagulation factors such as haemophilia. Diseases that cause prolonged bleeding time include thrombocytopenia, disseminated intravascular coagulation (DIC), Bernard-Soulier disease, and Glanzmann's thrombasthenia."}, {"id": "pubmed23n0726_15808", "title": "A longitudinal prospective study of bleeding diathesis in Egyptian pediatric patients: single-center experience.", "score": 0.011224489795918367, "content": "Keeping an updated registry of bleeding disorders is crucial for planning care and documenting prevalence. We aimed to assess the prevalence of various bleeding disorders including rare inherited coagulation and platelet disorders concerning their clinico-epidemiological, diagnostic data and bleeding manifestations severity. Patients suffering from manifestations of bleeding or coagulation disorders presented to Hematology Clinic during 16 years were included and prospectively followed up. Demographics, clinical characteristics, complete blood count, bleeding, prothrombin and activated partial thromboplastin times, platelet aggregation tests and bone marrow aspiration were recorded. Overall 687 patients with bleeding disorders from total 2949 patients were identified. Inherited coagulation defects were found in 27.2%; hemophilia A (70.6%), hemophilia B (13.9%), factor I deficiency (2.3%), factor V deficiency (1.6%), factor X deficiency (4.2%), factor VII deficiency (2.6%), factor XIII deficiency (1.1%), combined factor deficiency (2.1%) and unclassified coagulation disorders in 1.6% of studied patients. Overall 72.7% had diagnosed with platelet disorders; immune thrombocytopenia was the commonest (74.8%), and inherited conditions represent (25.2%) in the following order: Glanzman's thrombasthenia (11.2%), von Willebrand disease (6.6%), Bernard-Soulier syndrome (1%) and Chediak Higashi in 0.4% and unclassified in 6%. Median age of diagnosis of coagulation and platelet disorders were 33 and 72 months. Presenting symptoms of coagulation disorders were: 25.1% post circumcision bleeding, 22.5% ecchymosis, 20.9% hemoarthrosis and 15% epistaxis. Symptoms of rare coagulation disorders were postcircumcision bleeding (20%), bleeding umbilical stump (20%), epistaxis (12%), hemoarthrosis (8%) and hematomas (4%). Presenting symptoms in rare inherited platelet disorders were purpura, ecchymosis, epistaxis and bleeding gums, respectively. Analysis of the clinico-epidemiological data of patients with bleeding disorders is a useful tool for monitoring and improving their quality of care."}, {"id": "wiki20220301en026_78760", "title": "Bleeding diathesis", "score": 0.01085003144936652, "content": "Causes other than coagulation Bleeding diathesis may also be caused by impaired wound healing (as in scurvy), or by thinning of the skin, such as in Cushing's syndrome. Genetic Some people lack genes that typically produce the protein coagulation factors that allow normal clotting. Various types of hemophilia and von Willebrand disease are the major genetic disorders associated with coagulopathy. Rare examples are Bernard–Soulier syndrome, Wiskott–Aldrich syndrome and Glanzmann's thrombasthenia. Gene therapy treatments may be a solution as they involve in the insertion of normal genes to replace defective genes causing for the genetic disorder. Gene therapy is a source of active research that hold promise for the future. Diagnosis Comparing coagulation tests"}, {"id": "article-22748_37", "title": "Hemophilia -- Differential Diagnosis", "score": 0.010722125288456126, "content": "Other conditions can also present similarly with bleeding after minor trauma or spontaneous bleeds and require exclusion before confirming the diagnosis of hemophilia. Some of these conditions include von Willebrand disease, scurvy, diseases of platelet dysfunction, deficiency of other coagulation factors like V, VII, X, or fibrinogen, Ehlers-Danlos syndrome, Fabry disease, disseminated intravascular coagulation, and child abuse. In von Willebrand disease, bleeding symptoms can be similar to mild hemophilia, but patients with von Willebrand disease have more mucosal bleeding compared to musculoskeletal bleeding seen in hemophilia. Von Willebrand disease is diagnosed by checking for von Willebrand factor antigen or von Willebrand factor multimers. [40] Similarly, in scurvy, Ehlers-Danlos syndrome, and Fabry disease; also, the bleeding is usually mucosal, unlike hemophilia, where it is musculoskeletal. In scurvy, there is a deficiency of vitamin C. [41] In Ehlers-Danlos syndrome, the skin is hyperextensible, and joints are hypermobile. The diagnosis is usually through clinical features, genetic testing, and tissue biopsy. [42] Similarly, in Fabry disease, patients may also have other organs being affected, including kidneys and heart, and have skin lesions called angiokeratomas. They also have pain in the extremities. Fabry disease is usually diagnosed with clinical findings and genetic testing. [43] In cases of platelet dysfunction disorders, bleeding is usually mucocutaneous, unlike hemophilia. Usually, these disorders are diagnosed by platelet aggregation studies or platelet electron microscopy. [44] In the deficiency of other coagulation factors, musculoskeletal bleeding is uncommon. In fact, sometimes thrombosis can occur, especially in patients with factor VII or fibrinogen deficiency or in patients with combined factor V and VIII deficiency. Specific coagulation factor assays usually confirm the diagnosis. Disseminated intravascular coagulation (DIC) that mimics hemophilia is hard to differentiate, but usually, there is an underlying condition in DIC, for example, acute promyelocytic leukemia. Diagnosis is usually carried out by blood tests that show decreased platelet count and the absence of factor VIII autoantibodies. Child abuse can sometimes be misidentified and confused with hemophilia, and it is essential to find inconsistencies in the history of how trauma has occurred. Other signs of malnourishment require vigilance, and x-rays may reveal evidence of fractures of different ages. [45] [46]"}, {"id": "wiki20220301en100_5572", "title": "List of MeSH codes (C16)", "score": 0.010553058432675396, "content": "– anemia, hypoplastic, congenital – anemia, Diamond–Blackfan – fanconi anemia – ataxia telangiectasia – blood coagulation disorders, inherited – activated protein C resistance – afibrinogenemia – antithrombin III deficiency – Bernard–Soulier syndrome – factor V deficiency – factor VII deficiency – factor X deficiency – factor XI deficiency – factor XII deficiency – factor XIII deficiency – hemophilia A – hemophilia B – Hermansky–Pudlak syndrome – hypoprothrombinemias – protein C deficiency – thrombasthenia – Von Willebrand disease – Wiskott–Aldrich syndrome – CADASIL – cardiomyopathy, hypertrophic, familial – cherubism"}, {"id": "article-18225_15", "title": "Bernard-Soulier Syndrome -- History and Physical", "score": 0.010514704156593987, "content": "The International Society on Thrombosis and Haemostasis Bleeding Assessment Tool (ISTH-BAT) is useful for assessing bleeding disorders. Its utility was tested in a small study, including patients with known inherited platelet disorders. The study demonstrated a specificity of 100%, a positive predictive value of 90%, and a negative predictive value of 100% using this assessment tool. [17] In the setting of von Willebrand disease, a BAT score >6 repeatedly correlates to a 99% probability of an inherited platelet defect such as BSS. [8] ] Similarly, other bleeding assessment tools like Molecular and Clinical Markers for the Diagnosis and Management (MCMDM) of type 1 von Willebrand's disease (VWD) and the World Health Organization Bleeding Assessment Tool are similar tools. An electronic version of MCMDM-type-1 VWD was developed in 2010. [17]"}, {"id": "InternalMed_Harrison_26188", "title": "InternalMed_Harrison", "score": 0.01045157359454456, "content": "Hemophilia is a sex-linked recessive genetic disorder characterized by the absence or deficiency of factor VIII (hemophilia A, orclassic hemophilia) or factor IX (hemophilia B, or Christmas disease) (Chap. 141). Hemophilia A constitutes 85% of cases. Spontaneous hemarthrosis is a common problem with both types of hemophilia and can lead to a deforming arthritis. The frequency and severity of hemarthrosis are related to the degree of clotting factor deficiency. Hemarthrosis is not common in other disorders of coagulation such as von Willebrand disease, factor V deficiency, warfarin therapy, or thrombocytopenia. Hemarthrosis occurs after 1 year of age, when a child begins to walk 2241 and run. In order of frequency, the joints most commonly affected are theknees,ankles,elbows,shoulders,andhips.Smalljointsofthehands and feet are occasionally involved."}, {"id": "pubmed23n0597_16253", "title": "Hemarthrosis due to a rare cause of hemorrhagic diathesis: Ehlers-Danlos syndrome.", "score": 0.01044450504499591, "content": "The authors report a case of hemarthrosis complicated by severe anemia related to a congenital connective tissue disease: Ehlers-Danlos syndrome. A boy fell down and suffered tumefaction of both knees with bilateral rupture of the rotula tendon. He underwent surgical reinsertion of each tendon on the rotula. He later showed an unexpected ongoing hematic effusion, with severe anemia. He was screened for coagulation disorders with no results. On taking a more detailed history and investigating the patient's phenotypical features, the authors diagnosed Ehlers-Danlos syndrome, hypermobile variant. The hemarthrosis and anemia were thus concluded to be consequences of excessive tissue fragility due to a congenital connective tissue disease."}, {"id": "pubmed23n0709_6992", "title": "The role of ultrasonography in the diagnosis of the musculo-skeletal problems of haemophilia.", "score": 0.009900990099009901, "content": "Recurrent haemarthrosis is the final cause of haemophilic arthrosic disease in haemophilia patients. Therefore, it is essential to diagnose it early, both clinically and by imaging. In addition, haemophilia patients experience chronic synovitis, joint degeneration, muscle haematoma and pseudotumours. The objective of this article is to highlight the value of ultrasounds in the diagnosis and control of the evolution of musculo-skeletal problems in haemophilia patients. To this end, we have performed a literature search in the PubMed, Web of Science(®) (WOS) and SciVerse bases, using the following keywords: hemophilia or haemophilia and ultrasonography (US), ultrasound, echography and sonography. The search was limited to studies published in English between the years 1991 and 2011, finding a total of 221 references. After reviewing the title or abstract for evidence of the use of US for the diagnosis of musculo-skeletal lesions in haemophilia, we selected 24 of these references. We added data collected from our experience to the most important data found in the references. Our main conclusion is that US is highly valuable for the diagnosis of musculo-skeletal diseases in haemophilia. It is a fast, effective, safe, available, comparative, real-time technique that can help us confirm the clinical examination. It is particularly important in acute haemarthrosis, as it can be used to objectively identify the presence of blood in the joints, measure its size, pinpoint its location, assess its evolution and confirm its complete disappearance."}, {"id": "pubmed23n1125_14898", "title": "The value of ultrasonography in detecting early arthropathic changes and contribution to the clinical approach in patients of hemophilia.", "score": 0.00980392156862745, "content": "PURPOSE\\AIM: Hemophilia affects the blood clotting process, is a genetic disease characterized by recurrent bleeding. The hemophilia early arthropathy detection with ultrasound (HEAD-US) procedure and scoring method were designed for the detection of early changes in affected joints of patients. In this article, it was aimed to detect early arthropathic changes in the joints of hemophilia patients with the HEAD US scoring system and to investigate its clinical contribution. It was aimed to investigate the effectiveness of HEAD-US scoring in showing early joint damage in subclinical hemophilia cases and its contribution to treatment. The present study included 50 hemophilia patients who were admitted to Departments of Pediatric and Adult Hematology for routine follow-up. During routine follow-up controls, patients were scored by physical examination and HJHS 2.1 and by ultrasonography and HEAD US. Statistical tests were used to analyze joint health status and the results of US examination in the patient group. A total of 294 joints (elbow n = 100, knee n = 94, ankle n = 100) were evaluated by ultrasonography. The mean HJHS and HEAD-US scores of the patients were 14.94 ± 15.18 and 15.6 ± 12.6, respectively. HEAD-US is accepted to be more sensitive than HJHS in detecting early signs of arthropathy. Detection of early abnormalities by ultrasonography will enable the development of individualized treatment protocols and to the prevention of arthropathy development."}, {"id": "pubmed23n0318_20292", "title": "Nonsurgical synovectomy in the treatment of arthropathy in Von Willebrand's Disease.", "score": 0.009708737864077669, "content": "Von Willebrand's disease is the most common inherited bleeding disorder, with an overall prevalence in the general population of 0.8% to 1.3%. Hemarthrosis occurs mainly in the severest forms of the disease (type III), with a frequency of 3.5% to 11%, and can cause severe arthropathy similar to that seen in hemophilia. We retrospectively reviewed our experience with nonsurgical synovectomy in the treatment of recurrent hemarthrosis with arthropathy in patients with von Willebrand's disease. Four of our six patients had type III disease and the remaining two had type II disease. The age range was 13 to 63 years. The frequency of hemarthrosis prior to synovectomy was one to four per month. One (n = 2) or both (n = 1) knees were treated in 4 cases, one (n = 1) or both (n = 1) ankles in 3 cases and an elbow in one case. We used yttrium 90 in a dose of 5 mCi for one knee, rhenium 186 in a dose of 2 mCi for two ankles and the elbow and osmic acid for two knees and one ankle. Clinical and radiological results were evaluated six months after synovectomy using the World Federation of Hemophilia score. Radiologic lesions remained stable and clinical manifestations improved in every case (p < 0.05). Five patients achieved a complete remission. Safety was satisfactory. The clinical efficacy of synovectomy done, using radiocolloids or osmic acid in arthropathy due to von Willebrand's disease, seems similar to that in hemophilia."}, {"id": "pubmed23n0905_24616", "title": "Joint assessment in von Willebrand disease. Validation of the Haemophilia Joint Health score and Haemophilia Activities List.", "score": 0.009708737864077669, "content": "Assessment of clinical outcome after joint bleeding is essential to identify joint damage and optimise treatment, to prevent disability. However, disease-specific tools to assess the musculoskeletal status in patients with von Willebrand disease (VWD) are lacking. We aimed to determine validity and reliability of the Haemophilia Joint Health Score (HJHS) and Haemophilia Activities List (HAL) in patients with Von Willebrand disease (VWD). Ninety-six patients with VWD were included (mean age 46 years) of whom 27 had more than five documented joint bleeds. The HJHS was performed in all patients and all patients completed the HAL and Impact on Participation and Autonomy (IPA) questionnaires. Health-related quality of life (SF36) results were obtained from the prior 'Willebrand in the Netherlands' study. Joint X-rays of knees, elbows and ankles were scored according to Pettersson (PS). Internal consistency of the HJHS (Cronbach's α (α)=0.75) and HAL (α=0.89) were good. Inter-observer agreement of the HJHS was good (ICC 0.84; Limits of Agreement ± 10.3). The HJHS showed acceptable correlation with the X-ray PS (Spearman's r (r95%) for the diagnosis of Löfgren syndrome. When the triad is present, further testing with additional imaging and laboratory testing is unnecessary. Treatment NSAIDs (nonsteroidal anti-inflammatory drugs) are the usual recommended treatment for Löfgren syndrome. Colchicine or low-dose prednisone may also be used. Prognosis Löfgren syndrome is associated with a good prognosis, with > 90% of patients experiencing disease resolution within 2 years. In contrast, patients with the disfiguring skin condition lupus pernio or cardiac or neurologic involvement rarely experience disease remission."}, {"id": "pubmed23n1003_13224", "title": "Anti-MDA-5 Dermatomyositis Presenting with Rapidly Progressive Interstitial Lung Disease: A Cautionary Tale.", "score": 0.009615384615384616, "content": "A 68-year-old Indian man presented with a pruritic eruption on his neck, back, elbows, knees, and the dorsum of his hands. He was initially treated for possible Lyme's disease by his primary care physician, but without improvement. Then he developed daily chills and fevers up to 101 °F, as well as shortness of breath. A chest radiograph showed patchy airspace opacities suggestive of atypical pneumonia, and the patient was treated with levofloxacin and prednisone. Although prednisone diminished the eruption, the patient continued to experience fever, malaise, and generalized weakness, at which point he was hospitalized. Blood cultures and an antinuclear antibodies (ANA) were negative and extensive lab workup was only notable for an elevated erythrocyte sedimentation rate (ESR) (63 mm/hr, Reference Range 0-22), mild transaminitis (AST 77 U/L, Reference Range 10-40), hyponatremia (131 mEq/L, Reference Range 135-145) and elevated ferritin (440, Reference Range 20-500). The patient was discharged on 20 mg of prednisone, with referral to rheumatology and dermatology for possible autoimmune diseases."}, {"id": "pubmed23n0871_6759", "title": "Systemic Lupus Erythematosus With Acute Inflammatory Demyelinating Polyneuropathy: A Case Report and Review of the Literature.", "score": 0.009523809523809525, "content": "We recently encountered a patient with acute inflammatory demyelinating polyneuropathy (AIDP) that was associated with systemic lupus erythematosus (SLE). A 34-year-old Chinese female with a 3-year history of SLE presented with acute bilateral leg weakness and paraparesis, and lost the ability to walk 1 day after noticing bilateral leg numbness and pain for 12 days. Physical examination revealed bilateral facial muscle paralysis, muscle strength in the legs with graded 1/5 proximally and 2/5 distally bilaterally and absence of deep tendon reflex in both knees and ankles. Paresthesia was observed in distal limbs with glove and stocking distribution. Cerebrospinal fluid analysis demonstrated albuminocytologic dissociation. Electrophysiologic survey also indicated sensory-motor demyelinating polyneuropathy. The diagnosis of SLE was established based on her initial symptoms including intermittent fevers, hair loss, oral ulcers, malar rash and arthritis affecting the elbow, wrist and hand joints; positive immunologic findings for antinuclear antibody (ANA), anti-DNA antibody, anti-Smith (anti-Sm) antibody, low serum complement levels, and the kidney biopsy specimen showed glomerular mesangial proliferation with focal endothelial cell proliferation (ISN/PPS 2004 classification lupus nephritis, class III). Treatment with intravenous immunoglobulin, methylprednisolone and cyclophosphamide resulted in clinical and electrophysiological improvement. "}, {"id": "pubmed23n0702_18357", "title": "[Adult-onset Still's disease].", "score": 0.009433962264150943, "content": "Adult-onset Still's disease is a rare inflammatory systemic disease. Cardinal symptoms/manifestations are fever, arthralgias or arthritis, myalgias, the typical skin rash, sore throat, hepatosplenomegaly, lymphadenopathy and serositis. Several other symptoms and organ involvements are possible. The clinical picture is variable with mild to life-threatening courses. The disease is self-limiting, intermittently active or chronic. Because of the lack of a defined diagnostic test the diagnosis of AOSD can only be made after exclusion of several differential diagnoses in particular of infectious, malignant and autoimmune origin. For therapy non-steroidal anti-inflammatory drugs, glucocorticoids, disease modifying antirheumatic drugs and biologics can be used."}, {"id": "wiki20220301en111_27511", "title": "Pemphigoid", "score": 0.009345794392523364, "content": "Glucocorticoid sparing drugs For patients who require high dose of corticosteroids for clearing or maintenance, glucocorticoid sparing agents such as immunosuppressive drugs and anti-inflammatory drugs can be used as an adjunct therapy to reduce the systemic side effects of corticosteroids. Patients who have comorbidities and contraindications for corticosteroids may also consider these glucocorticoid sparing agents. Immunosuppressant drug Immunosuppressant drugs include azathioprine (1–3 mg/kg/day in two equally divided doses), mycophenolate mofetil (1000–3000 mg/day or 40 mg/kg/day in two divided doses), and methotrexate (10–15 mg/week)."}, {"id": "pubmed23n0688_16038", "title": "Adult onset Still's disease: review of 41 cases.", "score": 0.009345794392523364, "content": "To describe the clinical, laboratory and radiological features, treatment and prognosis of patients with adult onset Still's disease (AOSD). Specific clinical features were retrospectively recorded in 41 patients fulfilling the Yamaguchi criteria. Patients were reviewed in two academic hospitals with a referral area of 700,000-1,000,000 inhabitants. Laboratory tests including haemogram, ferritin, biochemistry and autoimmunity were reviewed. Radiological studies, treatment and ACR functional class were determined. Forty-one patients with AOSD were identified, 25 of whom were female. Mean age at diagnosis: 38.19 years (range 17-68). Feverish polyarthritis was the most common clinical presentation. Acute phase reactants were invariably high in all patients. Serum ferritin levels were elevated in 86% of patients. Anti-cyclic citrullinated peptide antibodies (anti-CCP antibodies) were negative in all patients except one. The course of the disease was monocyclic in 44% of the patients, polycyclic in 26%, and chronic articular in 30%. ACR class was as follows: 29 (72.5%) class I, 7 (17.5%) class II, 2 (5%) class III and 2 (5%) class IV. As for the treatment received, aspirin or NSAIDs controlled the disease in eight patients (19.5%) and high-dose corticosteroids (0.5-1 mg/kg/day) in 32 (78%). Almost half of the patients (49%) required an additional diseasemodifying agent, usually methotrexate. Finally, in seven of them (17%) a biological treatment with TNF-α or specially anti-IL-1 had to be added to control the disease. The clinical and laboratory findings were similar to previous studies. Anti-CCP antibodies were almost always negative. A monocyclic course was associated with a good prognosis. Most of the patients were in ACR functional class I and II. Biological agents were required in 7 patients (17%)."}, {"id": "wiki20220301en129_17765", "title": "Drug-induced lupus erythematosus", "score": 0.009259259259259259, "content": "Diagnosis Antinuclear antibodies are usually positive in drug-induced Lupus. Anti-Neutrophil Cytoplasmic antibodies (ANCA) can also be positive in association with certain drugs. Furthermore, Anti-Histone antibodies can also be positive in drug-induced lupus. Anti-Histone antibodies are positive in up to 95% of patients with drug induced lupus. The most common medications associated with drug induced lupus are hydralazine, procainamide, isoniazid, methyldopa, chlorpromazine, quinidine, and minocycline. Treatment It is important to recognize early that these drugs are causing DIL like symptoms and discontinue use of the drug. Symptoms of drug-induced lupus erythematosus generally disappear days to weeks after medication use is discontinued. Non-steroidal anti-inflammatory drugs (NSAIDs) will quicken the healing process. Corticosteroids may be used if more severe symptoms of DIL are present."}, {"id": "pubmed23n0652_14323", "title": "[Polyarteritis nodosa with a spontaneous recovery].", "score": 0.009259259259259259, "content": "The periarteritis nodosa (PAN) is a serious necrotizing vasculitis. Healing is, classically, obtained after a long-term treatment using corticosteroids and immunosuppressive agents. Reporting the case of a NAP or was spontaneous healing without having recourse to any immunosuppressive treatment. We report an observation of a patient aged 27 monitoring for manic-depressive psychosis hospitalized for a fever during the course associated with high blood pressure (hypertension), a weight loss, asthenia and myalgia. Biology was a biological inflammatory syndrome (SIB), a creatinine 115 micromol/l. The survey infectious and immunological balance were negative. The arteriography showed a strongly evocative of the NAP. We have seen no sign of poor prognosis. The recovery was spontaneous after a short period of evolution with an amendment clinical and biological weapons. The patient was put under anti-HTA and reviewed regularly to the consultation. An analogy is drawn between such developments and already reported in some cases of viral PAN. The actual frequency of self-limitting PAN is unknown. The scarcity of cases reported in the literature may in part be attributed to ignorance."}, {"id": "pubmed23n1132_18000", "title": "Can SARS-CoV-2 infection trigger rheumatoid arthritis? A case report.", "score": 0.009174311926605505, "content": "Inflammatory arthritis has been reported after SARS-COV-2 infection. We present a case of a 38-year-old female patient who developed polyarthralgia 1 month after SARS-COV-2 infection. Musculoskeletal examination was significant for synovitis of hands and wrists. Antinuclear antibody (ANA), rheumatoid factor (RF), and anti-cyclic citrullinated peptide (CCP) antibodies were positive. Magnetic resonance imaging of the hands showed synovitis of the metacarpophalangeal joints and proximal interphalangeal joints of the hands, wrist joints, and tendinitis with tenosynovitis. The patient was diagnosed with seropositive nonerosive rheumatoid arthritis (RA) and initiated on therapy using nonsteroidal anti-inflammatory agents and disease-modifying anti-rheumatic drug methotrexate leading to an improvement in symptoms."}, {"id": "pubmed23n0835_18846", "title": "[Microscopic polyangiitis associated with antineutrophil cytoplasmic antibodies: clinical features].", "score": 0.009174311926605505, "content": "To study the clinical features of early- and extended-stage microscopic polyangiitis (MPA) and its outcomes on the basis of a long-term follow-up in a rheumatologist's practice. The clinical features of early- and extended-stage MPA were studied in detail and the premorbid background and possible precipitating factors were analyzed in 70 patients with MPA and the proven hyperproduction of antineutrophil cytoplasmic antibodies (anti-proteinase-3 (anti-PR3) antibodies in 55% and anti-myeloperoxidase (anti-MPO) antibodies in 45%) who had been followed up for more than a year. There is evidence for the nosological unity of the two immunological types of MPA associated with anti-PR-3 or anti-MPO antibodies. MPA has been demonstrated to be an aggressive, polysyndromic disease prone to recurrences (52%), the typical manifestation of which is glomerulonephritis (94%) that is rapidly progressive in every four cases and accompanied by hemorrhagic alveolitis (69%) and involvement of other organs. ENT organs and lungs have been noted to be commonly involved in early-stage MPA, which was observed in 61% of the patients in the premorbid period, and to become the first manifestation of MPA (63%) concurrent with body temperature rises (64%), arthralgia or arthritis (41%). Respiratory tract involvement in MPA may be asymptomatic. Anti-PR-3-associated MPA may manifest itself more aggressively and in the first 2 years it is characterized by a poorer prognosis than of anti-MPO-associated MPA (survival rates, 82 and 94%, respectively; p = 0.04). With time, the differences were levelled off; recurrences in the patients with anti-PR-3 and anti-MPO develop equally frequently and proceed showing the similar clinical picture; the survival curves converge by age 3. Anti-MPO-associated MPA is characterized by the highest rate of lung involvement in the clinical phase of the disease (61%) and by a propensity to develop hemorrhagic alveolitis, diffuse interstitial (22%) or circumscribed pulmonary fibrosis in the outcome. CONCLUSION. The findings emphasize how important to diagnose MPA early and to prescribe long-term active treatment using the entire current arsenal of medications as soon as possible until severe injury to organs and systems develops. To specify regularities in the development of MPA may be of value for the better diagnosis of the disease and the further elaboration of optimal treatment policy."}, {"id": "wiki20220301en243_17832", "title": "Gero Hütter", "score": 0.00909090909090909, "content": "to express wildtype CCR5 (because they hadn't been replaced yet from bone marrow precursors), also had no detectable virus. After 600 days without antiretroviral drug treatment, the patient's blood, bone marrow and bowel HIV levels were below the limit of detection; the virus was thought to be present in other tissues. However, the patient actually had a brain biopsy, in addition to biopsies of his intestines, liver, lymph nodes, bone marrow—basically, every part of the body that can be biopsied. All were negative for virus. There is no virus in this person's body out to two and a half years off of all anti-HIV drugs. His antibody levels—called titers—are declining just the way expected if the patient was vaccinated against HIV and then the levels of antibodies were examined. They'd be very strong in the beginning, but would weaken if they are not re-exposed to the virus. It is believed this patient has no HIV in his body and therefore there is nothing to re-expose him, so the"}, {"id": "pubmed23n0506_104", "title": "Unusual presentation of lupus nephritis.", "score": 0.00909090909090909, "content": "We report a male patient who presented with pyrexia, generalized lymphadenopathy, hepatosplenomegaly, and pleural effusion with no cutaneous or musculoskeletal symptoms. Despite extensive investigation, no cause was detected. His initial serology was also negative for autoantibodies. The patient was placed on a trial of antitubercular treatment in view of a positive Mantoux test. His disease evolved into the full clinical picture of systemic lupus erythematosus with nephritis (World Health Organisation class IV) and strongly positive antinuclear antibody and dsDNA over a period of months. He was treated successfully with intravenous cyclophosphamide pulses along with oral prednisolone, and the disease was still in remission after 3 years of follow-up."}, {"id": "pubmed23n0293_20390", "title": "[Avellis syndrome in systemic rheumatoid vasculitis].", "score": 0.009009009009009009, "content": "A 74-year-old man presented sudden onset hoarseness and dysphagia. Two months before this event, he had developed arthralgia of the shoulders, elbows, hands and foot and pleuritis which had been alleviated by a treatment with prednisolone. On admission, the patient could not phonate nor swallow at all. His soft palate was elevated at the right side. The uvula moved left when the patient tried to speak. Laryngoscopic examination revealed the paralysis of right vocal cord. The erythrocyte sedimentation rate (79mm/1h), C-reactive protein (5.3mg/dl), rheumatoid factor (310 IU/ml) and Clq-binding immune complex (4.5 micrograms/ml) were elevated. Hepatitis C virus antibody titer was more than 10.8 IU/l. Anti-nuclear antibody was 1:20 (normal < 1:20) and anti-neutrophil cytoplasmic antibody (p-ANCA) was positive. Blood study also revealed the evidences of hemolytic anemia and hypoproteinemia. Hepatitis B virus markers, cryoglobulin, anti-ds DNA, anti-Sm, anti-RNP, anti-SS-A, anti-SS-B antibodies were negative. Magnetic resonance imaging of the brainstem was normal. A sural nerve biopsy revealed patchy demyelination of the fascicles. The teasing of nerve fibers showed segmental demyelination. Chest X-ray showed the interstitial pneumonia and pleuritis in the right lower lung. Otological examination revealed the bilateral secretory otitis media. A treatment with high dose prednisolone, ciclosporin and cyclophosphamide was partially effective. However we could not continue these medication because of the induction of liver damage. The patient died of multi-organ failure around a year after the emergence of aphonia and dysphagia. The autopsy specimen of the right vagus nerve showed the similar patchy damage of nerve fibers as was observed in the biopsied sural nerve. The present case was diagnosed as systemic rheumatoid vasculitis. The syndrome of aphonia and dysphagia due to paralysis of the unilateral soft palate and vocal cord is called Avellis syndrome. This syndrome has been reported mainly in relation with the infarction of lateral medulla. The present case shows that Avellis syndrome can be produced by mononeuritis of the vagus nerve."}, {"id": "pubmed23n0760_16452", "title": "[Anti-synthetase syndrome].", "score": 0.009009009009009009, "content": "Antysynthetase syndrome is considered as a group ofidiopathic inflammatory myositis with charcteristic serologic hallmark--antibodies which recognise the aminoacyl-tRNA synthetasses (ARS). Clinical picture of those patients contains myositis and/or intersticial lung disease (ILD) and/or arthritis and/or fever and/or Raynaud phenomenon and sometimes characteristic look of mechanic's hands. Myositis can be overt, sometimes even absent, while IBP is major cause of morbidity and determines the outcome of the disease. Untill now eight different any-synthetase autoantibodies are recognised, and most frequent are findings of anti-histidyl-tRNa synthetase antibodies. Patients with other ARS autoantibodies usually have severe ILD. Drug of choice are steroids in dosage of 1 mg/kg with immunosupresive agent (azatioprin or methotrexate) while in severe IBP cyclophosphamide is needed. Recently succsesful treatment with rituximab in combination with cyclophosphamide is reported."}, {"id": "wiki20220301en011_6033", "title": "Rheumatology", "score": 0.008928571428571428, "content": "Treatment Most rheumatic diseases are treated with analgesics, NSAIDs (nonsteroidal anti-inflammatory drug), steroids (in serious cases), DMARDs (disease-modifying antirheumatic drugs), monoclonal antibodies, such as infliximab and adalimumab, the TNF inhibitor etanercept, and methotrexate for moderate to severe rheumatoid arthritis. The biologic agent rituximab (anti-B cell therapy) is now licensed for use in refractory rheumatoid arthritis. Physiotherapy is vital in the treatment of many rheumatological disorders. Occupational therapy can help patients find alternative ways for common movements that would otherwise be restricted by their disease. Patients with rheumatoid arthritis often need a long term, coordinated and a multidisciplinary team approach towards management of individual patients. Treatment is often tailored according to the individual needs of each patient which is also dependent on the response and the tolerability of medications."}, {"id": "pubmed23n0853_10391", "title": "A 28-year-old man with chest and joint pains.", "score": 0.008928571428571428, "content": "A 28-year-old man with extensive travel history to developing countries was hospitalised for intermittent sharp chest pains, worst when supine and with inspiration. Two weeks prior to presentation, he had suffered a flu-like illness with a sore throat, which was resolving. Physical examination was notable for mild fever and tachycardia with cervical lymphadenopathy and painful bilateral knee and wrist effusions. Cardiac auscultation was remarkable for a soft early-peaking systolic murmur over the aortic area with a decrescendo early diastolic murmur along the left sternal edge. There was mild leucocytosis, elevation of serum troponin and acute-phase reactants with an ECG showing sinus tachycardia. Echocardiographic windows were extremely limited but suggested the presence of pericardial effusion and aortic regurgitation. Cardiac MRI was performed (figure 1). Viral, microbiological and autoimmune testing was remarkable only for significant elevation of antistreptolysin-O titres (1450 IU rising to 1940 IU, normal <200 IU). Pericardiocentesis revealed an exudative effusion, which was negative by cytology and microbiological analysis, including for tuberculosis and fungi. The most appropriate next step is? Coronary angiographyEndomyocardial biopsyTreatment with colchicine for 3 monthsTreatment with corticosteroidsTreatment with high-dose salicylates and long-term penicillinFor the answer see page 808For the question see page 769."}, {"id": "pubmed23n0913_15886", "title": "Crescentic glomerular nephritis associated with rheumatoid arthritis: a case report.", "score": 0.008849557522123894, "content": "Rheumatoid arthritis is a systemic disorder where clinically significant renal involvement is relatively common. However, crescentic glomerular nephritis is a rarely described entity among the rheumatoid nephropathies. We report a case of a patient with rheumatoid arthritis presenting with antineutrophil cytoplasmic antibody-negative crescentic glomerular nephritis. A 54-year-old Sri Lankan woman who had recently been diagnosed with rheumatoid arthritis was being treated with methotrexate 10 mg weekly and infrequent nonsteroidal anti-inflammatory drugs. She presented to our hospital with worsening generalized body swelling and oliguria of 1 month's duration. Her physical examination revealed that she had bilateral pitting leg edema and periorbital edema. She was not pale or icteric. She had evidence of mild synovitis of the small joints of the hand bilaterally with no deformities. No evidence of systemic vasculitis was seen. Her blood pressure was 170/100 mmHg, and her jugular venous pressure was elevated to 7 cm with an undisplaced cardiac apex. Her urine full report revealed 2+ proteinuria with active sediment (dysmorphic red blood cells [17%] and granular casts). Her 24-hour urinary protein excretion was 2 g. Her serum creatinine level was 388 μmol/L. Abdominal ultrasound revealed normal-sized kidneys with acute parenchymal changes and mild ascites. Her renal biopsy showed renal parenchyma containing 20 glomeruli showing diffuse proliferative glomerular nephritis, with 14 of 20 glomeruli showing cellular crescents, and the result of Congo red staining was negative. Her rheumatoid factor was positive with a high titer (120 IU/ml), but results for antinuclear antibody, double-stranded deoxyribonucleic acid, and antineutrophil cytoplasmic antibody (perinuclear and cytoplasmic) were negative. Antistreptolysin O titer <200 U/ml and cryoglobulins were not detected. The results of her hepatitis serology, retroviral screening, and malignancy screening were negative. Her erythrocyte sedimentation rate was 110 mm in the first hour, and her C-reactive protein level was 45 mg/dl. Her liver profile showed hypoalbuminemia of 28 g/dl. She was treated with immunomodulators and had a good recovery of her renal function. This case illustrates a rare presentation of antineutrophil cytoplasmic antibody-negative crescentic glomerular nephritis in a patient with rheumatoid arthritis, awareness of which would facilitate early appropriate investigations and treatment."}, {"id": "pubmed23n0594_19445", "title": "[Interstitial granulomatous dermatitis with arthritis: four cases].", "score": 0.008849557522123894, "content": "Interstitial granulomatous dermatitis with arthritis, or Ackerman's syndrome, is characterized by inflammatory articular flares of potentially destructive outcome and cutaneous lesions of varied patterns: cord-like lesions on the flanks or violaceous plaques on the trunk and limbs. Histologically, the main features are histiocytes laminated between collagen fibres, sometimes organized in rosettes comprising tiny granulomas surrounding fibres of homogeneous aspect. Four patients fitting this description were investigated. Cutaneous lesions displayed various patterns: cord-like lesions (one patient), brown plaques on the flanks edged with papules (one patient), grossly annular violaceous plaques on the back (one patient), and infiltrated plaques on the palms and thighs (one patient). Joint symptoms were present in all cases (one seropositive rheumatoid arthritis and one seronegative rheumatoid arthritis, both already known; acute rheumatoid arthritis in two cases). The histological pattern was quite similar in all cases, with laminated histiocytes in palisading layers mixed with swollen collagen fibres; \"rosette\" figures were present in two cases. Autoimmune response was seen with antinuclear antibodies (two cases) and/or rheumatoid factor (two cases). The short-term outcome of cutaneous lesions was rapidly favourable in most cases with systemic steroids (one case) or Non Steroidal Anti-Inflammatory Drugs (one case), and spontaneously in one case. This entity, defined by the presence of inflammatory arthritis and cutaneous lesions of various clinical types, either more specific but infrequent (cord-like), or less specific but more frequent (plaque-like), and featuring a peculiar histological pattern, chiefly affects women aged between 30 and 80 years. Pathophysiological mechanisms and nosological borders are debatable and authors consider this condition as either an autonomous disease or else a mere subset of vasculitis with palisading granulomas in collagen vascular disorders, despite the rarity of authentic vasculitis. A setting of autoimmunity is frequently present. Furthermore, a fairly similar histological pattern is sometimes seen in some lesions forming a subset of cutaneous side-effects of drugs. Articular outcome may be unfavourable with joint destruction in more than half of patients, whether or not in a setting of rheumatoid arthritis. Spontaneous resorption can occur. Treatment has not yet been codified and is based primarily on anti-inflammatory drugs."}, {"id": "wiki20220301en400_28934", "title": "Antisynthetase syndrome", "score": 0.008771929824561403, "content": "Diagnosis In the presence of suspicious symptoms a number of test are helpful in the diagnosis: Muscle enzymes are often elevated, i.e. creatine kinase Anti-Jo-1 antibody testing Electromyography Muscle biopsy Pulmonary function testing Lung biopsy Imaging such as High Resolution computed tomography In certain situations, testing of other antibodies, specific imaging (MRI, thoracic high resolution computed tomography), and swallowing evaluation may be needed. Treatment Unfortunately, treatment for the anti-synthetase syndrome is limited, and usually involves immunosuppressive drugs such as glucocorticoids. For patients with pulmonary involvement, the most serious complication of this syndrome is pulmonary fibrosis and subsequent pulmonary hypertension. Additional treatment with azathioprine and/or methotrexate may be required in advanced cases. Prognosis Prognosis is largely determined by the extent of pulmonary damage. References External links"}]}}}} {"correct_option": 1, "explanations": {"1": {"exist": true, "char_ranges": [[0, 187]], "word_ranges": [[0, 27]], "text": "Given this clinical picture, one possibility is dermatomyositis. Of the proposed tests, the determination of serum aldolase may be useful. Its elevation is characteristic of this disease."}, "2": {"exist": false, "char_ranges": [], "word_ranges": [], "text": ""}, "3": {"exist": true, "char_ranges": [[188, 239]], "word_ranges": [[27, 35]], "text": "The biopsy should be muscular and not subcutaneous."}, "4": {"exist": true, "char_ranges": [[240, 309]], "word_ranges": [[35, 44]], "text": "Anti-smooth muscle antibodies are not characteristic of this disease."}, "5": {"exist": false, "char_ranges": [], "word_ranges": [], "text": ""}}, "full_answer": "Given this clinical picture, one possibility is dermatomyositis. Of the proposed tests, the determination of serum aldolase may be useful. Its elevation is characteristic of this disease. The biopsy should be muscular and not subcutaneous. Anti-smooth muscle antibodies are not characteristic of this disease.", "full_answer_no_ref": "Given this clinical picture, one possibility is dermatomyositis. Of the proposed tests, the determination of serum aldolase may be useful. Its elevation is characteristic of this disease. The biopsy should be muscular and not subcutaneous. Anti-smooth muscle antibodies are not characteristic of this disease.", "full_question": "A 75-year-old woman consults for violaceous lesions on the hands and neck together with progressive muscle weakness of 3 months of evolution. What diagnostic tests, among those indicated, can be useful for the diagnosis?", "id": 153, "lang": "en", "options": {"1": "Determination of serum aldolase.", "2": "Electroencephalogram.", "3": "Biopsy of subcutaneous cellular tissue.", "4": "Determination of anti-smooth muscle antibodies.", "5": "Genetic study of their descendants."}, "question_id_specific": 74, "type": "RHEUMATOLOGY", "year": 2012, "rag": {"clinical_case_options": {"MedCorp": {"RRF-2": [{"id": "wiki20220301en065_36497", "title": "Hereditary inclusion body myopathy", "score": 0.013478025797482042, "content": "Diagnosis The most useful information for accurate diagnosis is the symptoms and weakness pattern. If the quadriceps are spared but the hamstrings and iliopsoas are severely affected in a person between ages of 20 - 40, it is very likely HIBM will be at the top of the differential diagnosis. The doctor may order any or all of the following tests to ascertain if a person has IBM2: Blood test for serum Creatine Kinase (CK or CPK); Nerve Conduction Study (NCS) / Electomyography (EMG); Muscle Biopsy; Magnetic Resonance Imaging (MRI) or Computer Tomography (CT) Scan to determine true sparing of quadriceps; Blood Test or Buccal swab for genetic testing;"}, {"id": "wiki20220301en032_90409", "title": "Becker muscular dystrophy", "score": 0.013436713485426646, "content": "In terms of the diagnosis of Becker muscular dystrophy symptom development resembles that of Duchenne muscular dystrophy. A physical exam indicates lack of pectoral and upper arm muscles, especially when the disease is unnoticed through the early teen years. Muscle wasting begins in the legs and pelvis, then progresses to the muscles of the shoulders and neck. Calf muscle enlargement (pseudohypertrophy) is quite obvious. Among the exams/tests performed are: Muscle biopsy (removes a small piece of muscle tissue, usually from the thigh, to check for dystrophin in muscle cells.) Creatine kinase test (checks the level of Creatine Kinase proteins in the blood. Creatine Kinase proteins are normally found inside of healthy muscle cells, but can be found in the blood when muscle cells are damaged.) Electromyography (shows that weakness is caused by destruction of muscle tissue rather than by damage to nerves.)"}, {"id": "pubmed23n1106_16797", "title": "[Anti-nuclear matrix protein 2 antibody-positive dermatomyositis with the preferential involvement of neck extensors: a case report].", "score": 0.012950971322849215, "content": "A 68-year-old man with a 2-month history of progressive weakness and spontaneous pain in proximal limb muscles presented to our hospital with a dropped head. He started experiencing progressive dysphagia several days before admission. On admission, he had muscle weakness of the limbs and neck extensors with edema and induration in distal extremities. Laboratory tests showed elevation of muscle enzymes. FDG-PET/CT demonstrated multiple hypermetabolic lymph nodes, but the primary site was not identified; thus, metastatic carcinoma of unknown primary origin was considered. The patient was diagnosed with anti-nuclear matrix protein 2 antibody-positive paraneoplastic myopathy based on serum tests. Histological findings of the left biceps brachii muscle biopsy revealed severe variation in fiber size and perifascicular myofiber atrophy. Myofibers exhibited myxovirus resistance protein A expression predominantly in the perifascicular region. Following intravenous methylprednisolone pulse therapy and intravenous immunoglobulin, the patient's muscle strength improved with normalization of muscle enzyme levels. The dropped head was considered to have resulted from the preferential involvement of neck extensors based on the observed FDG-PET/CT uptake in neck extensors."}, {"id": "wiki20220301en000_241100", "title": "Inclusion body myositis", "score": 0.01274876774265278, "content": "A diagnosis of inclusion body myositis was historically dependent on muscle biopsy results. Antibodies to cytoplasmic 5'-nucleotidase (cN1A; NT5C1A) have been strongly associated with the condition. In the clinical context of classic history and positive antibodies, a muscle biopsy might be unnecessary. Differential diagnosis IBM is often initially misdiagnosed as polymyositis. A course of prednisone is typically completed with no improvement and eventually, sIBM is confirmed. sIBM weakness comes on over months or years and progresses steadily, whereas polymyositis has an onset of weeks or months. Other forms of muscular dystrophy (e.g. limb girdle) must be considered as well."}, {"id": "wiki20220301en281_30724", "title": "Multi/minicore myopathy", "score": 0.012167874396135266, "content": "Other features include a long head, low set ears and a short neck. The respiratory muscles can be moderately to severely affected and problems with breathing are common. Genetics The most common causes are mutations in the RYR1 and SEPN1 genes. In these cases the inheritance is autosomal recessive. Less common recessive mutations causing this condition include those in the TTN, MEGF10 and CACNA1S genes. Automsomal dominant mutations associated with this disease include those in MYH7 and CACNA1S. The pathogenesis is not well understood at present. Diagnosis The diagnosis may be suspected on clinical grounds. On blood testing the creatinine kinase may be raised. Imaging with ultrasound or MRI will show abnormalities in the affected muscles but these changes are not diagnostic. The diagnostic test is a muscle biopsy."}, {"id": "wiki20220301en042_31047", "title": "Fukuyama congenital muscular dystrophy", "score": 0.009900990099009901, "content": "Diagnosis In terms of diagnosis of Fukuyama congenital muscular dystrophy, serum creatine kinase concentration and muscle biopsies can be obtained to help determine if the individual has FMCD. FKTN molecular genetic testing is used to determine a mutation in the FKTN gene after a serum creatine kinase concentration, muscle biopsies, and/or MRI imaging have presented abnormalities indicative of FCMD, the presence of the symptoms indicates Fukuyama congenital muscular dystrophy. The available genetic test include: Linkage analysis Deletion analysis Sequence analysis - exons Sequence analysis - entire coding region Treatment"}, {"id": "pubmed23n0093_16117", "title": "[A case of polymyositis with repeated dysphagia and muscle weakness associated with peculiar findings of skin].", "score": 0.009900990099009901, "content": "A case of polymyositis with repeated dysphagia and muscle weakness associated with peculiar findings of skin was reported. The patient was a 67-year-old man. His birth and development was normal. There was no family history of neuromuscular disease. On 26th March 1987 he was admitted to a hospital because of dysarthria and dysphagia after fever and diagnosed as having viral myositis. His conditions improved spontaneously with bed rest and he left hospital on 14th April. On 23rd April he had chill and sore throat with fever. On 27th he was admitted to the same hospital because of dysarthria and muscle weakness of the proximal portion of the upper limbs. These symptoms also improved with bed rest. He had repeated these symptoms several times and then he was admitted to our hospital on 12th June. On examination he showed the skin pigmentation under the right eye and the eruption in the back of hands and the buttocks. Muscle weakness was observed in the proximal portion of the upper limbs and the neck flexor. Laboratory tests in admission were as follows: sGOT 49 mU/ml, sGPT 104 mU/ml, LDH 1064 mU/ml, CPK 565 mM/ml, aldolase 25.2 IU/1/37 degrees C. Electromyography showed the typical myogenic changes and biopsy of left biceps brachii revealed inflammatory cells in the muscle fiber which are specific to polymyositis. Immuno-histochemical study is performed to analyse the subpopulation of mononuclear cells in biopsied muscle and skin. Mononuclear cells infiltrated into perimysium, endomysium and epidermis were positive for T11 and T8, but less positive for T4, B1 and Leu11. On the basis of these findings he was diagnosed as having \"polymyositis syndrome\"."}, {"id": "wiki20220301en093_17532", "title": "Neuromuscular disease", "score": 0.009849310094408134, "content": "Further causes of neuromuscular diseases are : Inflammatory muscle disorders Polymyalgia rheumatica (or \"muscle rheumatism\") is an inflammatory condition that mainly occurs in the elderly; it is associated with giant-cell arteritis(It often responds to prednisolone). Polymyositis is an autoimmune condition in which the muscle is affected. Rhabdomyolysis is the breakdown of muscular tissue due to any cause. Tumors Smooth muscle: leiomyoma (benign) Striated muscle: rhabdomyoma (benign) Diagnosis Diagnostic procedures that may reveal muscular disorders include direct clinical observations. This usually starts with the observation of bulk, possible atrophy or loss of muscle tone. Neuromuscular disease can also be diagnosed by various blood tests and using electrodiagnostic medicine tests including electromyography (measuring electrical activity in muscles) and nerve conduction studies. Genetic testing is an important part of diagnosing inherited neuromuscular conditions."}, {"id": "wiki20220301en601_17481", "title": "Calpainopathy", "score": 0.00980392156862745, "content": "With calpain 3 mutation, proteins typically found at the triad are reduced, including CaMKII (Ca2+/calmodulin-dependent protein kinase II). Decreased CaMKII activity impairs induction of slow oxidative gene expression, which in turn impairs genes involving the mitochondria and lipid metabolism. Diagnosis Genetic testing is the most definitive test. If genetic testing is not available, a muscle biopsy with protein immunoanalysis can be used. Biopsy shows general dystrophic features, such as areas of muscle death, variability in muscle size, nuclei in the center of muscle fibers, and disorganized muscle fibers within muscle cells. Serum creatine kinase, a nonspecific marker of muscle damage, can be elevated early in the disease. Facioscapulohumeral muscular dystrophy (FSHD) can present similarly, although facial weakness and asymetrical weakness is common in FSHD. Management As of 2019, no disease modifying pharmaceuticals are known."}, {"id": "pubmed23n0135_17694", "title": "[Subcutaneous localizations of Castleman's pseudolymphoma. Review of the literature apropos of a case].", "score": 0.00980392156862745, "content": "The angiofollicular lymphoid hyperplasia, first described in 1954 by Castleman in the mediastinum, is a quite rare pseudolymphoma where there are few subcutaneous localizations. Since 1954, more than 300 observations were published including mediastino-pulmonary forms (about 60 p. 100 of the cases) intra-abdominal forms (15 p. 100 of the cases) and superficial forms which represent 25 p. 100 of the cases and associate superficial ganglionic, intra-muscular and subcutaneous localizations. The authors report the observation of a 44-year-old negro who had a subcutaneous tumefaction of the left elbow which appeared recently without a functional sign nor a biological change. The histological findings allowed the diagnosis of Castleman's pseudolymphoma in a hyalino-vascular form or Flendrig's type II. The evolution was marked a few weeks later by a local recurrence of which a second surgery has secured the recovery. The detailed study of the 76 cases of Castleman's superficial pseudo-tumours published in the literature allows us to recall the features of this disease which affects especially the young adult without prevalence of sex at about 25 years old. The circumstances of discovery are univocal, isolated palpable subcutaneous tumefaction in most of the cases. The localizations are distributed by decreasing incidence as following: latero-cervical, axillary, sus-clavicular, inguinal, vulvar, abdominal wall, shoulder, arm, forearm with a few bifocal forms. The histological aspect associated a predominant lymphoid population and vessels with fibro-hyalinous wall which morphological variations have permitted to individualize three forms: a plasmocytic form or Flendrig's type I which should be a stage of beginning often associated with hematological changes, a hyalino-vascular form or Flendrig's type II more frequent and a mixed form or intermediary type. The immunofluorescence, histo-enzymology and immunohistochemistry studies reveal a changeable polyclonal plasmocytosis and a predominance of T-suppressors in the lymphocytic population. The histological differential diagnosis of the superficial forms of the Castleman's pseudolymphoma is rarely set with certain lymphoma in case of ganglionic localization. On the other hand isolated subcutaneous localizations must be distinguished of the Kimura's disease and of the angiolymphoid hyperplasia with eosinophils where the vessels have a different morphology. The evolution is favorable in most of the cases and surgical exeresis insures the recovery.(ABSTRACT TRUNCATED AT 400 WORDS)"}, {"id": "wiki20220301en572_13748", "title": "Muscle–eye–brain disease", "score": 0.009708737864077669, "content": "Diagnosis Medical diagnosis for the MEB usually involves the study of family history, measurement of serum CPK level, molecular testing, muscle biopsy and imaging study. Physical examination People with MEB have distinctive facial dysmorphisms. Rounded forehead, thin and drooping lip, micrognathia, midface retrusion, short nasal bridge are the possible indicative evidence for diagnosis. Assessment of motor and mental development, visual ability also provide clues. Genetic test Genetic test can analyze the genome of infants for confirmation of the specific genetic mutation. Mutation in the POMGNT1 is the determinant in the diagnosis of MEB. Several mutations like [c.1539+1G→A], [c.879+5G→T] are the prevalent nucleotide change found in affected people. The commonly used practices collect fetal DNA by chronic villus sampling, followed by linkage analysis and direct sequencing to conclude the POMGNT1 gene sequence."}, {"id": "pubmed23n0621_19682", "title": "[Diagnosis of a myopathic disease in adult].", "score": 0.009708737864077669, "content": "Strategy for the diagnosis of a muscle weakness includes two steps: to rely the weakness to a muscular origin and to find the aetiology. A muscular deficit is purely motor, without sensory signs, involving mainly axial and proximal muscles. The essential informations for aetiological characterization are: 1st) a family history, indicating a genetic origin, 2nd the chronological profile, 3rd) the clinical pattern (deficit topography, modification of muscle volume, fatigability, contractures, myotonia, oculobulbar, respiratory or cardiac involvement), 4th) investigations (CK level, EMG, muscle imaging muscle, biopsy, genetic testing). Two autosomal dominant myopathies begin in adulthood: Steinert's myotonic dystrophy characterized by myotonia facial and distal weakness and atrophy, plurisystemic involvement and facio-scapulo-peroneal dystrophy with asymmetric facial and scapulo-humeral weakness. If the evolution is rapid and family history absent, a curable myopathy (inflammatory, toxic, iatrogenic, and endocrine) is to be looked for. Inclusion body myositis is the most frequent myopathy after the age of 50 years."}, {"id": "pubmed23n0559_9392", "title": "Dermatomyositis with panniculitis.", "score": 0.009615384615384616, "content": "Case 1. A 23-year-old white housewife presented with an erythematous violaceous rash on her face, neck, chest, and limbs, particularly over the dorsum of the hands and fingers; diffuse alopecia; and an inability to climb stairs and get up from a low seat. The clinical examination showed red to violaceous well-demarcated plaques on sun-exposed areas on the dorsum of the fingers and hands, with periungual erythema and telangiectasia; facial erythema; and heliotrope rash. There was also symmetric involvement of proximal muscles of the limbs. Laboratory examination showed hypergammaglobulinemia, elevated serum aspartate aminotransferase, and serum alanine aminotransferase; normal activities of creatinokinase, lactate dehydrogenase, and aldolase; an antinuclear antibody titer of 1:40 with a speckled pattern; negative anti-DNA and anti-Scl70; and normal serum complement levels (C3, C4, and CH50). Urinalysis results were within normal limits. Skin biopsy histopathology showed hyperkeratosis, edema of the upper epidermis, scattered inflammatory infiltrate, and focal accumulation of mucin in the form of acid mucopolysaccharides. Deep asymptomatic nodules on the inner upper limbs appeared later. Histopathology of these lesions showed focal areas of lobular panniculitis in the subcutaneous tissue, with lymphoplasmocytic inflammatory infiltrate without vasculitis (Figure 1 and Figure 2). Case 2. A 29-year-old white housewife presented with an erythematous violaceous rash on her face, neck, chest, and lower extremities. Clinical examination showed red to violaceous well-demarcated aching plaques on the internal surface of the thighs and tips of the fingers; periungual erythema and digital petechiae; Raynaud's phenomenon; and bilateral ulnar and cervical enlarged lymph nodes. Laboratory examination showed elevated serum aspartate aminotransferase, alanine aminotransferase, creatinokinase, lactate dehydrogenase, and aldolase; negative venereal disease research test results; an antinuclear antibody titer of 1:1024 with speckled pattern; negative anti-DNA and anti-Scl70; and normal serum complement levels (C3, C4, and CH50). Urinalysis results were within normal limits. Histopathology of the deep asymptomatic nodule on the inner left thigh showed lobular panniculitis with a scattered inflammatory infiltrate and diffuse fat necrosis, in addition to calcium deposition between the lipocytes and microcysts without vasculitis (Figure 3)."}, {"id": "pubmed23n0060_9591", "title": "[Congenital muscular dystrophy: clinical study of 17 patients].", "score": 0.009615384615384616, "content": "We concur with the idea that congenital muscular dystrophy (CMD) is a distinct clinical entity, and report 17 patients (2 negroes and 15 whites; 12 M and 5 F; median age 6 years, range 1 to 24 years) with genetic, clinical, laboratorial, electrophysiological and histochemical studies. All our cases have an inheritance compatible with an autosomal recessive pattern. A decrease in fetal movements was reported by 57% of the mothers, generalized hypotonia at birth was present in 82%, limb girdle and neck weakness, absent or decreased deep tendon reflexes, and limb contractures were present in all. Severe muscular wasting was found in 41%. Calf pseudo-hypertrophy was observed in one patient. A patient was severely mentally retarded and another was borderline. During a 30-month follow-up, the muscle weakness of the majority remained essentially unchanged but the degree of motor activity deteriorated and was proportional to the worsening of the limb contractures. Serum CK levels were normal or increased to a maximum of 8 times. The electromyogram was myopathic in 74%, neurogenic in 13% and normal in 13%. CT scans showed a symmetrical white matter hypodensity in the hemispheres in 8 cases. All but 5 patients were operated upon to release the limb contractures and all were submitted to physical therapy. The contractures recurred in 4 patients submitted to surgery and were probably related to the cessation of physical therapy."}, {"id": "wiki20220301en244_24488", "title": "Metabolic myopathy", "score": 0.009523809523809525, "content": "Diagnosis The symptoms of a metabolic myopathy can be easily confused with the symptoms of another disease. In most cases, a Muscle biopsy is necessary for an accurate diagnosis of the cause of muscle weakness. A blood test can be done under normal circumstances to test for genetic differences and signs of tissue breakdown, or with an added cardio portion that can indicate if muscle breakdown is occurring. An electromyography is sometimes taken in order to rule out other disorders if the cause of fatigue is unknown. Differentiating between different types of metabolic myopathies can be difficult due to the similar symptoms of each type such as Myoglobinuria and exercise intolerance. It has to be determined whether the patient has fixed or exercise induced manifestations, and if exercise related what kind of exercise, before extensive exercise related lab testing is done to determine the underlying cause."}, {"id": "pubmed23n0043_12307", "title": "[A case of chronic multifocal myositis].", "score": 0.009523809523809525, "content": "A 61-year-old civil engineer began to have slowly progressive muscle atrophy in the right shoulder and the left arm at 56 years of age. Muscle wasting became manifest in the left thigh at 59 years and in the right thigh at 60 years. He had mild difficulty in climbing and descending stairs. On examination, although he had notable muscle atrophy in the right trapezius and proximal muscles in the upper and lower extremities, his muscle strength was relatively well preserved. The muscle atrophy was asymmetrical; the right periscapular region and the left upper and lower extremities were more markedly atrophic. In addition, multiple foci of the striking muscle atrophy were noted in the upper trunk and the proximal limb muscles. Fasciculation was not present. Deep tendon reflexes were normal with no pathologic reflexes. Except for a moderately elevated serum creatine kinase level of 709 Ul/l (normal 40-170) and mildly elevated serum myoglobin level of 100 ng/ml (normal < 60), no laboratory tests showed abnormal values suggesting an inflammatory process. Motor and sensory nerve conduction velocities were within normal limits. Electromyography disclosed myopathic and neuropathic changes. Computed tomography (CT) of skeletal muscles showed asymmetrical muscle atrophy and patchy low-density foci. In biopsied left quadriceps and right gastrocnemius muscles which showed partially low density on CT, there was marked variation in muscle fiber size, with necrotic and regenerating fibers, an increased number of centrally placed nuclei, and interstitial fibrosis. There were numerous foci of mononuclear inflammatory cellular infiltration, especially around the blood vessels.(ABSTRACT TRUNCATED AT 250 WORDS)"}, {"id": "Neurology_Adams_11294", "title": "Neurology_Adams", "score": 0.009522438402369297, "content": "In addition to these main issues of distinguishing PM and DM from IBM, currently aided by antibody testing, we call attention to the following problems that we have encountered in connection with diagnosis: 1. The patient with proximal muscle weakness is incorrectly diagnosed as having progressive muscular dystrophy (actually, the opposite pertains more often). Points in favor of myositis are (1) lack of family history (although many dystrophies have recessive inheritance); (2) older age at onset; (3) rapid evolution of weakness; (4) evidence, past or present, of other connective tissue diseases; (5) high serum CK values (again, can be high in certain dystrophies); (6) marked degeneration and regeneration in muscle biopsy; and, finally, if there is still doubt, (7) unmistakable improvement with corticosteroid therapy."}, {"id": "wiki20220301en128_50746", "title": "Equine polysaccharide storage myopathy", "score": 0.009433962264150943, "content": "A muscle biopsy may be taken from the semimembranosis or semitendinosis (hamstring) muscles. The biopsy is stained for glycogen, and the intensity of stain uptake in the muscle, as well as the presence of any inclusions, helps to determine the diagnosis of PSSM. This test is the only method for diagnosing Type 2 PSSM. Horses with Type 1 PSSM will usually have between 1.5-2 times the normal levels of glycogen in their skeletal muscle. While abnormalities indicating muscle damage can be seen on histologic sections of muscle as young as 1 month of age, abnormal polysaccharide accumulation may take up to 3 years to develop. Management"}, {"id": "pubmed23n0318_19140", "title": "[A case of fasciitis associated with Basedow's disease and polymyositis].", "score": 0.009433962264150943, "content": "A 39-year-old female suffered from diffuse goiter, palpitation, finger tremor and body weight loss for about one year. Then she developed acute onset of myalgia and swelling of calves, and muscle weakness of proximal limbs. She could not walk because of myalgia and muscle weakness, and was admitted to our hospital 4 days after the onset of muscle symptoms. On admission, her pulse was 110 per minute and she had finger tremor of 11-12 Hz. The thyroid gland was markedly and diffusely enlarged with an elastic soft surface. She presented muscle weakness of proximal limbs and neck, and had intermittent swelling and myalgia on calves. Deep tendon reflexes were increased in all extremities. The erythrocyte sedimentation rate was 22 mm per hour. Eosinophilia was not recognized. Serum CK level was elevated to 671 IU/l. Serum free T3 was higher than 21.7 pg/ml and free T4 was also elevated to 10.19 ng /dl. Serum TSH was lower than 0.05 microU/ml and thyroid stimulating antibody was 1,302.0%. Muscle biopsy of her left gastrocnemius muscle revealed markedly hypertrophic fascia with inflammatory cellular infiltration on HE staining. Inflammatory change was also recognized in muscle tissue and in perivascular region of perimysium. Variation of fiber size, necrotic fibers, and central nuclei were also seen. From these clinical and laboratory findings she was diagnosed as having Basedow's disease associated with fasciitis and polymyositis. Her thyroid function was improved by anti-thyroid drug, and swelling and myalgia of sural regions and weakness of proximal limbs were also improved by steroid therapy. Only one case of Basedow's disease associated with fasciitis and seven cases of that associated polymyositis have so far been reported. This is the first case report of fasciitis associated with Basedow's disease and polymyositis."}, {"id": "pubmed23n0076_3609", "title": "[A case of senile onset rimmed vacuole myopathy with proximally dominant involvement].", "score": 0.009345794392523364, "content": "A 73-year-old woman with progressive proximal-dominant muscular atrophy and weakness was described. She had been well until 70-year-old, when she found difficulty in standing up from sitting position. At age 72 years, she could not raise her arms. Neurological examination showed muscular wasting and weakness in the proximal parts of extremities, shoulder and pelvic girdle. In the thigh, the flexors and adductors were severely affected. Muscular weakness was also observed in m. tibialis anterior. Serum CK and aldolase were normal. Electromyography showed low voltage short duration motor unit potentials with positive sharp waves and fibrillations. Rimmed vacuoles were observed in 4.8% of muscle fibers in biopsy sample obtained from right m. quadriceps femoris. No inflammatory cells, PAS-positive materials and inclusion bodies were observed in the sample. This case differs from distal myopathy with rimmed vacuoles, because the onset was very late and her muscular weakness and atrophy was proximal dominant. This case also differs from inclusion body myositis, because muscle biopsy revealed no inflammatory cells or inclusion body."}, {"id": "wiki20220301en435_2629", "title": "Schwartz–Jampel syndrome", "score": 0.009259259259259259, "content": "Diagnosis Schwartz–Jampel syndrome is diagnosed on the basis of characteristic facial features, skeletal features and myotonia. Blood tests may show elevated serum creatine kinase or aldolase. X-rays, muscle biopsy or electromyography (EMG) may be useful. Genetic testing for the HSPG2 gene may confirm diagnosis. Treatment There is no cure for Schwartz–Jampel syndrome. Treatment is aimed at reducing muscle stiffness and cramping and may include massage, muscle warming and gradual strengthening exercises. Muscle relaxants or anti-seizure medications, especially carbamazepine, may be used. Eye symptoms such as blepharospasm might be relieved by Botox. Otherwise, a variety of surgical procedures have been found to be effective. Malignant hyperthermia, a potential complication of surgery, is a greater risk for people Schwartz–Jampel syndrome and an important consideration when considering surgery. Prognosis Most people with Schwartz–Jampel syndrome have a nearly normal life expectancy."}, {"id": "pubmed23n0220_14234", "title": "[Apropos of a case of myopathy with histologic and electrophysiologic findings of both a myogenic and neurogenic nature].", "score": 0.009259259259259259, "content": "The authors describe the case of a 36 years old woman suffering from muscular weakness with proximo-distal distribution to legs, and proximal to arms. The disease, appeared during the third decade of life, is slowly becoming more serious. Hematochemical analyses are all within a normal standard; EMG and histopathologic findings prove the existence of both a protopathic and neurogenic trouble in studied muscles. A therapy based on prednisone (50 mg/die) and ACTH (1 mg each 5 days) for a fourty days period doesn't cause essential changes in symptomatology. A cousin (on her mother's side) of our patient suffers from probable \"sporadic distal myopathy\". The authors discuss if the disease, shown by this patient, may be considered as an atypical form of SDM or if, what appears more probable, it must be nosographically framed as \"scapuloperoneal atrophy\"."}, {"id": "wiki20220301en330_16141", "title": "Brody myopathy", "score": 0.009174311926605505, "content": "Blood testing may be used to measure serum creatine kinase, which ranges from normal to slightly elevated in those with BD. Skeletal muscle biopsies are used to examine muscle fibers. Biopsies in individuals with BD often show variation in muscle fiber size, atrophied fast-twitch muscle fibers, and increased nuclei number. Electromyography (EMG) can be used in diagnosis to rule out myotonia, or muscle stiffness that is detected by EMG. Individuals with BD have stiff muscles but normal EMG results (pseudo-myotonia), where no myotonic discharges are detected. Genetic testing may also be used in the diagnosis of BD to look for mutations in ATP2A1. Since only some forms of the disease are associated with ATP2A1, results of genetic testing do not always confirm a diagnosis of BD, but are useful to rule out other similar disorders. Treatment"}, {"id": "pubmed23n0325_15644", "title": "[Diagnostic muscle MRI abnormality in a patient with inclusion body myositis].", "score": 0.009174311926605505, "content": "A 64-year-old woman was admitted to our hospital because of muscle weakness and atrophy in the extremities. Four years before admission, he was noticed to have elevated creatine kinase (CK) level, but had no further evaluation. Two years later, she became difficult in standing up and needed a wheelchair. Six months before admission, she noticed muscle wasting in the buttock, thigh, bilateral forearms, and weakness in the upper limbs. On neurologic examination, she had weakness in sternocleidomastoid and all limb muscles, predominantly in the distal portion of the upper extremities. Laboratory study revealed elevated CK, LDH, and aldolase levels, and myogenic change with fibrillation on needle EMG. Muscle biopsy showed myopathic changes with infiltration of mononuclear cells and rimmed vacuoles. The clinical manifestations as well as poor response to corticosteroids therapy were supportive of the diagnosis of inclusion body myositis. However, the distribution of muscle weakness in her wrist, weaker in the extensors than in the flexors, was not characteristic to IBM. This problem was solved by the right forearm MRI which showed a high signal intensity area in flexor muscles, but not in extensors on T1 and T2 weighted images. Accordingly, the muscle MRI of forearm was a diagnostic aid of IBM in this patient."}, {"id": "wiki20220301en082_3387", "title": "MELAS syndrome", "score": 0.00909090909090909, "content": "Diagnosis MRI: Multifocal infarct-like cortical areas in different stages of ischemic evolution, areas that do not conform to any known vascular territory. Initial lesions often occur in the occipital or parietal lobes with eventual involvement of the cerebellum, cerebral cortex, basal ganglia, and thalamus. Lactate levels are often elevated in serum and cerebrospinal fluid. MR spectroscopy may show an elevated lactate peak in affected and even unaffected brain areas. Muscle biopsy shows ragged red fibers. However, genetic evaluation should be done first, which eliminates the need for muscle biopsy in most cases. Diagnosis may be molecular or clinical: Stroke-like episodes before 40 years old Encephalopathy with seizures or dementia Blood lactic acidosis* or ragged red fibers on muscle biopsy"}, {"id": "pubmed23n0070_6846", "title": "[Hypothyroid myopathy. Clinico-pathologic study of 20 cases].", "score": 0.00909090909090909, "content": "20 patients afflicted with primary hypothyroidism were studied in order to evaluate the association of clinical or sub-clinical myopathy, detected by neurophysiological (electromyography) (EMG) or neuropathological methods (muscular biopsy with enzymatic study). 70% of the patients had muscular weakness (moderate in 30% and severe in 40%) of the scapular and pelvic muscles. 60% of the patients had muscular cramps. There was no myodema nor muscular atrophy or hypertrophy. Seric CPK was high in 70% of the cases. EMG was myopathic in 65%. All cases with weakness registered EMG alterations. The histological findings were import findings were important. The enzymatic techniques showed alterations of the fiber subtypes in 90% of the cases. The type I fibers had sarcolemmal and mitochondrial accumules in 85% and 70% had areas without oxidative activity, similar to \"core\". In this study, we did not find any correlation between the evolution time of hypothyroidism, hormonal levels, CPK increase, and muscular weakness. The EMG was myopathic in cases with severe weakness, however, in patients with moderate weakness it could also prove abnormal. There was no correlation between the electric myopathic pattern, CPK levels and thyroid hormones."}, {"id": "wiki20220301en051_31863", "title": "Nemaline myopathy", "score": 0.009009009009009009, "content": "There are several other identified kinds of mutations that lead to Nemaline Myopathies. One affects slow skeletal muscles, one leads to the formation of both nemaline bodies and other abnormal, core-like, structures forming in the patient's muscles. Diagnosis Electromyography or (EMG). This procedure determines if nerve or muscle cells are damaged. Since a common symptom of Nemaline Myopathy is muscle weakness this allows doctors to determine where and why the weakness is occurring. MRI of the Musculoskeletal System. MRI uses a magnetic field to take pictures of body structures and allows physicians to determine if a patient has a certain disease. Needle biopsy A needle biopsy allows a physician to test specific cells in the body. These cells are sent to a laboratory to undergo testing and can further determine why muscle weakness throughout the body could be occurring. This testing can confirm that muscle cells contain rod like structures."}, {"id": "pubmed23n0041_6300", "title": "Congenital muscular dystrophy: a clinico-pathological and follow-up study of 15 patients.", "score": 0.009009009009009009, "content": "Fifteen patients with a presumptive diagnosis of congenital muscular dystrophy were followed for up to 15 years. The diagnosis was based on clinical, enzyme, histological and neurophysiological examinations. The group formed nine per cent of the 160 children suffering from neuromuscular disorders seen at the same hospital during a period of ten years. The muscle weakness was generalized and also involved respiratory muscles and the face. 60 per cent of the children had congenital contractures; these were well amenable to treatment. However, there was a strong tendency for new contractures to form from the second to third year onwards. There were also other signs indicating that the disease process was changing with time. The deep tendon reflexes were present in the beginning but later were usually lost. The serum creatine kinase was raised even to high levels in the first one to two years and gradually sank to normal or near normal values. The histopathological findings changed with time from relatively slight changes compatible with a muscle destroying process to inactive type lesions characterized by fibrotic and particularly adipose tissue replacing muscle fibres. On the basis of these findings it can be assumed that the active disease process is at its height during intrauterine and early postnatal life and then wanes leaving an outburnt or cicatrical state in which new contractures easily develop causing possible deterioration with time. Active treatment is thus of great importance both to overcome neonatal contractures and to prevent new ones to develop."}, {"id": "wiki20220301en323_9175", "title": "Acquired non-inflammatory myopathy", "score": 0.008928571428571428, "content": "If needed, more advanced equipment can be used to help determine whether a patient is suffering from ANIM. This includes: Measurement of serum levels of muscle enzymes Electromyography (EMG) Magnetic Resonance Imaging (MRI) Muscle biopsy When examining the serum levels of muscle enzymes, the relative levels of creatine kinase, aldolase, aspartate aminotransferase, alanine aminotransferase, and lactate dehydrogenase are closely examined. Abnormal levels of these proteins are indicative of both inflammatory myopathy and ANIM."}, {"id": "pubmed23n0668_6610", "title": "[Inflammatory myopathy with an unusual evolution].", "score": 0.008928571428571428, "content": "Inflammatory myopathies are a heterogeneous group of conditions characterized by proximal muscle weakness, nonsuppurative inflammation of skeletal muscle, with elevated muscle enzyme levels and characteristic electromyography and muscle biopsy findings. The authors describe a clinical case of a young woman, admitted with a four day history of bilateral thigh myalgia. She was afebrile and without skin, mucosal or joint involvement. Thigh muscle palpation was painful. Complete blood count revealed leukopenia and thrombocytopenia. High levels of creatine kinase, serum aminotransferases and myoglobin were detected. Metabolic, toxic and drug-related causes were excluded as well as infectious diseases, malignant tumours and endocrine myopathies. Auto-antibodies for connective diseases were negative. Magnetic resonance imaging and electromyography of lower limbs were suggestive of inflammatory myopathy. Generalized muscle weakness and dysphagia were reported subsequently. Clinical and laboratorial improvement was seen after corticotherapy. Muscle biopsy revealed myopathy signs without inflammatory changes or vasculitis. After prednisolone reduction, presently without treatment, she remains asymptomatic with normal laboratorial findings. The authors emphasize in this case of inflammatory myopathy the unusual clinical and laboratory evolution and the importance of a cautious differential diagnosis."}, {"id": "pubmed23n0997_17604", "title": "[Smooth muscle tumors of uncertain malignant potential].", "score": 0.008849557522123894, "content": "To investigate microsatellite instability in smooth muscle tumors of uncertain malignant potential and to compare the results with clinical and morphological data. Histological and immunohistochemical studies were conducted in 26 patients aged 30-63 years (mean age, 37 years) with leiomyomatosis; which revealed intravenous leiomyomatosis in 20 cases, metastasizing leiomyoma in 2, disseminated peritoneal leiomyomatosis in 3, and smooth muscle tumor of uncertain malignant potential in 1 case. Microsatellite instability was studied by fragment analysis on a genetic analyzer using a test system of six markers: D10S1146, D10S218, D10S24, D10S1213, D3S1295, and D9S942. Microsatellite repeat changes characteristic of leiomyosarcomas (heterozygosity loss and/or microsatellite instability in at least one locus studied) were found in 6 patients; all were clinically and morphologically diagnosed as having intravenous leiomyomatosis. In 3 of these 6 cases, leiomyomatosis was accompanied by metastases to the lungs and spread to the peritoneum; heart damage was noted in 2 cases. The data analysis did not allow identification of any significant clinical and morphological criteria for this group. Leiomyomatosis is not a transitional form from benign leiomyoma to leiomyosarcoma, as evidenced by the difference in the status of molecular markers. Analysis of molecular genetic changes in DNA from tumor tissue samples cannot categorically clarify the nature of the disease by identifying the signs of genetic instability; however, there is a need for further accumulation of experience in studying tumors of this group and in identifying the possible association with disease prognosis."}, {"id": "wiki20220301en041_14148", "title": "Muscle biopsy", "score": 0.008812865707003625, "content": "In medicine, a muscle biopsy is a procedure in which a piece of muscle tissue is removed from an organism and examined microscopically. A muscle biopsy can lead to the discovery of problems with the nervous system, connective tissue, vascular system, or musculoskeletal system. Indications In humans with weakness and low muscle tone, a muscle biopsy can help distinguish between myopathies (where the pathology is in the muscle tissue itself) and neuropathies (where the pathology is at the nerves innervating those muscles). Muscle biopsies can also help to distinguish among various types of myopathies, by microscopic analysis for differing characteristics when exposed to a variety of chemical reactions and stains."}]}}}} {"correct_option": 2, "explanations": {"1": {"exist": false, "char_ranges": [], "word_ranges": [], "text": ""}, "2": {"exist": true, "char_ranges": [[0, 125]], "word_ranges": [[0, 17]], "text": "Systemic corticosteroids in these patients should be used primarily in acute exacerbations, not for control of exacerbations."}, "3": {"exist": false, "char_ranges": [], "word_ranges": [], "text": ""}, "4": {"exist": false, "char_ranges": [], "word_ranges": [], "text": ""}, "5": {"exist": false, "char_ranges": [], "word_ranges": [], "text": ""}}, "full_answer": "Systemic corticosteroids in these patients should be used primarily in acute exacerbations, not for control of exacerbations.", "full_answer_no_ref": "Systemic corticosteroids in these patients should be used primarily in acute exacerbations, not for control of exacerbations.", "full_question": "A 67-year-old man, ex-smoker, with a diagnosis of severe COPD (multidimensional index BODE 5, FEVl 38%, body mass index 23, dyspnea index according to the mMRC 3 scale, distance covered in the 6-minute walk test 260 m) who has had 3 hospital admissions for exacerbation of his COPD in the last 7 months, comes to the consultation. She also has a history of hypertension, ischemic heart disease with AMI 5 years ago and intermittent claudication. In the clinical examination, there is a decrease in vesicular murmur with expiratory wheezing in both lung fields and an oximetry saturation of 88%. Which of the following therapeutic strategies would NOT be recommended for this patient?", "id": 369, "lang": "en", "options": {"1": "Adjustment of inhaled therapy with long-acting bronchodilators combining anticholinergics and beta-2 adrenergics with inhaled glucocorticoids.", "2": "Start oral glucocorticoids for 6 months to control exacerbations.", "3": "Check that the patient performs the inhalation technique correctly.", "4": "Initiate chronic home oxygen therapy regimen.", "5": NaN}, "question_id_specific": 122, "type": "PNEUMOLOGY AND THORACIC SURGERY", "year": 2016, "rag": {"clinical_case_options": {"MedCorp": {"RRF-2": [{"id": "pubmed23n0794_1665", "title": "The clinical and integrated management of COPD.", "score": 0.015942028985507246, "content": "COPD is a chronic pathological condition of the respiratory system characterized by persistent and partially reversible airflow obstruction, to which variably contribute remodeling of bronchi (chronic bronchitis), bronchioles (small airway disease) and lung parenchyma (pulmonary emphysema). COPD can cause important systemic effects and be associated with complications and comorbidities. The diagnosis of COPD is based on the presence of respiratory symptoms and/or a history of exposure to risk factors, and the demonstration of airflow obstruction by spirometry. GARD of WHO has defined COPD \"a preventable and treatable disease\". The integration among general practitioner, chest physician as well as other specialists, whenever required, assures the best management of the COPD person, when specific targets to be achieved are well defined in a diagnostic and therapeutic route, previously designed and shared with appropriateness. The first-line pharmacologic treatment of COPD is represented by inhaled long-acting bronchodilators. In symptomatic patients, with pre-bronchodilator FEV1 < 60%predicted and ≥ 2 exacerbations/year, ICS may be added to LABA. The use of fixed-dose, single-inhaler combination may improve the adherence to treatment. Long term oxygen therapy (LTOT) is indicated in stable patients, at rest while receiving the best possible treatment, and exhibiting a PaO2 ≤ 55 mmHg (SO2<88%) or PaO2 values between 56 and 59 mmHg (SO2 < 89%) associated with pulmonary arterial hypertension, cor pulmonale, or edema of the lower limbs or hematocrit > 55%. Respiratory rehabilitation is addressed to patients with chronic respiratory disease in all stages of severity who report symptoms and limitation of their daily activity. It must be integrated in an individual patient tailored treatment as it improves dyspnea, exercise performance, and quality of life. Acute exacerbation of COPD is a sudden worsening of usual symptoms in a person with COPD, over and beyond normal daily variability that requires treatment modification. The pharmacologic therapy can be applied at home and includes the administration of drugs used during the stable phase by increasing the dose or modifying the route, and adding, whenever required, drugs as antibiotics or systemic corticosteroids. In case of patients who because of COPD severity and/or of exacerbations do not respond promptly to treatment at home hospital admission should be considered. Patients with \"severe or \"very severe COPD who experience exacerbations should be carried out in respiratory unit, based on the severity of acute respiratory failure. An integrated system is required in the community in order to ensure adequate treatments also outside acute care hospital settings and rehabilitation centers. This article is being simultaneusly published in Multidisciplinary Respiratory Medicine 2014; 9:25."}, {"id": "pubmed23n0519_20578", "title": "Contemporary issues in the care of patients with chronic obstructive pulmonary disease.", "score": 0.015476190476190477, "content": "Chronic obstructive pulmonary disease (COPD) is the fourth leading cause of death in the United States and is estimated to be responsible for 119,000 deaths in the year 2000 alone. Additionally, COPD places a tremendous burden on the health care system, with estimated annual costs of US 24 billion dollars in 2000, and it is generally expected that costs will continue to rise as more individuals are diagnosed. COPD was responsible for approximately 8 million physician outpatient visits, 1.5 million emergency department visits and 726,000 hospitalizations, also in the year 2000. The objective of this article is to review current, pertinent clinical issues in the management of patients with COPD, with estimates of their relative utility and efficacy. COPD is a disease characterized by airflow limitation that is not fully reversible. Patients with COPD may frequently experience symptoms of chronic cough with sputum production, dyspnea, and reduced exercise capacity. They may frequently experience exacerbations characterized by increased symptoms that often require medical intervention. The diagnosis of COPD is usually fairly straightforward and made in a cigarette smoker, with the aforementioned symptoms and airflow obstruction measured by spirometry. Spirometry should be performed in all patients in whom COPD is suspected, as it provides useful prognostic information and may be used to stage the disease. The Global Initiative for Chronic Obstructive Lung Disease (GOLD) has provided evidenced-based management guidelines for COPD. GOLD guidelines advocate staging COPD by spirometry and make specific treatment recommendations based on COPD stage. The most important risk factor for the development of COPD is cigarette smoking, and smoking cessation has been shown to reduce all-cause mortality and to alter the natural history of COPD. Smoking cessation strategies that employ both counseling and medications like buproprion and nicotine replacement are most effective, but relapse rates remain high. It has not been shown that medications like bronchodilators or inhaled steroids change the natural history of COPD, nor do they reduce mortality, but they can affect other important outcomes. Long-acting bronchodilators, including beta- 2-adrenergic agonists such as salmeterol and formoterol, and the anticholinergic agent tiotropium, improve lung function and exercise tolerance, reduce symptoms, and modestly reduce exacerbation rates. Long-acting bronchodilators are indicated for all COPD patients with chronic symptoms. Short-acting bronchodilators are indicated for rescue when acute symptoms occur. Inhaled corticosteroids minimally improve lung function, but, importantly, reduce exacerbation rates and are indicated in severe COPD or when exacerbations are frequent. Continuous oxygen therapy has been shown to reduce mortality when severe hypoxemia is present and can improve quality of life when moderate hypoxia is present. Finally, well-designed, multidisciplinary disease management programs and pulmonary rehabilitation can improve important disease outcomes in a cost-effective manner. COPD is a common, preventable disease that affects a significant number of people. It may be managed by utilizing various readily available medical therapies, as well as other nonpharmacologic interventions, such as pulmonary rehabilitation. Proper coordination of care is important in this disease, and efforts should be focused on improving quality of life and reduction of symptoms."}, {"id": "pubmed23n0801_9963", "title": "The clinical and integrated management of COPD. An official document of AIMAR (Interdisciplinary Association for Research in Lung Disease), AIPO (Italian Association of Hospital Pulmonologists), SIMER (Italian Society of Respiratory Medicine), SIMG (Italian Society of General Medicine).", "score": 0.015363786202024958, "content": "COPD is a chronic pathological condition of the respiratory system characterized by persistent and partially reversible airflow obstruction, to which variably contribute remodeling of bronchi (chronic bronchitis), bronchioles (small airway disease) and lung parenchyma (pulmonary emphysema). COPD can cause important systemic effects and be associated with complications and comorbidities. The diagnosis of COPD is based on the presence of respiratory symptoms and/or a history of exposure to risk factors, and the demonstration of airflow obstruction by spirometry. GARD of WHO has defined COPD \"a preventable and treatable disease\". The integration among general practitioner, chest physician as well as other specialists, whenever required, assures the best management of the COPD person, when specific targets to be achieved are well defined in a diagnostic and therapeutic route, previously designed and shared with appropriateness. The first-line pharmacologic treatment of COPD is represented by inhaled long-acting bronchodilators. In symptomatic patients, with pre-bronchodilator FEV1 < 60% predicted and ≥ 2 exacerbations/year, ICS may be added to LABA. The use of fixed-dose, single-inhaler combination may improve the adherence to treatment. Long term oxygen therapy (LTOT) is indicated in stable patients, at rest while receiving the best possible treatment, and exhibiting a PaO2 ≤ 55 mmHg (SO2 < 88%) or PaO2 values between 56 and 59 mmHg (SO2 < 89%) associated with pulmonary arterial hypertension, cor pulmonale, or edema of the lower limbs or hematocrit > 55%. Respiratory rehabilitation is addressed to patients with chronic respiratory disease in all stages of severity who report symptoms and limitation of their daily activity. It must be integrated in an individual patient tailored treatment as it improves dyspnea, exercise performance, and quality of life. Acute exacerbation of COPD is a sudden worsening of usual symptoms in a person with COPD, over and beyond normal daily variability that requires treatment modification. The pharmacologic therapy can be applied at home and includes the administration of drugs used during the stable phase by increasing the dose or modifying the route, and adding, whenever required, drugs as antibiotics or systemic corticosteroids. In case of patients who because of COPD severity and/or of exacerbations do not respond promptly to treatment at home hospital admission should be considered. Patients with \"severe\" or \"very severe\" COPD who experience exacerbations should be carried out in respiratory unit, based on the severity of acute respiratory failure. An integrated system is required in the community in order to ensure adequate treatments also outside acute care hospital settings and rehabilitation centers. This article is being simultaneously published in Sarcoidosis Vasc Diffuse Lung Dis 2014, 31(Suppl. 1);3-21. "}, {"id": "pubmed23n0816_19939", "title": "Three-month treatment response and exacerbation in chronic obstructive pulmonary disease.", "score": 0.015343915343915344, "content": "The aim of this study was to investigate relationships between acute exacerbation and Forced Expiratory Volume 1 second (FEV1) improvement after treatment with combined long-acting beta-agonist (LABA) and inhaled corticosteroid (ICS) in patients with chronic obstructive pulmonary disease (COPD). A total of 137 COPD patients were classified as responders or nonresponders according to FEV1 improvement after 3 months of LABA/ICS treatment in fourteen referral hospitals in Korea. Exacerbation occurrence in these two subgroups was compared over a period of 1 yr. Eighty of the 137 COPD patients (58.4%) were classified as responders and 57 (41.6%) as nonresponders. Acute exacerbations occurred in 25 patients (31.3%) in the responder group and in 26 patients (45.6%) in the nonresponder group (P=0.086). FEV1 improvement after LABA/ICS treatment was a significant prognostic factor for fewer acute exacerbations in a multivariate Cox proportional hazard model adjusted for age, sex, FEV1, smoking history, 6 min walk distance, body mass index, exacerbation history in the previous year, and dyspnea scale.Three-month treatment response to LABA/ICS might be a prognostic factor for the occurrence of acute exacerbation in COPD patients. "}, {"id": "pubmed23n0805_16697", "title": "Withdrawal of inhaled glucocorticoids and exacerbations of COPD.", "score": 0.014823717948717948, "content": "Treatment with inhaled glucocorticoids in combination with long-acting bronchodilators is recommended in patients with frequent exacerbations of severe chronic obstructive pulmonary disease (COPD). However, the benefit of inhaled glucocorticoids in addition to two long-acting bronchodilators has not been fully explored. In this 12-month, double-blind, parallel-group study, 2485 patients with a history of exacerbation of COPD received triple therapy consisting of tiotropium (at a dose of 18 μg once daily), salmeterol (50 μg twice daily), and the inhaled glucocorticoid fluticasone propionate (500 μg twice daily) during a 6-week run-in period. Patients were then randomly assigned to continued triple therapy or withdrawal of fluticasone in three steps over a 12-week period. The primary end point was the time to the first moderate or severe COPD exacerbation. Spirometric findings, health status, and dyspnea were also monitored. As compared with continued glucocorticoid use, glucocorticoid withdrawal met the prespecified noninferiority criterion of 1.20 for the upper limit of the 95% confidence interval (CI) with respect to the first moderate or severe COPD exacerbation (hazard ratio, 1.06; 95% CI, 0.94 to 1.19). At week 18, when glucocorticoid withdrawal was complete, the adjusted mean reduction from baseline in the trough forced expiratory volume in 1 second was 38 ml greater in the glucocorticoid-withdrawal group than in the glucocorticoid-continuation group (P<0.001); a similar between-group difference (43 ml) was seen at week 52 (P=0.001). No change in dyspnea and minor changes in health status occurred in the glucocorticoid-withdrawal group. In patients with severe COPD receiving tiotropium plus salmeterol, the risk of moderate or severe exacerbations was similar among those who discontinued inhaled glucocorticoids and those who continued glucocorticoid therapy. However, there was a greater decrease in lung function during the final step of glucocorticoid withdrawal. (Funded by Boehringer Ingelheim Pharma; WISDOM ClinicalTrials.gov number, NCT00975195.)."}, {"id": "pubmed23n1061_20996", "title": "Clinical features of Japanese patients with exacerbations of chronic obstructive pulmonary disease.", "score": 0.01459931798806479, "content": "Exacerbations are critical events in chronic pulmonary obstructive disease (COPD). The frequency of COPD exacerbations is associated with the prognosis, including mortality, but no useful biomarker has been established. The present retrospective study investigated 481 COPD patients. Clinical features in the stable period were compared between patients who experienced severe exacerbation (n = 88, 18.3%) and those who never experienced severe exacerbation (n = 393, 81.7%). In the patients who experienced exacerbations, clinical features were also compared between frequent exacerbators (exacerbation rate ≥ 2 times/year, n = 27, 30.7%) and infrequent exacerbators (1 time/year, n = 61, 69.3%). Compared to COPD patients who never experienced exacerbations, body mass index (BMI), serum albumin, and pulmonary functions were significantly lower, and the cardiovascular disease comorbidity rate, COPD assessment test score, modified Medical Research Council dyspnea scale, and use of long-term oxygen therapy, long-acting β 10%. [7] It is also important to note that malignant transformation such as choriocarcinoma or placental-site trophoblastic tumor can occur following any pregnancy."}, {"id": "pubmed23n0568_17404", "title": "Shortened duration of human chorionic gonadotrophin surveillance following complete or partial hydatidiform mole: evidence for revised protocol of a UK regional trophoblastic disease unit.", "score": 0.011123136123136123, "content": "Following hydatidiform mole, women are at increased risk of persistent gestational trophoblastic neoplasia (pGTN) and are therefore monitored using serum human chorionic gonadotrophin (hCG) concentration measurements. We retrospectively evaluated the policy of extended (2 year) follow up for women with hCG concentrations returning to normal >56 days after evacuation. Of 6701 women registered for hCG follow up, 422 (6%) developed pGTN, 412 (98%) of these women presented within 6 months after evacuation. Three developed pGTN at 402, 677 and 1267 days after evacuation following spontaneous normalisation of hCG levels. Only one woman was detected by routine extended follow up. Prolonged surveillance after molar pregnancy causes significant anxiety and is not cost-effective. Therefore, the current revised protocol comprises hCG follow up for 6 months after spontaneous return of hCG levels to normal for all women."}]}}}} {"correct_option": 4, "explanations": {"1": {"exist": false, "char_ranges": [], "word_ranges": [], "text": ""}, "2": {"exist": false, "char_ranges": [], "word_ranges": [], "text": ""}, "3": {"exist": false, "char_ranges": [], "word_ranges": [], "text": ""}, "4": {"exist": true, "char_ranges": [[143, 334]], "word_ranges": [[26, 57]], "text": "Microhematuria without associated symptoms may be a finding without pathologic significance and must be confirmed in a subsequent new sediment (although I do not know if at 15 days or later)."}, "5": {"exist": false, "char_ranges": [], "word_ranges": [], "text": ""}}, "full_answer": "The correct answer is 4. What is not very clear to me is \"routine urinalysis\" because it is not usually done in an asymptomatic healthy child. Microhematuria without associated symptoms may be a finding without pathologic significance and must be confirmed in a subsequent new sediment (although I do not know if at 15 days or later).", "full_answer_no_ref": "[HIDDEN] What is not very clear to me is \"routine urinalysis\" because it is not usually done in an asymptomatic healthy child. Microhematuria without associated symptoms may be a finding without pathologic significance and must be confirmed in a subsequent new sediment (although I do not know if at 15 days or later).", "full_question": "A 10-year-old girl comes for a health check-up. Physical examination is normal with weight and height in the 50th percentile and BP 109/65. A routine urinalysis shows a specific gravity of 1035 pH6 blood 2+ with no protein. Urine sediment shows 5-10 red blood cells per field. What would be the most appropriate course of action?", "id": 43, "lang": "en", "options": {"1": "Determination of creatinine and nitrogen in blood.", "2": "Refer the child for cystoscopy.", "3": "Determine antinuclear antibodies and complement.", "4": "Repeat urine sediment in 15 days.", "5": "Abdominal computed axial tomography."}, "question_id_specific": 151, "type": "PEDIATRICS", "year": 2011, "rag": {"clinical_case_options": {"MedCorp": {"RRF-2": [{"id": "pubmed23n0510_13943", "title": "[Acute tubulointerstitial nephritis in children].", "score": 0.017862838915470493, "content": "Acute tubulointerstitial nephritis (ATIN) is a rare renal disorder in children. Patients usually present non-specific symptoms and signs so that the diagnosis of ATIN is often delayed. The disease may be infection- or drug-induced or it may occur without a known cause. Early recognition and appropriated therapy usually lead to an excellent prognosis. The aim of the study was to describe clinical and laboratory findings and the course of ATIN in 21 patients, that are typical enough to enable early recognition of the disease as it is potentially reversible. Between 1986 and 1997 we observed 21 patients, aged 7-16 years (mean, 12.8), with acute tubulointerstitial nephritis, including eight with tubulointerstitial nephritis and uveitis (TINU syndrome). Laboratory studies included urinalysis, complete blood count, erytrocyte sedimentation rate (ESR), plasma creatinine, glomerular filtration rate (GFR), electrolytes, proteins, IgG, C3, C4 antinuclear-antibodies (ANA), antistreptolysin-O and antibodies to hantaviruses. Renal ultrasound was done in all patients. Renal biopsy was performed in 5 children. All children had previously been healthy. The symptoms of ATIN developed within a few days (Table 1). The most common initial symptoms were fatigue, fever, gastrointestinal disturbances, anorexia and weight loss. Less common complaints included headache, arthralgias and maculopapular rash. On addmition no patient had hypertension, oedema or evidence of acute infection. ESR, plasma urea and creatinine concentrations were increased in all, plasma proteins and IgG levels in the majority of patients. ANA were negative in 15 pts in whom this analysis was performed; C3 and C4 levels were normal. In two children recent strepococcal and in the other 6 hantavirus infection was serologicaly proved. All patients had non-oliguric acute renal failure (ARF): GFR was 21.7 +/- 8 9 in 14 pts and 67 +/- 9.7 in 7 pts. Low urine specific gravity (1003-1014), mild proteinuria (0.3-0.4 g/24 h), leukocyturia and/or haematuria were found in all patients; glycosuria, aminoaciduria and decreased tubular reaposrption of phosphate (TRP) were found in 12/21, 9/21 and 9/14 patients, respectively. Urine cultures were negative in all children. Renal US demonstrated enlarged hyperechoic kidneys in 11 pts, in remaining 10 pts no abnormalities were found. Renal biopsy, performed in 5 children, confirmed the diagnosis of ATIN. Eight patients subsequently developed anterior uveitis as part of TINU syndrome. Treatment included supportive therapy in all and six patients received prednisolone for 4-8 weeks (40-60 mg/m2/24 h for 10-14 days with subsequent reduction of dose over several weeks). Anterior uveitis was successfully treated with topical steroids. Renal function completely recovered in all patients: GFR (109 +/- 22.6 ml/min) within a mean interval of 47 +/- 33 days, concentration ability within 2-12 (mean 4.5) months. Common clinical features of ATIN are non-oliguric acute renal failure of various degrees, signs of tubular dysfunction, proteinuria, haematuria, leukocyturia and absence of hypertension. All our patients had normal blood pressure, non-oliguric renal failure, proteinuria, hypostenuria and abnormal urinary sediment; about half of them had glycosuria and/or other signs of proximal tubular dysfunction. The most important causes of ATIN in children reported in literature are systemic infections and drugs. However, the cause of ATIN in our patients was assessed as being related to infection only in 8 patients and to diclofenac in one. No infection, drug, toxin or other cause could be identified in 4, as well as in 8 pts with TINU syndrome. The prognosis of ATIN in children is considered to be favourable, but some patients may develop chronic renal failure. Renal function completely recovered in all our patients; that is consistent with outcome data from the most reports. Acute tubulointerstitial nephritis is an important cause of ARF in children, its aetiology may be different and it carries an excellent prognosis. ATIN should be suspected in a child who presents typical, although non-specific symptoms and signs, associated with lukocyturia and/or microhaematuria, signs of tubular dysfunction and unexplained renal failure. The diagnosis can be verified at renal biopsy. Early recognition of the disease is important to remove possible aetiologic agents and to treat them before chronic lesions are present to avoid long-term renal damage."}, {"id": "pubmed23n0417_5268", "title": "Cases from the Osler Medical Service at Johns Hopkins University. Antiglomerular basement membrane disease.", "score": 0.01609322974472808, "content": "A 47-year-old Taiwanese man with no notable medical history was admitted with low-grade fevers and night sweats that had persisted for 5 to 6 weeks. An extensive investigation at another hospital could not determine the cause of the fevers, but documented acute renal failure with a blood urea nitrogen level of 60 mg/dL and a serum creatinine level of 5.6 mg/dL. He was admitted to the Johns Hopkins Hospital for further evaluation.The patient, who had been living in the United States for the past 20 years, reported no recent travel and no behaviors that are associated with transmission of human immunodeficiency virus. He was not taking any medications, and he denied using herbal or nutritional supplements. He had no recent weight loss. There were no specific complaints on review of systems. On physical examination, he was a thin, middle-aged man in no distress. Vital signs included a temperature of 37.5 degrees C, a blood pressure of 166/86 mm Hg, a pulse of 70 beats per minute, a respiratory rate of 16 breaths per minute, and 99% oxygen saturation on room air. Sclera were anicteric, and he had no palpable adenopathy. His lungs were clear, and his heart rate was regular without extra sounds. His abdomen was thin, nontender, and without masses or organomegaly. There was no edema or signs of embolism in the extremities. Laboratory studies revealed a white blood cell count of 14,200/mL(3), a hematocrit of 23.1%, and a platelet count of 456,000/mL(3). Blood chemistries were notable for a blood urea nitrogen level of 61 mg/dL and a serum creatinine level of 7.6 mg/dL. Levels of aminotransferases, total bilirubin, and alkaline phosphatase were within normal limits. Urinalysis revealed large hemoglobin, 1+ protein, numerous red blood cells, and 3 to 5 white blood cells. Numerous red blood cell casts were seen on microscopic examination of the urine sediment. The patient's erythrocyte sedimentation rate was >130 mm/h, and his C-reactive protein level was elevated at 12.6 mg/dL. Serologies were negative for antinuclear antibodies and antineutrophil cytoplasmic antibodies; serum complement levels were normal. What is the diagnosis?"}, {"id": "pubmed23n0620_809", "title": "Microscopic hematuria is associated with low bone mineral density in aged women and men.", "score": 0.016064516129032258, "content": "Little is known concerning renal or urological risk factors for osteoporosis. The aim of this study was to explore an association between urinalysis and bone mineral density (BMD) in 4,835 Japanese adults. Participants were 4,835 individuals (female 3,683; male 1,152) aged 50 years and over who received a health check-up between January 1995 and March 2006 in Japan. BMD of the distal radius and ulna of the non-dominant forearm was measured by the dual-energy X-ray absorptiometry (DXA) method using a DTX-200 Dexacare osteometer (Osteometer MediTech A/S, Rødovre, Denmark). Urine variables were protein, and red blood cells (RBCs) and white blood cells (WBCs) in urine sediment (categorized as <1, 1-4, 5-9, or >or=10 cells per high-power microscopic field). Average age was 58.9 years (SD 5.6) in women, and 60.5 years (SD 6.2) in men. Simple linear regression analysis showed that urinary RBCs were associated with BMD in women (beta = or-5.88 x 10(-3), R (2) = 0.004, p < 0.0001) and men (beta = or-1.34 x 10(-2), R (2) = 0.013, p = 0.0001). These associations held when possible confounders were adjusted (beta = or-2.05 x 10(-3), R (2) = 0.001, p = 0.0338 for women and beta =or -5.67 x 10(-3), R (2) = 0.006, p = 0.0163 for men). Microscopic hematuria is associated with forearm BMD in women and men aged 50 years and over. Further studies should be conducted to determine the mechanisms underlying this association."}, {"id": "pubmed23n0324_186", "title": "Limited evaluation of microscopic hematuria in pediatrics.", "score": 0.015154994259471873, "content": "The purpose was to determine the value of the standard laboratory and radiologic evaluation of microscopic hematuria in children, and to determine the prevalence of idiopathic hypercalciuria in those children referred for evaluation of unexplained microscopic hematuria. This was a retrospective study of 325 children referred from 1985 to 1994 for the evaluation of asymptomatic microscopic hematuria. The diagnostic studies reviewed included serum creatinine, blood urea nitrogen, serum electrolyte studies, serum complement concentration, antinuclear antibody, urinalysis, urine calcium to creatinine ratios, urinary protein to creatinine ratio and/or 24-hour urinary protein excretion, renal ultrasounds, intravenous pyelograms, voiding cystourethrograms, and historical information. All creatinine and electrolyte values were normal for age, and none of the biochemical tests obtained in the children with hypercalciuria was abnormal. Of the 325 patients with idiopathic microscopic hematuria, only 18 had abnormal renal ultrasound examinations and 9 voiding cystourethrograms showed low-grade reflux. Hypercalciuria was found in 29 patients. The family history was positive for urolithiasis in 16% of patients without hypercalciuria compared with 14% of patients with hypercalciuria. A positive family history of hematuria was reported in 25% of patients; 62 patients did not have hypercalciuria and 4 of the patients had hypercalciuria. Microscopic hematuria in children is a benign finding in the vast majority of children. Our data demonstrate that a renal ultrasound, voiding cystourethrogram, cystoscopy, and renal biopsy are not indicated in the work-up of microscopic hematuria, and microhematuria in the otherwise healthy child is a minimal health threat, rarely indicative of serious illness."}, {"id": "pubmed23n1014_25815", "title": "[Adrenal hemorrhage in a patient with systemic lupus erythematosus].", "score": 0.014421936524503059, "content": "A 58-year-old female was referred to our department with intermittent suffocation for 1.5 years, aggravated for a month. 1.5 years before she developed oral ulcer, raynaud phenomenon, proteinuria, bilateral pleural effusion, ANA and anti-dsDNA positive. This patient was diagnosed with systemic lupus erythematosus (SLE). After given hormones, hydroxychloroquine sulfate (HCQ), her symptom relieved soon. The patient stopped her pills 1 year ago. One month ago, she had chest tightness, increased urine foam, and suffered from oliguria. Her admission medical examination: blood pressure (BP) 130/80 mmHg, conjunctiva pale, and lower lung breath sounds reduced. There was no tenderness, rebound pain and abdominal muscle tension in the abdomen. Liver and spleen rib inferior, mobile dullness negative, and lower extremity edema. Blood routine tests were performed with hemoglobin (HGB) 57 g/L. Urine routine: BLD (3+). 24-hour urinary protein 3.2 g. serum albumin 20.5 g/L, C-reactive protein (CRP) 12.85 mg/L, erythrocyte sedimentation rate (ESR) 140 mm/h. Antinuclear antibody (ANA) (H)1:10 000, anti-dsDNA antibody 1:3 200; anti-Smith antibody, anti-U1-snRNP/Sm antibody were positive, blood complement 3(C3) 0.43 g/L, complement 4(C4) 0.07 g /L. Anticardiolipin antibody (ACL), anti-β2-GP1, lupus anticoagulant (LA) were negative; HRCT suggested bilateral medial pleural cavity product liquid. Admission diagnosis: SLE lupus nephritis, anemia, pleural effusion, and hypoproteinemia. We treated her with methylprednisolone 1 000 mg×3 d, late to 48 mg/d and cyclophosphamide 1.0 g, HCQ 0.2 g bid, gamma globulin 10 g×5 d. Day 2 of treatment, this patient developed acute right upper quadrant pain, not accompanied by nausea, vomiting, blood stool and diarrhea. Antipyretic antispasmodic treatment was invalid, after the morning to ease their own abdominal pain. Day 4 of treatment, daytime blood HGB 77 g/L. Bilateral renal vascular ultrasound: bilateral renal artery blood flow velocity was reduced. The abdominal pain of the above symptoms recurred at night, BP was 120/80 mmHg, and no positive signs were found on abdominal examination. No abnormality was found in the vertical abdominal plain film. Blood routine examination: HGB 53 g/L, Plasma D dimer 2 515 μg/L, amylase in hematuria was normal, the stool occult blood was negative. Abdominal computed tomography (CT): normal structure of right adrenal gland disappeared, irregular mass shadow could be seen in adrenal region, CT value was about 50 HU. Morphological density of left adrenal gland was not abnormal. The retroperitoneum descended along the inferior vena cava to the right iliac blood vessel and showed a bolus shadow. The density of some segments increased. The lesion involved the right renal periphery and reached the left side of abdominal aorta. Most lesions surrounded the inferior vena cava, the right renal vein and part of the small intestine. The boundary between the upper lesion and the vena cava was unclear. Iodinecontaining contrast agent was taken orally. No sign of contrast agent overflowing was found in the abdominal cavity. Hematoma and exudative changes were considered in retroperitoneum. CONCLUSION of contrast-enhanced ultrasound of blood vessels: The retroperitoneal inferior vena cava (volume 3.5 cm×3.5 cm×1.5 cm) was hypoechoic and had no blood flow lesion. The adrenal gland had a high possibility of origin. Left renal vein thrombosis extended to inferior vena cava. According to the above data, it was analyzed that the cause of retroperitoneal hematoma of the patient was left adrenal vein thrombosis caused by hypercoagulable state, which led to vascular rupture and hemorrhage caused by increased vascular pressure in adrenal gland. Therefore, on the basis of continuing to actively treat the primary disease, and on the basis of dynamic observation of no active hemorrhage for 3 days, the anticoagulant therapy was continued with 10 mg/d of apixaban. Clinical symptoms were gradually eased, HGB did not decrease. Two weeks later, the ultrasonic examination showed that the irregular cluster hypoechoic range behind the inferior vena cava was significantly smaller than that before (1.8 cm×1.2 cm×0.7 cm). Abdominal CT examination after 1 month showed that there was no abnormal morphological density of bilateral adrenal glands and basic absorption of retroperitoneal exudation. Adrenal hemorrhage is uncommon. SLE with adrenal hemorrhage is rarer. In SLE patients, especially those complicated with APS, if abdominal pain accompanied by HGB decrease occurs, except after gastrointestinal hemorrhage, the possibility of adrenal hemorrhage should be warned."}, {"id": "pubmed23n0392_8074", "title": "Atherosclerotic Renovascular Disease.", "score": 0.014250493096646943, "content": "The patient, a 78-year-old Asian male, was brought to the hospital because of acute shortness of breath that had progressively worsened over the course of the day. He complained of a nonproductive cough and claudication after walking 1 block. His past medical history was significant for mild renal insufficiency (serum creatinine 1.5--2.0 mg/dl), the etiology of which was never explored. Although there was a recent history of mild to moderate hypertension, at presentation his blood pressure was noted to be 240/118 mm Hg in both arms. His physical exam at the time of admission was remarkable for grade II hypertensive retinopathy, an S4 gallop, periumbilical systolic bruits, audible femoral arterial bruits and absent distal lower extremity pulses. Initial complete blood count, serum electrolytes and cardiac enzymes (including lactate dehydrogenase) were normal. His blood urea nitrogen and serum creatinine concentrations were 51 and 3.6 mg/dl, respectively, and his urinalysis showed 1+ protein (both by dipstick and sulfasalicylic acid) with a \"benign\" sediment (0--1 WBC/HPE, 1--2 RBCs/HPF) with occasional granular casts. His electrocardiogram, apart from demonstrating left ventricular hypertrophy with secondary ST-T wave abnormalities, showed no acute changes; his chest X-ray demonstrated cardiomegaly and pulmonary vascular congestion. He was intubated and subsequently treated with increasing parenteral doses of furosemide (40--240 mg) and a nitroglycerine drip (up to 15 mcg/min). Over the course of the first 48 h, his blood pressure was gradually lowered to 170/100 mm Hg. His urine output increased from 20 ml/h to 125/ml/h, and his respiratory status improved, allowing him to be extubated. In spite of adequate control of his blood pressure in the ensuing days (150--170/80--90 mm Hg), his renal function continued to deteriorate. Renal sonography (without Doppler) demonstrated a right kidney of 9.6 cm and a left kidney of 9.3 cm in length without evidence of hydronephrosis. Both kidneys were noted to be echogenic. Assays for antinuclear antibodies and antineutrophilic cytoplasmic antibodies were negative, and the patient's serum complement levels were normal. For several days after his admission, his serum creatinine gradually rose to 10.7 mg/dl, and hemodialysis was initiated for uremic encephalopathy. Because of the high index of suspicion for renal artery stenosis as the case of both his hypertension and renal failure, a renal angiogram was performed. It revealed a 90% occlusion of the right renal artery with ostial involvement and a 70% occlusion of the left renal artery; both kidneys had poor distal renal vasculature and there was marked atherosclerotic disease of the aorta. After being hemodialyzed for 3 treatments, his renal function began to improve spontaneously. His serum creatinine returned to 3.4 mg/dl, and a subsequent 24-hour urine demonstrated a creatinine clearance of 20 ml/min and an excretion of 1.2 g of protein. Following his discharge from the hospital, his renal function remained unchanged for 3 years, and his blood pressure was easily controlled on monotherapy with a long-acting calcium channel blocker. He recently died from pneumonia."}, {"id": "article-22283_9", "title": "Diffuse Proliferative Glomerulonephritis -- Evaluation", "score": 0.01404869640163758, "content": "A complete blood count showing possible anemia and low platelet count followed by renal function tests with elevated serum creatinine (0.4 mg/dl above the upper limit), blood urea nitrogen levels, and urine analysis positive for urine sediments: red blood cells and casts, white blood cells, granular casts are indicative of a glomerular pathology. For further confirmation, a 24 hours urine protein to creatinine ratio and 24 hours urine sample for protein levels can be done. A protein count of greater than 3.5 g/day is suggestive of nephrotic range proteinuria, which is associated with a worse prognosis. A 24-hour urine sample can be used to calculate creatinine clearance to estimate the eGFR. Renal ultrasound can be done to see the size and confirm the presence of two kidneys and the absence of any obstructive pathology resulting in hydronephrosis. Serum complement (C3 and C4) levels help determine the etiology; low levels are associated with the presence of SLE, cryoglobulinemia, and infectious etiology. [12]"}, {"id": "pubmed23n0064_4794", "title": "Usefulness of scanning procedures for diagnosis of fever of unknown origin in children.", "score": 0.013066349906668929, "content": "During a 5-year study period, 109 patients were referred to a large children's hospital for evaluation of prolonged fever of unknown origin, defined as temperature greater than or equal to 38 degrees C (100.4 degrees F) for 3 weeks or longer and negative findings on initial examination. A two-phase protocol of outpatient followed by inpatient diagnostic studies was instituted for most patients. Confirmed diagnoses were achieved in just 36 of these children (33%) in the following disease categories: infectious, 24 (22%); autoimmune, 7 (6%); and neoplastic, 2 (2%). Scanning or special procedures and the number with positive results (in parentheses) were as follows: abdominal ultrasonography, 43 (8); abdominal computed tomography, 14 (3); indium scan 11 (5); gallium scanning, 4 (1), upper gastrointestinal tract series, 13 (2); technetium bone scanning 15 (2); bone marrow examination, 16 (1); and cranial computed tomography, 7 (0). These studies rarely led to an unsuspected diagnosis. It appears most appropriate in evaluating fever of unknown origin in children to obtain only basic laboratory studies such as a complete blood cell count, urinalysis and culture, chest radiograph, tuberculin skin test, and, in the older child, an antinuclear antibody titer. When these test results are negative, almost all children can be observed clinically for progression of illness or a focus that might then direct specific diagnostic procedures."}, {"id": "pubmed23n0075_21536", "title": "[Laboratory tests in primary care medicine: \"essential laboratory tests\" (1). Urinalysis].", "score": 0.013063266511482471, "content": "Japan Society of Clinical Pathology has formed a committee dealing with \"lab. tests in primary care medicine\". As the first step, they made \"Essential Lab. Tests\" which were composed of simple qualitative bed-side tests such as urinalysis, Complete blood count (CBC: Hb, Ht, WBC, RBC), CRP, or ESR (Erythrocyte sedimental rate), A/G ratio and biochemical tests if necessary (Table 1). We have performed \"Essential Lab. Tests\" on 1,026 outpatients who visited General Medicine Clinic for the first time. They consisted of 456 male (age 13-81), and 526 female (age 10-85). This report is the result of urinalysis from \"Essential Lab. Tests\" of 1,026 patients. 1) The result showed that overall positivity of the urinalysis was 21.3% (when more than one item of the qualitative tests was positive). 2) There was distinctive difference in the positivity of the urinalysis between the sex; i.e. protein and glucose were about twice frequently positive in male, where as occult blood and WBC (Esterase reaction) were 2-3 times more positive in female. 3) Urine protein shows positive in the individual 10-20 yrs old and more than 50 yrs old in both sexes. 4) Glucose was positive in over 40 yrs in male, and occult blood, 40-50 yrs or older in male. 5) WBC shows positive in all age groups in female and 50 yrs or older in male. 6) Positive WBC patients did not necessarily reflect urinary infection in female but nitrites roughly corresponded with urinary WBC in male of 50 yrs or older, meaning probable urinary infection associated with prostatic hypertrophy. 7) Abnormality of urinary sediment corresponded to the positive occult blood and WBC Erastase. 8) Urinalysis is an useful method of screening in primary care medicine."}, {"id": "wiki20220301en003_107194", "title": "Appendicitis", "score": 0.012315806644554401, "content": "Blood and urine test While there is no laboratory test specific for appendicitis, a complete blood count (CBC) is done to check for signs of infection. Although 70–90 percent of people with appendicitis may have an elevated white blood cell (WBC) count, there are many other abdominal and pelvic conditions that can cause the WBC count to be elevated. Due to its low sensitivity and specificity, on its own, WBC is not seen as a good indicator of appendicitis. A urinalysis generally does not show infection, but it is important for determining pregnancy status, especially the possibility of an ectopic pregnancy in women of childbearing age. The urinalysis is also important for ruling out a urinary tract infection as the cause of abdominal pain. The presence of more than 20 WBC per high-power field in the urine is more suggestive of a urinary tract disorder."}, {"id": "wiki20220301en022_61054", "title": "Hematuria", "score": 0.012287697533599172, "content": "The first step in evaluation of red or brown colored urine is to confirm true hematuria with urinalysis and urine microscopy, where hematuria is defined by three of more red blood cells per high power field. Although a urine dipstick test may be used, it can give false positive or false negative results. In gathering information, it is important to inquire about recent trauma, urologic procedures, menses, and culture-documented urinary tract infection. If any of these are present, it is appropriate to repeat a urinalysis with urine microscopy in 1 to 2 weeks or after treatment of the infection. If the results of the urinalysis and urine microscopy reveal a glomerular origin of hematuria (indicated by proteinuria or red blood cell casts), consultation of a nephrologist should be made. If the results of the urinalysis indicate a non-glomerular origin, a microbiological culture of the urine should be performed, if it has not been done already. If the culture is positive, treatment of the"}, {"id": "wiki20220301en022_61059", "title": "Hematuria", "score": 0.012061679040548258, "content": "a female less than 50 years old or a male less than 40 years old; has 3-10 red blood cells per high power field; has not had microscopic hematuria before; and has no other risk factors for urothelial cancer. To be in the intermediate risk category, one must satisfy any of the following criteria: Has smoked 10-30 pack-years; is a female 50-59 years old or a male aged 40-59 years old; has 11-25 red blood cells per high power field; or was previously a low-risk patient with persistent microscopic hematuria and has 3-25 red blood cells per high power field. To be in the high risk category, one must satisfy any of the following criteria: Has smoked more than 30 pack-years; is older than 60 years of age; or has above 25 red blood cells per high power field on any urinalysis. For the low risk category, the next step is to either repeat a urinalysis with urine microscopy in 6 months or perform a cystoscopy and renal ultrasound. For the intermediate risk category, the next step is to perform a"}, {"id": "wiki20220301en057_61758", "title": "Nephritic syndrome", "score": 0.011691002367797947, "content": "Blood urea nitrogen (BUN) - Also measured using a BMP or CMP, blood urea nitrogen is an indicator of how much nitrogen is in the blood at the time of the phlebotomy. The kidney is responsible for excreting nitrogenous substances in the urine, so an elevated BUN usually indicates that the kidney is not functioning appropriately. Urine Analysis (Urinalysis) - After the patient provides a urine specimen, it is sent to the lab for analysis using a variety of methods including urine dipstick testing and microscopic examination. Because the kidney is responsible for making urine, analyzing the urine directly can provide crucial data that can help the physician diagnose nephritic syndrome. Some findings on urinalysis that are consistent with nephritic syndrome include red blood cells (hematuria), red blood cell casts, proteinuria, and possibly white blood cells (pyuria)."}, {"id": "pubmed23n0396_2510", "title": "Clinical utility of a rapid test for uristatin.", "score": 0.011033478893740904, "content": "Uristatin is a trypsin inhibitor present in urine that is increased in most patients with bacterial or viral infections and in many with inflammatory disorders. We included the assay of uristatin as part of a screening program carried out by pediatricians on 4207 Japanese schoolchildren to judge the ability of uristatin to identify those with an infection and (or) inflammation of any cause. We used urine dipsticks for the assay of uristatin, creatinine, albumin, blood, leukocyte esterase, and protein. We also performed quantitative assays for uristatin and creatinine. Another aim was to estimate the reference range for uristatin in schoolchildren, ages 5 to 14 yr. We prepared dipstick pads that were impregnated with a chromogenic substrate for trypsin and measured the uristatin-caused inhibition of trypsin in urine. We measured creatinine so that the ratio of uristatin to creatinine could be calculated to correct for urine concentration. We obtained quantitative uristatin and creatinine results for 4207 children. Of these, 177 had an abnormal urine dipstick for albumin or blood or protein or leukocyte esterase or a combination of these. We used data from 3622 children to establish the reference range for the uristatin dipsticks. The 3622 were diagnosed by their pediatricians as free from an infection or inflammation of any cause and with normal urine dipstick tests. We recommend an upper reference limit for uristatin by dipstick of < or = 7.5 mg uristatin/g creatinine. The leftover 408 children ( [4207-3622-177] = 408) fell into two groups: 205 with diagnoses of no infection, possible infection, or possible inflammatory disorders. The remaining 203 children were renal disease follow-up cases. The diagnoses were based on a physical examination, microscopic urinalysis plus urine dipstick tests for albumin, blood, creatinine, protein, leukocyte esterase and a complete blood count. In the 205 children, 46 had an abnormal uristatin dipstick test, 39 had an abnormal uristatin by immunoassay, 41 had an abnormal erythrocyte sedimentation rate (ESR), 27 had an abnormal serum C-reactive protein (CRP), and one had an abnormal urine microscopic exam. For the first 938 children in the study, the agreement was 93% of negative dipstick uristatin results and immunoassays. The agreement of positive uristatin dipsticks with immunoassays was 85%. We assumed that the immunoassay results were correct. In the evaluation of 189 children with fever, 62 also had an abnormal uristatin by dipstick. A rapid dipstick test for uristatin read on a reflectance photometer gave values that compared well with a quantitative immunoassay method. The uristatin test is sensitive but not specific for any cause of infection or inflammation. Uristatin is easy to determine and appears to be a better indicator than fever, ESR, or CRP for the diagnosis of an infection or inflammation."}, {"id": "wiki20220301en001_26330", "title": "Nephrology", "score": 0.011021580068537557, "content": "Diagnosis History and physical examination are central to the diagnostic workup in nephrology. The history typically includes the present illness, family history, general medical history, diet, medication use, drug use and occupation. The physical examination typically includes an assessment of volume state, blood pressure, heart, lungs, peripheral arteries, joints, abdomen and flank. A rash may be relevant too, especially as an indicator of autoimmune disease. Examination of the urine (urinalysis) allows a direct assessment for possible kidney problems, which may be suggested by appearance of blood in the urine (hematuria), protein in the urine (proteinuria), pus cells in the urine (pyuria) or cancer cells in the urine. A 24-hour urine collection used to be used to quantify daily protein loss (see proteinuria), urine output, creatinine clearance or electrolyte handling by the renal tubules. It is now more common to measure protein loss from a small random sample of urine."}, {"id": "InternalMed_Harrison_1673", "title": "InternalMed_Harrison", "score": 0.01100948509485095, "content": "Fever >38.3°C (101°F) on at least two occasions 2. Illness duration of ≥3 weeks 3. 4. Diagnosis that remains uncertain after a thorough history-taking, physical examination, and the following obligatory investigations: determination of erythrocyte sedimentation rate (ESR) and C-reactive protein (CRP) level; platelet count; leukocyte count and differential; measurement of levels of hemoglobin, electrolytes, creatinine, total protein, alkaline phosphatase, alanine aminotransferase, aspartate aminotransferase, lactate dehydrogenase, creatine kinase, ferritin, antinuclear antibodies, and rheumatoid factor; protein electrophoresis; urinalysis; blood cultures (n = 3); urine culture; chest x-ray; abdominal ultrasonography; and tuberculin skin test (TST). The range of FUO etiologies has evolved over time as a result of changes in the spectrum of diseases causing FUO, the widespread Percentage of Cases Due to Indicated Cause"}, {"id": "wiki20220301en057_61744", "title": "Nephritic syndrome", "score": 0.010579710144927536, "content": "Signs and symptoms that are consistent with nephritic syndrome include: Hematuria (red blood cells in the urine) Proteinuria (protein in the urine) ranging from sub-nephrotic (<3.5 g/day) to >10 g/day, although it is rarely above nephrotic range proteinuria levels. Hypertension resting blood pressure is persistently at or above 130/80 or 140/90 mmHg. Blurred vision Azotemia (increased plasma Urea and Creatinine) Oliguria (low urine output <400 ml/day) Red blood cell casts (seen with urine analysis and microscopy) Pyuria (white blood cells or pus in the urine) Causes"}, {"id": "wiki20220301en012_140488", "title": "Nephrotic syndrome", "score": 0.010557563242127001, "content": "Along with obtaining a complete medical history, a series of biochemical tests are required in order to arrive at an accurate diagnosis that verifies the presence of the illness. In addition, imaging of the kidneys (for structure and presence of two kidneys) is sometimes carried out, and/or a biopsy of the kidneys. The first test will be a urinalysis to test for high levels of proteins, as a healthy subject excretes an insignificant amount of protein in their urine. The test will involve a 24-hour bedside urinary total protein estimation. The urine sample is tested for proteinuria (>3.5 g per 1.73 m2 per 24 hours). It is also examined for urinary casts, which are more a feature of active nephritis. Next a blood screen, comprehensive metabolic panel (CMP) will look for hypoalbuminemia: albumin levels of ≤2.5 g/dL (normal=3.5-5 g/dL). Then a Creatinine Clearance CCr test will evaluate kidney function particularly the glomerular filtration capacity. Creatinine formation is a result of"}, {"id": "InternalMed_Harrison_1704", "title": "InternalMed_Harrison", "score": 0.010181114453396159, "content": "Fever ˜38.3° C (101° F) and illness lasting ˜3 weeks and no known immunocompromised state History and physical examination Stop antibiotic treatment and glucocorticoids Obligatory investigations: ESR and CRP, hemoglobin, platelet count, leukocyte count and differential, electrolytes, creatinine, total protein, protein electrophoresis, alkaline phosphatase, AST, ALT, LDH, creatine kinase, antinuclear antibodies, rheumatoid factor, urinalysis, blood cultures (n=3), urine culture, chest x-ray, abdominal ultrasonography, and tuberculin skin test"}, {"id": "pubmed23n0076_14175", "title": "Routine serologic tests in the differential diagnosis of the adult nephrotic syndrome.", "score": 0.010161779575328614, "content": "From 1980 to 1985, we performed biopsies on 87 adults with nephrotic syndrome (NS). The patients were tested for whether serologic studies obtained routinely at biopsy added to clinical diagnostic accuracy. Using history, physical examination, complete blood cell count (CBC), chemistry panel, urinalysis, and urine creatinine and protein, four nephrologists each predicted whether the patient had primary NS (PNS) or secondary NS (SNS), and the most likely histopathologic entity. Six months later, each nephrologist used this information, with results of tests of sera for fluorescent antinuclear antibody (FANA), rheumatoid factor (RF), complement components, hepatitis B surface antigen (HBsAg), venereal disease research laboratory serology (VDRI), cryoglobulins and protein electrophoresis (SPEP), with an erythrocyte sedimentation rate (ESR) and protein electrophoresis of the urine (UPEP), to make identical predictions. Histopathology was established by renal biopsy. We analyzed the concordance between nephrologists' choices and biopsy results both before and after serologic tests were available with a kappa statistic. Preserology concordance was moderate (kappa = 0.52), and identical to postserology concordance (kappa = 0.51) for both PNS versus SNS and actual histopathology. Serologies were rarely abnormal without clinical suspicion. These results suggest routine serologic testing does not improve diagnostic accuracy in adult NS."}, {"id": "InternalMed_Harrison_3372", "title": "InternalMed_Harrison", "score": 0.010054301713897845, "content": "PART 2 Cardinal Manifestations and Presentation of Diseases HEMATURIA Proteinuria (>500 mg/24 h), Dysmorphic RBCs or RBC casts Pyuria, WBC casts Urine culture Urine eosinophils Hemoglobin electrophoresis Urine cytology UA of family members 24 h urinary calcium/uric acid IVP +/Renal ultrasound As indicated: retrograde pyelography or arteriogram, or cyst aspiration Cystoscopy Urogenital biopsy and evaluation Renal CT scan Renal biopsy of mass/lesion Follow periodic urinalysis Renal biopsy FIguRE 61-2 Approach to the patient with hematuria. ANCA, antineutrophil cytoplasmic antibody; ASLO, antistreptolysin O; CT, computed tomography; GBM, glomerular basement membrane; IVP, intravenous pyelography; RBC, red blood cell; UA, urinalysis; VDRL, Venereal Disease Research Laboratory; WBC, white blood cell."}, {"id": "InternalMed_Harrison_3385", "title": "InternalMed_Harrison", "score": 0.009993002711449313, "content": "FIguRE 61-3 Approach to the patient with proteinuria. Investigation of proteinuria is often initiated by a positive dipstick on routine urinalysis. Conventional dipsticks detect predominantly albumin and provide a semiquantitative assessment (trace, 1+, 2+, or 3+), which is influenced by urinary concentration as reflected by urine specific gravity (minimum, <1.005; maximum, 1.030). However, more exact determination of proteinuria should employ a spot morning protein/creatinine ratio (mg/g) or a 24-h urine collection (mg/24 h). FSGS, focal segmental glomerulosclerosis; RBC, red blood cell; UPEP, urine protein electrophoresis."}, {"id": "wiki20220301en218_35381", "title": "Cholesterol embolism", "score": 0.009900990099009901, "content": "Tests for inflammation (C-reactive protein and the erythrocyte sedimentation rate) are typically elevated, and abnormal liver enzymes may be seen. If the kidneys are involved, tests of kidney function (such as urea and creatinine) are elevated. The complete blood count may show particularly high numbers of a type of white blood cell known as eosinophils (more than 0.5 billion per liter); this occurs in only 60-80% of cases, so normal eosinophil counts do not rule out the diagnosis. Examination of the urine may show red blood cells (occasionally in casts as seen under the microscope) and increased levels of protein; in a third of the cases with kidney involvement, eosinophils can also be detected in the urine. If vasculitis is suspected, complement levels may be determined as reduced levels are often encountered in vasculitis; complement is a group of proteins that forms part of the innate immune system. Complement levels are frequently reduced in cholesterol embolism, limiting the use"}, {"id": "pubmed23n0237_310", "title": "[Asymptomatic microhematuria].", "score": 0.009900990099009901, "content": "Microscopic haematuria is an urinary finding more and more frequently observed in routine analysis in childhood; the diagnostic problems, most of which unresolved, are yet the principal questions of concern. We have studied 123 patients during five years from 1975 to 1980, and followed them for 1-8 years (mean 4 years). Our experience allowed us to consider \"normal\" or without remarkable pathologic significance a urinary finding less than or equal to 5.000 RBC/m' at the Addis count. Such findings were pointed out in 55 cases (44%); 27 patients (22%) had 5-10.000 RBC/m', 34 cases (27,6%) presented 10-50.000 RBC/m' and 7 cases (5,7%) had more then 50.000 RBC/m'. The familiar background, the clinical, biological and immunological data, the roentgenographic investigations and the renal biopsy carried out in the 4 groups of patients, led us to the following conclusions: 1) 26% of the 123 cases had a \"unexplained;; microscopic haematuria with complete lack of anamnestic data, symptomatology and with normal biological findings. 2) in 65,8% of the cases it was possible to discover frequent upper respiratory tract infections (and allergy in 5,6% of them). 3) in 26% of the patients we discovered a previous or actual genito-urinary (10,5% and 15,5% respectively) infection: 9,4% of 96 urography demonstrated a variable degree of nephro-urological abnormalities. 4) 23 children (18,7%) was selected for renal biopsy, primarily by the hypocomplementemia and positive anti-DNA ab. test, and secondly by elevated degree of microhematuria. The histological and immunohistochemical studies demonstrated the presence of mesangial proliferation glomerulonephritis with IgG-IgA-C3 deposits in 7 cases (30% of the cases biopsied and 5,6% of the total), 69,6% of the cases had only minimal charges with negative immunofluorescence. 5) A mean follow-up of 4 years in two groups of patients (less than or greater than 6 years of age) has demonstrated that microscopic haematuria remains unchanged in 18-19% of both groups. A more marked improvement or normalization has been documented in the children more than 6 years aged (p less than 0,001) while a worsening has been observed in the children less than 6 years aged (p less than 0,005), with a statistically significant difference between the two groups considered."}, {"id": "wiki20220301en074_14199", "title": "Microhematuria", "score": 0.009873537566781634, "content": "Microhematuria, also called microscopic hematuria (both usually abbreviated as MH), is a medical condition in which urine contains small amounts of blood; the blood quantity is too low to change the color of the urine (otherwise, it is known as gross hematuria). While not dangerous in itself, it may be a symptom of kidney disease, such as IgA nephropathy or Sickle cell trait, which should be monitored by a doctor. The American Urological Association (AUA) recommends a definition of microscopic hematuria as three or more red blood cells per high-power microscopic field in urinary sediment from two of three properly collected urinalysis specimens. Microhematuria is usually asymptomatic, and there are medical guidelines on how to handle asymptomatic microhematuria (AMH) so as to avoid problems such as overtreatment or misdiagnosis. See also Proteinuria Hematuria Myoglobinuria Hemoglobinuria References External links 2012 AUA Guidelines"}, {"id": "pubmed23n0235_2713", "title": "[Rationalization of urine analysis while maintaining diagnostic accuracy (author's transl)].", "score": 0.00980392156862745, "content": "Screening 720 morning urinary samples for WBC, RBC and protein by test-strip, the number of subsequent microscopic examinations of urinary sediment was reduced to about half, without missing any significant number of clinically significant findings (4.4%). On the other hand, a large number (21.3%) of cases with obviously false-negative sediment findings were revealed. These were largely due to lysis of WBC and RBC, as well as poorly standardized methods of examining urinary sediment. But they could also have been due to differences in subjective criteria employed by the technicians. Our results indicate that using test-strips for screening, clinical routine examination can be rationalized, taking about half the time needed for sediment examination, with more potentially significant findings being discovered than missed."}, {"id": "pubmed23n0043_16875", "title": "[Clinical and laboratory correspondence to outpatients with the extreme value of C-reactive protein].", "score": 0.009708737864077669, "content": "It is the policy of Tenri Hospital to notify the patient promptly whenever an extreme laboratory data value is detected. We investigated the utility of forwarding clinical and laboratory correspondence to outpatients with extreme value of C-reactive protein (CRP). Sixty-eight outpatients with CRP levels more than 20 mg/dl detected during 1986 were studied. CRP was measured by turbidometric method, and a sample with CRP level more than 15 mg/dl was diluted with CRP negative serum (CRP level less than 0.2 mg/dl) and was reanalyzed. Fifty-two of 68 patients (76%) had infectious diseases as the causal disease of high CRP, and eight (12%) had other diseases. In the remaining (12%) the causes were unknown. In most patients the causal diseases were diagnosed within one or two days, but diagnosis required more than 4 days in those with acute pyelonephritis, meningitis, liver abscess or renal abscess, as these diseases were diagnosed after the examination of urine or cerebrospinal fluid, or after ultrasonography. Thirty-seven of 58 patients (64%) who had appointments with their physician on the day of the laboratory examination were admitted the same day, and two of 10 patients (20%) who had appointments on the following day were admitted on that day. Seventeen of 25 patients (68%) with urea-N levels more than 30 mg/dl, cholinesterase levels less than 0.7 delta pH and albumin levels less than 3.5 g/dl required more than 15 days to recover, while 29 of 32 patients (91%) with only 2 or fewer of these laboratory values required less than 14 days. The prompt notification of extreme CRP value is an important aspect of medical care. The examination of urine and cerebrospinal fluid and ultrasonography are necessary screening techniques accompanying examination of blood and plain chest X-ray. Urea-N, cholinesterase and albumin values should be determined at the same time as CRP value to assess prognosis."}, {"id": "wiki20220301en254_18252", "title": "Urine test strip", "score": 0.009615384615384616, "content": "Urinary sediment During routine screening, if a positive test for leukocytes, blood, protein, nitrite, and a pH greater than 7 is identified, the urine sediment be microscopically analysed to further pinpoint a diagnosis. Automated analysers Automatic analysis of urine test strips using automated urine test strip analysers is a well-established practice in modern-day urinalysis. They can measure calcium, blood, glucose, bilirubin, urobilinogen, ketones, leukocytes, creatinine, microalbumin, pH, ascorbic acid and protein. References Further reading Compendium Urinalysis: Urinalysis with Test Strips. Dr E F Hohenberger, Dr H Kimling (2002)http://www.diavant.com/diavant/servlet/MDBOutput?fileId=1392 Urinalysis Strips Instructions Urine tests"}, {"id": "pubmed23n0213_1876", "title": "[Critical study of microscopic hematuria disclosed by screening tests].", "score": 0.009615384615384616, "content": "Asymptomatic microscopic haematuria detected by dipstick in 0.28 p. 100 of 95,200 men was confirmed in only 0.11 p. 100. These results lead to the study of 272 cases to determine the value of dipsticks as a screening test and the benefits of complementary investigations to exclude symptomless diseases. Microscopic haematuria was confirmed in 65 p. 100 but results of other tests reduce the false positive screening results to 25.3 p. 100. Addis counting of at least 5,000 RBC/min is preferable to RBC count/ml which is dilution dependent. 77 p. 100 of microscopic haematuria are intermittent and can only be diagnosed by repeated Addis counts. In the absence of confirmed haematuria, only investigations to exclude proteinuria and urinary infections are justified."}, {"id": "article-25100_41", "title": "Microalbuminuria -- Evaluation", "score": 0.009605144589170148, "content": "In order to determine the cause of albuminuria as the underlying glomerular disease, the following should be done: Urine microscopy – Presence of dysmorphic red blood cells/casts 24-hour urine collection for the quantification of albumin excretion Serum creatinine, blood glucose, albumin, and cholesterol Autoantibody panel - If indicated by the clinical picture, test for antinuclear antibody (ANA), antistreptolysin O titers, anti-DNA antibodies, antineutrophil cytoplasmic antibodies (ANCA), cryoglobulins, anti-glomerular basement membrane (anti-GBM) antibodies, and complement levels (C3 and C4) Imaging studies in albuminuria may include the following: Renal ultrasonography – To establish glomerular disease as the cause of microalbuminuria, it is imperative to look at the echogenicity and size of the kidneys [18] Chest X-ray or computed tomography (CT) scan – If guided by the clinical picture"}, {"id": "wiki20220301en254_18211", "title": "Urine test strip", "score": 0.009523809523809525, "content": "Blood Blood may be present in the urine either in the form of intact red blood cells (hematuria) or as the product of red blood cell destruction, hemoglobin (hemoglobinuria). Blood present in large quantities can be detected visually. Hematuria produces cloudy red urine, and hemoglobinuria appears as a clear red specimen. Any amount of blood greater than five cells per microliter of urine is considered clinically significant; visual examination cannot be relied upon to detect the presence of blood. Microscopic examination of the urinary sediment shows intact red blood cells, but free hemoglobin produced either by hemolytic disorders or lysis of red blood cells is not detected. Therefore, chemical tests for hemoglobin provide the most accurate mean for determining the presence of blood. Once blood has been detected, the microscopic examination can be used to differentiate between hematuria and hemoglobinuria."}, {"id": "pubmed23n0399_3685", "title": "Laboratory and imaging studies used by French rheumatologists to determine the cause of recent onset polyarthritis without extra-articular manifestations.", "score": 0.009433962264150943, "content": "The cause of recent onset polyarthritis can be difficult to identify. To determine which laboratory and imaging studies French rheumatologists recommend, not taking cost into account, for the diagnosis of recent onset polyarthritis without extra-articular manifestations. From the list of the French Society for Rheumatology, a random sample of 210 rheumatologists was selected, who were asked to complete a questionnaire on the laboratory and imaging studies they would recommend in two fictional cases of recent onset polyarthritis (possible rheumatoid arthritis (RA)-case 1 and probable RA-case 2). In case 1, the following were recommended by over 75% of respondents: hand radiographs, rheumatoid factors (RFs), and antinuclear antibodies (ANA) (92%, 98%, and 98%, respectively). 50-74% of respondents recommended radiographs of the feet, knees, and chest (50%, 57%, and 66%, respectively); blood cell counts, erythrocyte sedimentation rate (ESR), serum assays of C reactive protein (CRP), aspartate aminotransferase (AST) and alanine aminotransferase (ALT) (65%, 74%, 67%, and 62%, respectively). 25-49% recommended determination of creatinine and proteinuria, HLA-B27, antikeratin antibody, radiographs of the pelvis, and synovial fluid analysis. Several investigations were recommended less often in case 2 than in case 1. Nevertheless, some laboratory and imaging studies (radiographs of hand, feet, knees, chest x rays, blood cell counts, ANA, RF, antikeratin antibody, CRP, ESR, creatinine, AST and ALT, proteinuria, and joint aspiration) were recommended by more than 25% of respondents in both cases. Wide variations were found among rheumatologists, indicating a need for standardisation. Some laboratory and imaging studies are recommended by at least 25% of respondents in recent onset polyarthritis with or without clues suggesting RA. In contrast, many tests were considered useful by fewer than 25% of the respondents in both cases."}]}}}} {"correct_option": 2, "explanations": {"1": {"exist": false, "char_ranges": [], "word_ranges": [], "text": ""}, "2": {"exist": true, "char_ranges": [[0, 69]], "word_ranges": [[0, 11]], "text": "Orthosis. Cobb angle between 25º - 45º. Immature skeleton (Risser 0)."}, "3": {"exist": false, "char_ranges": [], "word_ranges": [], "text": ""}, "4": {"exist": false, "char_ranges": [], "word_ranges": [], "text": ""}, "5": {"exist": false, "char_ranges": [], "word_ranges": [], "text": ""}}, "full_answer": "Orthosis. Cobb angle between 25º - 45º. Immature skeleton (Risser 0).", "full_answer_no_ref": "Orthosis. Cobb angle between 25º - 45º. Immature skeleton (Risser 0).", "full_question": "A 13-year-old female, with no relevant history, with menarche 3 months ago, followed since the age of 10 years by idiopathic scoliosis that has worsened. In the physical examination she presents a hump of 7 degrees in the Adams test and in the scoliogram a thoracolumbar curve T4-L1 of 35 degrees of Cobb and a Risser 0. The correct attitude to take will be:", "id": 616, "lang": "en", "options": {"1": "Recommend swimming and revision in three months.", "2": "Prescribe a corset-type orthosis.", "3": "Refer to physiotherapy for spine elastification.", "4": "Review in 6 months with a new X-ray.", "5": NaN}, "question_id_specific": 119, "type": "TRAUMATOLOGY", "year": 2022, "rag": {"clinical_case_options": {"MedCorp": {"RRF-2": [{"id": "pubmed23n0573_15298", "title": "The natural history of idiopathic scoliosis.", "score": 0.019334952363160844, "content": "Background. The natural history of idiopathic scoliosis is a crucial issue in the planning and assessment of different treatment methods. This article presents the evaluation of scoliotic deformity in immature patients who have been in observation without any treatment. Material and methods. 159 patients (128 girls, 31 boys) were examined between 1971 and 2002. Skoliosis was diagnosed at a mean age of 6 years 4 months (range 2.1-8.10), and observation was concluded at a mean age of 16 years 11 months (range 14.6-20.3). The mean follow-up was 10 years 5 months. The prognostic factors analyzed were: age, sex, Cobb angle, Mehta angle, apical vertebral rotation, specific rotation, Risser test. The progression and regression of curvature was analyzed in different biological age periods, and was measured by calculating the difference in the Cobb angle on successive x-rays divided by the interval between x-rays. Results. The mean progression of curvature before age 5 was 5.7 degrees per year; in the 6-10 age bracket, 2.3 degrees per year; in the 11-15 age bracket, 7.4 degrees per year; in the >15 age bracket, 0.3 degrees per year. The mean progression for patients with Risser 1 was 8.8 degrees per year; Risser 2, 7.3 degrees per year; Risser 3, 5.1 per year; Risser 4, 2.1 degrees per year; Risser 5, 0.3 degrees per year. Conclusions. The progression of curvature in idiopathic scoliosis is variable, and is influenced by age. Knowledge of the natural history of idiopathic scoliosis is a crucial tool in predicting the development of spinal curvature. The Risser test and biological age are the only effective predictors of progression."}, {"id": "pubmed23n0825_810", "title": "Effectiveness of brace treatment for adolescent idiopathic scoliosis.", "score": 0.01899189918991899, "content": "Effectiveness of brace treatment for adolescent idiopathic scoliosis (AIS) was demonstrated by the BrAIST study in 2013. Objectives of this study were to confirm its effectiveness by analyzing our results and to clarify the factors affecting the results of the treatment. According to the Scoliosis Research Society AIS brace studies standardization criteria, patients with age 10 years or older, Risser 0 to II, less than 1 year post-menarche, curve magnitude 25 to 40 degrees before brace treatment and who received no prior treatment were included in the study. At skeletal maturity, the rate of the patients whose curve was stabilized, exceeded 45 degrees, and who were recommended or underwent surgery were investigated. Additionally, initial correction rate by the brace and factors affecting the results were investigated. A total of 33 patients (27 females and 6 males) could be followed-up until their skeletal maturity and included in the analysis. An average age was 11.9 years, average Cobb angle was 30.8°, and Risser sign was 0 in 13 patients, I in 5, and II in 15 patients before treatment. There were 13 thoracic curves, 14 thoracolumbar or lumbar curves, and 6 double curves. Initial correction rate by the brace was 53.8% for the total curves. In terms of curve pattern, 34.4% for thoracic curve, 73.9% for thoracolumbar or lumbar curve, and 48.8% for double curve. After an average follow-up period of 33 months, 8 patients improved in more than 6 degrees, change of 17 patients were within 6 degrees, and 8 progressed in more than 6 degrees. Therefore, totally, 76% (25/33) of the curves were stabilized by the treatment. Four curves (12%) exceeded 45 degrees and one patient (3%) underwent surgery. Our results were better than the reported natural history. Factors that affected the results were hump degree before treatment and initial correction rate by the brace. 76% of the curve with AIS could be stabilized by brace treatment. Brace treatment was effective for treatment of AIS. Factors affecting the results were hump degrees and initial correction rate."}, {"id": "pubmed23n0258_10528", "title": "[The Schroth scoliosis-specific back school--initial results of a prospective follow-up study].", "score": 0.017496229260935144, "content": "The prospective study reported here was instituted in 1987 to obtain more detailed data on the efficacy of scoliosis-specific spinal rehabilitation after Schroth. Inclusion criteria were 1) idiopathic scoliosis, 2) Risser stage < 4, 3) no treatment with corset or electrical stimulation, 4) first examination between 1 and 3 years postoperatively, 5) usable total X-rays taken with the patient standing not more than 6 months prior to admission. A total of 181 scoliosis patients with an average age of 12.76 years and an average Cobb angle of 27 degrees were included in the study. The average Risser's sign was 1.4 and the average follow-up period was 33 months. No cases of relative progression (annual increase in curvature of 5 degrees or more) were observed. For the purpose of comparison with the spontaneous course, the patients were grouped by age and severity of scoliosis. Both the absence of any relative progression as well as direct comparison of the development of scoliosis under therapy with the spontaneous course confirmed the efficacy of the stationary rehabilitation programme notably in cases with poor prognosis, i.e. with large scoliosis angles and unfavourable curvatures."}, {"id": "pubmed23n0726_2853", "title": "Effectiveness of the Charleston night-time bending brace in the treatment of adolescent idiopathic scoliosis.", "score": 0.01562595373252762, "content": "Part-time or night-time bracing has been introduced to address the poor compliance and psychological burden of full-time bracing. The results of various bracing methods vary, however, due to a lack of consistent inclusion criteria and definitions of brace effectiveness. We have evaluated the effectiveness of the Charleston night-time bending brace in the treatment of adolescent idiopathic scoliosis based on the new standardized criteria proposed by the Scoliosis Research Society. To be included in this study, patients met the following criteria proposed by the Scoliosis Research Society: diagnosis of adolescent idiopathic scoliosis, age 10 years and older when the orthosis was prescribed, Risser 0-2, a primary curve magnitude of 25 to 40 degrees, and no prior treatment. A total of 95 patients (87 girls, 8 boys) were included. At skeletal maturity, 80 patients (84.2%) had 5 degrees or less curve progression and 15 (15.8%) had 6 degrees or more progression. Seven patients (7.8%) were recommended to undergo or underwent surgery before skeletal maturity. Eleven patients (12.6%) progressed beyond 45 degrees. According to these 3 criteria, the Charleston night-time brace was successful in 74 patients (77.9%). Depending on curve type, we observed success rates of 78.3% (47/60) for double, 71.4% (15/21) for thoracic, 83.3% (5/6) for thoracolumbar, and 87.5% (7/8) for lumbar curves. Success rates of 80.0% (36/45) and 76.0% (38/50) were observed in patients with curve magnitudes at bracing of 25 to 30 degrees and 31 to 40 degrees, respectively. Patients with high apex curves had a 67.6% (23/34) success rate, and those with low apex curves had 83.0% (39/47) success rate. Brace success rates among patients with initial Risser signs of 0, 1, and 2 were 68.8% (22/32), 80.6% (25/31), and 84.4% (27/32), respectively. Compared with the results of previous natural history and conventional brace study, the Charleston night-time bending brace is effective for the treatment of adolescent idiopathic scoliosis. Level VI."}, {"id": "pubmed23n0321_19357", "title": "Preliminary results and worst-case analysis of in patient scoliosis rehabilitation.", "score": 0.01552757793764988, "content": "The purpose of this study was to assess the effectiveness of a scoliosis-specific rehabilitation programme as it is carried out in the Katharina Schroth Spinal Deformities Rehabilitation Center. Physiotherapy in the treatment of scoliosis patients is still regarded as ineffective since the study by the American Orthopedic Association in 1941, which showed that general exercises could not influence the natural history of scoliosis. However, specific exercise programmes were not known in the USA at that time. This preliminary study started in 1989 with the following inclusion criteria: (1) diagnosis of idiopathic scoliosis; (2) risser sign < 4; (3) no treatment other than physiotherapy; (4) first control after 1-3 years during repeated in patient treatment; (5) standing AP radiograph taken not more than 6 months before the first in patient treatment. A total of 181 scoliosis patients, with an average age of 12.7 years and an average angle of curvature of 27% according to Cobb, were included in this study. The average risser sign was 1.4, the average follow-up 33 months. The Cobb angle of the major curve was measured in a standardized way. The results of our preliminary study were compared to natural history as known from literature. For the worst-case analysis additionally a questionnaire was sent to the non-repeaters treated at our centre at the same time (1989 and 1990) as the patient sample described above, taking into account the same inclusion criteria for this patient sample except point 4. Results showed that progression as usually defined (increase in curvature of 5 degrees or more per year) has not been found in the preliminary study. The patient sample of this study was divided into different age groups and different groups of curve magnitude, for comparison with other studies. Additional to the patient sample of the preliminary study, 116 of the patients from the years 1989 and 1990 fulfilled the inclusion criteria of the preliminary study with the exception of point 4. These patients formed the questionnaire sample for the worst-case analysis showing that the progression rate of the 181 patients from the preliminary study and the 116 patients of the questionnaire sample together was still better than natural history even if all drop-outs were considered to be failures. The fact that there was no relative progression in our patients sample treated solely by physiotherapy (preliminary study), seems to show the effectiveness of the inpatient rehabilitation programme even in cases with a bad prognosis, severe angles of curvature and unfavourable curvature patterns. A worst-case analysis does not prevent this conclusion, even if all dropouts from the non-repeaters group were considered as failures."}, {"id": "pubmed23n0830_21807", "title": "Mild angle early onset idiopathic scoliosis children avoid progression under FITS method (Functional Individual Therapy of Scoliosis).", "score": 0.015423410743607788, "content": "Physiotherapy for stabilization of idiopathic scoliosis angle in growing children remains controversial. Notably, little data on effectiveness of physiotherapy in children with Early Onset Idiopathic Scoliosis (EOIS) has been published.The aim of this study was to check results of FITS physiotherapy in a group of children with EOIS.The charts of the patients archived in a prospectively collected database were retrospectively reviewed. The inclusion criteria were:diagnosis of EOIS based on spine radiography, age below 10 years, both girls and boys, Cobb angle between 118 and 308, Risser zero, FITS therapy, no other treatment (bracing), and a follow-up at least 2 years from the initiation of the treatment. The criterion for curve progression were as follows: the Cobb angle increase of 68 or more, for curve stabilization; the Cobb angle was 58 comparing to the initial radiograph,for curve correction; and the Cobb angle decrease of 68 or more at the final follow-up radiograph.There were 41 children with EOIS, 36 girls and 5 boys, mean age 7.71.3 years (range 4 to 9 years) who started FITS therapy. The curve pattern was single thoracic (5 children), single thoracolumbar (22 children) or double thoracic/thoracolumbar (14 children), totally 55 structural curvatures. The minimum follow-up was 2 years after initiation of the FITS treatment, maximum was 16 years, mean 4.8 years). At follow-up the mean age was 12.53.4 years. Out of 41 children, 10 passed pubertal growth spurt at the final follow-up and 31 were still immature and continued FITS therapy. Out of 41 children, 27 improved, 13 were stable, and one progressed. Out of 55 structural curves, 32 improved, 22 were stable and one progressed. For the 55 structural curves, the Cobb angle significantly decreased from 18.085.48 at first assessment to 12.586.38 at last evaluation,p<0.0001, paired t-test. The angle of trunk rotation decreased significantly from 4.782.98 to 3.282.58 at last evaluation, p<0.0001,paired t-test.FITS physiotherapy was effective in preventing curve progression in children with EOIS. Final postpubertal follow-up data is needed."}, {"id": "pubmed23n0345_12855", "title": "[Primary correction of scoliosis with the Wilmington corset].", "score": 0.015093273035613341, "content": "As part of our study, the effectiveness and patient's acceptance of the Wilmington-brace is to be evaluated. The effectiveness can be documented with the help of the primary correction achieved, especially in light of the fact, that the primary correction and the long-term results are directly dependant upon one another. We examined a total of 52 patients with an idiopathic scoliosis treated in a thermoplast brace. The group consisted of 38 female and 14 male patients (average age 11.6 years). The angulation was measured with the help of the Cobb-angle and the rotation with the method described by Nash and Moe. The skeletal age was classified according to Risser's-sign. The angle determinations were carried out at three separate points in time--at first presentation, prior to bracing and four to six weeks following bracing. The patients presented with an average angulation of 31 degrees. The average correction achieved in the Wilmington-brace was 41%. This corresponds to a correction of 13 degrees. The best primary correction (45%) was obtained in the thoracolumbar spine. Those patients with the smallest deformity at the onset of treatment showed the best results. The scoliosis with a large primary deformity and a marked rotation of the vertebral bodies responded poorly to correction. Advanced age or skeletal maturity were also limiting factors. Physical therapy had a positive influence on the amount of primary correction obtained. The Wilmington-brace (thermoplast) allows for a good primary correction of idiopathic scoliosis. The simplicity of application and the low production costs are also positive attributes."}, {"id": "pubmed23n0637_18882", "title": "Effectiveness of complete conservative treatment for adolescent idiopathic scoliosis (bracing and exercises) based on SOSORT management criteria: results according to the SRS criteria for bracing studies - SOSORT Award 2009 Winner.", "score": 0.014421936524503059, "content": "The SRS criteria give the methodological reference framework for the presentation of bracing results, while the SOSORT criteria give the clinical reference framework for an appropriate bracing treatment. The two have not been combined in a study until now. Our aim was to verify the efficacy of a complete, conservative treatment of Adolescent Idiopathic Scoliosis (AIS)according to the best methodological and management criteria defined in the literature. Study Design. Retrospective study. Population. We included all AIS patients respecting the SRS inclusion criteria (age 10 years or older; Risser test 0-2; Cobb degrees 25-40 degrees ; no prior treatment; less than one year post-menarchal) who had reached the end of treatment since our institute database start in 2003. Thus we had 44 females and four males, with an age of 12.8 +/- 1.6 at the commencement of the study. Methods. According to individual needs, two patients have been treated with Risser casts followed by Lyon brace, 40 with Lyon or SPoRT braces (14 for 23 hours per day, 23 for 21 h/d, and seven for 18 h/d at start), and two with exercises only (1 male, 1 female): these were excluded from further analysis. Outcome criteria. SRS (unchanged; worsened 6 degrees or more; over 45 degrees at the end of treatment; surgically treated; two years' follow-up); clinical (ATR, Aesthetic Index, plumbline distances); radiographic (Cobb degrees); and ISICO (optimal; minimal). Statistics. Paired ANOVA and t-test, Tukey-Kramer and chi-square test. Median reported compliance during the 4.2 +/- 1.4 treatment years was 90% (range 5-106%). No patient progressed beyond 45 degrees , nor was any patient fused, and this remained true at the two-year follow-up for the 85% that reached it. Only two patients (4%) worsened, both with single thoracic curve, 25-30 degrees Cobb and Risser 0 at the start. We found statistically significant reductions of the scoliosis curvatures (-7.1 degrees ): thoracic (-7.3 degrees ), thoracolumbar (-8.4 degrees ) and lumbar (-7.8 degrees ), but not double major. Statistically significant improvements have also been found for aesthetics and ATR. Respecting also SOSORT management criteria and thus increasing compliance, the results of conservative treatment were much better than what had previously been reported in the literature using SRS criteria only."}, {"id": "pubmed23n0805_1755", "title": "Low rate of surgery in juvenile idiopathic scoliosis treated with a complete and tailored conservative approach: end-growth results from a retrospective cohort.", "score": 0.014164173522812266, "content": "The main distinctive aspect of Juvenile Idiopathic Scoliosis (JIS) with respect to Adolescent Idiopathic Scoliosis (AIS) is the high risk of severe deformity and surgery. Approximately 70% of curves in patients with JIS progress and ultimately require surgery. There are presently very few studies with long-term follow-up of JIS and even fewer looking specifically at bracing Purpose To verify the effectiveness of a complete conservative treatment, including bracing and exercises, for JIS. Retrospective cohort observational study nested in a clinical prospective database of consecutive outpatients. Patient Sample Inclusion criteria: JIS, no previous treatment, all consecutive radiographies available from treatment start to end of growth (Risser sign 3). We found 30 patients, 27 females, 10 JIS type 1; mean age at first diagnosis was 7.8 +/-1.5 and mean treatment lasted 5.8 years. Cobb degrees 24.4+/-10 degrees, with 7 cases >30 degrees, and 2 > 45degrees. Outcome Measures Physiological measures. Radiographic and clinical data. Treatment (exercises alone, or elastic-rigid-highly rigid braces plus exercises) was tailored and continuously changed according to Cobb degrees, individual preferences, anthropometric characteristics, pubertal spurt, remaining growth, rotation, hump, lumbar curve take-off, and imbalance. The SOSORT Guidelines for patients' management have been followed. Funding and Conflict of Interest: no. 33.3% (95% Confidence Interval 16.4-50.2%) of patients worsened over the years. At the end of growth, 6.6% (0-15.5%) had surgical deformities (>45degrees). We observed a good correction in the first years of treatment until pubertal growth spurt, when progression was usually noted and treatment changed increasing corrective forces (hours or rigidity of bracing). 23 cases were followed up until they had two consecutive radiographies showing Risser sign 5 and showed stability. Conservative treatment initiated already in childhood may favorably change the natural history of JIS with the aim of reaching a curve as far as possible from surgical thresholds. Observation, physical exercises, braces can be useful tools in the hand of physicians, but they must be carefully utilized by a deep knowledge of JIS."}, {"id": "pubmed23n0668_24621", "title": "A comparison of thoracolumbosacral orthoses and SpineCor treatment of adolescent idiopathic scoliosis patients using the Scoliosis Research Society standardized criteria.", "score": 0.013440860215053764, "content": "SpineCor is a relatively new bracing system that uses dynamic bracing concepts in the treatment of adolescent idiopathic scoliosis (AIS). Limited data are available regarding its effectiveness. This study compared treatment outcomes of 2 groups of AIS patients treated via either a conventional rigid thoracolumbosacral orthoses (TLSO) or a SpineCor nonrigid orthosis. We identified 2 scoliosis patient cohorts: 35 patients treated with a TLSO and 32 patients treated with a SpineCor orthosis. All patients included in these groups conformed with the Scoliosis Research Society (SRS) standardized criteria for AIS bracing: (1) Risser < or =2, (2) curve magnitude 25 to 40 degrees, (3) age > or =10 years. Outcomes were SRS standardized with failure being defined as curve progression > or =6 degrees, or ever exceeding 45 degrees, or having surgery recommended before skeletal maturity. All patients were followed through the completion of brace treatment or attainment of other treatment end points. The Yates corrected chi test and unpaired t test were used for data analysis. The 35 patients (32 girls, 3 boys) in the TLSO group had an average age of 13 years (range: 11.1-16.8) and an average primary curve magnitude of 33 degrees (range: 25-40 degrees). Follow-up averaged 2 years (range: 8-61 m) from the beginning of brace treatment. The 32 patients (28 girls, 4 boys) in the SpineCor group had an average age of 13 years (range: 11-15.2) and an average primary curve magnitude of 31 degrees (range: 25-40 degrees). Follow-up for this group averaged 2 years and 6 months (range: 13-73 mo) from the beginning of brace treatment. No significant difference (P=0.75) was found using the more strict outcome measure (< or =5-degree curve progression) as the success rates were 60% (21/35) for TLSO and 53% (17/32) for SpineCor. Similarly, no significant difference (P=0.62) was found using the more liberal outcome measure (never reached 45 degrees) as the success rates were 80% (28/35) for TLSO and 72% (23/32) for SpineCor. We were unable to identify any significant differences in brace treatment outcomes when comparing TLSO and SpineCor treated patients."}, {"id": "wiki20220301en506_25267", "title": "Management of scoliosis", "score": 0.013235294117647059, "content": "Surgery is indicated by the Society on Scoliosis Orthopaedic and Rehabilitation Treatment (SOSORT) at 45 degrees to 50 degrees and by the Scoliosis Research Society (SRS) at a Cobb angle of 45 degrees. SOSORT uses the 45-degree to 50-degree threshold as a result of the well-documented, plus or minus five degrees measurement error that can occur while measuring Cobb angles. Scoliosis braces are usually comfortable for the patient, especially when it is well designed and fit; also after the 7- to 10-day break-in period. A well fit and functioning scoliosis brace provides comfort when it is supporting the deformity and redirecting the body into a more corrected and normal physiological position. The Scoliosis Research Society's recommendations for bracing include curves progressing to larger than 25°, curves presenting between 30 and 45°, Risser sign 0, 1, or 2 (an X-ray measurement of a pelvic growth area), and less than six months from the onset of menses in girls."}, {"id": "pubmed23n0239_2151", "title": "[Results of brace treatment in idiopathic scoliosis--evaluation of the patients treated for over 2 years or those who completed the treatment].", "score": 0.012824510797536447, "content": "The results of the brace treatment (Milwaukee brace, Thoraco-Lumbo-Sacral-Orthosis, Boston-Milwaukee brace) were studied in 509 patients with idiopathic scoliosis, who were braces for an average of 3 years and 3 months ranging from 2 to 10 years. Of these patients, 60 were followed up for about 24 months after the brace was discontinued. The distribution of these patients according to curve pattern was as follows: 319 had thoracic curve; 78 had lumbar curve (combining lumbar and thoracolumbar curves together as one group); and 112 had double major curve. The findings are summarized and conclusions were drawn as follows. 1. The best correction was obtained within a year after the initiation of bracing, followed by a gradual loss of correction. 2. There was a significant difference between the final correction rate and the best correction rate in the brace. 3. In the patients with ages ranging from 11 to 14 years and with curves of 30 degrees or less at the beginning of brace treatment, the curves were maintained within 30 degrees at the final stage. 4. Those patients who cooperated well in wearing their braces had a smaller loss of correction at the final stage. 5. In 63 per cent of the cases who showed progression in the brace, iliac apophysis did not appear at the beginning of brace treatment. 6. Moiré topography was used for the analysis of cosmesis and it was shown to be a valuable method of three dimensional evaluation. 7. In 60 patients who were followed up for 24 months on average after the brace was discontinued, the curves had improved an average of 1.6 degrees at the time of their last check up."}, {"id": "pubmed23n0730_6128", "title": "Bracing can reduce high degree curves and improve aesthetics immediately after the end of growth. Final results of a retrospective case series.", "score": 0.012481530445602303, "content": "Recently it has been shown that idiopathic scoliosis (IS) curves can be reduced with bracing, and it has been proposed that this could be useful in non-surgically treated high degree curves even after Risser 3. Moreover, bracing has been shown to be able to improve aesthetics, and this could be another reason to treat some patients with cosmetic needs. Our aim is to preliminary check if results can be obtained in IS patients after Risser 3. Design. Retrospective uncontrolled cohort study. Inclusion criteria. All IS patients treated on a voluntary basis for aesthetic reasons and/or for curve reduction; Risser 4-5 at start; end of treatment reached. Population. 34 females and 2 males, age 16.2±1.6 years, Cobb angle 27.6°±8.9°. Treatment. Lyon or SPoRT braces 18 to 24 hours/day, specific SEAS exercises, rapid weaning (2-3 hours every 6 months). Outcome criteria: SRS (unchanged; worsened over 6°; over 45° at the end of treatment; surgically treated), radiographic and clinical. Statistics. ANOVA and chi-test. The reported compliance during the 2.8 ± 1.1 treatment years was 95.1%, while residual growth was 0.9 ± 1.1 cm. Improvements were found in 39% of this cohort, (46% in curves over 30°). Only 1 patient progressed 6°. We found highly statistically significant reductions of maximal (-4.4°), thoracic (-6.0°) and thoracolumbar (-6.6°) curves. Statistically significant improvements were found for Aesthetic Index. Before 20 years of age, even in skeletally mature patients, it is possible to reach radiographic and aesthetic improvements, although not as good as during growth. Correction is based on bone growth, but ligaments and neuromuscular control of posture can also be involved."}, {"id": "pubmed23n0638_18760", "title": "Treatment of thoraco-lumbar curves in adolescent females affected by idiopathic scoliosis with a progressive action short brace (PASB): assessment of results according to the SRS committee on bracing and nonoperative management standardization criteria.", "score": 0.012043399638336347, "content": "The effectiveness of conservative treatment of scoliosis is controversial. Some studies suggest that brace is effective in stopping curve progression, whilst others did not report such an effect.The purpose of the present study was to effectiveness of Progressive Action Short Brace (PASB) in the correction of thoraco-lumbar curves, in agreement with the Scoliosis Research Society (SRS) Committee on Bracing and Nonoperative Management Standardisation Criteria. Fifty adolescent females (mean age 11.8 +/- 0.5 years) with thoraco-lumbar curve and a pre-treatment Risser score ranging from 0 to 2 have been enrolled. The minimum duration of follow-up was 24 months (mean: 55.4 +/- 44.5 months). Antero-posterior radiographs were used to estimate the curve magnitude (CM) and the torsion of the apical vertebra (TA) at 5 time points: beginning of treatment (t1), one year after the beginning of treatment (t2), intermediate time between t1 and t4 (t3), end of weaning (t4), 2-year minimum follow-up from t4 (t5). Three situations were distinguished: curve correction, curve stabilisation and curve progression.The Kruskal Wallis and Spearman Rank Correlation tests have been used as statistical tests. CM mean value was 29,30 +/- 5,16 SD at t1 and 14,67 +/- 7,65 SD at t5. TA was 12.70 +/- 6,14 SD at t1 and 8,95 +/- 5,82 at t5. The variation between measures of Cobb and Perdriolle degrees at t1,2,3,4,5 and between CM t5-t1 and TA t5-t1 were significantly different.Curve correction was accomplished in 94% of patients, whereas a curve stabilisation was obtained in 6% of patients. The PASB, due to its peculiar biomechanical action on vertebral modelling, is highly effective in correcting thoraco-lumbar curves."}, {"id": "pubmed23n0740_12959", "title": "Scoliosis detection, patient characteristics, referral patterns and treatment in the absence of a screening program in Norway.", "score": 0.011942959001782532, "content": "Early diagnosis of idiopathic scoliosis allows for observation and timely initiation of brace treatment in order to halt progression. School scoliosis screening programs were abolished in Norway in 1994 for lack of evidence that the programs improved outcome and for the costs involved. The consequences of this decision are discussed. To describe the detection, patient characteristics, referral patterns and treatment of idiopathic scoliosis at a scoliosis clinic during the period 2003-2011, when there was no screening and to compare treatment modalities to the period 1976-1988 when screening was performed. Patient demographics, age at detection, family history, clinical and radiological charts of consecutive patients referred for scoliosis evaluation during the period 2003-2011, were prospectively registered. Patients were recruited from a catchment area of about 500000 teenagers. Maturity was estimated according to Risser sign and menarcheal status. Severity of pain was recorded by a verbal 5-point scale from no pain to pain at all times. Physical and neurological examinations were conducted. The detector and patient characteristics were recorded. Referral patterns of orthopedic surgeons at local hospitals and other health care providers were recorded. Patient data was obtained by spine surgeons. Treatment modalities in the current period were compared to the period 1976-1988. We registered 752 patients with late onset juvenile and adolescent idiopathic scoliosis from 2003-2011. There were 644 (86%) girls and 108 (14%) boys. Mean age at detection was 14.6 (7-19) years. Sixty percent had Risser sign ≥ 3, whilst 74% were post menarche with a mean age at menarche of 13.2 years. Thirty-one percent had a family history of scoliosis. The mean major curve at first consultation at our clinic was 38° (10°-95°). About 40% had a major curve >40°. Seventy-one percent were detected by patients, close relatives, and friends. Orthopaedic surgeons referred 61% of the patients. The mean duration from detection to the first consultation was 20(0-27) months. The proportion of the average number of patients braced each year was 68% during the period with screening compared to 38% in the period without screening, while the proportion for those operated was 32% and 62%, respectively ( p=0.002, OR 3.5, (95%CI 1.6 to 7.5). In the absence of scoliosis screening, lay persons most often detect scoliosis. Many patients presented with a mean Cobb angle approaching the upper limit for brace treatment indications. The frequency of brace treatment has been reduced and surgery is increased during the recent period without screening compared with the period in the past when screening was still conducted."}, {"id": "wiki20220301en007_12922", "title": "Scoliosis", "score": 0.011295928500496523, "content": "For example, a person who is still growing with a 17° Cobb angle and significant thoracic rotation or flatback could be considered for nighttime bracing. On the opposite end of the growth spectrum, a 29° Cobb angle and a Risser sign three or four might not need to be braced because the potential for progression is reduced. The Scoliosis Research Society's recommendations for bracing include curves progressing to larger than 25°, curves presenting between 30 and 45°, Risser sign 0, 1, or 2 (an X-ray measurement of a pelvic growth area), and less than six months from the onset of menses in girls. Scoliosis braces are usually comfortable, especially when well designed and well fitted, also after the 7- to 10-day break-in period. A well fitted and functioning scoliosis brace provides comfort when it is supporting the deformity and redirecting the body into a more corrected and normal physiological position."}, {"id": "pubmed23n1138_9600", "title": "Does Risser Casting for Adolescent Idiopathic Scoliosis Still Have a Role in the Treatment of Curves Larger Than 40°? A Case Control Study with Bracing.", "score": 0.010638598873892992, "content": "15 years. In some cases with juveniles, bracing has reduced curves significantly, going from a 40 degrees (of the curve, mentioned in length above.) out of the brace to 18 degrees in it. Braces are sometimes prescribed for adults to relieve pain related to scoliosis. Bracing involves fitting the patient with a device that covers the torso; in some cases, it extends to the neck. The most commonly used brace is a TLSO, such as a Cheneau type brace, a corset-like appliance that fits from armpits to hips and is custom-made from fiberglass or"}, {"id": "pubmed23n0885_17329", "title": "Immediate effects of scoliosis-specific corrective exercises on the Cobb angle after one week and after one year of practice.", "score": 0.009523809523809525, "content": "We are unaware of any studies describing the immediate effects of scoliosis-specific exercises on the Cobb angle measured by radiograph. This study aimed to describe the differences between radiographs obtained with and without corrective exercises after initial training and after one year. A female with adolescent idiopathic scoliosis was first seen at age 13 years, 0 months with a Risser 0. She had a 43265 mg/dL"}, {"id": "pubmed23n0079_1903", "title": "[DBMP-85 was effective at diagnosis and LVP was effective at relapse in a case of acute mixed leukemia].", "score": 0.009259259259259259, "content": "A 16 year-old boy was admitted to our hospital in April 1985, because of bilateral submandibular swellings. Hematological examination revealed Hb was 7.3 g/dl, WBC was 89,000/microliters (76% blast), and platelet was 154,000/microliters. His bone marrow was hypercellular and consisted with 91% blasts. Myeloperoxidase staining was positive for 38% of blasts. Auer rods were seen in some of blasts. Thus, the diagnosis was M1 according to FAB classification. Cytogenetic studies of 20 marrow cells were performed and all cells had 46, XY, -1, -7, 3q-, 7q-, 17q+, +2mar. Eighty five percent of blasts expressed HLA-DR and 43% of blasts expressed CD2 and CD13 simultaneously. Thus, this leukemia was considered as the hybrid type of acute mixed leukemia by surface marker analysis. DBMP-85 regimen, the chemotherapy for AML, was started after admission and complete remission (CR) was attained in June 1985. After 4 courses of post remission chemotherapy, he discharged in December 1985 and was followed at our outpatient clinic without chemotherapy. His disease was relapsed in June 1986, and the combination chemotherapy with mitoxantrone, etoposide and Ara-C was applied to him but failed to attain CR. Then, LVP protocol, the chemotherapy for ALL, was started and CR was achieved. The blasts at relapse had morphologically myeloid features, and expressed HLA-DR, CD2 and CD13 as well as at diagnosis. Cytogenetic studies at relapse showed some karyotype except gaining 12p- anomaly. Therefore, same blasts were considered to emerge at relapse. Our case suggests that LVP therapy may be effective for AML expressing myeloid and lymphoid surface markers."}, {"id": "pubmed23n0792_7780", "title": "[Identification of splenic marginal zone lymphoma from B lymphoproliferative disorders by flow cytometry].", "score": 0.009174311926605505, "content": "The splenic marginal zone lymphoma (SMZL) is a relatively rare chronic B lymphoproliferative disease, which primarily manifest increase of peripheral blood lymphocyte count and/or scale, and splenomegaly, while the peripheral superficial lymph nodes are often not swollen. Therefore, the splenectomy are usually needed to confirm the diagnosis, but the majority of patients could not accept such management, resulting in early difficult diagnosis. This study was purposed to explore the more prior way for diagnosis based flow cytometry (FCM). Six patients with suspected diagnosis of SMZL were used as research objects, 10 healthy bone marrow donors and 10 cases of chronic lymphocytic leukemia (CLL), 3 cases of hairy cell leukemia (HCL), 3 cases of lymphatic plasma cell lymphoma/Waldenströ's macroglobulinemia (LPL/WM) were selected as control. The immunophenotype of bone marrow cells were analyzed and compared by FCM using a panel of antibodies including CD45, CD5, CD10, CD19, CD20, CD22, CD23, CD25, CD103, CD11c, CD123, κ,λ, Cyclin D1, and combined with bone marrow cell morphology. The results indicated that 6 cases of suspected SMZL showed a large increase of lymphocytes and splenomegaly. Because absence of peripheral lymphadenopathy, 6 patients did not suffer from lymph node biopsy, only 1 patient underwent diagnostic splenectomy. The immunophenotypes of bone marrow in patients and controls were analyzed by FCM, as a result, except for the healthy donors, varying degrees of abnormal mature B cell clones were found in bone marrow of all patients, and the further differentiation from other B-cell tumors was performed through CD5, CD10 expression and combination with other B-cell phenotype. All 6 cases of SMZL patients expressed CD19(+) and CD20(+), but CD10 expression was negative, 4 patients expressed CD5(-), 2 patients expressed CD5(+). The expressions of CD23, CD38, ZAP-70, CD11c, CD103, CD123, Cyclin D1 were negative. The morphological examination of bone marrow cells showed velutinous abnormal lymphocytes. Combined with clinical characteristics, 6 patients were diagnosed as SMZL, 1 patient suffered from splenectomy because of concurrent hypersplenism, and this postoperative pathologic examination confirmed the patient with SMZL. Ten cases of CLL mainly expressed CD5, CD23; 3 cases of HCL had more typical morphology of \"hair like\" in addition to CD11c, CD103 and CD123 positive; 3 cases of LPL/WM had significantly increased light chain restriction expression, IgM, plasmacytoid lymphocytes. It is concluded that the FCM immunophenotype analysis can be used as a powerful tools for clinical diagnosis of SMZL."}, {"id": "pubmed23n0347_22507", "title": "[Detection of PNH clones using flow cytometry in aplastic anemia and paroxysmal nocturnal hemoglobinuria].", "score": 0.009174311926605505, "content": "To detect and quantify by flow cytometry (FC) PNH clones in paroxysmal nocturnal haemoglobinuria (PNH) and aplastic anaemia (AA) patients. We have performed a flow cytometric analysis to determine the granulocyte expression of CD55 and CD59 from 29 patients with AA and 11 patients with PNH. In the 11 PNH patients the study showed 58 +/- 34% and 56 +/- 32% (mean +/- SD) CD55(-) y CD59(-) granulocytes. A good correlation was found between the results of FC and haemolysis. The follow-up study showed PNH clone progression in one case and stability in 5 cases. Among 11 AA patients studied at diagnosis, two presented a population of CD55(-) granulocytes (14% and 48%) with CD59 normal, this defect disappeared in both patients after immunosuppressive therapy. The FC study revealed PNH clones in 7 cases among the 26 analyzed after treatment (23 with ATG and/or CyA), in 3 cases with negative Ham's test (in two this became positive 6 and 12 months later). The mean values obtained in these 7 patients with PNH-AA syndrome were 26 +/- 15% y 36 +/- 30% (mean +/- SD) CD55(-) and CD59(-) granulocytes. The median time from diagnosis to detection of PNH phenomenon was 83 months. In the follow-up study, 4 cases had stability, one case had a decrease and one a progression of the abnormal clone. In a retrospective analysis, among the 7 patients with PNH-AA syndrome, 5 had a partial response after the initial treatment. The FC on granulocytes is a useful method to diagnose and characterize PNH. This test is good for early detection of PNH clones in AA patients at initial diagnosis and in long term survivors. In both diseases it permits measuring the extent of the abnormal clone and its follow up. The extent of the defect is more related to haemolysis than the haematopoietic deficiency. PNH development seems to be more frequent in AA patients with incomplete response after immunosuppressive therapy and in some cases the defect could be latent at the time of diagnosis."}, {"id": "wiki20220301en018_98872", "title": "Chronic lymphocytic leukemia", "score": 0.009094575519257915, "content": "Diagnosis The diagnosis of CLL is based on the demonstration of an abnormal population of B lymphocytes in the blood, bone marrow, or tissues that display an unusual but characteristic pattern of molecules on the cell surface. CLL is usually first suspected by a diagnosis of lymphocytosis, an increase in a type of white blood cell, on a complete blood count test. This frequently is an incidental finding on a routine physician visit. Most often the lymphocyte count is greater than 5000 cells per microliter (µl) of blood but can be much higher. The presence of lymphocytosis in an person who is elderly should raise strong suspicion for CLL, and a confirmatory diagnostic test, in particular flow cytometry, should be performed unless clinically unnecessary."}, {"id": "wiki20220301en254_37373", "title": "Monoclonal B-cell lymphocytosis", "score": 0.00909090909090909, "content": "MLB falls into three phenotypes that are distinguished based on the cell surface marker proteins which they express viz., the CLL/SLL, atypical CLL/SLL, and non-CLL/SLL phenotypes. These markers are: CD5, CD19, CD20, CD23, and immunoglobulins (Ig) (either Ig light chains or complete Ig, i.e. light chains bound to Ig heavy chains. Distinguishing between these phenotypes is important because they progress to different lymphocyte malignancies. The following table gives the markers for the three MBL phenotypes with (+) indicating the expression (either dim, moderate, or bright depending or the intensity of their expression), (−) indicating the absence of expression, and na indicating not applicable as determined using fluorescent probes that bind the marker proteins. Detection of fluorescent probe binding by the cells requires the use of flow cytometry preferably employing 6 to 8 different fluorescent probes that bind to different markers on 5 million cells from the patient's blood. The"}, {"id": "pubmed23n0682_19246", "title": "White blood cell count at diagnosis and immunoglobulin variable region gene mutations are independent predictors of treatment-free survival in young patients with stage A chronic lymphocytic leukemia.", "score": 0.00909090909090909, "content": "A comprehensive panel of clinical-biological parameters was prospectively evaluated at presentation in 112 patients with chronic lymphocytic leukemia (<65 years), to predict the risk of progression in early stage disease. Eighty-one percent were in Binet stage A, 19% in stages B/C. Treatment-free survival was evaluated as the time from diagnosis to first treatment, death or last follow up. In univariate analysis, advanced stage, hemoglobin, platelets, white blood cell, leukemic lymphocyte count, raised beta 2-microglobulin and LDH, unmutated immunoglobulin variable region genes, CD38, del(17p), del(11q) and +12, were significantly associated with a short treatment-free survival; the T/leukemic lymphocyte ratio was associated with a better outcome. Multivariate analysis of treatment-free survival in stage A patients selected a high white blood cell count and unmutated immunoglobulin variable region genes as unfavorable prognostic factors and a high T/leukemic lymphocyte ratio as a favorable one. At diagnosis, these parameters independently predict the risk of progression in stage A chronic lymphocytic leukemia patients."}, {"id": "wiki20220301en179_39478", "title": "B-cell prolymphocytic leukemia", "score": 0.009009009009009009, "content": "Immunophenotype This technique is used to study proteins expressed in cells using immunologic markers. In B-PLL patients there is strong expression of surface immunoglobulin – a membrane-bound form of an antibody, b-lymphocyte surface antigens CD19, CD20, CD22, CD79a and FMC7, and weak expression of CD5 and CD23. Due to the similarities among lymphoproliferative disorders, it is often difficult to diagnose patients. Immunophenotyping helps distinguish B-PLL from similar diseases, one of its key identifiers is the absence in expression of the surface antigens CD10, CD11c, CD25, CD103 and cyclin D1 – an important regulator of cell-cycle progression. A case has been described as CD20+, CD22+, and CD5-. It can also be CD5+. Another case was described as CD45+, CD19+, CD20+, CD5+, HLA-DR+, CD10-, CD23+/-, CD38+ and FMC7-."}, {"id": "pubmed23n0673_21307", "title": "[Minimal residual disease monitoring by flow cytometry in children with acute lymphoblastic leukemia].", "score": 0.009009009009009009, "content": "The cells that have avoided the action of antitumor drugs may be retained after remission achievement during induction therapy and consolidation. A combination of these cells is given the name minimal residual disease (MRD). Multicolor flow cytometry has recently attracted considerable interest as the most promising method for measuring the content of residual tumor blasts. This technique is based on the detection of the so-called leukemia-associated immunophenotype (LAIP), i.e., a tumor-specific combination of the expression of membrane and cytoplasmic markers. Flow cytometry may be successfully used to monitor MRD in 90-95% cases of acute lymphoblastic leukemia (ALL) and in 80-85% of patients with acute myelocytic leukemia. The sensitivity of flow cytometry, which is real for routine flow techniques, is a possibility of identifying one cell among 10(4)-10(5) cells. Multicolor flow cytometry (that involves the simultaneous analysis of the expression of a few markers) is the most reasonable tool for MRD monitoring. The monoclonal antibody panels recommended by different groups of investigators for MRD monitoring in B-lineage ALL include antibodies to the pan-B-cell antigen CD19, markers of different stages of differentiation of B-lineage precursors of CD10, CD34, and CD20 and leukemia-associated markers different for each panel, such as CD22, CD38, CD58, CD45, TdT, CD13, CD33. The hyperexpression of CD10, CD34, CD19, TdT, the decreased expression of CD38, CD45, CD22, CD19, the simultaneous expression of markers of different stages of differentiation of B lymphocytes, such as CD10 and CD20, and the lymphoblast coexpression of myeloid markers of CD13, CD33, CDS15 are the most frequently described immunophenotype aberrations in B-lineage ALL. The selection of combinations of markers for MRD monitoring in children with T-ALL is based on the simultaneous expression of combinations of the antigens characteristic for early stages of differentiation of normal T lymphocytes, namely TdT and cytoplasmic CD3. Some authors consider the use of CD99 versus TdT to be most appropriate. There is recent evidence that MRD-positive patients have a higher cumulative risk for recurrences as compared with those without residual blasts. Moreover, the longer the tumor cells are retained during therapy, the worse the prognosis is. Thus, for choice of the adequate intensity of antitumor therapy, it is necessary to qualitatively and quantitatively assess MRD by multicolor flow cytometry at different stages of therapy."}, {"id": "pubmed23n0749_1875", "title": "A case of chronic lymphocytic leukemia with massive ascites.", "score": 0.008928571428571428, "content": "An 81-year old woman with a history of chronic lymphocytic leukemia (CLL) was admitted with night sweats and abdominal distension. A complete blood count showed hemoglobin 5 g/dL, white blood cell (WBC) count 28.5×10(9)/L and platelets 38.4×10(9)/L. Peripheral blood smear examination showed a large number of smudge cells and lymphocytosis composed of mature-looking lymphocytes with clumped nuclear chromatin. Computed tomography scan demonstrated enlarged cervical, axillary, paraaortic, retroperitoneal and mesenteric lymph nodes with concomitant omental thickening and ascites. Also, the liver and the spleen were enlarged in the presence of multiple ill-defined hypoechoic areas in the latter. Histopathological analysis of the cervical lymph node biopsy was consistent with CLL. Bone marrow examination showed diffuse infiltration of the marrow with small lymphocytes. Analysis of the ascitic fluid revealed an exudate with WBC 1220 cells/mL. Cytocentrifuge preparation of the ascitic fluid showed small mature lymphoid cells containing hyperchromatic nuclei with coarsely granular chromatin. On flow cytometric analysis of the ascitic fluid, expression of CD5, CD19, CD20, CD22, CD23, CD45 and HLA-DR was compatible with a diagnosis of CLL, in accordance with the results of the peripheral blood analysis. The patient was treated with chemotherapy consisting of cyclophosphamide, vincristine and prednisolone but died within one month after development of non-chylous ascites."}, {"id": "pubmed23n0694_17017", "title": "[The limited possibility of using a simplified approach to detect minimal residual disease by the flow cytometry technique in children with precursor B-lineage acute lymphoblastic leukemia].", "score": 0.008928571428571428, "content": "Minimal residue disease (MRD) is a state in which the tumor cells remain in the patient in the amounts unrecognizable with the standard cytological techniques. Flow cytometry is one of the basic methods for evaluation of MRD in precursor B-lineage acute lymphoblastic leukemia (PBLALL). The so-called simplified three-color analysis using the combination of CD19/CD10/CD34 antibodies has been proposed to detect MRD in the midcourse of induction therapy. Four-to-nine-color is presently used to identify MRD. One hundred and thirty-four bone marrow samples taken at different stages of therapy in 55 children with PBLALL were examined to estimate the possibility of using the flow cytometry technique using the 3-color simplified approach to determining MRD. The results of the simplified and standard approaches were compared in the samples stained with 6-8 monoclonal antibodies in the combinations that always included CD19, Cd10 and CD34. The comparison revealed that MRD had been incorrectly identified by the simplified method in 8.0, 17.6, and 75.8% of the patients on therapy days 15, 36, and 85, respectively. In addition, the content of residual tumor cells with respect to the threshold values more frequently proposed to stratify patients was found to be incorrectly calculated in some true positive samples. Thus, when the simplified approach was applied using the results of MRD detection to stratify the patients into risk groups, 16.0, 27.4, and 81.8% of the samples would yield incorrect information on therapy days 15, 36, and 85, respectively. Thus, the simplified approach to identifying MRD is most applicable on day 15 of therapy; however, there may be mistakes in this point of observation. This method used on day 36 more frequently yields incorrect results and is inapplicable on day 85."}]}}}} {"correct_option": 1, "explanations": {"1": {"exist": true, "char_ranges": [[0, 351]], "word_ranges": [[0, 50]], "text": "This is a clear clinical case of systemic sclerosis (Raynaud's, esophageal involvement, skin induration) presenting with renal crisis. Apart from anti-proteinase 3 antibodies, which are not related to scleroderma (incorrect answer 3), the most likely ones would be antiRNA polymerase III antibodies, which appear in younger patients with renal crises."}, "2": {"exist": false, "char_ranges": [], "word_ranges": [], "text": ""}, "3": {"exist": true, "char_ranges": [[146, 234]], "word_ranges": [[20, 32]], "text": "anti-proteinase 3 antibodies, which are not related to scleroderma (incorrect answer 3),"}, "4": {"exist": false, "char_ranges": [], "word_ranges": [], "text": ""}, "5": {"exist": false, "char_ranges": [], "word_ranges": [], "text": ""}}, "full_answer": "This is a clear clinical case of systemic sclerosis (Raynaud's, esophageal involvement, skin induration) presenting with renal crisis. Apart from anti-proteinase 3 antibodies, which are not related to scleroderma (incorrect answer 3), the most likely ones would be antiRNA polymerase III antibodies, which appear in younger patients with renal crises.", "full_answer_no_ref": "This is a clear clinical case of systemic sclerosis (Raynaud's, esophageal involvement, skin induration) presenting with renal crisis. Apart from anti-proteinase 3 antibodies, which are not related to scleroderma ([HIDDEN]), the most likely ones would be antiRNA polymerase III antibodies, which appear in younger patients with renal crises.", "full_question": "A 45-year-old female patient with a history of gastroesophageal reflux has been presenting for the past year with episodes of pallor in some fingers with exposure to cold. She had recently been prescribed prednisone at a dose of 20 mg/day for joint pain and skin induration in the hands and arms. For the last 48 hours, she presented with general malaise and intense headache, for which she went to the emergency department. Examination revealed only a rhythmic tachycardia at 100 bpm, with no neurological focality. Blood pressure was 200/110 mmHg. The blood test shows only a creatinine level of 2.5 mg/dL. Indicate which of the following autoantibodies is best related to the process described:", "id": 534, "lang": "en", "options": {"1": "Anti-RNA polymerase III antibodies.", "2": "Anti-centromere antibodies.", "3": "Anti-proteinase 3 antibodies.", "4": "Anti-PM-Scl antibodies.", "5": NaN}, "question_id_specific": 119, "type": "RHEUMATOLOGY", "year": 2021, "rag": {"clinical_case_options": {"MedCorp": {"RRF-2": [{"id": "pubmed23n1102_22675", "title": "Case 294: Catastrophic Antiphospholipid Syndrome.", "score": 0.017940466613032984, "content": "History A 50-year-old woman presented to the emergency department of our hospital with a 2-day history of lower limb pain associated with unusual asthenia and diffuse arthralgia over the past 3 weeks. She was a native of Guinea and had lived in France for most of her life, working as a personal care assistant. Her only medical history of note was an occurrence of fetal death at 12 weeks gestation when she was 35 years old. She had bilateral lower limb swelling, without changes in skin temperature or color. All proximal and distal arterial pulses were felt. General physical examination findings were otherwise unremarkable. Her laboratory tests showed a decreased hemoglobin concentration of 8.9 g/dL (normal range, 12-16 g/dL), a decreased platelet count of 45 × 10 12 years in girls). ■If the patient is of short stature (bone age < 12 years) with normal growth velocity, constitutional growth delay (the most common cause of 1° amenorrhea) is the probable cause. ■If bone age is > 12 years but there are no signs of puberty, obtain LH/ FSH and consider where the problem is on the HPA axis (see Figure 2.12-3). ■↓ GnRH, ↓ LH/FSH, ↓ estrogen/progesterone at prepuberty levels: Points to constitutional growth delay (puberty has not yet started). ■↓ GnRH, ↓ LH/FSH, ↓ estrogen/progesterone: Hypogonadotropic hypogonadism. Suggests a hypothalamic or pituitary problem."}, {"id": "pubmed23n0284_7534", "title": "Aromatase deficiency in male and female siblings caused by a novel mutation and the physiological role of estrogens.", "score": 0.009009009009009009, "content": "The aromatase enzyme complex catalyzes the conversion of androgens to estrogens in a wide variety of tissues, including the ovary, testis, placenta, brain, and adipose tissue. Only a single human gene encoding aromatase P450 (CYP19) has been isolated; tissue-specific regulation is controlled in part by alternative promoters in a tissue-specific manner. We report a novel mutation in the CYP19 gene in a sister and brother. The 28-yr-old XX proband, followed since infancy, exhibited the cardinal features of the aromatase deficiency syndrome as recently defined. She had nonadrenal female pseudohermaphrodism at birth and underwent repair of the external genitalia, including a clitorectomy. At the age of puberty, she developed progressive signs of virilization, pubertal failure with no signs of estrogen action, hypergonadotropic hypogonadism, polycystic ovaries on pelvic sonography, and tall stature. The basal concentrations of plasma testosterone, androstenedione, and 17-hydroxyprogesterone were elevated, whereas plasma estradiol was low. Cyst fluid from the polycystic ovaries had a strikingly abnormal ratio of androstenedione and testosterone to estradiol and estrone. Hormone replacement therapy led to breast development, menses, resolution of ovarian cysts, and suppression of the elevated FSH and LH values. Her adult height is 177.6 cm (+2.5 SD). Her only sibling, an XY male, was studied at 24 yr of age. During both pregnancies, the mother exhibited signs of progressive virilization that regressed postpartum. The height of the brother was 204 cm (+3.7 SD) with eunuchoid skeletal proportions, and the weight was 135.1 kg (+2.1 SD). He was sexually fully mature and had macroorchidism. The plasma concentrations of testosterone (2015 ng/dL), 5 alpha-dihydrotestosterone (125 ng/dL), and androstenedione (335 ng/dL) were elevated; estradiol and estrone levels were less than 7 pg/mL. Plasma FSH and LH concentrations were more than 3 times the mean value. Plasma PRL was low; serum insulin-like growth factor I and GH-binding protein were normal. The bone age was 14 yr at a chronological age of 24 3/12 yr. Striking osteopenia was noted at the wrist. Bone mineral densitometric indexes of the lumbar spine (cancellous bone) and distal radius (cortical bone) were consistent with osteoporosis; the distal radius was -4.7 SD below the mean value for age- and sex-matched normal men; indexes of bone turnover were increased. Hyperinsulinemia, increased serum total and low density lipoprotein cholesterol, and triglycerides and decreased high density lipoprotein cholesterol were detected.(ABSTRACT TRUNCATED AT 400 WORDS)"}, {"id": "pubmed23n0419_17533", "title": "Growth without growth hormone: growth pattern and final height of five patients with idiopathic combined pituitary hormone deficiency.", "score": 0.009009009009009009, "content": "Growth without GH has been reported in patients with organic combined pituitary hormone deficiency (CPHD) after resection of craniopharyngiomas and hypothalamic tumours or in septo-optic dysplasia. This study describes the growth pattern and final height of five children (four boys, one girl) with idiopathic CPHD (GH, TSH, ACTH, LH and FSH) who maintained normal growth despite persistent GH deficiency throughout the growth period. Presenting findings were borderline small penis in two children diagnosed at ages 3 and 9 years, and absence of pubertal signs in three adolescents diagnosed at age 12.8-13.7 years. The latter three patients also exhibited acromegaloid features. The height of all patients was within the 10-25th percentiles, and weight at the 25-50th percentiles. Although they were moderately overweight, accelerated weight gain was not observed. Prepubertal growth rate was 4-5 cm/year. The pubertal growth period, starting after initiation of sex hormone therapy (chronological age 15.9-16.3 years and bone age 12.5-14.5 years) continued for 4-5.5 years. Total pubertal growth was 6-11.7 cm with reduced growth spurt. Final height, which was attained at an advanced age (19-22 years), was 170-179 cm in the boys and 164 cm in the girl, equal to or exceeding the target height range. Repeated hormonal evaluations revealed undetectable GH and IGF-I levels, and no evidence of hyperprolactinaemia or hyperinsulinism. Final height attainment within or above target height range may occur in patients with idiopathic CPHD despite persistent GHD. As this was not mediated by GH, IGF-I, insulin or prolactin, some other growth factors probably played a growth-promoting role."}, {"id": "Gynecology_Novak_5294", "title": "Gynecology_Novak", "score": 0.008950617283950617, "content": "Evaluation of Women with Amenorrhea Associated with the Absence of Secondary Sexual Characteristics A careful history and physical examination are necessary to appropriately diagnose and treat primary amenorrhea associated with hypogonadism. The physical examination may be particularly helpful in patients with Turner syndrome. A history of short stature but consistent growth rate, a family history of delayed puberty, and normal physical findings (including assessment of smell, optic discs, and visual fields) may suggest physiologic delay. Headaches, visual disturbances, short stature, symptoms of diabetes insipidus, and weakness of one or more limbs suggest central nervous system lesions (38). Galactorrhea may be seen with prolactinomas, a condition more commonly associated with secondary amenorrhea in the presence of normal secondary sexual characteristics. The diagnostic workup is summarized as follows: 1."}, {"id": "pubmed23n1065_5338", "title": "A Case Report Emphasizing the Importance of Early Diagnosis and Management of Intracranial Germinoma.", "score": 0.008928571428571428, "content": "Intracranial germ cell tumors (GCTs) account for 3%-5% of all intracranial tumors. They commonly manifest during first two decades of life. We are reporting a case of a young female, who presented with progressive visual loss, polyuria and polydipsia, harboring an intracranial GCT. She presented initially to a neurosurgery clinic and then to an endocrine clinic, with a history of chronic worsening headache and recent onset visual blurring along with polyuria with polydipsia. On further inquiry, she was found to have primary amenorrhea, easy fatigability, and failure of development of secondary sexual characteristics. On examination the patient had bitemporal hemianopia with breast development at tanner stage II and pubic and axillary hair at tanner stage I. Her initial hormonal workup was suggestive of panhypopituitarism with diabetes insipidus. MRI pituitary showed a sellar mass with suprasellar extension, so an initial impression of a pituitary macroadenoma was made and the patient underwent trans-sphenoidal surgery. The histopathology was suggestive of lymphoid hyperplasia. Follow up MRI showed significant residual tumor and her vision and pituitary function did not recover. Neurosurgery was planned as second surgery, but we requested a second opinion of histopathology report and it was suggestive of a germinoma. She was then started on chemotherapy followed by radiotherapy, after which her tumor size reduced significantly, though she still required pituitary hormone replacement therapy.  Pituitary stalk lesions are rare and their diagnosis is challenging as different etiologies present clinically and radiologically in a similar manner with tissue diagnosis being the gold standard. Germinoma is a radiosensitive tumor. In our patient it took a long time to reach the correct diagnosis and late diagnosis resulted in permanent visual field defect and panhypopituitarism. This case report emphasizes that we should guide and educate our patients to seek medical advice early in the course of disease. We should also keep differential diagnosis in mind before referring the patient for surgery."}, {"id": "pubmed23n0113_6062", "title": "[Long-term fate of pituitary dwarfs treated with growth hormone].", "score": 0.008928571428571428, "content": "A questionnaire having been mailed to 50 hypopituitary patients aged 18 to 36 years (m 21.7 +/- 3.4) previously treated with human growth hormone for at least 3 years, 44 answers have been received. The final height is 2.1 +/- 0.9 standard deviations below the average. However 57% of the patients consider it is sufficient. The smallness is felt as a handicap by 20% only of these adults, though 88% had suffered for it during their adolescence. The treatment is retrospectively considered as useful and acceptable by 68%, heavy but useful by 25%, heavy and useless by 7%, without correlation with the results. Only 41% are satisfied with their school achievements. However, more than two thirds of patients had severe school difficulties at the time of onset of the treatment. Actually 75% of the patients are professionally qualified, among whom 36% have achieved high school, and most have an educational level similar to that of their parents or even higher. But 41% only have an employment, 27% are still students and 32% are unemployed. The way of life of the young hypopituitary adults is severely affected: 6 only are married or living with a mate, 11 only write they have occasional sexual experiences, 16 remain completely alone. In contrast, leisure activities are good in more than 90%. A score taking all these data into consideration to evaluate their way of life shows, among the 43 complete answers to the questionnaire, 16% with excellent results, 49% with a rather good social status and 35% with poor final result.(ABSTRACT TRUNCATED AT 250 WORDS)"}]}}}} {"correct_option": 1, "explanations": {"1": {"exist": true, "char_ranges": [[25, 189]], "word_ranges": [[5, 33]], "text": "The statement perfectly defines a tolerance phenomenon in which the patient has become \"used to\" the dose and needs more in order to have the same analgesic effect."}, "2": {"exist": false, "char_ranges": [], "word_ranges": [], "text": ""}, "3": {"exist": false, "char_ranges": [], "word_ranges": [], "text": ""}, "4": {"exist": false, "char_ranges": [], "word_ranges": [], "text": ""}, "5": {"exist": false, "char_ranges": [], "word_ranges": [], "text": ""}}, "full_answer": "The correct answer is 1. The statement perfectly defines a tolerance phenomenon in which the patient has become \"used to\" the dose and needs more in order to have the same analgesic effect.", "full_answer_no_ref": "The [HIDDEN] The statement perfectly defines a tolerance phenomenon in which the patient has become \"used to\" the dose and needs more in order to have the same analgesic effect.", "full_question": "The patient comes to the home of an oncology patient whose pain has not been well controlled lately. On physical examination there appears to be no evidence of tumor progression, and no previously known data of interest is revealed. In the anamnesis, the main caregiver states that the patient has pain 8 hours after receiving the prescribed basal dose of morphine every 12 hours. This clinical situation is referred to as:", "id": 409, "lang": "en", "options": {"1": "Tolerance", "2": "Hyperalgesia", "3": "Dependency", "4": "Ineffectiveness", "5": NaN}, "question_id_specific": 191, "type": "PRIMARY CARE", "year": 2018, "rag": {"clinical_case_options": {"MedCorp": {"RRF-2": [{"id": "wiki20220301en289_33584", "title": "Patient-controlled analgesia", "score": 0.011736001729516044, "content": "With a PCA the patient is protected from overdose by the caregiver programming the PCA to deliver a dose at set intervals. If the patient presses the button sooner than the prescribed intake pressing the button does not operate the PCA. (The PCA can be set to emit a beep telling the patient a dose was NOT delivered). Dosage is also controlled when the patient is too sedated to press the button; preventing the patient from receiving needless doses and keeping the patient safe from overdosing. Epidural Patient-controlled epidural analgesia (PCEA) is a related term describing the patient-controlled administration of analgesic medicine in the epidural space, by way of intermittent boluses or infusion pumps. This can be used by women in labour, terminally ill cancer patients or to manage post-operative pain."}, {"id": "wiki20220301en354_20189", "title": "Cancer pain", "score": 0.010107455524643304, "content": "Analgesics should not be taken \"on demand\" but \"by the clock\" (every 3–6 hours), with each dose delivered before the preceding dose has worn off, in doses sufficiently high to ensure continuous pain relief. People taking slow-release morphine should also be provided with immediate-release (\"rescue\") morphine to use as necessary, for pain spikes (breakthrough pain) that are not suppressed by the regular medication. Oral analgesia is the cheapest and simplest mode of delivery. Other delivery routes such as sublingual, topical, transdermal, parenteral, rectal or spinal should be considered if the need is urgent, or in case of vomiting, impaired swallow, obstruction of the gastrointestinal tract, poor absorption or coma. Current evidence for the effectiveness of fentanyl transdermal patches in controlling chronic cancer pain is weak but they may reduce complaints of constipation compared with oral morphine."}, {"id": "wiki20220301en131_179", "title": "Opioid-induced hyperalgesia", "score": 0.009900990099009901, "content": "activation (desensitization of antinociceptive mechanisms) and opioid receptor down-regulation (internalization of membrane receptors). In opioid-induced hyperalgesia, sensitization of pronociceptive mechanisms occurs, resulting in a decrease in the pain threshold, or allodynia. In addition, what appears to be opioid tolerance can be caused by opioid-induced hyperalgesia lowering the baseline pain level, thus masking the drug's analgesic effects. Identifying the development of hyperalgesia is of great clinical importance since patients receiving opioids to relieve pain may paradoxically experience more pain as a result of treatment. Whereas increasing the dose of opioid can be an effective way to overcome tolerance, doing so to compensate for opioid-induced hyperalgesia may worsen the patient's condition by increasing sensitivity to pain while escalating physical dependence."}, {"id": "pubmed23n0590_13933", "title": "[Outpatient opiate therapy in cancer patients during their last days of life].", "score": 0.009900990099009901, "content": "Pain is one of the symptoms that many tumor patients are especially afraid of in the final phase of their illness. Symptoms can change rapidly, making quick adaptation of the therapy necessary. This poses particular challenges to organizational structures in outpatient treatment if the patients' desire to spend their last days of life in their accustomed surroundings is to be realized. Pain intensity and the associated symptoms in a WHO step III opiate therapy during the last 3 days of life were investigated retrospectively among 601 tumor patients who had received medical care from Home Care Berlin. Differences in gender, age, living conditions/care situation and place of death were evaluated with due consideration for the different pain medications administered and for the different forms and routes of administration. More than 80% of the patients reported freedom from pain or only moderate pain during the opiate therapy. Care provided by the hospice-at-home medical service Home Care Berlin allowed excellent control of tumor patients' symptoms with only moderate side-effects in their final days of life. Among patients receiving opioids by the transdermal route there were significantly more frequent complaints of pain (p=0.004) and nausea (p=0.001). During the last days of life continuous subcutaneous infusions containing opiates facilitated good analgesia within an acceptable spectrum of side-effects. Most problems with controlling symptoms in outpatients were encountered in younger patients. Morphine emerged as the drug of first choice in this investigation, because it can be given parenterally and also because of its price. The use of subcutaneous and intravenous administration systems such as PCA pumps requires trained nursing services and regular house visits by physicians experienced in palliative medicine."}, {"id": "wiki20220301en356_22715", "title": "Racotumomab", "score": 0.00980392156862745, "content": "Treatment administration and side effects Racotumomab is administered in patients who have previously received the oncospecific treatment established in the oncological therapeutic standards (surgery, chemotherapy and radiation therapy). A Racotumomab-alum solution is administered by intradermal injection every 14 days for the first 2 months (5 doses), followed by monthly booster doses. Racotumomab is well tolerated by patients. The overall toxicity of the vaccine has been classified as grade 1 and 2, according to the NCI Common Toxicity Criteria (version 3.0). Treatment is mostly associated with mild to moderate injection-site reactions (local erythema, induration and pain), which disappear within 24–48 hours. Systemic reactions, such as flu-like symptoms and chills are less frequent, reversible, and self-limited. Approvals and indications"}, {"id": "pubmed23n0328_5262", "title": "[Terminal care of the cancer patient at home--patient and family spending the last moments together].", "score": 0.00980392156862745, "content": "Some 129 cancer patients were taken care of at home by the visiting nurse team of our hospital since 1994 to 1997; 116 patients have already died, 50 of them at home (group A), and 66 at the hospital (group B). We analyze the differences the between two groups in nutritional assistance, pain control and manpower in looking after a patient. Twice as many patients in group A took nutritional medication (drip infusion or intravenous hyperalimentation, for example) than group B. More patients in group A used oral and suppository morphine than group B. In group A 70% of patients (56% in group B) had more than two persons looking after at home. The important factors for a good recuperation at home are: 1. Available nutritional assist available. 2. Good control. 3. Less burden and anxiety of the person looked after. To spend pain desirable last moments together, it is important for the visiting nurse team to take proper care and to confirm the wishes of the patient and family, considering the rapidly changing conditions of the individual patient."}, {"id": "wiki20220301en556_14481", "title": "Ubrogepant", "score": 0.009708737864077669, "content": "The effectiveness of ubrogepant for the acute treatment of migraine was demonstrated in two randomized, double-blind, placebo-controlled trials. In these studies, 1,439 adult patients with a history of migraine, with and without aura, received the approved doses of ubrogepant to treat an ongoing migraine. In both studies, the percentages of patients achieving pain freedom two hours after treatment (defined as a reduction in headache severity from moderate or severe pain to no pain) and whose most bothersome migraine symptom (nausea, light sensitivity or sound sensitivity) stopped two hours after treatment were significantly greater among patients receiving ubrogepant (19–21% depending on the dose) compared to those receiving placebo (12%). Patients were allowed to take their usual acute treatment of migraine at least two hours after taking ubrogepant. 23% of patients were taking a preventive medication for migraine."}, {"id": "pubmed23n0502_2110", "title": "Patterns of high-dose morphine use in a home-care hospice service: should we be afraid of it?", "score": 0.009708737864077669, "content": "Management of cancer pain is one of the most important goals of palliative care. Relieving pain is often problematic. High doses of morphine at home may be required to relieve patients' pain, and is therefore feared. The goals of the current study were to assess the feasibility of high-dose morphine use at home, to characterize the patients, and to examine whether the use of high-dose morphine might affect their survival. The authors retrospectively studied the medical charts of 661 outpatients, which were completed by a home-care hospice team. The authors collected data regarding demographic parameters, medical diagnosis, pain type, morphine dosage, use of rescue doses in addition to regular doses, use of coanalgesics and adjuvant treatments, and survival time as associated with morphine dosage. The authors also compared the data of patients receiving high-dose morphine with those of a group of patients receiving regular doses. The authors identified 435 patients (65.8%) who received morphine for pain relief. Of these, 396 patients (91%) received a dose of 5-299 mg of morphine per day), 32 patients (7.4%) received 300-599 mg of morphine per day), and 7 patients (1.6%) received very high doses (> or = 600 mg of morphine per day). Overall, 39 patients (9%) received > 299 mg per day. Morphine dosage was found to be inversely correlated (r) with age (r = -0.254; P < 0.001). Male patients required slightly higher dosages than female patients (62.5% of high-dose and 71% of very high-morphine groups, respectively). Primary gastrointestinal (P = 0.015) and lung (P = 0.027) carcinomas, as well as metastatic bone disease (P = 0.001), ovarian carcinoma (P = 0.037), and brain tumors (P = 0.0053) were associated with higher and very higher morphine dosages. Adverse effects were similar in the groups receiving regular, high, and very high doses of morphine. The median survival of patients treated with high doses of morphine was 27 days and was 37 days for those treated with very high doses. Patients treated with low doses of morphine survived for 18 days. Patients not treated with morphine survived for 22 days (P = 0.001 by Mantel-Cox analysis; P = 0.029 by Breslow analysis). The use of high and very high morphine doses at home proved safe and did not appear to affect the patients' life expectancy adversely. The use of high or very high-dose morphine should not be a barrier to providing palliative terminal care for home-care hospice patients."}, {"id": "wiki20220301en131_194", "title": "Opioid-induced hyperalgesia", "score": 0.009615384615384616, "content": "The use of an NSAID, especially some COX-2 inhibitors, or acetaminophen either as monotherapy or combination therapy is also suggested as a possible treatment option. Research needs It can be difficult to apply research into OIH to average patients, because some research focused on people taking very high doses or in methadone rehabilitation programs. Opioid-induced hyperalgesia has also been criticized as overdiagnosed among chronic pain patients, due to poor differential practice in distinguishing it from the much more common phenomenon of opioid tolerance. The misdiagnosis of common opioid tolerance (OT) as opioid-induced hyperalgesia (OIH) can be problematic as the clinical actions suggested by each condition can be contrary to each other. Patients misdiagnosed with OIH may have their opioid dose mistakenly decreased (in the attempt to counter OIH) at times when it is actually appropriate for their dose to be increased or rotated (as a counter to opioid tolerance)."}, {"id": "pubmed23n0749_24517", "title": "Opioid use and effectiveness of its prescription at discharge in an acute pain relief and palliative care unit.", "score": 0.009615384615384616, "content": "The aim of this study was to present how opioids are used in an acute pain relief and palliative care unit (APRPCU), where many patients with difficult pain conditions are admitted from GPs, home palliative care programs, oncology departments, other hospitals or emergency units, and other regional places. From a consecutive sample of cancer patients admitted to an APRPCU for a period of 6 months, patients who had been administered opioids were included in this survey. Basic information was collected as well as opioid therapy prescribed at admission and, subsequently, during admission and at time of discharge. Patients were discharged once stabilization of pain and symptoms were obtained and the treatment was considered to be optimized. One week after being discharged, patients or relatives were contacted by phone to gather information about the availability of opioids at dosages prescribed at time of discharge. One hundred eighty six of 231 patients were specifically admitted for uncontrolled pain, with a mean pain intensity of 6.8 (SD 2.5). The mean dose of oral morphine equivalents in patients receiving opioids before admission was 45 mg/day (range 10-500 mg). One hundred seventy five patients (75.7 %) were prescribed around the clock opioids at admission. About one third of patients changed treatment (opioid or route). Forty two of 175 (24 %), 27/58 (46.5 %), 10/22 (45.4 %), and 2/4 (50 %) patients were receiving more than 200 mg of oral morphine equivalents, as maximum dose of the first, second, third, and fourth opioid prescriptions, respectively. The pattern of opioids changed, with the highest doses administered with subsequent line options. The mean final dose of opioids, expressed as oral morphine equivalents, for all patients was 318 mg/day (SD 798), that is more than six times the doses of pre-admission opioid doses. One hundred eighty six patients (80.5 %) were prescribed a breakthrough cancer pain (BTcP) medication at admission. Sixty five patients changed their BTcP prescription, and further 27 patients changed again. Finally, eight patients were prescribed a fourth BTcP medication. Of 46 patients available for interview, the majority of them (n = 39, 84 %) did not have problems with their GPs, who facilitated prescription and availability of opioids at the dosages prescribed at discharge. For patients with severe distress, APRPCUs may guarantee a high-level support to optimize pain and symptom intensities providing intensive approach and resolving highly distressing situations in a short time by optimizing the use of opioids."}, {"id": "wiki20220301en422_29695", "title": "Mistresses (American season 1)", "score": 0.009547899006471607, "content": "Karen After her patient, Thomas Grey, reveals that he is dying of lung cancer and that he has been in love with her for a year, Karen begins having an affair with him. To help him end his life when the pain becomes too much, Karen prescribes Thomas fatal doses of morphine, which his wife, Elizabeth, helps to administrate. The insurance company begins to investigate Thomas' death, going so far as to approach Karen for the notes from her sessions with Thomas. However, Karen had destroyed anything relating to Thomas being her patient. While grieving his father's death, Thomas' son Sam turns to Karen for advice as he has discovered that his father was having an affair. Even though Savi advises Karen to cut all ties with the Grey Family, Karen begins helping Sam. Things take a dangerous turn after Sam falls in love with Karen and begins stalking her."}, {"id": "wiki20220301en175_2628", "title": "Agkistrodon contortrix phaeogaster", "score": 0.009523809523809525, "content": "CroFab antivenin has been used successfully to treat Osage copperhead bites, although a lack of complete cross-tolerance requires careful administration and close supervision during the full course of treatment to ensure that the lowest effective dose is administered (a lower dose would not fully treat the envenomation, and a higher dose may be particularly dangerous to children, the elderly, and infirm adults). Not uncommonly, opiate/opioid narcotic analgesics (ex. morphine, fentanyl), muscle relaxerss (ex. diazepam, tizanidine, orphenadrine), and broad-spectrum antibiotics are administered. A few days' supply of weaker analgesics and muscle relaxers may be prescribed for the patient to control pain after he or she returns home as the pain resolves completely within one to three days. Patients also receive a prescription for an intensive antibiotic therapy, which much be taken until the supplies are depleted, giving the drug enough time to fully treat any opportunistic infections"}, {"id": "pubmed23n0659_24409", "title": "[Opioid rotation in the home medical care service].", "score": 0.009523809523809525, "content": "Pain relief is a quite important subject for maintaining the home medical care of patients with terminal cancer. Therefore, the opioid rotation should be made in conjunction with an individual medical condition, which is of growing importance in a proper pain management. We considered what opioid rotation is desirable in the home medical care service by analyzing the cases at our clinic. The most important thing in the opioid rotation at home is to perform a rotation before exacerbation of pain becomes apparent. For this purpose, morphine hydrochloride injection is thought to be the best dosage form because it has advantages of: (1) quickness in varying the amount, (2) immediate rescue efficacy, and (3) usefulness in case of ingestion."}, {"id": "wiki20220301en001_39074", "title": "Oxycodone", "score": 0.009433962264150943, "content": "Oxycodone has a half-life of 4.5 hours. It is available as a generic medication. The manufacturer of OxyContin, a controlled-release preparation of oxycodone, Purdue Pharma, claimed in their 1992 patent application that the duration of action of OxyContin is 12 hours in \"90% of patients.\" It has never performed any clinical studies in which OxyContin was given at more frequent intervals. In a separate filing, Purdue claims that controlled-release oxycodone \"provides pain relief in said patient for at least 12 hours after administration.\" However, in 2016 an investigation by the Los Angeles Times found that \"the drug weans off hours early in many people,\" inducing symptoms of opiate withdrawal and intense cravings for OxyContin. One doctor, Lawrence Robbins, told journalists that over 70% of his patients would report that OxyContin would only provide 4–7 hours of relief. Doctors in the 1990s often would switch their patients to a dosing schedule of once every eight hours when they"}, {"id": "pubmed23n0597_2723", "title": "[Patient-controlled analgesia in outpatients with severe cancer pain.].", "score": 0.009433962264150943, "content": "In this case report, we describe continuous subcutaneous infusion of opiates as PCAO (patient controlled analgesia in outpatients) in one patient with metastatic carcinoma of the rectum (liver and bone metastases, partial bowel obstruction) with severe cancer pain and vomiting in the terminal phase. The parenteral administration of opioids extended over 58 days. The infusion was powered by an external portable clockwork-driven syringe pump (Perfusor M, Braun Medical/Germany). The open-accessible pump has a syringe volume of 10 ml, and its maximal infusion time is 24 h. The 27-G infusion needle (Sub-Q-Set, Baxter/USA) was inserted in the side of the abdomen and was left in the same position for 10 to 20 days. It took the patient and his family only 1.5 h to familiarize themselves with the use of the pump. They were trained in its use in our outpatient pain department. For pain control both the variable continuous infusion and the extra injection doses could be administered by the way of the syringe driver. The patient was given a stock of 120 ampoules of morphine for further treatment at home. For optimal pain control he decided to raise the daily dose of opioid infusion from the initial 60 mg to 240 mg morphine within 48 h. In this way, PCAO-besides rapid titration of the opioid dose to achieve analgesia-allows the use of opioids controlled by the patient himself. In the present case this procedure was also important when an outpatient radiation therapy became urgently necessary to prevent a fracture of the spine because of metastasis. The pain control by the patient himself was the main factor to get free of pain during the transport to the hospital. Even positioning for radiation was possible without pain. When he received outpatient radiation therapy the patient needed extra injection doses of up to 360 mg morphine a day. The PCAO procedure by continuous subcutaneous infusion with opiates is a safe and efficient method of pain management for outpatient patients suffering from severe cancer pain and intractable nausea in the terminal phase. Its validity has also been proven especially for radiation treatment of bone metastases."}, {"id": "wiki20220301en352_34206", "title": "Neutron capture therapy of cancer", "score": 0.009345794392523364, "content": "was well tolerated by the 30 patients who were enrolled in this study. All were treated with 2 fields, and the average whole brain dose was 3.2–6.1 Gy (weighted), and the minimum dose to the tumor ranged from 15.4 to 54.3 Gy (w). There has been some disagreement among the Swedish investigators regarding the evaluation of the results. Based on incomplete survival data, the MeST was reported as 14.2 months and the time to tumor progression was 5.8 months. However, more careful examination of the complete survival data revealed that the MeST was 17.7 months compared to 15.5 months that has been reported for patients who received standard therapy of surgery, followed by radiotherapy (RT) and the drug temozolomide (TMZ). Furthermore, the frequency of adverse events was lower after BNCT (14%) than after radiation therapy (RT) alone (21%) and both of these were lower than those seen following RT in combination with TMZ. If this improved survival data, obtained using the higher dose of BPA"}, {"id": "pubmed23n0349_8700", "title": "Intravenous titration with morphine for severe cancer pain: report of 28 cases.", "score": 0.009345794392523364, "content": "In a multicenter study, 28 patients with cancer pain and insufficient pain relief with analgesic treatment according to step II of the guidelines of the World Health Organization (WHO) were switched to oral slow-release morphine. Patients received intravenous morphine through a patient-controlled pump (PCA) for the first 24 hours (bolus = 1 mg, lockout interval = 5 minutes, maximum dose = 12 mg/hour). From day 2 patients were treated with oral slow-release morphine. Daily doses were calculated from the requirements of the day before. Breakthrough pain was treated with PCA until stable doses were reached (<2 boluses/day) and then with oral immediate-release morphine solution. Pain intensity was reported in a diary four times a day, in addition to mood, activity, and quality of sleep once daily. Mean duration until adequate pain relief reported (<30 on a 101-step numerical scale; NRS) was 5 hours (range = 80-620 minutes). Mean pain intensity was reduced from 67 NRS to 22 NRS. Mean doses of oral morphine were 133 mg/day initially and then 154 mg/day on day 14. Serious adverse events such as respiratory depression were not observed. Two patients terminated the study due to progressive symptoms of gastrointestinal obstruction. Seventy-five percent of the patients evaluated the effectiveness of the analgesic regime as good. Dose finding with intravenous PCA may be appropriate for a small minority of patients with severe pain. Higher treatment costs and the risk of complications are drawbacks of this method compared with conventional oral titration."}, {"id": "wiki20220301en544_13987", "title": "Bland embolization", "score": 0.009259259259259259, "content": "After embolization, patients are observed in the post-anesthesia care unit for several hours. Patients are discharged from the hospital when taking adequate nutrition by mouth, when pain is adequately controlled with oral narcotics and when the temperature is lower than 38.5 for 24 hours. Post-procedure evaluation Follow-up triple phase CT is performed 2 to 4 weeks after treatment is complete and reviewed for any evidence of persistent untreated disease. If there is no evidence of enhancement of the treated tumor, these patients are monitored with triple phase CT every 3 months for the first year and every 6 months thereafter. When there is evidence of untreated disease, recurrent disease, or new disease elsewhere within the liver the patient is scheduled for additional embolization. References Hepatology Diseases of liver"}, {"id": "pubmed23n0045_19047", "title": "Analgesic use in home hospice cancer patients.", "score": 0.009259259259259259, "content": "Pain control in hospice patients in the home may be compromised by concerns about overuse of analgesics, particularly narcotics. A retrospective chart audit of analgesic type and amount was performed on the medical records of 100 cancer patients receiving hospice care in the home. Different types and amounts of analgesics were converted to a common standard, an oral morphine equivalent (OME) relative to 1 mg of oral morphine sulfate. Descriptive statistics were used to characterize patient analgesic use during the entire course of hospice care and the last 5 days of life. Associations between analgesic use and select patient characteristics (age, sex, cancer site, metastases, and pain intensity at admission) were explored. Ninety-one percent of the sample had used analgesics at some time during hospice care. The proportion of patients using analgesics increased as death approached. The mean and median daily analgesic use over the entire period were 114 and 82 OMEs and during the last 5 days 140 and 84 OMEs, respectively. The range of mean daily analgesic use was between 10 and 735 OMEs. Individual variability in analgesic use was demonstrated. Not all patients required analgesics, and among those who did there was remarkable variation in the amount used. Large and even enormous doses of analgesics may sometimes be required to control cancer pain."}, {"id": "pubmed23n0800_14851", "title": "Breathlessness with pulmonary metastases: a multimodal approach.", "score": 0.009174311926605505, "content": "Case Study  Sarah is a 58-year-old breast cancer survivor, social worker, and health-care administrator at a long-term care facility. She lives with her husband and enjoys gardening and reading. She has two grown children and three grandchildren who live approximately 180 miles away. SECOND CANCER DIAGNOSIS  One morning while showering, Sarah detected a painless quarter-sized lump on her inner thigh. While she thought it was unusual, she felt it would probably go away. One month later, she felt the lump again; she thought that it had grown, so she scheduled a visit with her primary care physician. A CT scan revealed a 6.2-cm soft-tissue mass in the left groin. She was referred to an oncologic surgeon and underwent an excision of the groin mass. Pathology revealed a grade 3 malignant melanoma. She was later tested and found to have BRAF-negative status. Following her recovery from surgery, Sarah was further evaluated with an MRI scan of the brain, which was negative, and a PET scan, which revealed two nodules in the left lung. As Sarah had attended a cancer support group during her breast cancer treatment in the past, she decided to go back to the group when she learned of her melanoma diagnosis. While the treatment options for her lung lesions included interleukin-2, ipilimumab (Yervoy), temozolomide, dacarbazine, a clinical trial, or radiosurgery, Sarah's oncologist felt that ipilimumab or radiosurgery would be the best course of action. She shared with her support group that she was ambivalent about this decision, as she had experienced profound fatigue and nausea with chemotherapy during her past treatment for breast cancer. She eventually opted to undergo stereotactic radiosurgery. DISEASE RECURRENCE  After the radiosurgery, Sarah was followed every 2 months. She complained of shortness of breath about 2 weeks prior to each follow-up visit. Each time her chest x-ray was normal, and she eventually believed that her breathlessness was anxiety-related. Unfortunately, Sarah's 1-year follow-up exam revealed a 2 cm × 3 cm mass in her left lung, for which she had a surgical wedge resection. Her complaints of shortness of breath increased following the surgery and occurred most often with anxiety, heat, and gardening activities, especially when she needed to bend over. Sarah also complained of a burning \"pins and needles\" sensation at the surgical chest wall site that was bothersome and would wake her up at night. Sarah met with the nurse practitioner in the symptom management clinic to discuss her concerns. Upon physical examination, observable signs of breathlessness were lacking, and oxygen saturation remained stable at 94%, but Sarah rated her breathlessness as 7 on the 0 to 10 Borg scale. The nurse practitioner prescribed duloxetine to help manage the surgical site neuropathic pain and to assist with anxiety, which in turn could possibly improve Sarah's breathlessness. Several nonpharmacologic modalities for breathlessness were also recommended: using a fan directed toward her face, working in the garden in the early morning when the weather is cooler, gardening in containers that are at eye level to avoid the need to bend down, and performing relaxation exercises with pursed lip breathing to relieve anxiety-provoked breathlessness. One month later, Sarah reported relief of her anxiety; she stated that the fan directed toward her face helped most when she started to feel \"air hungry.\" She rated her breathlessness at 4/10 on the Borg scale. SECOND RECURRENCE: MULTIPLE PULMONARY NODULES  Sarah's chest x-rays remained clear for 6 months, but she developed a chronic cough shortly before the 9-month exam. An x-ray revealed several bilateral lung lesions and growth in the area of the previously resected lung nodule. Systemic therapy was recommended, and she underwent two cycles of ipilimumab. Sarah's cough and breathlessness worsened, she developed colitis, and she decided to stop therapy after the third cycle. In addition, her coughing spells triggered bronchospasms that resulted in severe anxiety, panic attacks, and air hunger. She rated her breathlessness at 10/10 on the Borg scale during these episodes. She found communication difficult due to the cough and began to isolate herself. She continued to attend the support group weekly but had difficulty participating in conversation due to her cough. Sarah was seen in the symptom management clinic every 2 weeks or more often as needed. No acute distress was present at the beginning of each visit, but when Sarah began to talk about her symptoms and fear of dying, her shortness of breath and anxiety increased. The symptom management nurse practitioner treated the suspected underlying cause of the breathlessness and prescribed oral lorazepam (0.5 to 1 mg every 6 hours) for anxiety and codeine cough syrup for the cough. Opioids were initiated for chest wall pain and to control the breathlessness. Controlled-release oxycodone was started at 10 mg every 12 hours with a breakthrough pain (BTP) dose of 5 mg every 2 hours as needed for breathlessness or pain. Sarah noted improvement in her symptoms and reported a Borg scale rating of 5/10. Oxygen therapy was attempted, but subjective improvement in Sarah's breathlessness was lacking. END OF LIFE  Sarah's disease progressed to the liver, and she began experiencing more notable signs of breathlessness: nasal flaring, tachycardia, and restlessness. Opioid doses were titrated over the course of 3 months to oxycodone (40 mg every 12 hours) with a BTP dose of 10 to 15 mg every 2 hours as needed, but her breathlessness caused significant distress, which she rated 8/10. The oxycodone was rotated to IV morphine continuous infusion with patient-controlled analgesia (PCA) that was delivered through her implantable port. This combination allowed Sarah to depress the PCA as needed and achieve immediate control of her dyspneic episodes. Oral lorazepam was also continued as needed. Sarah's daughter moved home to take care of her mother, and hospice became involved for end-of-life care. As Sarah became less responsive, nurses maintained doses of morphine for control of pain and breathlessness and used a respiratory distress observation scale to assess for breathlessness since Sarah could no longer self-report. A bolus PCA dose of morphine was administered by Sarah's daughter if her mother appeared to be in distress. Sarah died peacefully in her home without signs of distress. "}, {"id": "pubmed23n0363_6533", "title": "[Emergency pain treatment--ambulatory intravenous morphine titration in a patient with cancer pain].", "score": 0.009174311926605505, "content": "Even when the guidelines for cancer pain management are followed, acute severe pain requiring immediate treatment will occur in some patients. Titration with intravenous morphine may provide fast and efficient pain relief and give an indication of the amount of opioid necessary for continuous treatment. In cooperation with a general practitioner we performed an intravenous morphine titration in a patient with severe cancer pain at home. Adequate analgesia was reached with 20 mg intravenous morphine. Blood pressure, cardiac frequency and oxygen saturation did not change. No side effects were reported during the titration, and the previous regimen with tramadol 150 mg per day was switched to slow release morphine 300 mg per day. This medication was prescribed by the general practitioner and provided good pain relief until the patient died two weeks later. We conclude that intravenous morphine titration may be performed even in cancer patients at home, adequate monitoring, however, should be available."}, {"id": "wiki20220301en020_61862", "title": "Opioid", "score": 0.00909090909090909, "content": "The initial 24 hours after opioid administration appear to be the most critical with regard to life-threatening OIRD, but may be preventable with a more cautious approach to opioid use. Patients with cardiac, respiratory disease and/or obstructive sleep apnoea are at increased risk for OIRD. Increased pain sensitivity Opioid-induced hyperalgesia – where individuals using opioids to relieve pain paradoxically experience more pain as a result of that medication – has been observed in some people. This phenomenon, although uncommon, is seen in some people receiving palliative care, most often when dose is increased rapidly. If encountered, rotation between several different opioid pain medications may decrease the development of increased pain. Opioid induced hyperalgesia more commonly occurs with chronic use or brief high doses but some research suggests that it may also occur with very low doses."}, {"id": "pubmed23n0582_16576", "title": "Intravenous morphine consumption in outpatients with cancer during their last week of life--an analysis based on patient-controlled analgesia data.", "score": 0.00909090909090909, "content": "Studies on opioid use in terminally ill cancer patients have shown a prefinal dose increase in the majority of patients. Mostly oral opioids were used. Due to the pharmacokinetic properties of opioids, it is rather difficult to get a reliable estimate of the true opioid need from those results. Retrospectively, we analyzed opioid use during the last week of life of 30 consecutive outpatients with cancer on intravenous (i.v.) morphine patient-controlled analgesia (PCA). A dose increase (decrease) was defined as an increase (decrease) of the patient's individual daily dose by at least 30% with respect to their prior daily dose. We also analyzed circadian variations in morphine use. Thirty patients fulfilled the primary study inclusion criteria. Fulfilling the exclusion criteria, seven patients had to be excluded from analysis (n = 3, on PCA for less than 7 days; n = 4, PCA was finished before death). Twenty-three patients with a total of 161 treatment days were analyzed. The patients' median age was 57 years (range, 4 to 72). The median duration of intravenous morphine PCA was 19 days (range, 8 to 58). The median daily intravenous morphine dose during the last week of life was 96 to 115 mg, without significant change over time/from day to day (Friedman test). On 144/161 days (89.2%), morphine dose remained stable. On 9 treatment days (5.6%), the dose increased, and on 8 days (5.0%), it decreased. In three patients, only dose increases, and in four patients, only dose decreases were observed. In four patients, both dose increases and decreases were observed. Twelve patients showed no change in daily morphine dose. Opioid use lacked a diurnal pattern. During their end-of-life phase, cancer patients on i.v. morphine PCA showed a stable daily opioid need."}, {"id": "wiki20220301en027_47835", "title": "Buprenorphine", "score": 0.009009009009009009, "content": "Both buprenorphine and methadone are medications used for detoxification and opioid replacement therapy, and appear to have similar effectiveness based on limited data. Both are safe for pregnant women with opioid use disorder, although preliminary evidence suggests that methadone is more likely to cause neonatal abstinence syndrome. In the US and European Union, only designated clinics can prescribe methadone for opioid use disorder, requiring patients to travel to the clinic daily. If patients are drug free for a period they may be permitted to receive \"take home doses,\" reducing their visits to as lilttle as once a week. Alternatively, up to a month's supply of buprenorphine has been able to be prescribed by clinicians in the US or Europe who have completed a basic training (8–24 hours in the US and received a waiver/licence allowing prescription of the medicine. In France, buprenorphine prescription for opioid use disorder has been permitted without any special training or"}, {"id": "pubmed23n0299_3086", "title": "[How to palliate the symptoms of terminally ill patients at home].", "score": 0.009009009009009009, "content": "We reported the kind of symptoms and how they could be palliated in terminally ill patients at home based on our experience of about 9 years. Cancer pain, which was the most frequent symptom, appeared in 67 among 126 patients receiving home care, and it could be effectively controlled with morphine; no patient returned to the hospital because of aggravation of pain. Very few patients stayed in the hospital and never returned home due to uncontrollable pain. Home parenteral infusion was done for 63 patients who were unable to eat or drink because of peritonitis carcinomatosa or cancer cachexia. High fever in the tumor mass was controlled by glucocorticoid hormone, and ascites was drained continuously when the patients suffered from abdominal distension. From analysis of the cases in which home care was interrupted or those in which patients were unable to transfer to home care, symptoms that were difficult to palliate at home were nausea caused by bowel obstruction, acute symptoms (bleeding, disturbance of consciousness, and so on), and dyspnea. But if the patients and family are eager for home care and an adequate medical support system is in place, home care may be possible despite these symptoms."}, {"id": "wiki20220301en300_9122", "title": "Oxytrex", "score": 0.008928571428571428, "content": "A phase III clinical trial has shown that the combination of ULD (ultra-low dose) naltrexone with oxycodone has been nearly as effective in providing pain relief while causing less physical dependence than oxycodone alone. The trial has been criticized for its confounding and limiting factors which include the huge dropout rate (54%), lack of demographic stratification of pain intensity amongst the study arms, and that patients received varying amounts of oxycodone which would influence their overall dependence. The lack of an on-board break-through-pain agent also hurts their external validity and likely contributed to their high dropout rate. References Webster et al., \"Oxytrex Minimizes Physical Dependence While Providing Effective Analgesia: A Randomized Controlled Trial in Low Back Pain\", The Journal of Pain, Vol 7, No 12, 2006:937-946 Expert Opin. Investig. Drugs (2007) 16(8):1277-1283 Mu-opioid receptor agonists Semisynthetic opioids Combination drugs Experimental drugs"}, {"id": "pubmed23n0252_10171", "title": "[Cancer-pain management in home care].", "score": 0.008928571428571428, "content": "Patient was 58 year old female with severe uterus cancer pain who refused to take the oral morphine in hospital because doctor should be hesitated to discharged patient whose pain is particularly difficult to manage in home. A few days later, she returned back home with her family irritably and recovered from hallucination by morphine-intake in home. The oral morphine is the prepared route of analgesic administration in home care. When patient can not take medications orally, continuous intravenous infusion provides the most consistent level of analgesia. In future, transdermal route offers a practical alternative in the hospice and home."}, {"id": "pubmed23n0090_18219", "title": "Pharmacokinetics and clinical efficacy of oral morphine solution and controlled-release morphine tablets in cancer patients.", "score": 0.008849557522123894, "content": "Twenty-three adult patients with chronic pain due to cancer completed a double-blind, randomized, two-phase crossover trial comparing plasma morphine concentrations and analgesic efficacy of oral morphine sulfate solution (MSS) and controlled-release morphine sulfate tablets (MS Contin [MSC], Purdue Frederick, Inc., Toronto, Ontario, Canada). MS Contin was given every 12 hours to all patients except those whose daily morphine dose could not be equally divided into two 12-hour doses with the tablet strengths available. MSS was given every 4 hours. Patients received both of the test drugs for at least 5 days, and, on the final day of each phase, peripheral venous blood samples for morphine analysis were obtained. Eighteen patients received MSC every 12 hours, and five received it every 8 hours. The same total daily morphine dose was given in both phases. In the 18 patients who received MSC every 12 hours, the daily morphine dose was 183.9 +/- 140.0 mg (mean +/- SD). In this group, the mean area under the curve (AUC) with MSC was 443.6 +/- 348.4 ng/ml/hour, compared with 406.8 +/- 259.7 ng/ml/hour for MSS (P greater than 0.20). Mean maximum morphine concentrations (Cmax) for MSC and MSS were 67.9 +/- 42.1 and 58.8 +/- 30.3 ng/ml, respectively (P greater than 0.05). Mean minimum morphine concentrations (Cmin) were 17.0 +/- 17.7 and 18.3 +/- 15.0, respectively (P greater than 0.30). There was a significant difference (P less than 0.001) between the two drugs in time required to reach maximum morphine concentration (Tmax). Mean Tmax after MSC occurred at 3.6 +/- 2.3 hours. After MSS, it occurred at 1.3 +/- 0.4 hours. In the five patients who received MSC every 8 hours, the findings paralleled those in the principal group, with no significant differences between MSC and MSS in Cmax or Cmin and a highly significant difference between the two in Tmax. However, in this small group of patients, the AUC with MSC was significantly (P = 0.04) greater than that with MSS. All patients had very good pain control throughout the study and both formulations were well tolerated. There were no significant differences between MSC and MSS in pain scores or side effects. Under the conditions of this study there was no clinically significant difference in bioavailability between MSC and oral MSS. When given on a 12-hourly basis in individually titrated doses, the MSC provided therapeutic plasma morphine concentrations throughout the dosing interval."}, {"id": "pubmed23n0299_3083", "title": "[What can we do for cancer patients with pain to go home].", "score": 0.008849557522123894, "content": "When patients suffering from cancer pain wish to stay at home, the most important condition is to relieve the cancer pain. But it is not enough; decreasing the fear of pain is another important problem. From these viewpoints, we have tried to assist these patients to go home. In this paper we report two cases of 61-year-old and 57-year-old women with severe leg pain caused by pelvic recurrence of cancer of the uterus and rectum. At first, we needed high doses of morphine by intravenous administration all day long. To control their mental condition, we talked personally with them to understand their distress and asked them to put in writing anything about their anxiety in a special notebook. After that, we advised them on how to use anodyne, how to deal with side effects of the drugs and how to use various social services in their home. Then, after their cancer pain was controlled by radiotherapy and appropriate anodyne, they overcame their anxiety and at last they could go home."}, {"id": "pubmed23n0478_8599", "title": "Patient-reported utilization patterns of fentanyl transdermal system and oxycodone hydrochloride controlled-release among patients with chronic nonmalignant pain.", "score": 0.008771929824561403, "content": "Although use of long-acting opioid analgesics has increased for chronic nonmalignant pain management, little is known about patient-reported utilization patterns. To assess patient-reported utilization patterns of fentanyl transdermal system and oxycodone hydrochloride (HCl) controlled-release among patients with chronic nonmalignant pain and to compare these patterns to standard dose administration guidelines recommended in the manufacturers. prescribing information (PI). Cross-sectional, observational, multicenter study of English-speaking patients who were seeking chronic nonmalignant pain management from 6 outpatient pain clinics. The inclusion criteria for the study were (1) diagnosis of chronic nonmalignant pain, (2) prescription for and current use of either transdermal fentanyl or oxycodone HCl controlled-release, and (3) duration of use for either transdermal fentanyl or oxycodone HCl controlled-release of at least 6 weeks. Patients completed either an oxycodone HCl controlled-release or transdermal fentanyl utilization questionnaire. A conversion table was used to standardize opioid analgesic doses from transdermal fentanyl or oxycodone HCl controlled-release to daily oral morphine equivalents. The principal outcome measures were the average interval between oxycodone HCl controlled-release administrations, the number of days the current transdermal fentanyl patch would be worn, and the percentage of oxycodone HCl controlled-release and transdermal fentanyl patients whose administration frequency exceeded the standard recommendation in the manufacturer.s PI (every 12 hours for oxycodone HCl controlled-release or every 72 hours for transdermal fentanyl). Other outcome measures included the number of oxycodone HCl controlled-release tablets per administration, the daily dose of long-acting opioid, the duration of adequate pain relief, and the difference in daily oral morphine equivalents between transdermal fentanyl and oxycodone HCl controlled-release patients, after adjusting in a multivariate regression model for demographic and clinical characteristics. A total of 690 patients were enrolled in this study; 437 (63.4%) received oxycodone HCl controlled-release and 253 (36.6%) received transdermal fentanyl. Oxycodone HCl controlled-release patients reported taking a median of 1 tablet 3 times per day or a median of 3 tablets per day. A mean of 1.6 tablets per administration and 4.6 tablets per day were taken. The average interval between administrations of oxycodone HCl controlled-release was 7.8 hours, and the median daily dose was 80.0 mg (mean 155.6 mg). Among oxycodone HCl controlled-release patients, 17.5% had an average interval between administrations of 12 or more hours, whereas 1.9% reported the duration of pain relief as 12 or more hours. Transdermal fentanyl patients reported wearing the patch, on average, for 2.5 days (median 2.5),and 41.2% reported wearing the patch for at least 3 days, whereas 14.1% reported the duration of pain relief as at least 3 days. The median daily dosage strength of transdermal fentanyl was 75.0 mcg/hour. In the multivariate regression analysis, oxycodone HCl controlled-release patients had, on average, roughly 22 mg additional oral morphine equivalents per day relative to transdermal fentanyl patients (not statistically significant); the probability that oxycodone HCl controlled-release patients had higher oral morphine equivalents was 82.6%, which suggests a trend toward higher oral morphine equivalents per day in the oxycodone HCl controlled-release group. Transdermal fentanyl and oxycodone HCl controlled-release both appear to be used by patients in a manner that is inconsistent with the standard recommendation in the manufacturers' PI;however, the difference between patient-reported utilization and the PI recommendation is more pronounced with oxycodone HCl controlled-release."}, {"id": "pubmed23n0287_8829", "title": "[Continuous spinal analgesia in home care of oncologic pain].", "score": 0.008771929824561403, "content": "The authors in this study, after a short survey of the most important therapeutic techniques for cancer pain, report their results in the treatment of 18 patients suffering from incurable disease. It was impossible to dismiss them from hospital care on account of a painful symptomatology not controllable by oral morphine or owing to excessive collateral morphinic consequences. The analgetic technique employed was continuous intrathecal infusion of morphine, clonidine, droperidol and, in 10 cases, bupivacaine. Drug delivery systems, totally internalized, except infusion pump, were always utilized. Adequate pain relief was obtained, within - 5 days, in all the patients. Family membres, in the same period, learnt the infusion circuit action. At this point the patients were dismissed and treated with home care. The average time of assistance was 140 days, and very moderate variations in posology were necessary. Hospital reentrance, really little numerous, happened only when no member of palliative care service was present. Reasons were no bodily pain, but the total suffering of cancer disease. No complication nor collateral consequences were never found."}, {"id": "wiki20220301en522_29798", "title": "Opioid rotation", "score": 0.00873015873015873, "content": "Opioid rotation or opioid switching is the process of changing one opioid to another to improve pain control or reduce unwanted side effects. This technique was introduced in the 1990s to help manage severe chronic pain and improve the opioid response in cancer patients. In order to obtain adequate levels of pain relief, patients requiring chronic opioid therapy may require an increase in the original prescribed dose for a number of reasons, including increased pain or a worsening disease state. Over the course of long term treatment, an increase in dosage cannot be continued indefinitely as unwanted side effects of treatment often become intolerable once a certain dose is reached, even though the pain may still not be properly managed. One strategy used to address this is to switch the patient between different opioid drugs over time, usually every few months. Opioid rotation requires strict monitoring in patients with ongoing levels of high opioid doses for extended periods of time,"}]}}}} {"correct_option": 2, "explanations": {"1": {"exist": true, "char_ranges": [[0, 195]], "word_ranges": [[0, 31]], "text": "In the presence of isolated thrombocytopenia we must not lose sight of the physical examination; thus, the presence of lymphadenopathy forces us to rule out the presence of lymphoma, for example."}, "2": {"exist": true, "char_ranges": [[210, 422]], "word_ranges": [[34, 69]], "text": "When performing a bone marrow biopsy-aspirate it is seen that the number of megakaryocytes is normal or even increased, the failure is not in the marrow but in the peripheral blood, where platelets are destroyed."}, "3": {"exist": false, "char_ranges": [], "word_ranges": [], "text": ""}, "4": {"exist": false, "char_ranges": [], "word_ranges": [], "text": ""}, "5": {"exist": false, "char_ranges": [], "word_ranges": [], "text": ""}}, "full_answer": "In the presence of isolated thrombocytopenia we must not lose sight of the physical examination; thus, the presence of lymphadenopathy forces us to rule out the presence of lymphoma, for example. 1 is correct. When performing a bone marrow biopsy-aspirate it is seen that the number of megakaryocytes is normal or even increased, the failure is not in the marrow but in the peripheral blood, where platelets are destroyed. So. The false answer is 2.", "full_answer_no_ref": "In the presence of isolated thrombocytopenia we must not lose sight of the physical examination; thus, the presence of lymphadenopathy forces us to rule out the presence of lymphoma, for example. [HIDDEN] When performing a bone marrow biopsy-aspirate it is seen that the number of megakaryocytes is normal or even increased, the failure is not in the marrow but in the peripheral blood, where platelets are destroyed. So. [HIDDEN]", "full_question": "A 33-year-old woman consults for repeated epistaxis, petechiae and ecchymosis. Laboratory tests show thrombocytopenia with a platelet count of 4000 platelets/microliter. The initial presumptive diagnosis is chronic immune thrombocytopenic purpura (ITP). Which of the following statements is FALSE regarding the diagnosis of ITP?", "id": 179, "lang": "en", "options": {"1": "The presence of lymphadenopathy or splenomegaly in the physical examination suggests a different diagnosis of ITP.", "2": "Bone marrow analysis shows a decreased number of megakaryocytes without other alterations.", "3": "Complete blood count shows isolated thrombocytopenia with often large platelets, without anemia unless there is significant bleeding or associated autoimmune hemolysis (Evans syndrome).", "4": "The diagnosis of ITP is established by exclusion of other processes causing thrombocytopenia.", "5": "The determination of antiplatelet antibodies is not accurate to establish the diagnosis."}, "question_id_specific": 98, "type": "HEMATOLOGY", "year": 2013, "rag": {"clinical_case_options": {"MedCorp": {"RRF-2": [{"id": "wiki20220301en016_42764", "title": "Immune thrombocytopenic purpura", "score": 0.01951265943270512, "content": "Despite the destruction of platelets by splenic macrophages, the spleen is normally not enlarged. In fact, an enlarged spleen should lead to a search for other possible causes for the thrombocytopenia. Bleeding time is usually prolonged in ITP patients. However, the use of bleeding time in diagnosis is discouraged by the American Society of Hematology practice guidelines and a normal bleeding time does not exclude a platelet disorder. Bone marrow examination may be performed on patients over the age of 60 and those who do not respond to treatment, or when the diagnosis is in doubt. On examination of the marrow, an increase in the production of megakaryocytes may be observed and may help in establishing a diagnosis of ITP. An analysis for anti-platelet antibodies is a matter of clinician's preference, as there is disagreement on whether the 80 percent specificity of this test is sufficient to be clinically useful."}, {"id": "wiki20220301en016_42759", "title": "Immune thrombocytopenic purpura", "score": 0.01837336704593342, "content": "ITP is an autoimmune disease with antibodies detectable against several platelet surface structures. ITP is diagnosed by identifying a low platelet count on a complete blood count (a common blood test). However, since the diagnosis depends on the exclusion of other causes of a low platelet count, additional investigations (such as a bone marrow biopsy) may be necessary in some cases. In mild cases, only careful observation may be required but very low counts or significant bleeding may prompt treatment with corticosteroids, intravenous immunoglobulin, anti-D immunoglobulin, or immunosuppressive medications. Refractory ITP (not responsive to conventional treatment or constant relapsing after splenectomy) requires treatment to reduce the risk of clinically significant bleeding. Platelet transfusions may be used in severe cases with very low platelet counts in people who are bleeding. Sometimes the body may compensate by making abnormally large platelets."}, {"id": "pubmed23n0319_6679", "title": "[Idiopathic thrombocytopenic purpura in children].", "score": 0.017158294392523366, "content": "Idiopathic (immune) thrombocytopenic purpura (ITP) is the most frequent hemorrhagic disease in children. It represents the acquired megakaryocytic thrombocytopenia with the shortened life of platelets because of immunologic damage (antibodies absorbed by platelets). In the case of this acquired hemorrhagic disorder, in spite of compensatory increased function of the bone marrow, there is a reduced number of platelets because of their increased destruction by the reticuloendothelial system (destructive thrombocytopenia). There are three forms of ITP: acute, chronic and intermittent. The acute form occurs in 80-90% of cases with bleeding episodes lasting a few days or weeks, but no longer than 6 months. The chronic form occurs in 10-15% of children, while the rarest-intermittent form is characterized by periods of normalization in regard to the number of platelets but also with relapse in intervals of 1-3 months. The disease is caused by an immunological disorder in the sense of an imbalanced immune response. Immunologic damages of platelets cause shortening of the opsonized platelets life span. The most frequent platelet opsonins are the immumoglobulin G (IgG) antibodies directed at the platelet membrane in the form of autoantibodies, alloantibodies or possibly absorbed antigen caused by microorganism infection or drug intake. It is typical for the phenomenon of bleeding that it starts suddenly and without any other sign of illness. The most frequent acute form appears between the second and fourth year, and is characterized by seasonal occurrence usually after acute viral infections. Children older than 10 years of age, like adults, often have the chronic form associated with other immunologic disorders. The disease affects girls more often than boys (about three times more often) with moderate and constant increase of antiplatelet antibodies. Hemorrhagic manifestations include: petechiae, purpura, epistaxis, gastrointestinal and genitourinary bleeding. They depend on the grade of thrombocytopenia, although there is no strict correlation between the number of platelets and volume of bleeding. Low mortality of the acute ITP is almost exclusively due to intracranial hemorrhage. LABORATORY STUDIES: Thrombocytopenia represents a decrease in the number of blood platelets being a basic abnormality of the blood count. The half-life of platelets in ITP is shortened. Detection of antiplatelet antibodies is connected with technical difficulties, so they are established in about 30% of cases. Bleeding time is prolonged and so is the coagulum retraction which may be completely missed. The Rumpel-Leede test is positive. Clinical differentiation of drug-induced thrombocytopenia is not possible. However, other differential diagnostic possibilities are thrombotic-thrombocytopenic purpura and hemolytic-uremic syndrome. A child with aplastic anemia or acute leukemia, beside thrombocytopenia, has a characteristic finding of white and red blood cell count. Thrombocytopenia with absent radii syndrome is associated with skeletal system abnormalities. New knowledge about the role of the immune system in ITP determines the modern therapeutic modalities. In cases of acute ITP in children, there are two therapeutic options or therapies of choice: corticosteroids and high doses of intravenous immunoglobulin. Immunosupressive therapy means anti Rh(D) immunoglobulin, cyclosporine, cytostatics, danazol, loaded platelets. In cases of distinctive hemorrhagic syndrome there are also indications for platelet transfusion. Nowadays splenectomy is more restricted, because one third of cases is unsuccessful, whereas plasmapheresis is rarely used in children because of possible complications. ITP is the most frequent hemorrhagic disease in children. The disease is basically caused by an immunologic disorder with platelet destruction due to increased immunoglobulin on their membrane. (ABSTRACT TRUNCATED)"}, {"id": "pubmed23n0526_14831", "title": "Initial laboratory findings useful for predicting the diagnosis of idiopathic thrombocytopenic purpura.", "score": 0.016695652173913042, "content": "To identify initial laboratory findings useful for the later diagnosis of idiopathic thrombocytopenic purpura (ITP) in adult patients with thrombocytopenia. We studied 62 consecutive adult patients who had thrombocytopenia and whose peripheral blood film was normal except for thrombocytopenia at presentation. Each patient underwent physical examination and routine laboratory tests and was prospectively followed for 22.5 +/- 9.8 months (range, 8 to 41 months). The frequency of antiglycoprotein (GP) IIb/IIIa antibody-producing B cells, the presence of platelet-associated and plasma anti-GPIIb/IIIa antibodies, the percentage of reticulated platelets, and the plasma thrombopoietin level were examined at the first visit. The final diagnosis was based on the clinical history, physical examination, complete blood test, bone marrow findings, and the clinical course at last observation. Forty-six patients were diagnosed as having ITP and 16 as having another disorder, including myelodysplastic syndrome, aplastic anemia, amegakaryocytic thrombocytopenia, and reduced platelet production, with or without other cytopenias, and without dysplasia or evidence for destruction. Six initial laboratory findings discriminated ITP from other diagnoses: the absence of anemia, absence of leukocytopenia, increased frequency of anti-GPIIb/IIIa antibody-producing B cells, increased platelet-associated anti-GPIIb/IIIa antibodies, elevated percentage of reticulated platelets, and a normal or slightly increased plasma thrombopoietin level. Three or more of these ITP-associated findings were found at presentation in 44 patients (96%) with thrombocytopenia later diagnosed as ITP, compared with only 1 patient (6%) whose disorder was non-ITP. Initial laboratory findings can well predict future diagnosis of ITP. Further studies prospectively evaluating these same diagnostic criteria on another, independent set of patients are necessary."}, {"id": "pubmed23n0396_7459", "title": "The role of the bone marrow examination in the diagnosis of immune thrombocytopenic purpura: case series and literature review.", "score": 0.01579740729574917, "content": "The need for a bone marrow examination was assessed in patients with clinical and laboratory features consistent with ITP; the literature was reviewed. The records of all patients undergoing a bone marrow examination between January 1988 to January 1998 were retrospectively reviewed to determine which were motivated by the suspicion of ITP. Data were collected from hospital and outpatient medical and pathology records. Eighty-six patients with isolated thrombocytopenia (i.e., normal white blood cell count, hemoglobin, peripheral smear and clotting studies) were studied. The bone marrow was consistent with ITP in 82 patients, (i.e., normal or increased megakaryocytes and other hemopoietic lineages normal.) Four patients had decreased megakaryocytes, but all patients responded to corticosteroids. All 86 patients were followed up for a median of 22 months after bone marrow aspiration (range, 2-76 months.) During that time, none of the patients developed features to suggest an alternative diagnosis to ITP. The initial clinical and laboratory findings of 99 patients with acute leukemia were also reviewed; all had features atypical of ITP. These data suggest that routine performance of a bone marrow examination for the diagnosis of ITP is not necessary, provided that a thorough history and physical examination are performed and that the complete blood cell count, peripheral blood smear, and routine clotting studies show no abnormalities apart from thrombocytopenia. The findings of seven prior retrospective studies, two in adults and five in children are consistent with the previous findings. However, the value of marrow investigation in ITP remains unresolved and data from a large prospective study would be helpful."}, {"id": "wiki20220301en020_74489", "title": "Thrombocytopenia", "score": 0.015160118101294572, "content": "Immune thrombocytopenic purpura Many cases of immune thrombocytopenic purpura (ITP) also known as idiopathic thrombocytopenic purpura, can be left untreated, and spontaneous remission (especially in children) is not uncommon. However, counts under 50,000 are usually monitored with regular blood tests, and those with counts under 10,000 are usually treated, as the risk of serious spontaneous bleeding is high with such low platelet counts. Any patient experiencing severe bleeding symptoms is also usually treated. The threshold for treating ITP has decreased since the 1990s; hematologists recognize that patients rarely spontaneously bleed with platelet counts greater than 10,000, although exceptions to this observation have been documented."}, {"id": "pubmed23n1121_25608", "title": "Assessment and Management of Immune Thrombocytopenia (ITP) in the Emergency Department: Current Perspectives.", "score": 0.01510676965015902, "content": "Immune thrombocytopenia (ITP) is characterized by a platelet count less than 100 × 10^9/L without anemia or leukopenia. Patients with ITP may be asymptomatic, or they may have mild bleeding like petechiae, purpura, or epistaxis. In rare cases, they may present to the emergency department (ED) with life-threatening bleeding as a result of their thrombocytopenia. The emergency physician should thus be prepared to diagnose ITP and treat the bleeding that can result from it. The diagnosis of ITP requires excluding secondary causes of thrombocytopenia, and in the ED, the bare minimum workup for ITP includes a complete blood count and a peripheral blood smear. The peripheral blood smear should show a small number of large platelets with normal morphology, and there should not be an increased number of schistocytes. Many patients with ITP require no emergent treatment. However, if a patient with suspected ITP presents to the ED with critical hemorrhage, the emergency physician should initiate treatment with a platelet transfusion, corticosteroids, and intravenous immune globulin (IVIG) as soon as possible. For less severe bleeding, platelet transfusions are not recommended, and the treatment consists of corticosteroids by themselves or in conjunction with IVIG."}, {"id": "pubmed23n0416_22915", "title": "[The diagnosis of ITP].", "score": 0.014509285029885476, "content": "Idiopathic thrombocytopenic purpura(ITP) is a hematologic disorder which causes thrombocytopenia. The diagnosis of ITP is based on the history, physical examination and, complete blood count, and examination of the peripheral smear. The diagnostic criteria of ITP established by the Ministry of Health, Welfare, and Labor in Japan requires the bone marrow examination and the measurement of platelet associated IgG, but those tests are not always necessary according to the guidelines developed by the American Society of Hematology. The appropriate strategies for the diagnosis of ITP need to be established. In this paper, some new examinations which may help the diagnosis of ITP are also demonstrated."}, {"id": "wiki20220301en016_42758", "title": "Immune thrombocytopenic purpura", "score": 0.013782051282051282, "content": "Immune thrombocytopenic purpura (ITP), also known as idiopathic thrombocytopenic purpura or immune thrombocytopenia, is a type of thrombocytopenic purpura defined as an isolated low platelet count with a normal bone marrow in the absence of other causes of low platelets. It causes a characteristic red or purple bruise-like rash and an increased tendency to bleed. Two distinct clinical syndromes manifest as an acute condition in children and a chronic condition in adults. The acute form often follows an infection and spontaneously resolves within two months. Chronic immune thrombocytopenia persists longer than six months with a specific cause being unknown. ITP is an autoimmune disease with antibodies detectable against several platelet surface structures."}, {"id": "pubmed23n0980_11344", "title": "Immune thrombocytopenic purpura.", "score": 0.0135122838944495, "content": "Immune thrombocytopenic purpura (ITP) is a bleeding disorder characterized by isolated thrombocytopenia (platelet count <150,000 u/L), which is not associated with a systemic illness. ITP is reported in approximately 2 per 100,000 adults. The mean age of diagnosis is 50 years. ITP is more common in females of childbearing age and in pregnancy. In adults, the course is more chronic although spontaneous remission can also occur within months of initial diagnosis. A thorough and timely workup of thrombocytopenia is imperative to rule out other differentials of ITP as it is considered a diagnosis of exclusion. Primary care physicians encounter patients who exhibit signs of thrombocytopenia such as petechiae or purpura on a regular basis. A high index of clinical suspicion is required to accurately diagnose ITP and commence the appropriate treatment including glucocorticoids to increase the chances of a favorable prognosis as described by the authors."}, {"id": "pubmed23n1009_24869", "title": "Teetering on a liver's edge: a case report highlighting clinical decision-making in thrombocytopenia.", "score": 0.013461369814960975, "content": "This report illustrates the importance of a detailed history and physical exam and careful analysis of hematologic parameters when diagnosing ITP. This case demonstrates that even with subtle deviations from typical ITP findings one must promptly reevaluate the diagnosis. This case also highlights the importance of peripheral smear review by an expert in pediatric hematopathology. A previously healthy 10 year-old Asian boy presented with 2 months of easy bruising. Review of systems was negative for any constitutional symptoms. On examination, he appeared well but had numerous large ecchymoses. He had no appreciable lymphadenopathy or splenomegaly. The liver was palpable 1.5 cm below the costal margin. A complete blood count (CBC) showed: platelets = 17 × 109/L, hemoglobin = 128 g/L, white blood cell count = 5.43 × 109/L, and neutrophils = 1.63 × 109/L. A blood smear was reported as normal. Urate was 370 umol/L and lactate dehydrogenase (LDH) was 803 U/L. The child was admitted with a presumptive diagnosis of immune thrombocytopenic purpura (ITP) and treated with intravenous immunoglobulin. The following day, the blood smear was reviewed by a hematopathologist who identified blasts. A bone marrow aspiration (BMA) confirmed the diagnosis of precursor B-cell acute lymphoblastic leukemia. In children presenting with suspected ITP, leukemia should always be considered. A BMA was historically performed on all patients with presumed ITP to rule out leukemia. In 2011, the American Society of Hematology (ASH) stopped recommending routine BMA in patients suspected of having ITP. ASH advises in cases with unusual findings on history, physical examination or CBC, it is reasonable to perform a BMA. Our patient had mild hepatomegaly, which may have qualified him for a BMA. He also had an elevated LDH and urate, which are not listed as criteria for BMA by ASH but were considered atypical for ITP by the clinical team. A literature search did not reveal any primary data assessing these markers. While corticosteroids are a first line treatment in ITP, they must be reserved for when clinicians are confident that the patient does not have leukemia. Steroid administration prior to diagnosing leukemia results in delayed diagnosis and may increase the risk of complications and decrease survival."}, {"id": "pubmed23n0233_63", "title": "Present-day problems of diagnosis and treatment in the idiopathic thrombocytopenic purpura.", "score": 0.013105389171252153, "content": "A review is made of the data in the literature and of the authors' experience regarding the etiology and pathogenesis, diagnosis and therapy of the chronic form of idiopathic thrombocytopenic purpura (ITP). The mechanisms of production (after McMillan et al.) are presented schematically and the five criteria of ITP diagnosis suggested by Karpatkin are discussed: 1) decreased blood platelet count with direct or indirect signs of thrombocytolysis; 2) increased number of megakaryocytes in the bone marrow and/or signs of intramedullary thrombocytolysis; 3) direct or indirect signs of antiplatelet autoantibody presence in the plasma; 4) exclusion of a primary disorder and 5) absence of splenomegaly. The results of corticotherapy, splenectomy, platelet transfusion and immunosuppression (including \"target\" immunosuppressive therapy) in 188 patients with ITP, admitted to the clinic of Hematology--Bucharest between 1966 and 1978, are presented and analysed."}, {"id": "wiki20220301en016_42781", "title": "Immune thrombocytopenic purpura", "score": 0.012842166569598136, "content": "It is recommended that pregnant women with thrombocytopenia or a previous diagnosis of ITP should be tested for serum antiplatelet antibodies. A woman with symptomatic thrombocytopenia and an identifiable antiplatelet antibody should be started on therapy for their ITP which may include steroids or IVIG. Fetal blood analysis to determine the platelet count is not generally performed as ITP-induced thrombocytopenia in the fetus is generally less severe than NAIT. Platelet transfusions may be performed in newborns, depending on the degree of thrombocytopenia. It is recommended that neonates be followed with serial platelet counts for the first few days after birth."}, {"id": "First_Aid_Step2_374", "title": "First_Aid_Step2", "score": 0.01283068783068783, "content": "ITP is associated with a range of conditions, including lymphoma, leukemia, SLE, HIV, and HCV. The clinical presentation is as follows: Acute: Abrupt onset of hemorrhagic complications following a viral illness. Commonly affects children 2–6 years of age, with males and females affected equally. Chronic: Insidious onset that is unrelated to infection. Most often affects adults 20–40 years of age; women are three times more likely to be affected than men. A diagnosis of exclusion, as the test for platelet-associated antibodies is a poor one. Once other causes of thrombocytopenia have been ruled out, a diagnosis can be made on the basis of the history and physical, a CBC, and a peripheral blood smear showing normal RBC morphology. Most patients do not require bone marrow biopsy, which would show ↑ megakaryocytes as the only abnormality. Most patients with acute childhood ITP spontaneously remit, but this is rarely the case in chronic ITP."}, {"id": "wiki20220301en016_42763", "title": "Immune thrombocytopenic purpura", "score": 0.012776222526654356, "content": "Diagnosis The diagnosis of ITP is a process of exclusion. First, it has to be determined that there are no blood abnormalities other than a low platelet count, and no physical signs other than bleeding. Then, secondary causes (5–10 percent of suspected ITP cases) should be excluded. Such secondary causes include leukemia, medications (e.g., quinine, heparin), lupus erythematosus, cirrhosis, HIV, hepatitis C, congenital causes, antiphospholipid syndrome, von Willebrand factor deficiency, onyalai and others. All patients with presumed ITP should be tested for HIV and hepatitis C virus, as platelet counts may be corrected by treating the underlying disease. In approximately 2.7 to 5 percent of cases, autoimmune hemolytic anemia and ITP coexist, a condition referred to as Evans syndrome."}, {"id": "pubmed23n0950_231", "title": "Immune Thrombocytopenic Purpura Detected with Oral Hemorrhage: a Case Report.", "score": 0.012767425810904072, "content": "Immune thrombocytopenic purpura (ITP) is an immune-mediated acquired disease found in both adults and children. It is characterized by transient or persistent decreases in the platelet count. We report a case of ITP detected based on oral hemorrhagic symptoms. The patient was a 79-year-old female with no significant past medical history. She presented with sudden onset of gingival bleeding and hemorrhagic bullae on the buccal mucosa. Gingival bleeding was difficult to control. Laboratory tests revealed severe thrombocytopenia with a platelet count as low as 2000/μL. Under a provisional diagnosis of a hematological disorder, she was referred to a hematologist. A peripheral smear showed normal-sized platelets. A bone marrow examination revealed increased numbers of megakaryocytes without morphologic abnormalities. The patient was diagnosed with ITP and treated with a combination of pulsed steroid therapy and high-dose immunoglobulin therapy. However, her severe thrombocytopenia was refractory to these treatments. Then, a thrombopoietin receptor agonist was begun as a second-line treatment. Her platelets rapidly increased, and no bleeding complications were reported. Because oral symptoms can be one of the initial manifestations of ITP, dentists should be familiar with the clinical appearance of ITP, and attention must be paid to detect and diagnose unidentified cases."}, {"id": "pubmed23n1154_18040", "title": "Development and internal validation of a clinical prediction model for the diagnosis of immune thrombocytopenia.", "score": 0.012743506493506494, "content": "Immune thrombocytopenia (ITP) is a diagnosis of exclusion that can resemble other thrombocytopenic disorders. To develop a clinical prediction model (CPM) for the diagnosis of ITP to aid hematogists in investigating patients presenting with undifferentiated thrombocytopenia. We designed a CPM for ITP diagnosis at the time of the initial hematology consultation using penalized logistic regression based on data from patients with thrombocytopenia enrolled in the McMaster ITP registry (n = 523) called the Predict-ITP Tool. The case definition for ITP was a platelet count less than 100 × 1030,000/μL appear not to have increased mortality related to the thrombocytopenia."}, {"id": "wiki20220301en016_42780", "title": "Immune thrombocytopenic purpura", "score": 0.012015221017514597, "content": "Pregnancy Anti-platelet autoantibodies in a pregnant woman with ITP will attack the patient's own platelets and will also cross the placenta and react against fetal platelets. Therefore, ITP is a significant cause of fetal and neonatal immune thrombocytopenia. Approximately 10% of newborns affected by ITP will have platelet counts <50,000/uL and 1% to 2% will have a risk of intracerebral hemorrhage comparable to infants with neonatal alloimmune thrombocytopenia (NAIT). No lab test can reliably predict if neonatal thrombocytopenia will occur. The risk of neonatal thrombocytopenia is increased with: Mothers with a history of splenectomy for ITP Mothers who had a previous infant affected with ITP Gestational (maternal) platelet count less than 100,000/uL"}, {"id": "wiki20220301en010_97374", "title": "Platelet", "score": 0.011666458930574598, "content": "Disorders Adapted from: The three broad categories of platelet disorders are \"not enough\"; \"dysfunctional\"; and \"too many\". Thrombocytopenia Immune thrombocytopenias (ITP) – formerly known as immune thrombocytopenic purpura and idiopathic thrombocytopenic purpura Splenomegaly Gaucher's disease Familial thrombocytopenia Chemotherapy Babesiosis Dengue fever Onyalai Thrombotic thrombocytopenic purpura HELLP syndrome Hemolytic–uremic syndrome Drug-induced thrombocytopenic purpura (five known drugs – most problematic is heparin-induced thrombocytopenia (HIT) Pregnancy-associated Neonatal alloimmune associated Aplastic anemia Transfusion-associated Pseudothrombocytopenia idiopathic thrombocytopenic purpura Vaccine induced immune thrombocytopenia Gilbert's syndrome"}, {"id": "wiki20220301en046_45191", "title": "Evans syndrome", "score": 0.011654511654511654, "content": "Causes Although Evans syndrome seems to be a disorder of immune regulation, the exact pathophysiology is unknown, but a gradual loss of self-tolerance is postulated. Autoantibodies targeted at different antigenic determinants on red cells and platelets are assumed to cause isolated episodes of hemolytic anemia and thrombocytopenia, respectively. Diagnosis The diagnosis of primary Evans syndrome is made upon blood tests to confirm not only hemolytic anemia and immune thrombocytopenic purpura, but also a positive direct antiglobulin test (DAT) and an absence of any known underlying cause. In 27% to 50% of cases there is an associated malignancy or a predisposing autoimmune disease (e.g. systemic lupus erythematosus), it is then common to denote it as secondary Evans syndrome. Other antibodies may occur directed against neutrophils and lymphocytes, and \"immunopancytopenia\" has been suggested as a term for this syndrome."}, {"id": "pubmed23n0660_1139", "title": "[Myelodysplastic syndrome mimicking idiopathic thrombocytopenic purpura].", "score": 0.011628300037183518, "content": "In patients with isolated thrombocytopenia, but without significant dysplasia, diagnosis of idiopathic thrombocytopenic purpura (ITP) rather than myelodysplastic syndrome (MDS) may be taken into account. It is important to make an accurate diagnosis because different treatments are used for ITP and MDS. The purpose of this study was to investigate the clinical and hematologic features of patients who were initially diagnosed as ITP but had cytogenetic abnormalities. We retrospectively reviewed cytogenetic studies of 100 patients who were diagnosed as ITP from 2004 to 2009 at Mokdong Hospital of Ewha Womans University based on clinical features and hematologic studies. Bone marrow pathology was re-evaluated based on 2008 WHO classification. Cytogenetic analysis was performed by 24-48 hr culture of bone marrow aspirates without using mitogens and 20 metaphases were analyzed. Of the 100 patients diagnosed as ITP initially, three patients (3%) had cytogenetic abnormalities. They had no thrombocytopenia-related symptoms and thrombocytopenia was found accidentally. The numbers of megakaryocytes in bone marrow were increased and dysplasia was not found in megakaryocyte, erythroid, and myeloid cell lineages. The proportion of blasts was within normal limits. Clonal chromosomal abnormalities found were der(1;7)(q10;p10), add(9)(q12), or t(7;11)(p22;q12). Presumptive diagnosis of MDS or diagnosis of idiopathic cytopenia of undetermined significance (ICUS) was made according to 2008 WHO classification. During the follow up, disease progression was not found. In patients with suspected ITP, cytogenetic analysis should be done. If specific clonal chromosomal abnormality is found, presumptive diagnosis of MDS has to be considered and close follow up is needed."}, {"id": "wiki20220301en192_930", "title": "Harrington–Hollingsworth experiment", "score": 0.011321078361302495, "content": "The Harrington–Hollingsworth experiment was an experiment that established the autoimmune nature of the blood disorder immune thrombocytopenic purpura. It was performed in 1950 by the academic staff of Barnes-Jewish Hospital in St. Louis, Missouri. Experiment The experiment was undertaken in 1950 by William J. Harrington and James W. Hollingsworth, who postulated that in patients with idiopathic thrombocytopenic purpura (ITP), it was a blood factor that caused the destruction of platelets. To test this hypothesis, Harrington received 500 ml of blood from a patient with ITP. Within three hours, his platelets dropped to dangerously low levels and he experienced a seizure. His platelet count remained extremely low for four days, finally returning to normal levels by the fifth day. Bone marrow biopsy from Harrington's sternum demonstrated normal megakaryocytes, the cells necessary for platelet production."}, {"id": "Pediatrics_Nelson_3266", "title": "Pediatrics_Nelson", "score": 0.01130952380952381, "content": "Diagnosis. The diagnosis of ITP usually is based on clinical presentation and the platelet count and does not often require a bone marrow examination. If atypical findings are noted, however, marrow examination is indicated to rule out an infiltrative disorder (leukemia) or an aplastic process (aplastic anemia). In ITP, an examination of the bone marrow reveals increased megakaryocytes and normal erythroid and myeloid elements."}, {"id": "wiki20220301en016_42777", "title": "Immune thrombocytopenic purpura", "score": 0.011308439587128112, "content": "Epidemiology A normal platelet count is considered to be in the range of 150,000–450,000 per microlitre (μl) of blood for most healthy individuals. Hence one may be considered thrombocytopenic below that range, although the threshold for a diagnosis of ITP is not tied to any specific number. The incidence of ITP is estimated at 50–100 new cases per million per year, with children accounting for half of that number. At least 70 percent of childhood cases will end up in remission within six months, even without treatment. Moreover, a third of the remaining chronic cases will usually remit during follow-up observation, and another third will end up with only mild thrombocytopenia (defined as a platelet count above 50,000). A number of immune related genes and polymorphisms have been identified as influencing predisposition to ITP, with FCGR3a-V158 allele and KIRDS2/DL2 increasing susceptibility and KIR2DS5 shown to be protective."}, {"id": "wiki20220301en020_74487", "title": "Thrombocytopenia", "score": 0.011154115746807667, "content": "In severe thrombocytopenia, a bone marrow study can determine the number, size, and maturity of the megakaryocytes. This information may identify ineffective platelet production as the cause of thrombocytopenia and rule out a malignant disease process at the same time. Treatment Treatment is guided by the severity and specific cause of the disease. Treatment focuses on eliminating the underlying problem, whether that means discontinuing drugs suspected to cause it or treating underlying sepsis. Diagnosis and treatment of serious thrombocytopenia is usually directed by a hematologist. Corticosteroids may be used to increase platelet production. Lithium carbonate or folate may also be used to stimulate platelet production in the bone marrow. Platelet transfusions Platelet transfusions may be suggested for people who have a low platelet count due to thrombocytopenia."}, {"id": "wiki20220301en435_34729", "title": "Upshaw–Schulman syndrome", "score": 0.011148522259633371, "content": "Diagnosis A diagnosis of TTP is based on the clinical symptoms with the concomitant presence of thrombocytopenia (platelet count below 100×109/L) and microangiopathic hemolytic anemia with schistocytes on the blood smear, a negative direct antiglobulin test (Coombs test), elevated levels of hemolysis markers (such as total bilirubin, LDH, free hemoglobin, and an unmeasurable haptoglobin), after exclusion of any other apparent cause. USS can present similar to the following diseases, which have to be excluded: fulminant infections, disseminated intravascular coagulation, autoimmune hemolytic anemia, Evans syndrome, the typical and atypical form of hemolytic uremic syndrome, HELLP (hemolysis, elevated liver enzymes, low platelets) syndrome, pre-eclampsia, heparin-induced thrombocytopenia, cancer that is often accompanied with metastasis, kidney injury, antiphospholipid antibody syndrome, and side effects from hematopoietic stem cell transplantation."}, {"id": "article-29382_20", "title": "Physiology, Spleen -- Clinical Significance", "score": 0.010940976312001048, "content": "ITP appears to be an autoimmune condition resulting in thrombocytopenia, petechiae/purpura, and bleeding from mucosal surfaces. There have been many antibodies implicated in the pathogenesis of ITP with the two main autoantibodies being against IgG and the glycoprotein (GP) IIb/IIIa complex on platelets. Platelets tagged by these autoantibodies then get broken down by phagocytes in the spleen, resulting in thrombocytopenia. Without platelets to form the platelet plug, which is the first step of hemostasis, the patient becomes more susceptible to bleeding. Most patients with ITP only have minor bleeds, such as nosebleeds or conjunctival bleeds. However, these patients are at risk of severe bleeds, and that risk helps guide management. ITP tends to affect children around five years old or older adults. In children, ITP typically presents with a sudden onset, usually following a microbial illness and is brief. In adults, it typically presents with insidious onset and is a chronic condition. In both patient populations, there must be a high index of suspicion, and ITP is usually a diagnosis of exclusion. A CBC and peripheral blood smear are typically performed in the initial evaluation. In adults, it is essential to rule out ITP secondary to other autoimmune diseases, so an anti-nuclear antibody (ANA) and autoantibodies specific for other conditions may be tested as well. A more specific test for ITP would be autoantibodies against IgG and GP IIb/IIIa. In children, treatment is generally conservative, as ITP is typically transient. Adults usually require treatment. Prednisone is the first-line treatment in adults and can also be used to treat children. IVIG can also be used to reduce or eliminate causative autoantibodies and is typically used before steroids in children and after steroids in adults. In cases refractory to medical management, a splenectomy is an option as most platelet breakdown occurs there. [28]"}, {"id": "pubmed23n0896_11785", "title": "Role of Helicobacter pylori Eradication Therapy on Platelet Recovery in Chronic Immune Thrombocytopenic Purpura.", "score": 0.010840824960338447, "content": " 1.6 mEq/L). Which of the following therapeutic options would be most indicated?", "id": 507, "lang": "en", "options": {"1": "Aminophylline associated with a cathartic.", "2": "Activated charcoal.", "3": "Hemodialysis.", "4": "Forced diuresis.", "5": NaN}, "question_id_specific": 89, "type": "PSYCHIATRY", "year": 2020, "rag": {"clinical_case_options": {"MedCorp": {"RRF-2": [{"id": "pubmed23n0522_8152", "title": "[Acute intoxication with sustained-release lithium carbonate tablets. A propos of a case].", "score": 0.019232547387887194, "content": "To describe the case of a patient who ingested 50 sustained release lithium carbonate 400 mg tablets, and reached a late peak concentration above 3 mEq/L. A 32-year-old male with bipolar mood disorder ingested 50 sustained-release lithium carbonate tablets. Upon admission to the emergency room, a gastric wash was performed,from which several tablet remnants were obtained, as well as an intestinal lavage using activated carbon. good general status, no fever, blood pressure 160/90 mm Hg, no edemas. Neurologic, pulmonary, and cardiac examinations were normal. CBC and the chemistry panel were normal. The patient's psychopathological examination suggested a stable status with no apparent manifestations arising from a decompensated mood disorder. Five hours after his massive lithium ingestion the drug's plasma levels were 0.75 mEq/L. At 22 hours post-ingestion a chemistry panel was obtained, which showed serum creatinin at 1.38 mg/dL and a lithium plasma concentration of 3.15 mEq/L. A hemodyalisis trial was attempted for 4 hours. At 73 hours post-ingestion, lithium plasma levels were 0.6 mEq/L, that is, within therapeutic range. The patient was hemodynamically stable and serial blood tests were normal; he was discharged. Acute lithium intoxication with plasma levels above 3 mEq/l can be fatal or result in irreversible neurologicsequelae in almost one third of cases, with persistent cerebellar dysfunction in association with dementia of variable degree, andrenal, blood, and liver disturbances. Sustained-release tablets may prolong absorption and delay peak plasma concentrations. In such cases, therefore, it is recommended that drug plasma concentrations be monitored during 48-72 hours post-ingestion."}, {"id": "pubmed23n0968_28", "title": "Lithium intoxication presenting as altered consciousness and arrhythmia with cardiogenic shock: A case report.", "score": 0.018350930115636, "content": "Lithium has been used to treat bipolar disorder. Lithium has a narrow therapeutic index, with a therapeutic level between 0.6 and 1.5 mEq/L. The possible complications of lithium overdose include altered mental status, hand tremor, muscle weakness, nausea, vomiting, diarrhea, seizure, syncope, and arrhythmia. Lithium intoxication can be fatal and is difficult to diagnose in patients without a history of lithium intake. The occurrence of serious cardiac arrhythmias is rare in lithium intoxication. An 81-year-old man was brought to the emergency department because of consciousness disturbance for 2 days. According to his daughter, he had a history of hypertension and diabetes. Recently, his family also observed slurring of speech and easy choking. The physical examination findings were unremarkable. Blood examination only revealed impaired renal function. Twelve-lead electrocardiography revealed sinus rhythm with first-degree atrioventricular block. Chest radiography revealed mediastinal widening. The blood pressures obtained from the 4 limbs showed no significant differences. Subsequently, brain computed tomography revealed no obvious intracranial lesion. A neurologist was consulted, and a recent ischemic stroke could not be ruled out. While in the observation area, his systolic blood pressure decreased to <90 mm Hg and he showed bradycardia, and 12-lead electrocardiography revealed an AV block and long pulse. Contrast-enhanced chest computed tomography revealed no evidence of aortic dissection. Another family member reported a history of lithium intake for bipolar disorder for >30 years. Blood examination revealed a lithium concentration of 2.65 mEq/L. A nephrologist was consulted, and emergency hemodialysis was indicated. Dopamine was administered for his shock status via a right neck central venous catheter. His lithium level gradually declined after the hemodialysis, and blood pressure and consciousness level improved subsequently. The patient was discharged 9 days later in a stable condition. If an emergency physician encounters a patient with altered consciousness and arrhythmia with cardiogenic shock, the patient's drug intake history should be carefully reviewed to rule out cardiovascular problems on the basis of the patient's clinical condition."}, {"id": "pubmed23n0660_22478", "title": "Prolonged neurological burden in severe lithium intoxication.", "score": 0.01778827233372688, "content": "A 53-year-old woman was brought to the Emergency Department for a persistent state of stupor, tremors, fever and oliguria. The patient had been under treatment for depression. The electrocardiogram showed a wider QRS complex; laboratory tests were as follows: urea 110 mg/dL, creatinine 3 mg/dL, sodium 135 mEq/L, potassium 4.5 mEq/L, and lithium 8.0 mEq/L. Renal replacement was initiated to normalize plasma lithium levels; both stupor and speech impairment persisted for several days after dialysis. Complete recovery was achieved several days afterwards."}, {"id": "pubmed23n0691_20683", "title": "[Difficulty in determining when to end continuous hemodialysis for lithium intoxication: case report].", "score": 0.016677089847821555, "content": "The patient was a 26-year-old woman who ingested a total of 230 tablets, including 160 lithium carbonate tablets (200 mg), in a suicide attempt, and was brought to our hospital 5 hr later. After arriving at the hospital, her only complaint was mild nausea, and no neurological abnormalities or renal dysfunction was observed. We were unable to learn the blood concentration of lithium immediately. A forced diuresis was performed after admission, but the nausea persisted. Continuous hemodialysis (CHD) was performed for 23.5 hr starting 19 hr after ingesting the tablets because the patient had ingested a large amount of lithium carbonate, 32,000 mg. Since the nausea resolved after the CHD was started and no manifestations of lithium intoxication had developed as of 91 hr after ingestion, the patient was discharged. The blood lithium concentrations (mEq/L) revealed at a later date showed that the concentration 5 hr (at the time of the initial examination), 19 hr (start of CHD), 44.5 hr (end of CHD), and 91 hr after ingestion (at the time of discharge) was 4.08, 3.30, 1.09, and 0.38, respectively. Blood purification is said to be effective in treating serious lithium intoxication, but it is difficult to judge when to stop. A favorable outcome of treatment of acute lithium intoxication in a patient with normal renal function appears to have been achieved by performing CHD guided by the clinical manifestations, intravascular redistribution times, etc."}, {"id": "pubmed23n0743_22851", "title": "Lithium toxicity precipitated by thyrotoxicosis due to silent thyroiditis: cardiac arrest, quadriplegia, and coma.", "score": 0.016196721311475412, "content": "Lithium is widely used to treat bipolar disorders. Lithium toxicity is generally caused by inappropriately high doses of lithium or impaired lithium excretion. Most lithium is eliminated via the kidneys and, since thyroid hormone increases tubular reabsorption of lithium, thyrotoxicosis could contribute to the development of lithium toxicity. We report a case of severe lithium toxicity that was apparently precipitated by the onset of thyrotoxicosis resulting from silent thyroiditis and dehydration. The patient was a 64-year-old woman who was admitted for muscle weakness in the lower extremities, diarrhea, and palpitations. She had bipolar disorder and was being treated with lithium carbonate, which she discontinued one week before admission. Her circulating lithium levels had been monitored yearly. Early in her admission she was dehydrated and had febrile episodes, paroxysmal atrial fibrillation, and muscle weakness. Initially, fluid therapy was started, but she lost consciousness and had a cardiac arrest for 2 minutes due to prolonged sinus arrest. Chest compression and manual artificial ventilation were performed, and body surface pacing was started. Serum lithium was markedly elevated to 3.81 mEq/L (therapeutic range, 0.4-1.0 mEq/L), and thyroid hormone levels were increased (free triiodothyronine, 8.12 pg/mL; free thyroxine, 4.45 ng/dL), while thyrotropin (TSH) was suppressed (<0.01 μIU/mL). Hemodialysis was performed, and a temporary pacemaker was inserted for severe sinus bradycardia. The serum thyroglobulin was 4680 ng/mL (reference range, <32.7 ng/mL). A TSH receptor antibody test was negative. Glucocorticoid therapy and inorganic iodine (100 mg) were administered and continued until day 11. However, her neurological symptoms deteriorated with floppy quadriplegia and deep coma. She gradually recovered. On day 36, she was discharged without any neurological symptoms or thyrotoxicosis. A 64-year-old woman taking lithium for bipolar disorder developed lithium toxicity in the setting of what seemed likely to be a recent onset of thyrotoxicosis due to silent thyroiditis. Thyrotoxicosis may be a contributing cause of lithium toxicity, particularly if it is abrupt in onset and even with cessation of lithium therapy if renal function is compromised. Thyroid function should be assessed immediately in patients with suspected lithium toxicity."}, {"id": "pubmed23n0479_3914", "title": "[Reversible choreoathetosis associated with lithium intoxication].", "score": 0.01606699751861042, "content": "Several reports have been published in the literature of choreoathetosis associated with lithium intoxication, but little is known about choreoathetosis without concurrent antipsychotic treatment. We report a 65-year-old woman with lithium intoxication whose choreoathetosis completely recovered without sequela following decrease of her serum lithium level. She had been treated elsewhere for bipolar II disorder and also for hypertension, chronic hepatitis type C and diabetes mellitus. As she became hypomanic, lithium carbonate at 600 mg/day was commenced, which was increased to 1200 mg/day due to unfavorable therapeutic response. She began to manifest disorientation and abnormal involuntary movement and was therefore referred to our Department of Psychiatry. Her clinical symptoms at admission included consciousness disturbance with marked bilateral symmetrical slow-wave activity in her EEG and choreoathetosis was observed in her face and upper and lower extremities. Cerebellar symptoms were minimal with only mild ataxic gait and finger-to-nose test did not show dysmetria or intention tremor. Her serum lithium level was 3.52 mEq/L, which was clearly in the toxic range. She demonstrated no metabolic abnormalities including hyperglycemia, and was diagnosed with lithium intoxication and treated with water loading and mannitol for forced diuresis. On the 14th day after admission her consciousness disturbance and choreoathetosis resolved, but EEG abnormalities still persisted. On the 23rd day after admission, she was discharged with clinical remission and normal EEG background activity. Although she developed mild renal dysfunction, hemodialysis was not indicated. Hypersensitivity of dopamine receptor in the nigrostriatal pathways may contribute to choreoathetosis in association with the patient's vulnerability. Choreoathetosis can be a sign of lithium intoxication and prompt treatment is required following careful differential diagnosis."}, {"id": "pubmed23n1080_6407", "title": "[Lithium sulfate poisoning treated with hemodialysis in a patient with normal renal function: a case report].", "score": 0.015763076188201577, "content": "Lithium is the milestone of psychiatric patients' therapy, in particular in bipolar disorder. Despite its high therapeutic efficacy, there are several side effects (renal, thyroid, parathyroid, dermatological) and management problems linked to its narrow therapeutic range, which exposes patients to a high risk of toxicity. We describe the case of a male patient with bipolar disorder in therapy with lithium sulfate who developed a severe acute-on-chronic intoxication. He came to our attention in a somnolent state with lithemia >3 mEq/L and therefore underwent hemodialysis. In view of the high toxicity of lithium, a timely and correct therapeutic choice is important to improve the patient's outcome. In this context, considering lithemia, but also kidney function and the patient's clinical status, it is necessary to consider extracorporeal treatments, of which hemodialysis is the most preferable."}, {"id": "wiki20220301en487_12076", "title": "Lithium toxicity", "score": 0.013792040682667489, "content": "Gastric lavage and whole bowel irrigation may be useful if done early. Activated charcoal is not effective. For severe toxicity hemodialysis is recommended. The risk of death is generally low. Acute toxicity generally has better outcomes than chronic toxicity. In the United States about 5,000 cases are reported to poison control centers a year. Lithium toxicity was first described in 1898. Signs and symptoms Symptoms of lithium toxicity can be mild, moderate, or severe. Mild symptoms include nausea, feeling tired, and tremor occur at a level of 1.5 to 2.5 mEq/L. Moderate symptoms include confusion, an increased heart rate, and low muscle tone occur at a level of 2.5 to 3.5 mEq/L. Severe symptoms include coma, seizures, low blood pressure and increased body temperature which occur at a lithium concentration greater than 3.5 mEq/L. When lithium overdoses produce neurological deficits or cardiac toxicity, the symptoms are considered serious and can be fatal."}, {"id": "wiki20220301en096_2387", "title": "Treatment of bipolar disorder", "score": 0.01339437738847478, "content": "Potential side effects from lithium include gastrointestinal upset, tremor, sedation, excessive thirst, frequent urination, cognitive problems, impaired motor coordination, hair loss, and acne. Excessive levels of lithium can be harmful to the kidneys, and increase the risk of side effects in general. As a result, kidney function and blood levels of lithium are monitored in patients being treated with lithium. Therapeutic plasma levels of lithium range from 0.5 to 1.5 mEq/L, with levels of 0.8 or higher being desirable in acute mania. Lithium levels should be above 0.6 mEq/L to reduce both manic and depressive episodes in patients. A recent review concludes that the standard lithium serum level should be 0.60-0.80 mmol/L with optional reduction to 0.40-0.60 mmol/L in case of good response but poor tolerance or an increase to 0.80-1.00 mmol/L in case of insufficient response and good tolerance."}, {"id": "pubmed23n0316_15490", "title": "Antacid-induced hypermagnesemia in a patient with normal renal function and bowel obstruction.", "score": 0.01329004329004329, "content": "To report a case of severe hypermagnesemia caused by magnesium hydroxide in a woman with normal renal function. A 42-year-old Hispanic woman with schizophrenia and bipolar affective disorder was transported from jail to the emergency department with confusion, abdominal pain, vomiting, and constipation. She had been treated in jail with magnesium hydroxide, ordered as milk of magnesia 30 mL po each night and Maalox 30 mL po three times daily. Additional medications included lithium carbonate 300 mg po three times daily, chlorpromazine 150 mg po three times daily, benztropine mesylate 1 mg po twice daily, and docusate sodium 100 mg po each morning. Her temperature was 35.1 degrees C, blood pressure 108/58 mm Hg, heart rate 112 beats/min, and respiratory rate 24 breaths/min. She would respond only briefly to voice or painful stimuli. Her abdomen was distended and diffusely tender. Laboratory tests included serum magnesium concentration 9.1 mEq/L (normal 1.3-2), blood urea nitrogen 16 mg/dL (8-22), creatinine 0.9 mg/dL (0.5-1.1), calcium 3.9 mEq/L (4.2-5.2), and lithium 1.0 mEq/L. A laparotomy was performed, and an adhesive band from a previous oophorectomy was found to be compressing the sigmoid colon. Hypermagnesemia, hypothermia, and hypotension continued in the intensive care unit. Despite successful treatment of the hypermagnesemia with calcium, intravenous fluids, and furosemide, the patient's cardiac rhythm degenerated into fatal, pulseless electrical activity on postoperative day 2. This case of severe hypermagnesemia from magnesium hydroxide ingestion illustrates many of the risk factors for hypermagnesemia in patients with normal renal function. People using magnesium-containing medications for relief of gastrointestinal distress may be at increased risk for hypermagnesemia. A brief review of magnesium physiology, clinical effects, and treatment is provided. Frequent use of the laboratory to identify hypermagnesemia is encouraged because it is often a clinically unexpected finding and responds well to early treatment."}, {"id": "pubmed23n1156_1115", "title": "[Diagnostic trap: Lithium neurotoxicity with normal lithemia].", "score": 0.013289183222958058, "content": "We describe here the case of a 54-year-old bipolar woman, followed in psychiatry and treated with lithium and a selective serotonin reuptake inhibitor (escitalopram) and lamotrigine, presenting a lithium poisoning with an altered state of consciousness caused by a supposed mismanagement of her treatment. Lithium poisoning was suggested based on neurological clinical features, but the blood test brought out a lithium concentration within the therapeutic values at 1,2 mmol/L (N: 0,6-1,2 mmol/L). The classic biological complications related to lithium poisoning (hypercalcemia, diabetes insipidus) confirmed the diagnosis. The patient has been transferred to our nephrology department where she got two hemodialysis sessions conducting to clinical and biological improvement, confirming the diagnosis of lithium poisoning despite the normal blood levels. Later, she was transferred to the psychiatry department for follow-up and for treatment adjustment."}, {"id": "InternalMed_Harrison_3617", "title": "InternalMed_Harrison", "score": 0.013096965927154605, "content": "A 63-year-old man was admitted to the intensive care unit (ICU) with a severe aspiration pneumonia. Past medical history included schizophrenia, for which he required institutional care; treatment had included neuroleptics and intermittent lithium, the latter restarted 6 months before admission. The patient was treated with antibiotics and intubated for several days, with the development of polyuria (3–5 L/d), hypernatremia, and acute renal insufficiency; the peak plasma Na+ concentration was 156 meq/L, and peak creatinine was 2.6 mg/dL. Urine osmolality was measured once and reported as 157 mOsm/kg, with a coincident plasma osmolality of 318 mOsm/kg. Lithium was stopped on admission to the ICU. On physical examination, the patient was alert, extubated, and thirsty. Weight was 97.5 kg. Urine output for the previous 24 h had been 3.4 L, with an IV intake of 2 L/d of D5W."}, {"id": "pubmed23n0590_20860", "title": "Lithium intoxication-induced acute parkinsonism complicated with hyperparathyroidism and nephrogenic diabetes insipidus: report of a case.", "score": 0.012099083619702175, "content": "To describe a patient with lithium intoxication presenting as acute parkinsonism, adverse metabolic effects and nephrogenic diabetes insipidus (DI). We report a case of a 67-year-old woman with a bipolar affective disorder who was treated with lithium for 10 years. Under concomitant renal insufficiency and urinary tract infarction, she experienced progressive hand tremor, bradykinesia, and unsteady gait. Laboratory results revealed hypercalcemia and hypermagnesiemia. A high serum lithium level (3.6 mEq/L) was found; thus lithium was discontinued. She was found to have a high serum level of intact parathyroid hormone: 135.0 pg/ml and a suspicious parathyroid adenoma. Polyuria with hypernatremia was also noted. A water deprivation test confirmed nephrogenic diabetes insipitces. After correction of electrolyte imbalance and reduction of lithium level, her consciousness recovered. Her parkinsonian features were responsive to levodopa 400 mg/day in 2 divided doses. One month later, apart from the residual extrapyramidal symptoms and mania, her condition was otherwise stationary. Tremor is the most frequent movement disorder associated with lithium therapy, while severe parkinsonism has been rarely reported. It should be kept in mind in differential diagnosis of acute parkinsonism especially in elder patients who receive a chronic lithium carbonate therapy."}, {"id": "pubmed23n0917_11038", "title": "[Reversible alterations in the dentate nuclei and rapid-onset cerebral atrophy due to neurotoxicity caused by lithium].", "score": 0.011761914446478205, "content": "Treatment with lithium can cause several neurological side effects, even at therapeutic levels. We report the case of a 49-year-old woman, with bipolar disorder and depression, undergoing treatment with lithium, antidepressants and antipsychotics, who was admitted to hospital due to a clinical picture of visual hallucinations with an elevated lithaemia of 2.1 mEq/L (therapeutic range: 0.6-1.2 mEq/L). The patient developed a severe encephalopathy that required the use of assisted ventilation in the intensive care unit. Initial magnetic resonance imaging showed a reversible bilateral symmetrical hyperintensity in the dentate nuclei in T2 and T2-FLAIR sequences. Over the following months she gradually developed a pancerebellar syndrome with evidence of a marked loss of bilateral volume in the cerebellum, above all at the expense of the vermis, which was accompanied by a permanent and disabling cerebellar syndrome. Although treatment with lithium can cause a variety of neurological side effects, they are usually reversible. However, they occasionally give rise to permanent and disabling sequelae, as in the case of the patient reported here, with a marked and progressive cerebellar atrophy, accompanied by permanent sequelae in the form of a disabling cerebellar syndrome. The cerebellar neurotoxicity of lithium must be taken into account in the broad differential diagnosis of cerebellar ataxia in adults."}, {"id": "pubmed23n0915_18044", "title": "Use of the anion gap and intermittent hemodialysis following continuous hemodiafiltration in extremely high dose acute-on-chronic lithium poisoning: A case report.", "score": 0.011430064334933543, "content": "A 35-year-old woman intentionally took 40,000 mg of lithium carbonate, and she was transferred to our hospital with nausea, vomiting, and diarrhea. She was diagnosed as having bipolar disorder 10 years ago and was receiving oral lithium therapy. Blood test results on arrival were remarkable for a negative anion gap of -2.1 and later, the serum lithium level turned out to be as high as 15.4 mEq/L. Intubation was required because of disrupted consciousness, and continuous hemodiafiltration (CHDF) was immediately started in the intensive care unit to obtain constant removal of lithium. After adding intermittent hemodialysis (IHD) twice during the daytime to accelerate the lithium clearance, CHDF became unnecessary on day 4, and she was extubated on day 6 with complete recovery of consciousness. Close monitoring of the patient data showed recovery of the decreased anion gap as indicator of the serum lithium level reduction. On day 36, she was discharged without any complication and sequela. The current case highlighted the effective use of CHDF between IHD sessions to prevent the rebound elevation of lithium and the role of the anion gap as a surrogate marker of serum lithium concentration during the treatment."}, {"id": "wiki20220301en000_324536", "title": "Mood stabilizer", "score": 0.011349477219784387, "content": "Mineral Lithium – Lithium is the \"classic\" mood stabilizer, the first to be approved by the US FDA, and still popular in treatment. Therapeutic drug monitoring is required to ensure lithium levels remain in the therapeutic range: 0.6 or 0.8-1.2 mEq/L (or millimolar). Signs and symptoms of toxicity include nausea, vomiting, diarrhea, and ataxia. The most common side effects are lethargy and weight gain. The less common side effects of using lithium are blurred vision, a slight tremble in the hands, and a feeling of being mildly ill. In general, these side effects occur in the first few weeks after commencing lithium treatment. These symptoms can often be improved by lowering the dose."}, {"id": "wiki20220301en415_39692", "title": "Lithium (medication)", "score": 0.010789014821272886, "content": "Lithium concentrations in whole blood, plasma, serum or urine may be measured using instrumental techniques as a guide to therapy, to confirm the diagnosis in potential poisoning victims or to assist in the forensic investigation in a case of fatal overdosage. Serum lithium concentrations are usually in the range of 0.5–1.3 mmol/L (0.5–1.3 mEq/L) in well-controlled people, but may increase to 1.8–2.5 mmol/L in those who accumulate the drug over time and to 3–10 mmol/L in acute overdose. Lithium salts have a narrow therapeutic/toxic ratio, so should not be prescribed unless facilities for monitoring plasma concentrations are available. Doses are adjusted to achieve plasma concentrations of 0.4 to 1.2 mmol /L on samples taken 12 hours after the preceding dose. Given the rates of thyroid dysfunction, thyroid parameters should be checked before lithium is instituted and monitored after 3–6 months and then every 6–12 months."}, {"id": "wiki20220301en487_12081", "title": "Lithium toxicity", "score": 0.010607759587351424, "content": "Gastric lavage. A tube is placed through the nose or mouth into the stomach. The tube is used to remove lithium that has not been digested yet. It may also be used to put medicines directly into the stomach to help stop lithium from being absorbed. Use of an artificial kidney to clean the blood (dialysis). This is usually done only in the most severe cases. Diuretic medications such as furosemide and hydration via intravenous normal saline appear to be effective in speeding the removal of lithium and also rehydrate patients who've lost fluids. Hemodialysis. Hemodialysis is widely advocated as a means of reducing the risk of permanent neurological sequelae following lithium poisoning. Although hemodialysis clearly enhances the elimination of lithium, it is unclear whether this translates into improved patient outcomes. People may be sent home once their lithium level is less than 1.5 mEq/L and they have no symptoms. See also"}, {"id": "InternalMed_Harrison_3685", "title": "InternalMed_Harrison", "score": 0.010577236383687997, "content": "A stuporous 22-year-old man was admitted with a history of behaving strangely. His friends indicated he experienced recent emotional problems stemming from a failed relationship and had threatened suicide. There was a history of alcohol abuse, but his friends were unaware of recent alcohol consumption. The patient was obtunded on admission, with no evident focal neurologic deficits. The remainder of the physical examination was unremarkable. Na+ 140 meq/L K+ 5 meq/L Cl− 95 meq/L HCO3− 10 meq/L Glucose 125 mg/dL BUN 15 mg/dL Creatinine 0.9 mg/dL Ionized calcium 4.0 mg/dL Plasma osmolality 325 mOsm kg/H2O Urinalysis revealed crystalluria, with a mixture of envelope-shaped and needle-shaped crystals."}, {"id": "Neurology_Adams_9710", "title": "Neurology_Adams", "score": 0.010146056015667398, "content": "With blood levels of lithium in the upper therapeutic range (therapeutic 0.6 to 1.2 mEq/L), it is not uncommon to observe a fast-frequency action tremor or asterixis, together with nausea, loose stools, fatigue, polydipsia, and polyuria. These symptoms usually subside with time. Above a level of 1.5 to 2 mEq/L, particularly in patients with impaired renal function or in those taking a thiazide diuretic, serious intoxication becomes manifest—clouding of consciousness, confusion, delirium, dizziness, nystagmus, ataxia, stammering, diffuse myoclonic twitching, and nephrogenic diabetes insipidus. Vertical (downbeating) nystagmus and opsoclonus (see Chap. 13) may also be prominent. A variety of skin problems is common including worsening of acne vulgaris. An uncommon toxic effect is the development of goiter but most patients remain euthyroid although the thyroid-stimulating hormone (TSH) levels may increase slightly. The goiter usually requires no treatment but it is possible to"}, {"id": "Neurology_Adams_12202", "title": "Neurology_Adams", "score": 0.01009266720386785, "content": "becomes effective, usually a matter of 4 or 5 days. The usual dosage of lithium is 1,200 to 2,400 mg daily in divided oral doses, which produces a desired serum level of 0.9 to 1.4 mEq/L. The serum level of lithium must be checked frequently, both to ensure that a therapeutic dose is being taken and to guard against toxicity (see later)."}, {"id": "InternalMed_Harrison_31726", "title": "InternalMed_Harrison", "score": 0.010035035035035035, "content": "In the treatment of acute mania, lithium is initiated at 300 mg bid or tid, and the dose is then increased by 300 mg every 2–3 days to achieve blood levels of 0.8–1.2 meq/L. Because the therapeutic effect of lithium may not appear until after 7–10 days of treatment, adjunctive usage of lorazepam (1–2 mg every 4 h) or clonazepam (0.5–1 mg every 4 h) may be beneficial to control agitation. Antipsychotics are indicated in patients with severe agitation who respond only partially to benzodiazepines. Patients using lithium should be monitored closely, since the blood levels required to achieve a therapeutic benefit are close to those associated with toxicity. Valproic acid may be better than lithium for patients who experience rapid cycling (i.e., more than four episodes a year) or who present with a mixed or dysphoric mania. Tremor and weight gain are the most common side effects; hepatotoxicity and pancreatitis are rare toxicities."}, {"id": "InternalMed_Harrison_3656", "title": "InternalMed_Harrison", "score": 0.009956492637215528, "content": "A 32-year-old man was admitted to the hospital with weakness and hypokalemia. The patient had been very healthy until 2 months previously when he developed intermittent leg weakness. His review of systems was otherwise negative. He denied drug or laxative abuse and was on no medications. Past medical history was unremarkable, with no history of neuromuscular disease. Family history was notable for a sister with thyroid disease. Physical examination was notable only for reduced deep tendon reflexes. Sodium 139 143 meq/L Potassium 2.0 3.8 meq/L Chloride 105 107 meq/L Bicarbonate 26 29 meq/L BUN 11 16 mg/dL Creatinine 0.6 1.0 mg/dL Calcium 8.8 8.8 mg/dL Phosphate 1.2 mg/dL Albumin 3.8 meq/L TSH 0.08 μIU/L (normal 0.2–5.39) Free T4 41 pmol/L (normal 10–27)"}, {"id": "pubmed23n0035_294", "title": "[Origin and treatment of the hypokalemic paresis (author's transl)].", "score": 0.009900990099009901, "content": "The abuse of laxatives and a prolonged treatment with diuretics has to be brought into consideration as the most common cause for renal or intestinal loss of potassium. Besides characteristical alterations at the E.C.G. and besides intestinal disturbances there do occur again and again acute, life-threatening aspects of cases connected with tetraplegias and a respiratory failure. By means of 3 cases from our hospital and 27 casuistics in literature symptomatology and dynamic in the development of hypokaliemia is discussed. The mean of potassium in the serum of the 16 patients, those having quadriplegias, ran up to 1,7 mval/l (range 1,4-2,5). Paralysis develops peracutely in 4 of the cases within hours and in 12 of the cases within days. In the anamnesis symptoms of adynamia could be traced with nearly every patient. A functional psychosis (reversible physically founded psychosis) couldn't be detected in any of the cases. With the help of a administration of potassium one could achieve a total retrogression of the symptoms. Besides this a normalisation of the acide-base equilibrium is required because of a metabolic alkalosis, detectable in most of the cases."}, {"id": "pubmed23n0422_8094", "title": "[Aetiologies of lithium overdose: 10-year experience of Marseille poison centre].", "score": 0.00980392156862745, "content": "Lithium is used for control of bipolar disorders. In order to precise the different circumstances at the origin of poisonings, the authors present the cases of lithium intoxication observed in the Marseille poison centre between January 1991 and December 2000. Retrospective study. Three hundred and four cases were observed during the studied period (1 patient a case), concerning 6 different circumstances. For 3 of them, the symptoms were mild: accidental ingestion with children (13 cases); mistakes on the quantities of ingested tablets (43 cases); elevation of lithium blood level due to diuretic therapy (8 cases). For 2 other circumstances, the clinical signs were more severe: treated patients who developed renal failure (15 cases, 6 patients managed in intensive care unit [ICU], 1 death) or dehydration (35 cases, 8 patients treated in ICU and 1 death). Finally, the most severe cases were collected with suicide attempts. Fifty-six percent of the patients were managed in ICU, 5% needed haemodialysis, 10% had cardiac (repolarization disturbances) or neurological (seizures) complications, 2% died. The severity of lithium poisonings depends of the circumstances. Ingestion of high quantities of sustained released tablets is the most dangerous situation. Accidental ingestion, even with children, must be considered as less severe situations."}, {"id": "Neurology_Adams_9712", "title": "Neurology_Adams", "score": 0.009769961808331113, "content": "The myoclonic state, particularly when combined with confusion and sharp waves in the EEG, may mimic Creutzfeldt-Jakob disease (see Chap. 32) but there should be no problem in diagnosis if the setting of the illness and the administration of lithium are known. At blood lithium concentrations above 3.5 mEq/L, these symptoms are replaced by stupor and coma, sometimes with convulsions, and may prove fatal. Discontinuing lithium in the intoxicated patient, which is the initial step in therapy, does not result in immediate disappearance of toxic symptoms. This may be delayed by a week or two, and the diabetes insipidus may persist even longer. Fluids, sodium chloride, aminophylline, and acetazolamide promote the excretion of lithium. Lithium coma may require hemodialysis, which has proved to be the most rapid means of reducing the blood lithium concentration."}, {"id": "wiki20220301en000_287929", "title": "Lithium carbonate", "score": 0.009708737864077669, "content": "Lithium carbonate is used as a psychiatric medication to treat mania, the elevated phase of bipolar disorder. Prescription lithium carbonate from a pharmacy is suitable for use as medicine in humans while industrial lithium carbonate is not since the latter may, for example, contain unsafe levels of toxic heavy metals or other toxicants. After ingestion, lithium carbonate is dissociated into pharmacologically active lithium ions (Li+) and (non-therapeutic) carbonate, with 300 mg of lithium carbonate containing approximately 8 mEq (8 mmol) of lithium ion. According to the Food and Drug Administration (FDA), 300–600 mg of lithium carbonate taken two to three times daily is typical for maintenance of bipolar I disorder in adults, where the exact dose given varies depending on factors such as the patient's serum lithium concentrations, which must be monitored by a physician to avoid lithium toxicity and potential kidney damage (or even failure) from lithium-induced nephrogenic diabetes"}, {"id": "article-24349_30", "title": "Lithium -- Toxicity -- Recommendations", "score": 0.009705540488182875, "content": "The guidelines established by EXtracorporeal TReatments In Poisoning (EXTRIP) are listed below. Initiate extracorporeal treatment for patients with severe lithium poisoning presenting with coma, myoclonus, convulsions, or cardiopulmonary collapse. Initiate extracorporeal treatment when impaired kidney function is evident, and the lithium concentration surpasses 4 mEq/L. Hemodialysis is also indicated in patients with altered consciousness, seizures, or life-threatening dysrhythmias, regardless of the lithium concentration. Consider extracorporeal treatment if the lithium concentration exceeds 5 mEq/L, significant confusion is evident, or the projected duration for reducing the lithium concentration below 1 mEq/L extends beyond 36 hours. Continue extracorporeal treatment until clinical improvement or lithium concentration drops below 1 mEq/L. In cases where lithium concentration measurement is unattainable, maintain extracorporeal therapies for at least 6 hours."}, {"id": "wiki20220301en487_12075", "title": "Lithium toxicity", "score": 0.009641537774530102, "content": "Lithium toxicity, also known as lithium overdose, is the condition of having too much lithium. Symptoms may include a tremor, increased reflexes, trouble walking, kidney problems, and an altered level of consciousness. Some symptoms may last for a year after levels return to normal. Complications may include serotonin syndrome. Lithium toxicity can occur due to excessive intake or decreased excretion. Excessive intake may be either a suicide attempt or accidental. Decreased excretion may occur as a result of dehydration such as from vomiting or diarrhea, a low sodium diet, or from kidney problems. The diagnosis is generally based on symptoms and supported by a lithium level of greater than 1.2 mEq/L."}, {"id": "pubmed23n0802_12127", "title": "Case report on lithium intoxication with normal lithium levels.", "score": 0.009615384615384616, "content": "An 18-year old female was admitted to a psychiatric hospital with an initial episode of mania. Treated with routine dosages of lithium bicarbonate, her symptoms resolved after two weeks; she was discharged on a dosage of 250mg lithium bid. Five days after discharge she was taken to the emergency department of a general hospital with loss of appetite and disturbed consciousness. The general hospital physicians were unable to diagnose the problem so she was transferred back to the psychiatric hospital. At that time she had a lithium blood level of 0.57 mmol/L (i.e., at the lower end of the therapeutic range) but was, nevertheless, clearly experiencing lithium intoxication with anuria, trembling extremities, blurred speech, muscle rigidity and hyperactive tendon reflexes. Treated with intravenous mannitol, her acute symptoms resolved quickly. The case highlights the need to monitor clinical symptoms of intoxication in all patients taking lithium, regardless of their blood level, and to inform patients, family members, and general physicians about the symptoms and management of lithium intoxication."}, {"id": "pubmed23n0607_14684", "title": "[Analysis of the poisonings by lithium in a department of internal medicine].", "score": 0.009345794392523364, "content": "Lithium salts have been mainly used in the treatment of bipolar disorder. Because of its narrow therapeutic range, and several well characterised adverse effects, serum lithium levels must be monitored regularly in patients given lithium treatment in order to prevent intoxication. To describe the clinic and toxic characteristics in inpatients at our Clinic Toxicologic Unit. Descriptive and retrospective study of lithium intoxications in 150 inpatients between 2003 and 2006. Patients were classified based on their neuropsychiathric symptom profile and serum lithium levels. Sixteen of 150 inpatients had lithium intoxication: 58.3% women and 43.8% men; 49.19% +/- 18.49% years old. Lithium was used as treatment of bipolar disorder in 87.5% of cases. The most frequent cause of intoxication was attempted suicide. Using neuropsychiatric parametres, intoxication was moderate in 50% of cases, and mild in 25% and severe or very severe in 25%. Using serum lithium levels, intoxication was very severe in 31.35%, severe in 25%, and slight-moderate in 43.7%. Conservative measures were used as the most frequent treatment (50%), and haemodialfiltration was needed in 37.5%. Mean stay was 4,8 days in acute intoxication, and 11.2 days in chronic. Sequelaes were found in two patients (ataxia). Death was not present. Lithium intoxications can involve severe complications, even death. Narrow control is encouraged in polymedicated and elderly patients, and in concommitant treatment with antidepressant and neuroleptics."}, {"id": "pubmed23n1041_9231", "title": "Lithium toxicity with prolonged neurologic sequelae following sleeve gastrectomy: A case report and review of literature.", "score": 0.009259259259259259, "content": "Lithium is the first-line medication for bipolar disorder, given a narrow therapeutic window of 0.8 to 1.2 mEq/L. Change of lithium pharmacokinetics following bariatric surgery may lead to lithium toxicity, which is particularly concerned. We presented a 39-year-old man with morbid obesity and bipolar affective disorder for 20 years, who was treated with lithium. He developed serious lithium toxicity following sleeve gastrectomy and prolonged neurologic sequelae. He suffered from persistent watery diarrhea, general weakness, and then drowsy consciousness. Lithium level was checked immediately to be 3.42 mEq/L and lithium toxicity was diagnosed. After 3 courses of hemodialysis, his serum lithium level subsequently declined to 0.63 mEq/L, while his consciousness returned normal. Lithium was replaced by lamotrigine. The patient was discharged thirty-five days after admission, while his serum lithium declined to 0.06 mEq/L. Neurologic sequelae were noted by muscle weakness and pain sensation in both feet. The nerve conduction test revealed sensorimotor polyneuropathy with conduction block. He was advised to keep a passive range of motion exercise. Although the consensus guideline remains lacking, our report reviewed cases of relevance in the literature and highlighted the awareness of the potential risk of lithium toxicity following bariatric surgery. We suggest close monitoring of the lithium levels and perhaps a dosage adjustment for the postoperative period."}]}}}} {"correct_option": 1, "explanations": {"1": {"exist": true, "char_ranges": [[0, 221]], "word_ranges": [[0, 33]], "text": "There is no doubt, they are not meeting the diagnostic criteria of Kawasaki disease (fever of several days of evolution, exanthema, conjunctival injection, red lips, raspberry tongue, edema of acral parts and adenopathy)."}, "2": {"exist": false, "char_ranges": [], "word_ranges": [], "text": ""}, "3": {"exist": false, "char_ranges": [], "word_ranges": [], "text": ""}, "4": {"exist": false, "char_ranges": [], "word_ranges": [], "text": ""}, "5": {"exist": false, "char_ranges": [], "word_ranges": [], "text": ""}}, "full_answer": "There is no doubt, they are not meeting the diagnostic criteria of Kawasaki disease (fever of several days of evolution, exanthema, conjunctival injection, red lips, raspberry tongue, edema of acral parts and adenopathy).", "full_answer_no_ref": "There is no doubt, they are not meeting the diagnostic criteria of Kawasaki disease (fever of several days of evolution, exanthema, conjunctival injection, red lips, raspberry tongue, edema of acral parts and adenopathy).", "full_question": "A 4-year-old girl presenting with a high fever of 6 days' evolution. On clinical examination she presents an erythematous maculopapular rash on the trunk and genital area, with a tendency to confluence, without becoming scarlatiniform; conjunctival injection without secretions and red lips with raspberry tongue. She also presents erythema with edema in hands and feet and a unilateral cervical adenopathy of 2 cm in diameter. The most likely clinical diagnosis of suspicion is:", "id": 552, "lang": "en", "options": {"1": "Kawasaki disease.", "2": "Measles.", "3": "Rubella.", "4": "Scarlet fever.", "5": NaN}, "question_id_specific": 89, "type": "DERMATOLOGY", "year": 2022, "rag": {"clinical_case_options": {"MedCorp": {"RRF-2": [{"id": "pubmed23n0735_10590", "title": "Catastrophic Kawasaki disease unresponsive to IVIG in a 3-month-old infant: a diagnostic and therapeutic challenge.", "score": 0.018596642272922945, "content": "The present report describes the severe evolution of Kawasaki disease in a three-month-old infant. The ailment was initially atypical in its presentation, with the patient exhibiting only persistent fever in association with a progressive lethargy and maculopapular rash on the face, trunk and limbs erroneously diagnosed as roseola infantum. On the 10th day of the condition, mainly due to the unexplained persistence of fever, the infant was admitted to a local hospital. The typical features of KD appeared only on the 14th day of illness with the relapse of the maculopapular rash in association with non-purulent conjunctivitis; dry, reddish and fissured lips; tongue with reddish and hypertrophic papillae; erythema and edema of the palms and soles. During the following days, the ailment rapidly evolved to a catastrophic clinical picture characterized by generalized vasculitis, splenic infarction, pulmonary thrombosis, giant right and left coronary aneurysms, dilatation of common and internal iliac arteries and progressive ischemia of the distal third of the feet resulting in necrotic lesions of both halluces. Appropriate therapy was initiated, but repeated administration of intravenous immunoglobulin G (IVIG) followed by three days of administration of methylprednisolone did not abate the intense inflammatory activity. The remission of inflammation and regression of vascular lesions were only achieved during the following five weeks after the introduction of methotrexate associated with etanercept. The report of this case aims to draw attention to severe forms of KD that exhibit an unfavorable evolution and can be extremely refractory to the conventional therapy."}, {"id": "pubmed23n0985_24440", "title": "Late Treatment and Recurrence of Kawasaki Disease in a Moroccan Infant.", "score": 0.017862838915470493, "content": "While the diagnosis of typical form of Kawasaki disease (KD) is obvious, this multifaceted disease continues to surprise us. We report the case of a recurrent Kawasaki disease in an infant. At the age of 13 months, the infant was diagnosed with complete Kawasaki disease; he presented with prolonged fever, bilateral conjunctivitis, enanthem, exanthema, edema of the lower limb, peeling, and biological inflammatory syndrome. He was treated with intravenous immunoglobulin (IVIG) associated with a high dose of aspirin and then an antiplatelet dose with a good clinical-biological evolution. The echocardiography was normal. Seven months later, the patient was again admitted, in a similar picture: a prolonged fever evolving for 7 days, bilateral conjunctivitis, enanthem, cervical adenopathy of 1.5 cm/1 cm, scarlatiniform erythema, pruriginous of the trunk and limb, and peeling of the toes, with indurated edema of the hands and feet. The rest of the examination was normal except the irritability. The diagnosis of recurrent KD was made according the five criteria of the American Heart Association. The echocardiography was normal again. The infant received IVIG with good outcome. Despite its rarity, the possibility of recurrence of KD should be known by clinicians, so as not to delay the specific management of vasculitis whose stakes in terms of prevention of coronary artery lesions are well known. Our case confirms the possibility of this recurrence."}, {"id": "InternalMed_Harrison_4123", "title": "InternalMed_Harrison", "score": 0.017588325652841783, "content": "The cutaneous eruption in Kawasaki disease (Chap. 385) is polymorphous, but the two most common forms are morbilliform and scarlatiniform. Additional mucocutaneous findings include bilateral conjunctival injection; erythema and edema of the hands and feet followed by desquamation; and diffuse erythema of the oropharynx, red strawberry tongue, and dry fissured lips. This clinical picture can resemble TSS and scarlet fever, but clues to the diagnosis of Kawasaki disease are cervical lymphadenopathy, cheilitis, and thrombocytosis. The most serious associated systemic finding in this disease is coronary aneurysms secondary to arteritis. Scarlatiniform eruptions are also seen in the early phase of SSSS (see “Vesicles/Bullae,” above), in young adults with Arcanobacterium haemolyticum infection, and as reactions to drugs."}, {"id": "pubmed23n0271_3624", "title": "[Acute renal insufficiency in Kawasaki disease].", "score": 0.016865079365079364, "content": "Kawasaki disease is an acute inflammatory condition characterized by various combinations of features but renal involvement is rare. This report is of a case of Kawasaki disease complicated by acute kidney failure. A 10 year-old girl was admitted because of acute renal failure with fever. She developed a high fever, and her general condition was poor; she had developed a macular erythematous rash 10 days earlier for which she was given cefadroxil. At admission, she remained febrile and had strawberry tongue, pharyngitis, dry erythematous lips, bilateral conjunctivitis, cervical lymphadenopathy and desquamation of the skin on her hands. She was anemic (hemoglobin = 9.6 g%), leukocytotic (33,100/mm3), but with no burr, fragmented red blood cells or thrombocytopenia. Her plasma C-reactive protein level was 236 mg/l; her blood urea was 9.5 mmol/l, her creatininemia 288 mumol/l and proteinuria was 0.5 g/l without hematuria. Urine cultures did not grow. Her blood transaminase and gammaglutamyltransferase activities were elevated. Ultrasonography of kidneys and coronary arteries was normal. Kidney biopsy performed one day after admission showed no vascular or glomerular changes, but renal tubular necrosis, indicating urinary excretion of pigments. Tests for myoglobinemia, myoglobinuria and blood muscle enzyme activities were all positive. The renal failure disappeared within 10 days but the fever and inflammatory manifestations persisted for 1 1/2-2 months despite two treatments of intravenous gammaglobulins and continuous salicylate administration. The patient developed arthralgias at the end of the first month of disease, but recovered without renal or vascular complications. Several cases of renal involvement have been reported during the course of Kawasaki disease. They have been rarely documented by histological examination so that the vascular origin of changes has not been demonstrated. Myoglobinuria, as seen in muscular crush injury, and in our case possibly due to malignant hyperthermia, may be responsible for the transient acute renal failure."}, {"id": "article-23847_9", "title": "Kawasaki Disease -- History and Physical", "score": 0.016260162601626018, "content": "In 2014, the American Heart Association (AHA) published the criteria needed to establish a diagnosis. [1] However, it is important to note that children who fall short of the full criteria but have cardiac abnormalities on echocardiogram meet the diagnosis of KD. [1] The patient must have fevers for five or more days, with at least four of the following criteria (either all at once or over a series of days): Bilateral painless bulbar conjunctival injection without exudate Erythematous mouth and pharynx, strawberry tongue or red, cracked lips Polymorphous exanthem (morbilliform, maculopapular, or scarlatiniform) Swelling of hands and feet with erythema of the palms and soles Cervical lymphadenopathy (over 1.5 cm in diameter)"}, {"id": "pubmed23n0482_11745", "title": "An unusual pattern of arthritis in a child with Kawasaki syndrome.", "score": 0.015617403986387944, "content": "Arthritis is reported in one-third of cases with Kawasaki syndrome. It may have an early or a late onset form. We present a 15-month-old-girl who had been referred with complaints of pain and swelling in her left shoulder. Physical examination revealed bulbar conjunctival injection, erythematous lips and pharynx, strawberry tongue, erythematous rash, edema and erythema of the left shoulder, left knee, right elbow and right wrist, and moderate distress in the left shoulder and left hip. She was diagnosed with Kawasaki syndrome, and intravenous immunoglobulin infusion (IVIG) 2 g/kg and aspirin (100 mg/kg/day) were instituted. The patient had two additional episodes of arthritis involving the hip joint on the 8th day, and the shoulder and metacarpophalangeal (MCP) and interphalangeal (IP) joints of her right hand on the 15th day. Turbid material was aspirated in both instances; Gram and Wright's staining of this material showed many leukocytes but no bacteria. A second dose of IVIG (1 g/kg) was given. At the end of the third week all extremities were painless, with a normal range of motion. Arthritis in our patient was the presenting sign, having a 'septic arthritis mimicking' and 'biphasic' pattern. Although the patient presented with severe and recurrent arthritis, which is significantly correlated with severe multisystem disease and the presence or development of coronary artery aneurysm, the response to IVIG was excellent."}, {"id": "pubmed23n1039_17116", "title": "A Case of Kawasaki Disease with Intussusception.", "score": 0.015503875968992248, "content": "Kawasaki disease (KD) is a systemic vasculitis of unknown cause and is associated with various digestive disorders, although only a few cases of intussusception associated with KD have been reported. We describe a case of intussusception followed by KD in a 3-year-old boy. The patient was admitted to our hospital for evaluation of severe abdominal pain. Because the target sign was seen on ultrasonography, intussusception was diagnosed and hydrostatic reduction was performed. On the second day after admission, he developed a high fever (38°C) and an irregular rash over his whole body. On the fourth day after admission, the high fever continued, and bilateral nonexudative conjunctivitis, erythema of the lips and oral mucosa, strawberry tongue, indurated edema of the dorsa of the hands and feet, and diffuse erythema of the palms and soles appeared, and KD was ultimately diagnosed. He was treated with intravenous immunoglobulin 2 g/kg, aspirin 30 mg/kg/day, and prednisolone 2 mg/kg/day. The high fever and other clinical symptoms resolved immediately after the start of treatment. There was no relapse of KD symptoms after initial treatment, and periungual desquamation was observed on the 10th day after admission. He was discharged on the 15th day, without abnormalities such as coronary dilatation, 3 months after the onset of KD symptoms. Patients with intussusception and KD were older (≥3 years vs <3 years) than those with intussusception alone. In addition, the site of intussusception in KD was mainly colonic rather than ileocolic. If intussusception precedes development of the characteristic clinical symptoms of KD, diagnosis of KD may be delayed. KD should be considered in children older than 3 years with intussusception at a colonic site."}, {"id": "pubmed23n0327_56", "title": "Mediastinal lymphadenopathy: a variant of incomplete Kawasaki disease.", "score": 0.01501052398360474, "content": "A 14-month-old girl presented with a 4-d history of fever and generalized exanthema. Four characteristic symptoms of incomplete Kawasaki disease (KD) were present on admission (fever, rash, non-purulent conjunctival injection, oropharyngeal changes) and then followed by oedema of the hands and feet and mild plantar desquamation. The typical laboratory features of KD, such as elevated erythrocyte sedimentation rate, leukocytosis, thrombocytosis, and positive C-reactive protein were also seen. Ultrasound examination of the mediastinum revealed the presence of a lymph node, 30 mm in diameter, below the tracheal carina. Thoracic CT scan confirmed the mediastinal lymph node. The patient was treated with aspirin and intravenous gamma-globulin. Ultrasound study of the mediastinum, which was carried out 6 weeks after hospital discharge, showed that the lymph node had disappeared. This case illustrates that lymph nodes other than cervical lymphadenopathy should be sought when the diagnosis of classical or atypical KD is suspected."}, {"id": "pubmed23n0723_13682", "title": "Unusual manifestations of Kawasaki disease with retropharyngeal edema and shock syndrome in a Taiwanese child.", "score": 0.014988265414977598, "content": "We report a 3-year-old girl with Kawasaki disease who presented with retropharyngeal edema and shock syndrome. This is the first reported case in Taiwan. The patient initially presented with fever, cough, and pyuria followed by rapidly progressive enlarged bilateral cervical lymphadenopathy. On the third day of the fever, computed tomography for airway compression sign found widening of the retropharyngeal space mimicking a retropharyngeal abscess. Later, an endotracheal tube was inserted for respiratory distress. A skin rash over her trunk was also noted. On the fifth day of the fever, the clinical course progressed to hypotension and shock syndrome. Because of more swelling of bilateral neck lymph nodes, computed tomography was arranged again and revealed partial resolution of the edematous changes in the retropharyngeal space. Edema of the hands and feet, bilateral bulbar conjunctivitis, and fissured lips were subsequently found. The diagnosis of Kawasaki disease was confirmed on the eighth day of fever. There was no evidence of bacterial infection. She was administered intravenous immunoglobulin (2 mg/kg) and high dose aspirin (100 mg/kg/day). One day later, the fever subsided, and her blood pressure gradually became stable. Heart echocardiography on the Day 13 revealed dilated left coronary artery and mitral regurgitation. Follow-up echocardiography six months later showed normal coronary arteries. To date, the patient has not experienced any complications. This case illustrates that retropharyngeal edema and shock syndrome can be present in the same clinical course of Kawasaki disease. Clinicians and those who work in intensive care units should be aware of unusual presentations of Kawasaki disease to decrease rates of cardiovascular complications. "}, {"id": "pubmed23n0631_10241", "title": "[A case report of acute Q fever showing Kawasaki disease-like symptoms in a 9-year-old girl].", "score": 0.014983164983164984, "content": "A 9-year-old girl developing fever and hyperemia of both bulbar conjunctiva 5 days before admission to the Saitama Children's Medical Center after antibiotics proved ineffective was found on admission to have general fatigue and a temperature of 39 degrees C. Physical examination showed hyperemia of the bulbar conjunctiva, fissures of the lips, redness of the pharynx, and swelling of the cervical lymph nodes. Laboratory tests detected neutrophilia (11,200/microL), mild anemia (11.4g/dL), thrombocytopenia (110,000/microL), and elevated serum aspartate aminotransferase (242IU/L), alanine aminotransferase (328IU/L), and C-rective protein (25.2 mg/dL). Autoantibodies such as anti-nuclear, anti-SS-A/Ro, and anti-Jo-1 were also found. Echocardiography showed no abnormality of the coronary arteries. She was diagnosed as having incomplete Kawasaki disease on day 7 of illness, necessitating that a high dose of immunoglobulin be given intravenously. Her temperature dropped temporarily to 37 degrees C, but she developed erythema of the cheek and fever. Intravenous immunoglobulin was restarted, and minocycline introduced because her daily contact with a pet cat indicated richettsial infection such as Q fever. Mild fever, muscle pain, and elevated C-reactive protein did not improve, but clinical signs and symptoms gradually lessened after ibuprofen was given, then disappeared. A definitive diagnosis of Q fever was made through an over 4-fold rise in phase II IgG antibody titers against Coxiella burnetii, titer of less than 1 : 16 on day 14 of illness, and titer of 1 : 256 on day 34. This case study describes on atypical case of Q fever with clinical manifestations mimicking Kawasaki disease."}, {"id": "pubmed23n1010_25280", "title": "Uncommon erythema multiforme in small children: experience of a single Romanian pediatric unit: Two case reports.", "score": 0.014605543710021322, "content": "Erythema multiforme (EM) is an immune-mediated disease with mucocutaneous localization and plurietiologic determinism. The term \"multiforme\" refers to the variety of aspects that the lesions can take from patient to patient and during evolution in a single patient. We have selected 2 cases of small children diagnosed with different etiology of EM to illustrate the importance of a correct and fast diagnosis. Case 1 involves a 2-year-old girl from a rural area who presented with fever and pruritic erythematous papular eruption. The onset of the symptoms was 3 days before presentation with fever and ulcerative lesions on the oral and labial mucosa, followed by the appearance of erythematous macular lesions, with progressive confluence to intense pruritic patches. The 2nd involves a 2-year-old boy with fever, loss of appetite, productive cough, and petechiae. He had corticosensible immune thrombocytopenia from the age of 6 months, with many recurrences. The patient received treatment with ampicillin/sulbactam and symptomatics for an erythemato-pultaceous angina. During the 2nd day of treatment the patient developed an erythematous macular eruption on the face, scalp, trunk, and limbs, with bullae formation. The 1st patient was diagnosed based on biologic findings: positive inflammatory syndrome, elevated level of anti-Mycoplasma pneumoniae immunoglobulin M antibodies and immunoglobulin E. Histopathologic examination described papillary dermal edema, inflammatory infiltrate, and lymphocyte exocytosis. In the 2nd case, the hemoleucogram identified 12,000/mm platelets and the medulogram aspect was normal. Serology for Epstein-Barr virus was negative. The diagnosis was EM secondary to M pneumoniae infection in case 1 and secondary to administration of ampicillin/sulbactam in case 2. In both cases, etiopathogenic treatment consisting of steroidal antiinflammatory drugs, antihistamines was administered. Because of specific etiology, the 1st case received antibiotics. The evolution was favorable in 10 to 14 days; the patients were discharged after etiopathogenic treatment consisting of steroidal antiinflammatory drugs, antihistamines, and/or antibiotics. Performing a detailed clinical examination, medical history of drug use, infection or general diseases can establish a good diagnosis of EM. Histopathologic examination can help. The treatment is etiologic, pathogenic, and symptomatic. EM usually has a self-limited evolution."}, {"id": "pubmed23n0520_17294", "title": "Concomitant dengue infection and Kawasaki disease in an infant: a case report and literature review.", "score": 0.014150487781319424, "content": "A previously healthy 11-month-old girl presented with fever and rash for 6 days. Physical examination revealed an irritable infant with a high fever, injected conjunctivae, red cracked lips, posterior auricular lymphadenopathy, hepatomegaly, generalized erythematous maculopapular rash and petechial hemorrhage on trunk, face and extremities. Complete blood count showed atypical lymphocytosis and thrombocytopenia. Dengue infection was initially diagnosed. The persistent fever and clinical manifestations of Kawasaki disease (KD) were observed on day 8 with high erythrocyte sedimentation rate (56 mm/hr). Treatment of KD included intravenous immunoglobulin on day 9 of the illness. Desquamation of the fingers was found on day 15 of the illness. Ectasia of left coronary artery with small aneurysmal dilatation was detected by echocardiography on day 15 of the illness. Hemagglutination-inhibition test and enzyme-linked immunosorbent assay for dengue virus eventually showed a four-fold rising. According to the literature review, this is the second reported case of dengue infection concomitant with KD. The natural course of each disease may be modified and causes some difficulties in diagnosis and management."}, {"id": "First_Aid_Step2_934", "title": "First_Aid_Step2", "score": 0.014141613924050632, "content": "Cervical lymphadenopathy (often painful and unilateral, with at least one node > 1.5 cm). Diffuse mucous membrane erythema (e.g., “strawberry tongue”); dry, red, chapped lips. Erythema of the palms and soles; indurative edema of the hands and feet; late desquamation of the fingertips (in the subacute phase). Other manifestations include sterile pyuria, gallbladder hydrops, hepatitis, and arthritis. Untreated Kawasaki disease can lead to coronary aneurysms and even myocardial infarction! Conjunctivitis Rash Adenopathy Strawberry tongue Hands and feet (red, swollen, f aky skin) BURN (fever > 40°C for ≥ 5 days) Subacute phase: Begins after the abatement of fever and typically lasts for an additional 2–3 weeks. Manifestations are thrombocytosis and elevated ESR. Untreated children may begin to develop coronary artery aneurysms (40%); all patients should be assessed by echocardiography at diagnosis."}, {"id": "pubmed23n1118_16380", "title": "Nontypical presentation of a common disease: a case report.", "score": 0.014009009009009008, "content": "Kawasaki disease is an idiopathic medium-sized vasculitis that occurs primarily in infants and children younger than 5 years of age. Atypical Kawasaki disease applies to patients who do not fulfill the complete criteria of fever of 5 days or more with at least four of five features: bilateral conjunctival injection, changes in the lips and oral cavity, cervical lymphadenopathy, extremity changes, and polymorphous rash. Acute kidney injury is defined as a sudden decline in kidney function within hours, including structural injuries and loss of function. Acute kidney injury is extremely common in hospitalized pediatric patients. However, it is rarely documented in Kawasaki disease. Acute kidney injury is underestimated in Kawasaki disease due to the lack of a clear definition of age-specific normal serum creatinine levels and routine renal functions. This report describes a case who presented with clinical features suggestive of atypical Kawasaki disease and developed acute kidney injury. A 2-year-old Saudi girl had a history of high-grade fever for 5 days, moderate dehydration, dry cracked lips, poor appetite, and generalized erythematous rash; therefore, she was diagnosed to have incomplete Kawasaki disease. Laboratory investigations revealed normochromic normocytic anemia, leukocytosis, thrombocytosis, high inflammatory markers, and acute kidney injury stage III. An echocardiogram showed a 4-mm dilatation on the left main coronary artery and a 3-mm dilatation on the right. A renal biopsy was not performed to identify the cause of the injury as it showed improvements after the start of the specific therapy for Kawasaki disease; intravenous immune globulin at a dose of 2 g/kg, aspirin at a high dosage of 80 mg/kg/day, and prednisolone at 2 mg/kg. In addition to the acute kidney injury management, normal saline boluses were followed by furosemide at a 2 mg/kg dose. Her urine output increased, and her renal functions normalized. She was discharged in good condition after 10 days. It is valuable to check renal function tests in a confirmed case of Kawasaki disease to reduce the negative consequences of late acute kidney injury discovery. Early detection and intervention make a substantial difference in acute kidney injury management."}, {"id": "article-83816_25", "title": "Rubeola (Measles) -- Differential Diagnosis", "score": 0.014009009009009008, "content": "Measles should be distinguished from similar presenting exanthemic diseases of childhood, autoimmune processes, and adverse drug reactions. Rubella causes a rash similar to measles with head to caudal distribution, mild respiratory symptoms, the absence of conjunctivitis. Still, it is accompanied by the presence of adenopathies - which is characteristic of this disease. Roseola is characterized by an illness beginning with a high fever, which subsides after a few days, accompanied by the appearance of a rash in the central part of the body, without the presence of Koplik's points. Mononucleosis is a febrile viral disease, a characteristic course with few symptoms during childhood, contrary to what happens in more advanced ages.  Mononucleosis manifests itself by pharyngeal compromise, polyadenopathy, and hepatosplenomegaly, and the rash can have different forms of presentation. In Kawasaki disease, there is an ocular compromise with the presence of conjunctivitis without exudate, and the respiratory compromise is not part of this pathology. Group A Streptococcus (particularly Scarlet fever) may present with a similar rash (a coarse, sandpaper-like, blanching, erythematous) to measles in association with pharyngitis. [5] [14]"}, {"id": "pubmed23n0516_19231", "title": "Kawasaki disease.", "score": 0.01399703374119392, "content": "Kawasaki disease (KD) is a common vasculitic disorder usually seen in children below 5 years of age. The disease can present with protean clinical manifestations which include high grade fever (for at least 5 days), rash, redness of the lips and a typical strawberry tongue, cervical lymph node enlargement (often unilateral), swelling over the hands/feet and, later a characteristic peripheral desquamation over the fingers and toes. These clinical features appear sequentially and the findings may change from day-to-day. Thus, all these features may not be seen together at any one point of time. The diagnosis rests on the recognition of this characteristic temporal sequence of clinical events, none of which are, by themselves, pathognomonic. Establishing a diagnosis of KD may be further complicated by the occurrence of several other, seemingly unrelated, clinical features. These include irritability, neck stiffness, sterile pyuria, pneumonitis, hydrops of the gallbladder and hepatitis among many others. There is no laboratory test that can help in confirming a diagnosis of KD. Left untreated, up to 20% of children with KD can develop coronary aneurysms with catastrophic long term sequelae. It is important to diagnose KD in the first 10 days of the illness so that appropriate therapy with intravenous immunoglobulin and aspirin can be Initiated. All paediatricians, and physicians looking after children, need to be aware of this condition which is now being increasingly recognized in India."}, {"id": "pubmed23n0048_5013", "title": "Kawasaki syndrome.", "score": 0.01369047619047619, "content": "Kawasaki syndrome, also known as mucocutaneous lymph node syndrome, is an acute vasculitis of infants and young children. We describe a four-year-old girl who presented with fever, a diffuse erythematous maculopapular rash, bilateral nonpurulent bulbar conjunctivitis, dry, red, fissured lips, a tongue with a strawberry \"appearance\", an erythematous pharynx, indurative erythema, and edema and desquamation of the face, hands and feet. She probably developed mitral valve prolapse during the course of the disease. The diagnosis of Kawasaki syndrome was arrived at by excluding other diseases and by the presence of all the clinical criteria for Kawasaki syndrome. Since this syndrome is rarely encountered in Turkey, this case is presented and the literature regarding the syndrome is reviewed."}, {"id": "wiki20220301en033_29253", "title": "Kawasaki disease", "score": 0.013599788447735257, "content": "Classically, five days of fever plus four of five diagnostic criteria must be met to establish the diagnosis. The criteria are: erythema of the lips or oral cavity or cracking of the lips rash on the trunk swelling or erythema of the hands or feet red eyes (conjunctival injection) swollen lymph node in the neck of at least 15 mm Many children, especially infants, eventually diagnosed with Kawasaki disease, do not exhibit all of the above criteria. In fact, many experts now recommend treating for Kawasaki disease even if only three days of fever have passed and at least three diagnostic criteria are present, especially if other tests reveal abnormalities consistent with Kawasaki disease. In addition, the diagnosis can be made purely by the detection of coronary artery aneurysms in the proper clinical setting. Investigations A physical examination will demonstrate many of the features listed above."}, {"id": "pubmed23n0245_6121", "title": "Kawasaki disease in healthy young adult.", "score": 0.013448969331322273, "content": "This report describes a 26-year-old woman who fulfills the criteria for the diagnosis of Kawasaki disease or mucocutaneous lymph node syndrome, an acute febrile illness that usually afflicts young children. The diagnosis is made in persons with fever lasting 5 or more days when four of the following criteria are met: bilateral injection of ocular conjunctivae; the involvement of the mucous membranes of the upper respiratory tract consisting of any combination of the following--redness and fissuring of lips; \"strawberry tongue,\" or erythema of the pharynx; involvement of the peripheral extremities characterized in the early stages by an indurative erythematous rash of palms and soles followed by membranous desquamation; polymorphous nonvesicular truncal exanthem; and acute nonsuppurative enlargement of cervical lymph nodes. An added stipulation is that the illness must not be attributable to a known disease process."}, {"id": "pubmed23n0118_16510", "title": "Kawasaki disease in a 4-year-old boy.", "score": 0.01331028522039758, "content": "A 4-year-old boy experienced sudden fever, followed by a rash on the trunk and extremities and erythema of the pharynx. Five days later, the fever remained and erythema appeared on the oropharynx, tongue, and lips. The skin of the palms and soles became erythematous and indurated, and both conjunctivae became injected. Desquamation of the skin occurred on both thumbs and one finger, and an anterior cervical lymph node was found to be enlarged. The patient was diagnosed as having Kawasaki disease, and treatment with aspirin was started. The desquamation progressed to involve the entire surface of the palms and soles, and then symptoms resolved. Twenty years after recognition of Kawasaki disease, this enigmatic illness continues to defy attempts to understand its etiology and pathogenesis. Most experts agree that the cause is either an environmental toxin or an infectious agent, but other possible causative agents may need to be proposed and investigated."}, {"id": "pubmed23n1001_25845", "title": "Twenty-year-old woman presenting with typical Kawasaki disease.", "score": 0.013254843865736228, "content": "We describe adult-onset Kawasaki disease (KD) and review clinical manifestations and treatment guidelines. Our patient is a 20-year-old female who initially presented to an outside hospital for fever, cervical lymphadenopathy, malaise, exudative tonsillitis, and skin eruption. She received antibiotics for suspected exudative pharyngitis, but experienced continued fevers and presented to the UCLA emergency room one week later. She had diffuse petechial macules coalescing into reticulated patches, fingertip peeling, conjunctival injection, oral erosions, and tongue swelling. Despite her age, given her constellation of symptoms, a diagnosis of typical KD was favored. She was started on high dose aspirin and IVIG, with improvement of rash and conjunctivitis. She was discharged on 325mg of aspirin daily with close follow-up. This case highlights the challenge of diagnosing KD in adults. Although this patient had classic symptoms, she was likely misdiagnosed because KD is rare in adults and without validated criteria. Our patient met the pediatric criteria, suggesting these should be considered when clinical suspicion for adult-onset KD is high. Adult-onset KD is most commonly misdiagnosed as toxic shock syndrome or drug-induced hypersensitivity syndrome and these are important to rule-out. Treatment with high-dose aspirin and IVIG is well established and should be initiated promptly."}, {"id": "pubmed23n0694_19426", "title": "An adult case of kawasaki disease in a pregnant Japanese woman: a case report.", "score": 0.013141025641025641, "content": "Kawasaki disease is an acute febrile disease predominantly seen in young children. We report a case of Kawasaki disease in a 32-year-old pregnant woman. She developed a generalized erythematous skin rash accompanied by high fever. Bilateral conjunctival congestion, tender cervical lymphadenopathy, an edematous lower lip and peripheral edema followed by desquamation were observed. She was successfully treated with aspirin and intravenous gammaglobulin (1 g/kg/day). Her course was not complicated by coronary artery aneurysm and she delivered a healthy baby. To the best of our knowledge, this is the first case of Kawasaki disease in a pregnant woman. We suggest that Kawasaki disease should be included in the differential diagnosis of a generalized, erythematous skin rash accompanied by high fever in adults."}, {"id": "wiki20220301en249_18856", "title": "Systemic vasculitis", "score": 0.013075592227841364, "content": "Kawasaki disease. Usually in children (age<4), it affects large, medium, and small vessels, prominently the coronary arteries. Associated with a mucocutaneous lymph node syndrome. Diagnosis requires fever lasting five days or more with at least four out of five criteria: Bilateral conjunctival injection Injected or fissured lips, injected pharynx, or strawberry tongue Erythema of palms/soles, edema of hands/feet, periungual desquamation Polymorphous rash Cervical lymphadenopathy (at least one node > 1.5 cm) Isolated cerebral vasculitis. Affects medium and small arteries over a diffuse CNS area, without symptomatic extracranial vessel involvement. Patients have CNS symptoms as well as cerebral vasculitis by angiography and leptomeningeal biopsy. Small vessel vasculitis There are several vasculitides that affect small vessels."}, {"id": "InternalMed_Harrison_11793", "title": "InternalMed_Harrison", "score": 0.012918282821642233, "content": "in recent years, although strains of GAS that produce pyrogenic exotoxins continue to be prevalent in the population. The symptoms of scarlet fever are the same as those of pharyngitis alone. The rash typically begins on the first or second day of illness over the upper trunk, spreading to involve the extremities but sparing the palms and soles. The rash is made up of minute papules, giving a characteristic “sandpaper” feel to the skin. Associated findings include circumoral pallor, “strawberry tongue” (enlarged papillae on a coated tongue, which later may become denuded), and accentuation of the rash in skinfolds (Pastia’s lines). Subsidence of the rash in 6–9 days is followed after several days by desquamation of the palms and soles. The differential diagnosis of scarlet fever includes other causes of fever and generalized rash, such as measles and other viral exanthems, Kawasaki disease, TSS, and systemic allergic reactions (e.g., drug eruptions)."}, {"id": "pubmed23n0076_2863", "title": "[Kawasaki disease complicated with hemorrhagic enteritis mimicking intestinal obstruction: report of one case].", "score": 0.01290853031860226, "content": "A 6-month-old little boy presented with fever and cervical lymphadenopathy for four days. On admission, he was found to have conjunctival congestion of both eyes, reddened and fissured lips, straw-berry tongue, macular rash at the trunk and erythematous change on the BCG injection site. The diagnosis of Kawasaki disease was made and aspirin therapy (100 mg/kg/day) was started. On the 3rd hospital day, he developed abdominal distention, jaundice, poor activity and tachypnea. Bowel sound was silent on auscultation. Nasogastric tube was inserted for decompression and bilious material was drained out continuously. In addition, bloody, mucoid stool passage was noted. An abdominal radiography revealed dilatation of the small bowel and paucity of the colon gases. On the abdominal ultrasound, hydrops of gallbladder, marked ascites and silent and dilated bowel loops were found. Despite of supportive care, abdominal symptoms persisted and condition deteriorated. Laparotomy was done on the following day. At operation, it was found that the patient developed severe petechia on the long segment of small intestine from 15cm distal to the Treitz ligament to ileocecal valve and some fibrin plagues on the terminal ileum at 25cm proximal to the ileocecal valve were also found."}, {"id": "pubmed23n1119_3182", "title": "14-month-old girl with prolonged fever, a desquamative rash, and a new left hemiplegia.", "score": 0.012878787878787878, "content": "A young child with fever, erythematous rash, and conjunctivitis in sub-Saharan Africa is usually a case of measles. We report a-14-month-old girl with a prolonged fever, a desquamating erythematous rash, and a new left hemiplegia. This was initially diagnosed as measles but her correct final diagnosis was Kawasaki disease (KD); she very rapidly defervesced and regained normal function of her limbs after appropriate treatment. We believe this is the first reported case in Liberia of KD in a child younger than two years of age."}, {"id": "pubmed23n0905_1989", "title": "Atypical desquamation in a 2.5-year-old boy with Kawasaki disease: A case report.", "score": 0.012766474831806759, "content": "Kawasaki disease (KD) is a vasculitis that mostly affects children under 5 years of age. This article presents a 2.5-year-old boy who presented with 6 days of fever, generalized maculopapular rash, bilateral non-exudative conjunctivitis, cracked lips, right cervical lymphadenopathy, erythematous extremities, and perianal desquamation. Laboratory studies showed leukocytosis and sterile pyuria. Because diagnosis of KD was proved, oral acetylsalicylic acid with the anti-inflammatory dose and intravenous immunoglobulin were started for him. On the seventh day of admission time, he developed desquamation and erythema on the site of his right cervical lymphadenopathy as well as periungual scaling. About three weeks after starting the treatment, scaling of the cervical lymphadenopathy and periungual area stopped. Echocardiography was performed for him three times: at the time of diagnosis, four weeks, and 6 months later and revealed normal coronary arteries. We report this sign, desquamation on the site of cervical lymphadenopathy, as a new finding."}, {"id": "First_Aid_Step2_953", "title": "First_Aid_Step2", "score": 0.012711716430941156, "content": "Rubella Rubella virus Prodrome: Asymptomatic or tender, generalized lymphadenopathy. Rash: Presents with an erythematous, tender maculopapular rash that also starts on the face and spreads distally. In contrast to measles, children with rubella often have only a low-grade fever and do not appear as ill. Polyarthritis may be seen in adolescents. Encephalitis, thrombocytopenia (a rare complication of postnatal infection). Congenital infection is associated with congenital anomalies. Roseola infantum HHV-6 Prodrome: Acute onset of high fever (> 40°C); no other symptoms for 3–4 days. Rash: A maculopapular rash appears as fever breaks (begins on the trunk and quickly spreads to the face and extremities) and often lasts < 24 hours. Febrile seizures may occur as a result of rapid fever onset. Varicella Varicella-zoster virus (VZV) Prodrome: Mild fever, anorexia, and malaise precede the rash by 24 hours. Rash: Generalized, pruritic, “teardrop” vesicular periphery; lesions are often at"}, {"id": "article-28741_8", "title": "Scarlet Fever -- History and Physical", "score": 0.012658680493473986, "content": "Typically, scarlet fever is associated with acute pharyngitis. As a result, fever, sore throat, pain with swallowing, and cervical adenopathy is present. If there is no pharyngitis, the source of infection can be a wound or burn which is infected with GAS. The two vectors of infection can both cause scarlet fever and are not distinguishable from one another. The rash itself is a blanching, papular rash. It is distinguished from the macular rash found an allergic reaction by its insidious emergence and lack of confluence of the lesions. This lack of confluence is the primary reason it feels like sandpaper. Also of note, there are no vesicles or pustules present. Vesicles are more associated with the “dew on a rose petal” appearance of chickenpox in its initial stages. Pustules are more indicative of a local infection such as impetigo or erysipelas. The rash develops within 2 to 3 days after infection but can be delayed up to 7 days. The trunk, underarms, and groin are affected first, and then it spreads to the extremities. Usually, the palms and soles are spared. The circumoral area is also spared, making it pallor-like. The “strawberry tongue” begins with a white coating of the tongue with hyperplastic papillae. As the white coating resolves, the papules remain, giving the appearance of a strawberry. Pastia lines are found in the folds of the skin such as the neck, antecubital fossa, and groin. This appears as a linear accumulation of papules around pressure points. After the initial rash begins to resolve, a period of desquamation can occur and last up to two weeks in some cases."}, {"id": "wiki20220301en033_29237", "title": "Kawasaki disease", "score": 0.01245511030040472, "content": "The course of the disease can be divided into three clinical phases. The acute febrile phase, which usually lasts for one to two weeks, is characterized by fever, conjunctival injection, erythema of the oral mucosa, erythema and swelling of the hands and feet, rash, cervical adenopathy, aseptic meningitis, diarrhea, and hepatic dysfunction. Myocarditis is common during this time, and a pericardial effusion may be present. Coronary arteritis may be present, but aneurysms are generally not yet visible by echocardiography. The subacute phase begins when fever, rash, and lymphadenopathy resolve at about one to two weeks after the onset of fever, but irritability, anorexia, and conjunctival injection persist. Desquamation of the fingers and toes and thrombocytosis are seen during this stage, which generally lasts until about four weeks after the onset of fever. Coronary artery aneurysms usually develop during this time, and the risk for sudden death is highest."}, {"id": "article-83816_10", "title": "Rubeola (Measles) -- History and Physical", "score": 0.012015221017514597, "content": "The clinical picture of measles can be divided into three stages:  prodromal, eruptive, and convalescent and should be suspected in patients with the classic triad of the three “Cs”:  cough, conjunctivitis, and coryza. The primary or prodromal phase lasts four to six days and is characterized by the presence of high fever, malaise, coryza, conjunctivitis, palpebral edema, and dry cough.  Most cases show the characteristic Koplik spots of the disease, located in the buccal mucosa at the height of the second molar, and appear two to three days before the rash and disappear on the third day. The second phase, the eruptive, is characterized by the appearance of a maculopapular rash, initially fine that subsequently becomes confluent. The rash begins behind the auricle and along the hair implantation line, and extends downward to the face, trunk, and extremities. The third phase or convalescence occurs after three to four days when the rash begins to disappear, in the same order in which it appeared, leaving brown spots and producing a thin peeling of the skin. The fever disappears two to three days after the rash begins, as does the general malaise."}, {"id": "wiki20220301en004_55557", "title": "Scarlet fever", "score": 0.011755432946740231, "content": "Drug eruption: These are potential side effects of taking certain drugs such as penicillin. The reddened maculopapular rash which results can be itchy and be accompanied by a fever. Kawasaki disease: Children with this disease also present a strawberry tongue and undergo a desquamative process on their palms and soles. However, these children tend to be younger than 5 years old, their fever lasts longer (at least five days), and they have additional clinical criteria (including signs such as conjunctival redness and cracked lips), which can help distinguish this from scarlet fever. Toxic shock syndrome: Both streptococcal and staphylococcal bacteria can cause this syndrome. Clinical manifestations include diffuse rash and desquamation of the palms and soles. It can be distinguished from scarlet fever by low blood pressure, lack of sandpaper texture for the rash, and multi-organ system involvement."}]}}}} {"correct_option": 3, "explanations": {"1": {"exist": false, "char_ranges": [], "word_ranges": [], "text": ""}, "2": {"exist": false, "char_ranges": [], "word_ranges": [], "text": ""}, "3": {"exist": true, "char_ranges": [[0, 446]], "word_ranges": [[0, 75]], "text": "In this patient, he presents with a granulomatosis with polyangiitis formerly known as Wegener's granulomatosis. In the clinical case we are told about the otorhinolaryngologic involvement that is present in 92% of patients. At the pulmonary level, we were told that mucus with a clot was present in 85% of the patients. Also at the end of the case, they comment us the urinalysis with a glomerulonephritis that is present in 77% of the patients."}, "4": {"exist": false, "char_ranges": [], "word_ranges": [], "text": ""}, "5": {"exist": false, "char_ranges": [], "word_ranges": [], "text": ""}}, "full_answer": "In this patient, he presents with a granulomatosis with polyangiitis formerly known as Wegener's granulomatosis. In the clinical case we are told about the otorhinolaryngologic involvement that is present in 92% of patients. At the pulmonary level, we were told that mucus with a clot was present in 85% of the patients. Also at the end of the case, they comment us the urinalysis with a glomerulonephritis that is present in 77% of the patients.", "full_answer_no_ref": "In this patient, he presents with a granulomatosis with polyangiitis formerly known as Wegener's granulomatosis. In the clinical case we are told about the otorhinolaryngologic involvement that is present in 92% of patients. At the pulmonary level, we were told that mucus with a clot was present in 85% of the patients. Also at the end of the case, they comment us the urinalysis with a glomerulonephritis that is present in 77% of the patients.", "full_question": "A 67-year-old man presents with 3 months of asthenia and febrile fever, with nasal obstruction and mucus emission with some clots in the last month. In the last few days she noticed pain in the right eye and asymmetry with respect to the contralateral eye. Physical examination reveals proptosis of the right eyeball and inspection of the nostrils reveals an erythematous mucosa with serohematic crusts. The rest of the examination was normal. Blood tests (hemogram, renal and hepatic function) are normal, except for an ESR of 65 mm/h; urinalysis shows microhematuria and proteinuria of 520 mg/24h. What is the most probable initial diagnosis?", "id": 598, "lang": "en", "options": {"1": "Eosinophilic granulomatosis with polyangiitis.", "2": "Microscopic polyangiitis.", "3": "Granulomatosis with polyangiitis.", "4": "Polyarteritis nodosa.", "5": NaN}, "question_id_specific": 181, "type": "RHEUMATOLOGY", "year": 2022, "rag": {"clinical_case_options": {"MedCorp": {"RRF-2": [{"id": "wiki20220301en249_18855", "title": "Systemic vasculitis", "score": 0.019417475728155338, "content": "Medium vessel vasculitis These conditions are sometimes considered together with the small vessel vasculitides. Polyarteritis nodosa (PAN). Systemic necrotizing vasculitis and aneurysm formation affecting both medium and small arteries. If only small vessels are affected, it is called microscopic polyangiitis, although it is more associated with granulomatosis with polyangiitis than to classic PAN. At least 3 out of 10 criteria yields sensitivity and specificity of 82 and 87%: Unexplained weight loss > 4 kg Livedo reticularis Testicular pain Myalgias, weakness Abdominal pain, diarrhea, and GI bleeding Mononeuropathy or polyneuropathy New onset diastolic blood pressure > 90 mmHg Elevated serum blood urea nitrogen (> 40 mg/dL) or serum creatinine (> 1.5 mg/dL) Hepatitis B infection Arteriographic abnormalities Arterial biopsy showing polymorphonuclear cells"}, {"id": "pubmed23n1147_6293", "title": "Eosinophilic granulomatosis with polyangiitis as a rare cause of the syndrome of inappropriate antidiuretic hormone secretion.", "score": 0.017629698664181424, "content": "Eosinophilic granulomatosis with polyangiitis (EGPA, Churg-Strauss syndrome) is a rare multisystem necrotizing vasculitis that involves small- to medium-sized blood vessels. We report a rare case of syndrome of the inappropriate antidiuretic hormone (ADH) secretion (SIADH) secondary to EGPA. A 53-year-old man applied with complaints of pain in the large joints and morning stiffness in knee for 2 months. The patient had the history of impaired fasting glucose, asthma, nasal polyps, and urticaria. Physical examination revealed intrinsic muscle atrophy and weakness in the right hand. Peripheral eosinophil count was 9.78 × 109/L (0.02-0.5), erythrocyte sedimentation rate 39 mm/h (0-20), and C-reactive protein 5.77 mg/dL (0-0.5). Migratory ground-glass pulmonary opacities had been reported in previous chest computed tomography scans. Echocardiography revealed findings compatible with eosinophilic involvement. Electroneuromyographic evaluation showed acute distal axonal neuropathy of right ulnar nerve. EGPA was considered. Oral methylprednisolone treatment was initiated. Intravenous immunoglobulin (IVIG) and cyclophosphamide treatment and gradual tapering of oral steroids were planned. In 24-h urine analysis, sodium was 387 mEq, creatinine was 1156 mg, and volume was 3000 mL. When his medical records were investigated, it was observed that hyponatremia was present for nearly 2 years. While serum osmolality was 270, urine osmolality was 604 mOsm/kg H1.58 mg/dl or 140 μmol/l) Proteinuria (>1 g/24h) Gastrointestinal hemorrhage, infarction, or pancreatitis Involvement of the central nervous system Cardiomyopathy Having none of these factors indicates milder case, with a five-year mortality rate of 11.9%. The presence of one factor indicates severe disease, with a five-year mortality rate of 26%, and three or more indicate very severe disease: 46% five-year mortality rate."}, {"id": "wiki20220301en249_1086", "title": "Pauci-immune", "score": 0.010096237970253719, "content": "In the setting of systemic vasculitis as described above, proliferative nephritis is associated with antineutrophil cytoplasmic antibodies (ANCA). Because of this, an ANCA test should always follow a negative immunofluorescence result to have the highest accuracy for confirming pauci-immune vasculitis-driven proliferative nephritis. Some cases of pauci-immune proliferative nephritis have no explanation and are thus deemed \"idiopathic.\" Peak incidences in 50- to 60-year-olds symptoms include intermittent fever / weight loss / shortness of breath / joint pain. See also Systemic vasculitis#Pauci-immune Goodpasture Syndrome and Poststrep Glomerulonephritis Microscopic polyangiitis, Eosinophilic granulomatosis with polyangiitis or Granulomatosis with polyangiitis References External links wikt:paucity Vascular-related cutaneous conditions"}, {"id": "article-76416_14", "title": "Granulomatosis With Polyangiitis -- History and Physical -- Organ System Involvement", "score": 0.010012562578516116, "content": "Upon presentation, renal involvement is noted in only 10%-20%, but glomerulonephritis eventually develops in 80% of patients within two years of disease onset. The most common manifestation is rapidly progressive crescentic glomerulonephritis leading to chronic kidney disease or end-stage renal disease. Eye involvement:"}, {"id": "pubmed23n0056_3942", "title": "[A case of polyarteritis nodosa with bilateral hilar lymphadenopathy].", "score": 0.009900990099009901, "content": "A 55-year-old male was admitted with non productive cough and fever which had continued for 6 weeks. The patient had symptoms of peripheral neuralgia. Chest X-ray revealed bilateral hilar lymphadenopathy (BHL) and reticular shadows in both lung fields. Other laboratory abnormalities included hematuria, RBC cast, high BUN, leukocytosis and thrombocytosis. Destruction of the internal membrane of arterioles was observed in a livedo reticularis on the right lower extremity. Renal angiography showed irregularity in the diameter, discontinuation and narrowing of peripheral arteries of both kidneys. These findings suggested the existence of \"angiitis\". These data were compatible with the diagnosis of polyarteritis nodosa (PN). Prednisolone (60 mg/day) administration resulted in the improvement of his symptoms and laboratory findings. A case of PN with lymph node swelling has been reported, however PN with BHL has not yet been reported. This is the first report of PN with BHL."}, {"id": "pubmed23n0302_21179", "title": "Classical polyarteritis nodosa and microscopic polyarteritis with medium vessel involvement--a comparison of the clinical and laboratory features.", "score": 0.00980392156862745, "content": "Microscopic polyarteritis may involve medium-sized and small blood vessels as well as arterioles, venules and capillaries. We have compared the clinical and laboratory features in patients with microscopic polyarteritis and medium vessel involvement, with the features found in patients with polyarteritis nodosa affecting medium vessels alone. In a 9-year period, 21 patients presented to our hospital with a form of polyarteritis. Seven had microscopic polyarteritis demonstrated histologically (6/7, 86%) and associated with dysmorphic urinary red cells (7/7, 100%), as well as medium vessel vasculitis demonstrated histologically (7/7) or by angiography (1/7, 14%). Five patients had polyarteritis nodosa with medium vessel vasculitis demonstrated histologically (3/5, 60%) or by angiography (2/5, 40%); and no evidence of a glomerular vasculitis on biopsy (2/7, 29%) or in the urinary sediment (0/7, 0%). The remaining 9 patients had microscopic polyarteritis but medium vessel involvement was not excluded by angiography. All patients with microscopic polyarteritis and medium vessel involvement had glomerular hematuria (> 100,000 glomerular RBC/ml), proteinuria > 0.5 g/24 hours), and an elevated serum creatinine (0.166 to 0.811 mmol/l). Other symptoms included fever (6/7, 86%), night sweats (5/7, 71%), gastrointestinal bleeding (4/7, 57%), proximal myopathy (3/7, 43%) and peripheral neuropathy (3/7, 43%). One patient (1/7, 14%) had hypertension. Anemia (6/7, 86%), a raised ESR (6/7, 86%), thrombocytosis (6/7, 86%), hypoalbuminemia (6/7, 86%) and abnormal liver function tests (6/7, 86%) were common. Two patients (29%) had an eosinophilia. All 5 individuals who were tested for ANCA were positive (2cANCA, 2pANCA and one pattern not described). In contrast, in patients with polyarteritis nodosa and medium vessel involvement alone, an elevated ESR was common (4/5, 80%) but fever (1/5, 20%), night sweats (0/5, 0%), proximal myopathy (1/5, 20%) and peripheral neuropathy (1/5, 20%) were seen infrequently; hypertension (1/5, 20%) and eosinophilia (1/5, 20%) were also uncommon; and ANCA were not demonstrated (0/3, 0%). Medium-sized vessel involvement is common in patients with microscopic polyarteristis, and these patients are more likely to have renal involvement and systemic symptoms, and be ANCA-positive, than patients with polyarteritis nodosa alone. Gastrointestinal symptoms are often seen in both groups."}, {"id": "wiki20220301en249_18864", "title": "Systemic vasculitis", "score": 0.009740590895182403, "content": "Mononeuritis multiplex. Also known as asymmetric polyneuropathy, in a non-diabetic this is suggestive of vasculitis. Palpable purpura. If patients have this in isolation, it is most likely due to cutaneous leukocytoclastic vasculitis. If the purpura is in combination with systemic organ involvement, it is most likely to be Henoch–Schönlein purpura or microscopic polyangiitis. Pulmonary-renal syndrome. Individuals who are coughing up blood and have kidney involvement are likely to have granulomatosis with polyangiitis, microscopic polyangiitis, or anti-GBM disease (Goodpasture syndrome). Diagnosis A detailed history is important to elicit any recent medications, any risk of hepatitis infection, or any recent diagnosis with a connective tissue disorder such as systemic lupus erythematosus (SLE). A thorough physical exam is needed as usual."}, {"id": "pubmed23n0309_9355", "title": "[Microscopic polyangiitis with eosinophilia--an overlap syndrome or separate disease entity? A case report and review of the literature].", "score": 0.009685286806434969, "content": "Systemic vasculitides are potentially life-threatening diseases. Early and appropriate diagnosis based on case history, clinico-pathological features, and laboratory parameters, such as the presence of anti-neutrophil cytoplasmic antibodies (ANCA), is crucial for starting appropriate and, often, life-saving therapeutic measures. We report a 50-year-old female patient who presented with fever, arthralgias and hemoptysis. Skin signs included disseminated hemorrhagic pustules, ulcerations of oral and genital mucosa, subcutaneous nodules on arms and legs, and a pyoderma gangrenosum-like lesion on the right leg. Laboratory investigations revealed a peripheral eosinophilia and a positive cANCA titer. Histopathologic analysis of various biopsy specimens showed a granulomatous vasculitis in the subcutis, a nongranulomatous vasculitis with massive eosinophil infiltration in the lungs, and a segmental, necrotizing glomerulonephritis in the kidneys. Differential diagnosis included Wegener's granulomatosis, microscopic polyangiitis (MPA) and Churg-Strauss syndrome. MPA was diagnosed based on clinical and histopathological criteria. An interesting feature of this case was marked peripheral and tissue eosinophilia. Therapy consisted of cyclophosphamide and methylprednisolone. The patient went into a long-lasting clinical remission one month after starting therapy."}, {"id": "pubmed23n1079_2973", "title": "Wegener's granulomatosis with orbital involvement: case report and literature review.", "score": 0.009615384615384616, "content": " 80%)—deposition of yellowish extracellular material in and between Bruch's membrane and retinal pigment epithelium (“drusen”) with gradual loss in vision. Wet (exudative, 10–15%)—rapid loss of vision due to bleeding secondary to choroidal neovascularization. Etymology Gk, meta + morphe, form, opsis, sight See also Dysmorphopsia Hallucination References Visual disturbances and blindness"}, {"id": "pubmed23n0791_18770", "title": "Spontaneous closure of a fully developed macular hole in a severely myopic eye.", "score": 0.011612089844397995, "content": "Purpose. Myopic macular holes can be difficult to close with surgery and are frequently associated with retinal detachment. We report on a case of a macular hole in a severely myopic eye that underwent spontaneous closure. Methods. An observational case study. Results. A 55-year-old female was referred to Ophthalmology for a central scotoma and metamorphopsia in the right eye. Visual acuity was 1/20 in both eyes. Fundus examination showed loss of the foveal depression, with a small yellow ring in the center of the fovea in the right eye, and a tilted optic disc and peripapillary staphyloma bilaterally. Spectral domain optical coherence tomography (SD-OCT) revealed a fully developed macular hole with a rim of thickened and slightly elevated retina in the right eye. The patient refused surgery. After 4 years of follow-up, her visual acuity improved to 20/40 in the right eye, and SD-OCT revealed spontaneous sealing of the macular hole without bare retinal pigment epithelium. Conclusions. Myopic macular holes represent a challenge regarding their management, and the prognosis is often poor. "}, {"id": "pubmed23n0325_11826", "title": "Symptomatic age-related macular degeneration in Asian patients.", "score": 0.01158031346710592, "content": "To characterize retinal and fluorescein angiographic findings of Asian patients with symptoms secondary to age-related macular degeneration (ARMD). We retrospectively reviewed 453 consecutive medical records corresponding to fluorescein angiograms performed between November 1992 and November 1995 to identify Asian patients with symptomatic ARMD. Presenting visual symptoms, best-corrected Snellen visual acuities, and retinal examination findings were determined from the medical records. Fundus photos and fluorescein angiograms were reviewed. There were 26 symptomatic eyes in 19 Asian patients with a median age of 73 years. Presenting visual symptoms included decreased visual acuity (19 eyes), metamorphopsia (5 eyes), or scotoma (2 eyes). Retinal findings included occult choroidal neovascularization (CNV) in 5 (19%) eyes, serous pigment epithelial detachment (PED) in 8 (31%) eyes, PED with CNV in 5 (19%) eyes, drusen in 5 (19%) eyes, retinal pigment epithelial atrophy in 1 (4%) eye, vitreous hemorrhage in 1 (4%) eye, and a disciform scar in 1 (4%) eye. In this cohort of Asian patients with ARMD, the majority of symptomatic eyes had either CNV (46%) or serous PED (31%)."}, {"id": "pubmed23n0678_14026", "title": "[Clinical observations of macular hole with and without retinal detachment in high myopic eyes].", "score": 0.009900990099009901, "content": "To study the clinical features and the pathogenesis of macular hole with and without retina detachment (RD) in high myopic eyes. It was a retrospective series case study. The charts of high myopic patients with macular hole at our hospital from June 2006 to February 2007 were retrospectively reviewed and analyzed. Patients were divided into two groups (the RD group and non-RD group) depending on the presence of RD or not. Their clinical data and optic coherence tomography (OCT) results were further analyzed. SPSS 13.0 was used for the statistic analysis. When comparing the quantitative aspects like age, axial length and refraction, t-test was used. Categorical data, such as sex ratio, occurrence of vitreous traction, posterior staphyloma and retinoschisis were compared by using χ(2) test. Fisher's test was used in comparing eye laterality, incidence of white hole, visual acuity and posterior vitreous detachment (PVD). During this period, there were 43 patients fitting the including criteria. Among them, 36 patents (37 eyes) were in the RD group and 7 patients (7 eyes) in the no-RD group. In the RD group, the average age was 56.1, 24.3% of them (9/37) were male; percentage of left and right eyes was (11/37) and 70.3% (26/37), respectively; average refraction was (-8.9 ± 2.2) D; average axial length was (28.7 ± 2.0) mm. Visual acuity was ≤ 0.05 (72.2%) in 26 eyes and 0.05 - 0.2 (27.8%) in 10 patients. The incidence of complete and non-complete PVD was 89.2% (33/37) and 10.8% (4/37), respectively. White hole presented in 35.1% (3/37) patients. Vitreous traction and retinoschisis presented in 27.0% (10/37) and 35.1% (13/37) patients, respectively. In the non-RD group, the average age was 47.6; 16.7% of them (1/7) were male; left and right eyes were involved in 42.9% (3/7) and 57.1% (4/7), respectively. Average refraction was (-9.0 ± 1.9) D; average axial length was (28.9 ± 1.5) mm. Vision acuity was ≤ 0.05 in 3 patients (42.9%); between 0.05 - 0.2 in 3 eyes (42.9%) and ≥ 0.2 in 1 eye (14.3%). Incidence of complete and non-complete PVD was 85.7% (6/7) and 14.3% (1/7), respectively. White hole was observed in 14.3% (1/7) patients; 42.9% (3/7) patients were accompanied with vitreous traction and 71.4% (5/7) with retinoschisis. B-scan ultrasonography showed posterior staphyloma in all 44 eyes. The results of statistical analysis showed that the gender (χ(2) = 0.008) and eye laterality (χ(2) = 0.449) as well as refraction (t = 0.193), axial length (t = -0.25) and visual acuity (χ(2) = 4.509) of these two groups were similar (P > 0.05). The incidences of vitreous traction (χ(2) = 0.709), white hole (χ(2) = 1.179), PVD (χ(2) = 0.071) and retinoschisis (χ(2) = 3.207) were also similar (P > 0.05). But the age of the non-RD group is significantly younger than the RD group (t = 1.66, P < 0.05). Various pathogenesis may involved in the occurrence of retinal detachment in highly myopic eyes with macular hole. Further study is required to improve our understanding of this entity."}, {"id": "pubmed23n1133_19399", "title": "Atypical Chronic Central Serous Chorioretinopathy Mimicking Vogt-Koyanagi-Harada Disease: Full Therapeutic Response to Half-Fluence Photodynamic Therapy", "score": 0.00980392156862745, "content": "The aim of this case report is to describe a case of atypical central serous chorioretinopathy (CSCR) definitively diagnosed after 8 years. A 44-year-old woman presented with reduced visual acuity in her left eye. Her visual acuity was light perception with projection in the right eye and 0.15 in the left. She described a similar decline in vision in her right eye 8 years ago. At that time, she had exudative retinal detachment and was treated with systemic immunosuppressive therapy for a presumed diagnosis of Vogt-Koyanagi-Harada disease. Despite resolution of the exudative retinal detachment, macular scarring developed. Eight years later, she developed inferior exudative retinal detachment in the left eye. A diagnosis of atypical CSCR was made with the help of multimodal imaging and her left eye was successfully treated with eplerenone and half-fluence photodynamic therapy (hf-PDT). In conclusion, early diagnosis and treatment of atypical CSCR may prevent subretinal fibrosis formation and permanent vision loss. Hf-PDT and eplerenone are successful treatment options for atypical CSCR."}, {"id": "pubmed23n1025_4637", "title": "An Unusual Case: Self-separation of an Idiopathic Epiretinal Membrane", "score": 0.00980392156862745, "content": "Self-separation or peeling of an idiopathic epiretinal membrane (ERM) in an eye with partial posterior vitreous detachment (PVD) is a rare event. A 56-year-old woman presented to our clinic with complaints of floaters in her right eye. Best-corrected visual acuity (BCVA) was 9/10 in this eye. Fundus examination and Spectral domain optical coherence tomography (SD-OCT) revealed an idiopathic ERM and Grade 3 PVD in this eye. Four months later, she had complaints of metamorphopsia in her right eye. BCVA was 7/10, while SD-OCT images of the right macula were similar to previous images. One week after the last visit, she presented again due to the sudden disappearance of her metamorphopsia complaints. BCVA had improved to 10/10. Fundus examination demonstrated that the ERM had spontaneously separated from the retinal surface as a flap floating in the vitreous and the foveal contour had returned to normal. The etiologic mechanism may be explained as the contracting forces within an immature ERM being stronger than its adhesion to the retina."}, {"id": "pubmed23n0697_1369", "title": "[Long-term results of the treatment of optic disc pit associated with serous macular detachment: a review of 20 cases].", "score": 0.009708737864077669, "content": "The pathogenesis of the macular serous retinal detachment (SRD) associated with congenital optic disc pit remains controversial. The treatment is also discussed. Through this study, which includes the majority of the techniques available, we report our experiment in the treatment of this pathology. This was a retrospective single-centre study of 20 patients who presented with macular SRD associated with optic disc pit between 1983 and 2009. Various treatments were provided. At the beginning of the study, patients were treated only by juxtapapillary laser photocoagulation. After laser failure then as first-line treatment, laser photocoagulation was associated with intravitreal gas (C3F8) injection with postoperative facedown positioning for 2 weeks. During the past few years, all patients have been systematically treated with vitrectomy with or without internal limiting membrane (ILM) peeling, laser, and gas (C2F6) tamponade. This series consisted of 20 patients: nine men and 11 women. The patients' mean age at presentation was 29 years (range, 9-60 years). The mean time between the onset of the decrease in visual acuity (VA) and treatment was 6.1 months. None of these patients had a posterior vitreous detachment at the time of diagnosis. Six patients were treated by laser photocoagulation alone, which was successful only in two cases. Eleven patients (with laser treatment failure in three) were treated by laser and intravitreal gas injection, with a 72% success rate. We performed vitrectomy with posterior hyaloid dissection, laser, and gas tamponade in eight cases (with laser-gas treatment failure in two) with 87% success rate and no recurrence. Five of these patients had ILM peeling during the vitrectomy. The mean follow-up period was 60 months (range, 2 months to 17 years). This study shows that early treatment of macular SRD associated with optic disc pit by vitrectomy, ILM peeling, juxtapapillary photocoagulation, and gas tamponade is followed by good anatomical and functional results. This treatment is superior to the other less invasive procedures. Optical coherence tomography is an important exam for diagnosis and postoperative follow-up of patients."}, {"id": "pubmed23n0856_18120", "title": "[Atypical presentation of central serous choroidopathy. Case report].", "score": 0.009615384615384616, "content": "Central serous choroidopathy is a macular disease, usually with a self-limited and benign course, and predominantly affects male patients between 20 and 45 years old. A 68 year-old female patient complained of decreased visual acuity of her right eye of approximately 3 weeks of onset. Best corrected visual acuity in her right eye was 20/100. Fundus examination revealed a macular serous detachment involving its centre, as well as the presence of multiple calcified drusen. Fluorescein angiography showed late parafoveal leakage in a \"smokestack\" pattern in the right macular area. Optical coherence tomography showed a dome-shape macular detachment, also in the right eye. The patient was observed every 2 weeks and spontaneous resolution of the macular detachment was seen a month later. Based on these clinical features, a diagnosis was made of central serous choroidopathy of atypical presentation. Atypical presentation cases of serous central choroidopathy might be seen occasionally. Hence, it is an important differential diagnosis of age related macular degeneration in patients older than 60 years."}, {"id": "pubmed23n0348_19038", "title": "Use of a macular buckle in the treatment of exudative age-related macular degeneration.", "score": 0.009615384615384616, "content": "To evaluate a macular buckle for exudative choroidal neovascularization secondary to age-related macular degeneration (ARMD). Forty-two eyes with choroidal neovascular membranes (CNVM) secondary to ARMD underwent surgical placement of a macular buckle. A Gore-Tex strip (2.0-2.5 mm wide) was button-holed through a 5 mm diameter silicone sponge (9 mm long) and placed behind the macula underneath the CNVM by the same surgeon (Dr Peyman) in all cases. Follow-up ranged from 7-76 months (mean, 20.9 months). Of 12 eyes with classic subfoveal CNVM: 4 (33%) gained 2 or more lines of Snellen visual acuity; 3 (25%) gained 1 line, remained the same, or lost 1 line; and 5 (42%) lost 2 or more lines (range + 6 to - 6 lines). Of 22 eyes with ill-defined subfoveal CNVM: 12 (54%) gained 1 line, remained the same, or lost 1 line; and 10 (46%) lost 2 or more lines (range + 1 to - 8 lines). Eight eyes with ill-defined juxtafoveal CNVM had the following visual acuity outcomes: 5 eyes (62%) maintained the same level of Snellen visual acuity (gained 1, 0, or lost 1 line); and 3 (38%) got worse (lost 2 or more lines of Snellen visual acuity, range + 1 to - 6 lines). Ten eyes (24%) bled subretinally during the follow-up period (average 11.5 months, range 14 days to 27 months), all outside the area of indentation of the macular buckle. The macular buckle treatment for exudative subretinal choroidal neovascular membranes in ARMD stabilized visual decline and displaced significant subfoveal hemorrhage."}, {"id": "pubmed23n0888_25831", "title": "CYSTOID MACULAR EDEMA AND CYSTOID MACULAR DEGENERATION AS A RESULT OF MULTIPLE PATHOGENIC FACTORS IN THE SETTING OF CENTRAL SEROUS CHORIORETINOPATHY.", "score": 0.009523809523809525, "content": "To report the pathogenic factors that account for cystoid macular edema and cystoid macular degeneration in chronic central serous chorioretinopathy (CSC). The clinical course and multimodal imaging findings, including fundus color photography, fundus autofluorescence, spectral-domain optical coherence tomography, and fluorescein angiography, of one eye with cystoid macular edema due to chronic CSC was documented. A 44-year old woman with a history of chronic CSC presented with progressive visual decline in the right eye. Best-corrected visual acuity was 20/40. Funduscopic examination revealed diffuse retinal pigment epithelial changes and macular edema. Fluorescein angiography demonstrated perifoveal microaneurysms and leakage in a petaloid configuration. Spectral-domain optical coherence tomography demonstrated cysts at the level of the inner nuclear layer, an epiretinal membrane, vitreomacular traction, and an attenuated retinal pigment epithelial band. Central subfield thickness was 486 μm. Three intravitreal injections of aflibercept were administered over 16 weeks following which there was resolution of leakage, release of vitreomacular traction, and resolution of microaneurysms. Central subfield thickness reduced to 379 μm, but persistent intraretinal cysts were observed. There was subjective improvement in visual symptoms, but Snellen acuity remained at 20/40. Intraretinal cystic changes in chronic CSC may be the result of multifactorial pathogenic factors and may represent the coexistence of cystoid macular edema and cystoid macular degeneration. Anti-vascular endothelial growth factor may play an important role in the treatment of cystoid macular edema caused by CSC."}, {"id": "pubmed23n0723_18591", "title": "[Patient with recurrent central serous chorioretinopathy who developed multiple evanescent white dots and serous retinal detachment immediately following bevacizumab administration].", "score": 0.009523809523809525, "content": "A patient developed choroidal neovascularization (CNV) in one eye during treatment for bilateral recurrent central serous chorioretinopathy (CSC) and was intravitreously injected with bevacizumab; she developed multiple evanescent white dots and serous retinal detachment(SRD). A 50-year-old women had a history of CSC OD at the age of 29 years. On initial examination, CSC OD was noted, and multiple detachments of the retinal pigment epithelium OU were observed. While the CSC in the right eye was successfully treated by laser photocoagulation, it spread to both eyes following this episode. Examination of the right eye by optical coherence tomography (OCT) following the recurrence of the CSC showed slight elevation of the retinal pigment epithelial layer in the central fovea, but this finding disappeared with the resolution of the CSC. However, as the CSC combined with CNV (Gass type 2) recurred within 1 year, the patient was intravitreously injected with bevacizumab. On the day following the injection, SRD OD occurred, and on the 7th day following the injection many white lesions varying in size appeared in the deep layer of the retina, but they healed 3 weeks later, leaving only the CNV. The CNV was cured later by additional photodynamic therapy. Since the lesions of the fundus observed immediately after the bevacizumab administration resolved spontaneously without sequelae, they were retrospectively diagnosed as a white dot syndrome-like disease. The white dot syndrome-like disease is suggested as a rare complication of bevacizumab."}, {"id": "pubmed23n0855_6275", "title": "Unilateral central serous chorioretinopathy in a pregnant Nigerian woman.", "score": 0.009433962264150943, "content": "Central serous chorioretinopathy (CSCR) is an idiopathic condition characterized by serous detachment of the neurosensory retina in the macular region. It is relatively uncommon in Africans and though pregnancy is a known risk factor, there are no previous reports of CSCR in pregnant African women. We report the case of a 35-year-old pregnant woman who presented to our clinic at gestational age of 29 weeks with a 4 months history of blurring of vision in her left eye. Examination revealed visual acuity of 6/4 on the right eye and 6/9 on the left eye. She had normal anterior segments bilaterally and a normal posterior segment on the right. However, she had left macular edema with exudates. There was no significant refractive error. Her blood pressure was normal. Investigations including electrolytes and urea, urinalysis, and blood sugar profile were all normal. She was managed conservatively, and symptoms resolved 2 weeks prior to delivery. This is a case report of CSCR in a pregnant Nigerian woman with spontaneous resolution of symptoms prior to delivery. Pregnant women should be educated about the possibility of visual problems accompanying pregnancy. "}, {"id": "pubmed23n1120_22745", "title": "Three cases of macular hole that occurred in inferior scleral staphyloma associated with tilted disc syndrome: a case series.", "score": 0.009433962264150943, "content": "The objective is to examine the clinical characteristics of three patients with macular hole that occurred in inferior posterior staphyloma associated with tilted disc syndrome. This study involved three eyes of three Japanese female patients (mean age 76.0 years, range 73-84 years) with macular hole occurring in inferior posterior staphyloma associated with tilted disc syndrome. One of the three eyes was slightly myopic, while the other two eyes were highly myopic. In all three eyes, the macular hole was found to be located in or near the margin of the inferior posterior staphyloma. In one eye, the extent of retinoschisis was rather broad in the margin of the macular hole, and another eye had a history of treatment for choroidal neovascularization. As surgical treatment, the internal limiting membrane in areas surrounding the macular hole was detached after producing artificial posterior vitreous detachment, and a gas tamponade was performed. It was found during surgery that the extensibility of the retina in the margin of the MH was decreased in the three eyes as compared with a usual macular hole. Although the macular hole was successfully closed in all three cases post surgery, the layer structure of the central retina was poorly repaired in all three cases and choroidal neovascularization remained in one case. In all three cases, corrected visual acuity remained at 0.3-0.5 post surgery. Our findings showed poor improvement of visual acuity in all three cases post surgery, even if closure of the macular hole is achieved, thus suggesting that in cases of macular hole associated with tilted disc syndrome and inferior posterior staphyloma, the presence of macular dysfunction existing prior to the onset of macular hole affects visual prognosis."}, {"id": "pubmed23n0900_10038", "title": "Cystoid macular edema associated with preservative-free latanoprost after uncomplicated cataract surgery: case report and review of the literature.", "score": 0.009345794392523364, "content": "Cystoid macular edema associated with latanoprost administration has been reported in patients after complicated cataract surgery with coexisting risk factors. We present the first case of preservative free latanoprost associated cystoid macular edema that occurred many months after uncomplicated cataract surgery. A 65-year old Caucasian female presented in the Outpatients Clinic complaining of reduced vision and metamorphopsia in the right eye. She had undergone uneventful phacoemulsification 19 months ago in the right eye and was under treatment with preservative free latanoprost eye drops for the last 7 months for ocular hypertension. Her remaining medical and ocular history were otherwise unremarkable. Cystoid macular edema with serous retinal detachment was diagnosed in the right eye using optical coherence tomography and fluorescein angiography. Latanoprost was discontinued and brinzolamide and nepafenac eye drops were administered in the right eye. Two months later, cystoid macular edema completely resolved with restoration of visual acuity. Nepafenac eye drops were administered for another 2 months. Eight months after latanoprost cessation optical coherence demonstrated no sign of cystoid macular edema whereas a subtle epiretinal membrane was noted. Cystoid macular edema may potentially occur in patients receiving preservative free latanoprost. More interestingly, in our case it was diagnosed in a patient with a long standing pseudophakia after uncomplicated phacoemulsification. No obvious risk factor for macular edema development was recognized. Prompt diagnosis and latanoprost discontinuation resulted in complete resolution of the cystoid macular edema and functional restoration of the eye."}, {"id": "pubmed23n0722_21615", "title": "Vitrectomy for optic disk pit with macular schisis and outer retinal dehiscence.", "score": 0.009345794392523364, "content": "To describe the outcomes of vitrectomy for optic disc pit-related maculopathy with central outer retinal dehiscence. This prospective interventional case series included seven patients with optic disc pit with macular schisis and central outer retinal dehiscence who underwent vitrectomy with internal limiting membrane peeling, barrage laser photocoagulation, and gas tamponade and were followed for at least 6 months. The surgical outcomes in terms of restoration of macular anatomy and visual improvement were recorded at each visit by fundus photography and optical coherence tomography. The mean age of the patients was 21.3 ± 8.6 years (range, 10-35 years), and the mean duration of defective vision was 6.7 ± 8.5 months (range, 1-24 months). Preoperatively, the median best-corrected visual acuity (BCVA) was 20/60 (range, 20/40 to 20/120). Full-thickness macular holes were noticed in 4 patients 1 month postoperatively. Gas tamponade was repeated in two patients with large macular holes. By the final follow-up, macular holes had closed and BCVA improved in all patients except one. Final mean central macular thickness was 176.83 ± 55.74 μ, the range being 109 μ to 256 μ. The median postoperative BCVA was 20/30 (range, 20/20 to 20/80). Six of 7 patients (85.7%) had improvement in BCVA postoperatively (mean, +2 lines; range, 1-4 lines). Five patients (71%) achieved a postoperative BCVA of ≥20/30. Best-corrected visual acuity dropped by one line in the patient with persistent macular hole. Vitrectomy with internal limiting membrane peeling can achieve excellent final surgical outcomes in optic pit maculopathy with outer retinal dehiscence despite the potential for macular hole formation."}, {"id": "wiki20220301en150_35346", "title": "Epiretinal membrane", "score": 0.009266409266409266, "content": "Epidemiology This ocular pathology was first described by Iwanoff in 1865, and it has been shown to occur in about 7% of the population. It can occur more frequently in the older population with postmortem studies showing it in 2% of those aged 50 years and 20% in those aged 75 years. Culture In 1996, Spalding Gray (June 5, 1941 – ca. January 10, 2004), an American actor, screenwriter and playwright, released Gray's Anatomy, a film monologue describing his experiences dealing with a macular pucker and his decision to undergo surgery. In the 2011 film Paul, Ruth had Epiretinal membrane complicated by macular edema in her left vitreous cavity. See also Eye surgery References Additional references External links Macular Pucker Resource Guide from the National Eye Institute (NEI). Disorders of choroid and retina"}, {"id": "wiki20220301en155_35383", "title": "Optic pit", "score": 0.009260053159564435, "content": "Treatment for optic pit-associated macular detachment involves photocoagulation of the retina by use of an ion laser (either krypton or argon). This procedure works by burning one or more rows in between the optic disc and areas of serous retinal detachment. In most cases, macular reattachment results and visual acuity can be restored to about 20/80. This procedure may also be utilized prior to macular detachment in order to help prevent the future development of macular detachment. Other treatments for optic pit-associated macular detachment include macular buckling, gas tamponade, or vitrectomy. Some experts feel that the best results can be attained when the use of any of the above-mentioned modalities (laser photocoagulation, macular buckling, gas tamponade, and vitrectomy) are used in combination. Diagnosis Optic pits should be diagnosed by an eye care professional who can perform a thorough exam of the back of the eye using an ophthalmoscope."}, {"id": "pubmed23n0918_21332", "title": "Microperimetry - A New Tool for Assessing Retinal Sensitivity in Macular Diseases.", "score": 0.009259259259259259, "content": "Macular disease is the leading cause of low vision in the Western world. Drusen and pigmentary irregularities are common among the rural Northern Indian population. The disease process leads to loss of central vision, metamorphopsia, macropsia or micropsia and colour vision defect. To study the retinal sensitivity changes in macular diseases using microperimetry. It was an observational study, conducted in the Department of Ophthalmology at a rural tertiary care hospital. This study was started from December 2014 until June 2016, in all patients with macular disease above the age of 20 years attending the outpatient department. Microperimetry was done for 84 eyes of 52 patients with macular disease. Mean retinal Sensitivity (MS) and fixation stability was evaluated. The statistical analysis of mean retinal sensitivity, central 2° and 4° fixation was done by calculating the mean and standard deviation using 95% confidence interval. The range of age was between 20-81 years. Majority were 32 males (62%) and 20 females (38%). Out of the 84 eyes studied, majority of the macular disease were Age-Related Macular Degeneration (AMD) (50%). Rest 50% were other macular diseases. The mean retinal sensitivity (dB) shown by microperimetry was 10.83 in AMD, 9.12 in Cystoid Macular Oedema (CME), 10.34 in Epiretinal Membrane (ERM), 10.74 in Pigment Epithelial Detachment (PED), 8.96 in Central Serous Chorioretinopathy (CSCR), 6.43 in macular dystrophy, 7.15 in Lamellar Hole (LMH), 9.8 in Pseudomacular Hole (PMH), 3 in geographic atrophy, 11.1 in macular telangiectasia, 5.6 in Berlin oedema, 12.3 in macular scar and 15.2 in haemorrhage in macula. The study showed 64% of the eyes had stable 2° central fixation, 35% had relatively unstable fixation and 1% had unstable fixation. No significant correlation between retinal sensitivity and retinal thickness in AMD was found. This study shows that microperimetry can be a useful tool for objective evaluation of macular function and progression of the disease."}, {"id": "pubmed23n0359_7066", "title": "[Role of vitreoretinal interface in the pathogenesis and therapy of macular disease associated with optic pits].", "score": 0.009259259259259259, "content": "Although the relationship between optic pits and macular lesions was described nearly a century ago, the pathology and pathogenesis of macular detachment remain unclear. Recent OCT studies have shown schisislike spaces in connection with the disc. None of the hypotheses of pathogenesis proposed so far could have been proven. Besides the hypothesis of exudation, the role of the vitreoretinal interface is kept in the background of discussion. We describe a case of macular detachment associated with optic pit that regained full vision after pars plana vitrectomy with laser coagulation and gas tamponade over a follow-up of 26 months. The purpose of our case report is to emphasize the role of the vitreoretinal interface in the pathogenesis of macular detachment associated with optic pits. A 32-year-Caucasian woman developed macular detachment associated with an optic pit on her right eye. The vision deteriorated to 24/60. A standard three-port pars plana vitrectomy was performed. After creating a posterior vitreous detachment, all vitreoretinal adhesions were removed carefully. Peripapillary laser coagulation and gas tamponade with 15% C2F6-air mixture followed. For 26 months after surgery the macula has been flat. The vision is 60/60. Besides the exudative component of macular detachment, the vitreoretinal interface seems to play an underestimated role in the pathogenesis of maculopathy associated with optic pits. Tractional forces could explain the delay of macular detachment in young adulthood and the frequency of treatment failure after laser coagulation and gas tamponade. Pars plana vitrectomy with complete removal of all vitreoretinal adhesions should be a suitable technique in the treatment of macular detachment associated with optic pits."}, {"id": "wiki20220301en067_41615", "title": "Metamorphopsia", "score": 0.00917604966852573, "content": "Age-related macular degeneration Epiretinal membrane and vitreomacular traction Posterior vitreous detachment Macular hole Diagnosis Tests used for diagnosis of Metamorphopsia mostly make use of subjective assessments of how a person views regular patterns. Many of these tests have a poor ability to accurately diagnose or identify a person with the disease (i.e.,poor sensitivity). The use of assessments such as a psychophysical test called preferential hyperacuity perimetry, which assesses a person’s ability to any misalignments of visual objects, may permit a more sensitive diagnosis of Metamorphopsia. Treatment and Prognosis"}, {"id": "pubmed23n0818_11670", "title": "Bilateral macular edema in a patient treated with tamoxifen: a case report and review of the literature.", "score": 0.009174311926605505, "content": "We present a case of a 41-year-old female patient with progressive bilateral visual loss. On examination, her best corrected visual acuity (BCVA) in her right eye was 3/10 and her BCVA in her left eye was 2/10. Fundus and optical coherence tomography examination revealed severe bilateral macular edema. She had been diagnosed with breast cancer 6 years ago and was receiving tamoxifen at a dosage of 20 mg/day ever since. Tamoxifen therapy was discontinued, and the patient received 250 mg of acetazolamide three times a day for a period of 1 month. Both foveae regained their normal contour within 2 months, and her vision was restored to 10/10 BCVA 3 months later. To our knowledge, this is the first case reported where bilateral intraretinal macular edema is the only retinal manifestation in a patient on oral tamoxifen. "}, {"id": "pubmed23n0302_4975", "title": "[Surgical removal of submacular neovascularization membranes].", "score": 0.009174311926605505, "content": "New techniques for vitreoretinal surgery enable the removal of submacular neovascular membranes inaccessible to laser coagulation. The present study describes the anatomical and functional results following membrane removal. Between January 1994 and June 1995, submacular neovascular membranes were removed from 56 eyes with age-related exudative macular degeneration. Thirty-one eyes had classic subfoveal membranes; 19, occult membranes; and 6 eyes, disciform fibrotic changes that had already persisted for more than 6 months. Surgery led to improved vision in 20 of the 56 eyes (36%). In 26 of the 56 eyes (44%) vision at the last follow-up was no different from preoperative vision: in 10 eyes (17%) visual acuity declined. In the group of 6 eyes with a 6-month history of exudative maculopathy, surgery produced a slight improvement in vision in most patient. Complications arose in 8 of the 56 eyes (14%), namely choroidal hemorrhage (2 eyes), a macular hole (1 eye), and peripheral retinal detachment (5 eyes). Recurrence of the membrane was seen in 5 eyes (9%). Surgical removal of submacular membranes can often lead to stabilization or even improvement of vision. Owing to the relatively high rate of complications, however, the indications must be rigorously appraised on a case-by-case basis."}, {"id": "pubmed23n0059_11482", "title": "[Preventive treatment using laser of age-related macular degeneration of the contralateral eye after age-related macular degeneration of the first eye].", "score": 0.00909090909090909, "content": "Since 1982, and with informed patient consent, we have photocoagulated confluent drusen and limited serous pigment epithelium detachment (SPED) in the fellow eye of ten patients suffering from advanced, disciform type, age-related macula degeneration (ARMD). This treatment was only carried out on appearance of metamorphopsia. Photocoagulation was performed with either the green ray of the argon laser, or the yellow ray of a dye laser. Spots of about 200 microns were placed in a grid-like fashion among the drusen. No complications were observed due to the treatment. The follow-up period on these ten patients, eight women and two men, mean age 77 years, was two to eight years, and the three patients have died. The drusen disappeared completely in three patients and partially in one. The functional results seemed favorable in three cases. In one case of confluent drusen associated with SPED and serous retinal detachment, vision improved remarkably from 0.3 to 0.5 with a Parinaud 2, with a follow-up of five years. In another case, the improvement was from 0.4 to 0.7 but the patient died after only a few months. In another case, vision has been stable for five years. The vision of the seven remaining patients deteriorated; three cases showed central areolar sclerosis, and one case a localised new vessel with vision less than 0.1. In three cases vision dropped to 0.2 and Parinaud 6, but they have been stable for at least four years (eight years for one patient).(ABSTRACT TRUNCATED AT 250 WORDS)"}, {"id": "pubmed23n1124_9880", "title": "A case of Nd:YAG laser-induced traumatic macular hole with good visual prognosis after vitrectomy with inverted internal limiting membrane technique.", "score": 0.009009009009009009, "content": "To report an accidental case of traumatic macular hole caused by Nd:YAG laser in a dermatology clinic. A 24-year-old woman sustained a laser injury to her right eye while practicing a dermatologic treatment using a Nd:YAG laser without wearing protective goggles. She noticed sudden-onset and progressing visual loss in her right eye and consulted an ophthalmologist 2 days after injury. The best-corrected visual acuity (BCVA) of her right eye decreased to 20/133. Fundus examination showed white parafoveal flecks with a central retinal hemorrhage and underlying serous retinal detachment. The retinal sensitivity in this lesion deteriorated. Two weeks later, a full-thickness macular hole (FTMH) developed in the affected eye. She was referred to Nagoya City University Hospital where the laser damage described was observed. The BCVA was 20/67. She underwent pars plana vitrectomy performed using the inverted internal limiting membrane (ILM) flap technique and gas tamponade. One week postoperatively, the FTMH closed, the BCVA in her right eye improved to 20/50, and the retinal sensitivity in the macular area mostly improved. The BCVA gradually improved and reached 20/25 9 months after the injury. Protective goggles must be worn when using an Nd:YAG laser in the laboratory or clinical setting. In the unfortunate event of a FTMH, early vitrectomy with an inverted ILM flap technique can be helpful to achieve a good visual prognosis."}, {"id": "pubmed23n0597_21915", "title": "Fellow eye findings of highly myopic subjects operated for retinal detachment associated with a macular hole.", "score": 0.009009009009009009, "content": "To identity anatomic risk factors involved in the onset of retinal complications causing decrease of visual acuity (VA) in the fellow eyes of highly myopic patients operated for retinal detachment with macular hole (RDMH). Cohort study. Ninety-eight patients (mean age, 51.5+/-8.0 years) with bilateral high myopia (mean myopia of the fellow eye, 20.4+/-5.5 diopters) affected by RDMH in the other eye at baseline. Evaluation of the anatomic features at baseline and during 84+/-2.7 months of follow-up by biomicroscopic examination, indirect binocular ophthalmoscopy, B-scan ultrasonography, and optical coherence tomography. Detection of anatomic features associated with onset of retinal complications causing decrease of VA during the follow-up period. The fellow eyes were divided into 2 groups according to the clinical features of the RDMH eyes: Group 1, presence of posterior vitreous detachment (PVD); and Group 2, presence of posterior vitreous schisis (PVS). At baseline, the incidence of PVD in group 1 was 31 of 47 eyes (65.9%) and the incidence of PVS in Group 2 was 42 of 51 eyes (82.3%). At the end of follow-up, group 1 eyes had a lower incidence of retinal complications causing visual decrease than group 2 eyes (group 1, 2/47 eyes; group 2, 9/51 eyes). Fellow eyes of RDMH cases with higher degree of myopia and peculiar vitreoretinal features including PVS, posterior epiretinal membrane, severe posterior staphyloma, and chorioretinal atrophy are more likely to develop retinal complications causing decrease of VA."}]}}}} {"correct_option": 3, "explanations": {"1": {"exist": true, "char_ranges": [[107, 186]], "word_ranges": [[19, 34]], "text": "We all agree that the patient seems to have Cushing's syndrome (we rule out 1)."}, "2": {"exist": true, "char_ranges": [[1119, 1191]], "word_ranges": [[184, 199]], "text": "answer 2 is false (abdominal CT is not a test to rule out s. Cushing's);"}, "3": {"exist": true, "char_ranges": [[1192, 1285]], "word_ranges": [[199, 215]], "text": "an imaging test should only be done once a clear diagnosis of hypercortisolism has been made."}, "4": {"exist": true, "char_ranges": [[323, 620]], "word_ranges": [[54, 100]], "text": "If hypercortisolism is confirmed, the origin should be sought; the ACTH measurement indicates the origin: suppressed in hypercortisolism of adrenal origin or prolonged use of corticoids and elevated or normal if the origin is pituitary or by ectopic ACTH secretion. (Therefore, 4 and 5 are false)."}, "5": {"exist": true, "char_ranges": [[323, 620]], "word_ranges": [[54, 100]], "text": "If hypercortisolism is confirmed, the origin should be sought; the ACTH measurement indicates the origin: suppressed in hypercortisolism of adrenal origin or prolonged use of corticoids and elevated or normal if the origin is pituitary or by ectopic ACTH secretion. (Therefore, 4 and 5 are false)."}}, "full_answer": "This question is a bit convoluted for the diagnosis of Cushing's syndrome but easy if the steps are clear. We all agree that the patient seems to have Cushing's syndrome (we rule out 1). For the diagnosis we have to measure urinary free cortisol, do a dexamethasone suppression test or measure nocturnal salivary cortisol. If hypercortisolism is confirmed, the origin should be sought; the ACTH measurement indicates the origin: suppressed in hypercortisolism of adrenal origin or prolonged use of corticoids and elevated or normal if the origin is pituitary or by ectopic ACTH secretion. (Therefore, 4 and 5 are false). If ACTH is low, a CT scan of the abdomen should be performed to locate the origin. If ACTH is normal or high, a pituitary MRI should be performed (pituitary adenomas responsible for Cushing's syndrome are very small and CT of the pituitary is less sensitive than MRI). It is very important to perform the order well to arrive at a proper diagnosis: 1st: confirm hypercortisolism or s. Cushing's syndrome, 2nd: measure ACTH to orient etiology. 3rd: imaging test according to ACTH levels. Therefore, answer 2 is false (abdominal CT is not a test to rule out s. Cushing's); an imaging test should only be done once a clear diagnosis of hypercortisolism has been made. Therefore, the true one is 3.", "full_answer_no_ref": "This question is a bit convoluted for the diagnosis of Cushing's syndrome but easy if the steps are clear. We all agree that the patient seems to have Cushing's syndrome (we [HIDDEN]). For the diagnosis we have to measure urinary free cortisol, do a dexamethasone suppression test, or measure nocturnal salivary cortisol. If hypercortisolism is confirmed, the origin should be sought; the ACTH measurement indicates the origin: suppressed in hypercortisolism of adrenal origin or prolonged use of corticoids, and elevated or normal if the origin is pituitary or by ectopic ACTH secretion ([HIDDEN]). If ACTH is low, a CT scan of the abdomen should be performed to locate the origin. If ACTH is normal or high, a pituitary MRI should be performed (pituitary adenomas responsible for Cushing's syndrome are very small and CT of the pituitary is less sensitive than MRI). It is very important to perform the order well to arrive at a proper diagnosis: 1st: confirm hypercortisolism or s. Cushing's syndrome, 2nd: measure ACTH to orient etiology. 3rd: imaging test according to ACTH levels. Therefore, answer 2 is [HIDDEN] (abdominal CT is not a test to [HIDDEN]); an imaging test should only be done once a clear diagnosis of hypercortisolism has been made. Therefore, the true one is [HIDDEN].", "full_question": "A 56-year-old female patient consulted for dorso-lumbar spine pain and progressive difficulty in performing usual tasks. In the last 5 years she gained weight, she has ecchymosis easily and arterial hypertension was detected. Physical examination: Obesity of central predominance, rounded facies, increased supraclavicular fat, decreased proximal muscle strength and some reddish striae in the abdomen. She has a blood glucose of 136 mg/dL and the radiological study showed osteoporosis and vertebral crushing. What do you think is the most coherent interpretation and attitude?", "id": 186, "lang": "en", "options": {"1": "Postmenopausal osteoporosis, type 2 diabetes mellitus and essential hypertension, with decreased strength due to diabetic polyneuropathy.", "2": "It is necessary to rule out Cushing's disease by dexamethasone suppression test and perform a cranial CT scan.", "3": "Suggest Cushing's. Determine urinary free cortisol and basal ACTH, which serves to orient its etiology and select the most appropriate imaging technique.", "4": "It looks like Cushing's. If basal ACTH is high, it may be due to corticosteroid use or an adrenal tumor, and an MRI should be performed.", "5": "Probably has Cushing's. If the basal ACTH is low, he probably has a pituitary micro-adenoma, and a cranial CT scan should be performed."}, "question_id_specific": 67, "type": "ENDOCRINOLOGY", "year": 2013, "rag": {"clinical_case_options": {"MedCorp": {"RRF-2": [{"id": "pubmed23n0356_4633", "title": "Subclinical Cushing's syndrome.", "score": 0.019049346879535557, "content": "Classic Cushing's syndrome is a rare disease with an estimated incidence of 1 case per 100,000 persons. With routine use of imaging techniques such as ultrasound and CT, adrenal masses are being detected with increased frequency. A substantial percentage of these incidentalomas are hormonally active, with 5% to 20% of the tumors producing glucocorticoids. Autonomous glucocorticoid production without specific signs and symptoms of Cushing's syndrome is termed subclinical Cushing's syndrome. With an estimated prevalence of 79 cases per 100,000 persons, subclinical Cushing's syndrome is much more common than classic Cushing's syndrome. Depending on the amounts of glucocorticoids secreted by the tumor, the clinical spectrum ranges from slightly attenuated diurnal cortisol rhythm to complete atrophy of the contralateral adrenal gland with lasting adrenal insufficiency after unilateral adrenalectomy. Patients with subclinical Cushing's syndrome lack the classical stigmata of hypercortisolism but have a high prevalence of obesity, hypertension, and type 2 diabetes. All patients with incidentally detected adrenal masses scheduled for surgery must undergo testing for subclinical Cushing's syndrome to avoid postoperative adrenal crisis. The best screening test to uncover autonomous cortisol secretion is the short dexamethasone suppression test. Because the adrenal origin of a pathologic cortisol secretion is anticipated, the author prefers a higher dexamethasone dose (3 mg instead of 1 mg) to reduce false-positive results. A suppressed serum cortisol level of less than 3 micrograms/dL (80 nmol/L) after dexamethasone excludes significant cortisol secretion by the tumor. A serum cortisol level greater than 3 micrograms/dL requires further investigation, including confirmation by high-dose dexamethasone (8 mg) suppression testing, a CRH test, and analysis of diurnal rhythm. Determination of urinary free cortisol is less useful because increased values are a late finding usually associated with emerging clinical signs of Cushing's syndrome. Patients with suppressed plasma ACTH in response to CRH generally have adrenal insufficiency after surgery and require adequate perioperative and postoperative substitution therapy. Whether patients with subclinical Cushing's syndrome should undergo adrenalectomy is a matter of debate. The author performs surgery in young patients (< 50 years), in patients with suppressed plasma ACTH, and in patients with a recent history of weight gain, substantial obesity, arterial hypertension, diabetes mellitus, and osteopenia. In completely asymptomatic patients with normal plasma ACTH concentrations and in patients older than 75 years, the author recommends a nonsurgical approach. A large prospective randomized study is necessary to evaluate the benefits of surgery versus conservative treatment in patients with subclinical Cushing's syndrome."}, {"id": "pubmed23n0817_15153", "title": "Screening for Cushing's syndrome: is it worthwhile?", "score": 0.017095234703326447, "content": "Cushing's syndrome (CS) is a rare disease characterized by a collection of signs and symptoms, also common in the general population without elevated cortisol secretion. During the last years more patients with CS are identified earlier and with milder disease. Many of these patients are diagnosed during screening efforts performed for certain or isolated complaints like weight gain, diabetes mellitus (DM), hypertension, osteoporosis, elevated white blood cell counts and more. In this review article the most popular screening test performed in the studies cited was the 1-mg dexamethasone suppression test. Cushing is not frequent enough to support the use of routine screening in patients with morbid obesity and type 2 DM. Also only 1% of hypertensive patients have secondary hypertension due to CS. However, screening should be considered in young patients with resistant DM and/or hypertension. Among patients with osteoporosis and vertebral fractures up to 5% were diagnosed with subclinical hypercortisolism; most of these had adrenal adenoma. Screening for CS is important in subjects with adrenal incidentaloma, and many studies show a high prevalence (~10%) of Cushing or subclinical CS in these patients."}, {"id": "pubmed23n0783_14121", "title": "Cushing's disease: establishing the diagnosis and management approach.", "score": 0.015518967627099789, "content": "A 64 year old lady, with a background history of type 2 diabetes mellitus and hypertension, presented with general deterioration of general health, poor glycemic control, difficulty in controlling blood pressure and difficulty in walking. She had past medical history of adenocarcinoma of the oesophagus, treated with surgery and subsequent chemotherapy. General examination revealed high blood glucose and blood pressure and a Cushingoid facies. Overnight dexamethasone suppression test and urinary free cortisol levels confirmed Cushing's syndrome and High dose dexamethasone suppression test showed partial suppression. CT scan of the abdomen showed bilateral hyperplasia of the adrenals with nodularity on the left side, raising the possibility of an adrenal adenoma. ACTH levels were elevated thereby ruling out autonomously functioning adrenal nodule, however increasing the possibility of ectopic ACTH secretion due to the previous medical history. MRI of the pituitary confirmed the presence of an adenoma, thereby pointing to the diagnosis of pituitary dependant Cushing's disease. The patient could not undergo further invasive investigation or surgery due to septicaemia. Medical management of Cushing's syndrome was resorted to in the interim with Ketoconazole, showing excellent response. This case depicts the need for a high index of suspicion for the diagnosis, the importance of organizing specific investigations in the appropriate order to arrive at a diagnosis and an effective management plan."}, {"id": "pubmed23n1115_13389", "title": "[Morbidity and mortality in Cushing's syndrome].", "score": 0.015270734976977327, "content": "Endogenous Cushing's syndrome is a rare endocrine disorder that is fatal if left untreated. It can be distinguished into adrenocorticotropic hormone (ACTH)-dependent (central and ectopic Cushing's syndrome) and ACTH-independent subtypes (unilateral or bilateral adrenal adenomas). The clinical presentation of patients includes typical stigmata of cortisol excess with physical symptoms of catabolic metabolism (myopathy, striae, parchment skin, osteoporosis) and components of metabolic syndrome (diabetes mellitus, obesity, arterial hypertension, hypercholesterolemia). Biochemical diagnosis is performed in three steps: 1. Confirmation of the diagnosis by 1‑mg dexamethasone suppression test, 24‑h urine free cortisol, and measurement of late-night salivary cortisol. 2. Differentiation of ACTH-dependent Cushing's syndrome from ACTH-independent adrenal Cushing's syndrome by measurement of plasma ACTH. 3. Further subtyping by corticotropin-releasing hormone (CRH) test, inferior petrosal sinus sampling, and imaging modalities. Therapeutic decisions are made on an interdisciplinary basis. First-line therapy for all subtypes is surgery when possible; additional options for all forms include drug therapy and bilateral adrenalectomy. Despite adequate treatment, Cushing's syndrome is associated with increased long-term morbidity and mortality. Interdisciplinary and multimodal therapy management is necessary in the long term to positively influence mortality and reduced quality of life."}, {"id": "pubmed23n1121_2204", "title": "Coexistence of Cushing Disease With a Solitary Adrenocorticotrophic Hormone-Dependent Adrenal Adenoma.", "score": 0.015079365079365081, "content": "We report a 49-year-old woman who had minimal features of Cushing syndrome and an incidentally discovered adrenal adenoma. She was subsequently diagnosed with pituitary-dependent Cushing syndrome. Laboratory and imaging studies including serum cortisol, plasma adrenocorticotrophic hormone (ACTH), high dose dexamethasone test, corticotropin-releasing hormone test, computed tomography (CT) scan, and magnetic resonance imaging were performed. A 49-year-old woman was admitted for urosepsis. An abdominal CT scan performed during the urosepsis workup showed a 2.7-cm right adrenal adenoma. She denied any abdominal striae or other symptoms. Physical examination showed normal vital signs, minimal facial fullness without central obesity, and striae. Laboratory results were as follows: 24-hour-urine cortisol 294 μg (reference 4.0-50.0), midnight serum cortisol 23.0 μg/dL (reference < 7.5), and plasma ACTH level 39 pg/mL (reference 5-27). A corticotropin-releasing hormone stimulation test showed >20% rise in serum cortisol and >35% rise in ACTH levels. A pituitary magnetic resonance image showed a 5 mm pituitary lesion. The patient underwent transsphenoidal pituitary surgery, which confirmed an ACTH-secreting lesion. Postoperatively, she required hydrocortisone replacement for the next 10 months. A follow-up adrenal CT performed 6 months later showed a decrease in the size of the adrenal adenoma (1.8 cm). This case highlights the importance of recognizing the coexistence of ACTH-dependent Cushing disease with an adrenal adenoma and partial ACTH dependency of the adrenal adenoma."}, {"id": "pubmed23n0634_13742", "title": "[Prevalence, etiology and clinical findings of Cushing's syndrome].", "score": 0.015040515040515042, "content": "Endogenous Cushing's syndrome is a very rare entity, with an incidence of 2-4 cases per million inhabitants per year. Cases caused by ectopic ACTH secretion are under-diagnosed. Cushing's disease is the most frequent cause of endogenous Cushing's syndrome, which is 5 or 6 times more frequent than adrenal Cushing's syndrome, with an incidence of between 1.2 and 2.4 cases per million inhabitants per year. Cushing's disease is 3-8 times higher in women than in men. The frequency of adrenal tumors is 3 times higher in women, while that of Cushing's syndrome due to adrenal tumors is 3-5 times higher. Age at diagnosis of Cushing's syndrome varies according to the etiology. Most cases of Cushing's disease are due to a pituitary adenoma, although the tumor may not be visible on the available imaging techniques. ACTH-independent Cushing's syndrome is found in 20% of cases and is most frequently due to adenomas (10%) or adrenal carcinomas (8). Bilateral micronodular hyperplasia and macronodular hyperplasia are infrequent entities, representing less than 10% of all cases of ACTH-independent Cushing's syndrome. Both familial and sporadic forms exist: the familial form, or Carney complex, and ACTH-independent bilateral macronodular hyperplasia, in which the size of the adrenal glands is considerably enlarged. The signs and symptoms of Cushing's syndrome are a direct result of long-term exposure to excessive glucocorticoids. Most signs and symptoms are highly prevalent in the general population (hypertension, central obesity, diabetes mellitus or carbohydrate intolerance, osteoporosis, and characteristic phenotypical alterations)."}, {"id": "pubmed23n0270_19530", "title": "[Cushing's syndrome: diagnostic exploration].", "score": 0.01435472739820566, "content": "The diagnosis of Cushing's syndrome is one of the most perplexing and controversial problems in endocrinology. However, significant advances in the diagnosis procedures have been made in the past decade. The diagnostic studies involved in the evaluation of patients with suspected Cushing's syndrome fall into two categories: confirming the presence of true hypercortisolism and establishing the precise aetiology. Diagnosis of Cushing's syndrome: ambulatory screening relies on the overnight 1 mg dexamethasone test. Negative tests are confirmed by measuring cortisol in two 24-hour urine samples. If cortisol excretion is slightly above normal, a 48-hour low-dose dexamethasone suppression test or an intravenous infusion dexamethasone suppression test are required. Diagnosis of the aetiology of Cushing's syndrome: the first step is to establish if the hypercortisolism is ACTH-dependent or not. This step is solved by measuring plasma ACTH and cortisol in the late afternoon. Computed tomography scanning of the adrenal glands is required in ACTH-independent Cushing's syndrome. A unilateral tumour will be demonstrated in most of cases. If bilateral lesions are found, dynamic testing using cortisol releasing factor and/or metyrapone must be performed to confirm the ACTH-independency of the syndrome. In ACTH-dependent Cushing's syndrome, the major difficulty is to distinguish between a pituitary source and an ectopic source of ACTH secretion. Magnetic resonance imaging of the pituitary with gadolinium enhancement must be preferred to computed tomography scanning but its sensitivity is not better than 70-80% and false positives can occur. When no macroscopic pituitary lesion can be detected, bilateral inferior petrosal sinus sampling coupled to CRH injection for ACTH measurement will indicate the source of ACTH secretion. If this test indicates the patient has Cushing's disease, pituitary trans-sphenoidal surgery can be performed. If the test indicates the patient has ectopic ACTH-secretion, a cervico-thoraco-abdominal scanning is necessary to identify the tumour. In the case of occult tumour the hypercortisolism must be controlled by pharmacological agents and the imaging investigations must be repeated at appropriate intervals."}, {"id": "pubmed23n0591_1986", "title": "[Management of hypercortisolism].", "score": 0.01404394825447457, "content": "Cushing's syndrome is a rare but frequently considered disease. Its diagnosis can lead to some difficulties, including confirming the effective endogenous hypercortisolism and determining its cause. The severity of this disease, the diversity of its complications and the multiple therapeutic options make its management challenging. The aim of this review is to present the most recent data about management of Cushing's syndrome, especially diagnostic approaches and therapeutic options. Our references were obtained by screening MEDLINE database from 1996 to 2006. We also included some anterior reviews and consensus statements. We retained the following points: midnight salivary cortisol is a useful tool in the diagnosis of Cushing's syndrome; the desmopressin test can help to distinguish between Cushing's syndrome and \"pseudoCushing's\" due to alcohol consumption or psychiatric disorders; cavernous sinus and inferior petrosal sinus sampling is indicated in the evaluation of ACTH-dependent Cushing's syndromes when pituitary imaging is normal or equivocal or when dynamic tests are contradictory; multislice computed-tomography of the chest and the abdomen and somatostatin analogue scintigraphy, eventually combined, are the best imaging procedures in occult ectopic ACTH syndromes; patients with Cushing's disease should be referred to a neurosurgeon experienced in corticotroph adenomas surgery; metabolic consequences of Cushing's syndrome, such as cardiovascular risk factors and osteoporosis need an aggressive treatment. The incidence of Cushing's syndrome is only 1/100000 per year. However, hypercortisolism is diagnosed by systematic evaluation in 2 to 5% of patients with poorly controlled type 2 diabetes and adrenal incidentalomas. Endocrinological management of the disease improves metabolic disorders in these patients. If these results are confirmed, screening for Cushing's syndrome should be systematically performed in these populations."}, {"id": "wiki20220301en004_55231", "title": "Cushing's syndrome", "score": 0.013584706856325731, "content": "When Cushing's syndrome is suspected, either a dexamethasone suppression test (administration of dexamethasone and frequent determination of cortisol and ACTH level), or a 24-hour urinary measurement for cortisol offers equal detection rates. Dexamethasone is a glucocorticoid and simulates the effects of cortisol, including negative feedback on the pituitary gland. When dexamethasone is administered and a blood sample is tested, cortisol levels >50 nmol/l (1.81 μg/dl) would be indicative of Cushing's syndrome because an ectopic source of cortisol or ACTH (such as adrenal adenoma) exists which is not inhibited by the dexamethasone. A novel approach, recently cleared by the US FDA, is sampling cortisol in saliva over 24 hours, which may be equally sensitive, as late-night levels of salivary cortisol are high in cushingoid patients. Other pituitary hormone levels may need to be ascertained. Performing a physical examination to determine any visual field defect may be necessary if a"}, {"id": "wiki20220301en015_42166", "title": "Cushing's disease", "score": 0.01347210957916554, "content": "ACTH blood test Once Cushing's syndrome has been diagnosed, the first step towards finding the cause is measuring plasma adrenocorticotropic hormone (ACTH) concentration. A concentration consistently below 1.1 pmol/L is classified as corticotropin-independent and does not lead to a diagnosis of Cushing's disease. In such cases, the next step is adrenal imaging with CT. If plasma corticotropin concentrations are consistently above 3.3 pmol/L, then corticotropin-dependent Cushing's syndrome is most likely. Any intermediate values need to be cautiously interpreted and a corticotropin-releasing hormone (CRH) test is advised in order to confirm corticotropin dependency. If corticotropin-dependent Cushing's syndrome is determined then the next step is to distinguish between Cushing's disease and ectopic corticotropin syndrome. This is done via a combination of techniques including CRH, high-dose DST, BIPSS, and pituitary MRI."}, {"id": "pubmed23n0392_16373", "title": "A natural history of adrenocorticotropin-independent bilateral adrenal macronodular hyperplasia (AIMAH) from preclinical to clinically overt Cushing's syndrome.", "score": 0.013290356764394207, "content": "A 49-year-old man was referred to our hospital for the treatment of gallstones in 1993. Bilateral adrenal nodular masses were detected incidentally by abdominal computed tomography. He had no clinical signs of Cushing's syndrome such as central obesity, striae of skin and diabetes mellitus. We performed cholecystectomy and partial adrenalectomy of right adrenal gland as a biopsy, and diagnosed him as preclinical Cushing's syndrome due to adrenocorticotropin-independent bilateral adrenal macronodular hyperplasia (AIMAH) based on endocrinological and histological examinations. We followed him up for 7 years. During the observation period, the sizes of both adrenal glands increased gradually, and finally serum cortisol level increased beyond normal range, and he showed a Cushingoid appearance such as moon face and central obesity. His skin became atrophic and very fragile, and the bone mineral density of his lumbar spine was extremely low. Serum cortisol level was elevated, and plasma ACTH level was always suppressed. Urinary excretion of 17-hydroxycorticosteroid and free cortisol were increased. Diurnal rhythm of cortisol and ACTH was completely lost and high dose (8 mg/day) dexamethasone did not suppress urinary 17-hydroxycorticosteroid excretion. He became clinically overt Cushing's syndrome. We recommended total adrenalectomy, but he refused it. It is important to know the natural history of preclinical Cushing's syndrome due to AIMAH when choosing an adequate treatment."}, {"id": "wiki20220301en000_30619", "title": "Adrenal gland", "score": 0.013189269746646796, "content": "Corticosteroid overproduction Cushing's syndrome Cushing's syndrome is the manifestation of glucocorticoid excess. It can be the result of a prolonged treatment with glucocorticoids or be caused by an underlying disease which produces alterations in the HPA axis or the production of cortisol. Causes can be further classified into ACTH-dependent or ACTH-independent. The most common cause of endogenous Cushing's syndrome is a pituitary adenoma which causes an excessive production of ACTH. The disease produces a wide variety of signs and symptoms which include obesity, diabetes, increased blood pressure, excessive body hair (hirsutism), osteoporosis, depression, and most distinctively, stretch marks in the skin, caused by its progressive thinning."}, {"id": "pubmed23n0750_19523", "title": "Challenges in the diagnostic work-up and management of patients with subclinical Cushing's syndrome and bilateral adrenal masses.", "score": 0.013122829685859646, "content": "To review the challenges encountered in the diagnostic work-up and management of patients with subclinical Cushing's syndrome (SCS) and bilateral adrenal masses to aid in the case description of a patient with SCS and adrenocorticotropic hormone (ACTH)-independent macronodular adrenal hyperplasia (AIMAH). We describe our experience managing a patient with AIMAH and SCS. This case report is followed by an extensive review of the literature regarding differential diagnoses, work-up including adrenal venous sampling (AVS), and treatment of SCS with bilateral adrenal masses. A 51-year-old female who was diagnosed with recent onset hypertension and diabetes mellitus type 2 was evaluated for adrenal masses discovered incidentally on computed tomography (CT). She did not have any Cushingoid features. Magnetic resonance imaging (MRI) of abdomen was performed for further evaluation. Hormonal evaluation came back consistent with SCS. The AVS results were consistent with bilateral autonomous cortisol hypersecretion without lateralization. Collectively, the findings favored the diagnosis of bilateral AIMAH. A left adrenalectomy was performed, and the patient's clinical response was favorable with improvement in blood pressure (BP) accompanied by significant weight loss. Follow-up hormonal testing for autonomous cortisol hypersecretion was within the target range. AIMAH is a rare cause of SCS. AVS is a useful diagnostic tool that helps localize the source of autonomous cortisol hypersecretion in ACTH-independent SCS with bilateral adrenal masses, especially if radiological features are inconclusive. Patients undergoing unilateral adrenalectomy should be followed for monitoring of clinical response, as well as progression of AIMAH in the contralateral adrenal gland."}, {"id": "wiki20220301en023_25617", "title": "Corticotropic cell", "score": 0.012933710422846106, "content": "Associated diseases Cushing's Disease Corticotropic cells can have detrimental effects on the body if they express too much or too little ACTH. One such example is Cushing's disease, which can result from overproduction of ACTH in the corticotropes due to pituitary tumors known as corticotroph adenomas; this is the cause for roughly two-thirds of those diagnosed with Cushing's disease. It is also possible that this disease can result from production of ACTH in a non-pituitary tumor, known as ectopic production, or the adrenal glands can overproduce cortisol due to an adrenal tumor. This overproduction of ACTH causes an increase in cortisol levels due to increased glucocorticoid synthesis in the adrenal cortex resulting in several associated symptoms. Symptoms of Cushing's disease include: Fatty deposits in the neck or back Stretch marks (striae) Fatigue Osteoporosis Weakened immune system Hypertension"}, {"id": "wiki20220301en004_55233", "title": "Cushing's syndrome", "score": 0.01276003870343493, "content": "When any of these tests is positive, CT scanning of the adrenal gland and MRI of the pituitary gland are performed to detect the presence of any adrenal or pituitary adenomas or incidentalomas (the incidental discovery of harmless lesions). Scintigraphy of the adrenal gland with iodocholesterol scan is occasionally necessary. Occasionally, determining the ACTH levels in various veins in the body by venous catheterization, working towards the pituitary (petrosal sinus sampling) is necessary. In many cases, the tumors causing Cushing's disease are less than 2 mm in size and difficult to detect using MRI or CT imaging. In one study of 261 patients with confirmed pituitary Cushing's disease, only 48% of pituitary lesions were identified using MRI prior to surgery. Plasma CRH levels are inadequate at diagnosis (with the possible exception of tumors secreting CRH) because of peripheral dilution and binding to CRHBP."}, {"id": "pubmed23n0555_1604", "title": "Cushing's disease.", "score": 0.012324395701133403, "content": "Cushing's disease, i.e., pituitary ACTH-secreting adenoma causing excess glucocorticoid secretion, is a rare disease with significant mortality and morbidity. Timely diagnosis and appropriate treatment can alter the course of the disease and are therefore mandatory. First step of the diagnostic work-up is the endogenous glucocorticoid excess by measurement of urinary free cortisol, cortisol circadian rhythmicity or suppression by low doses of dexamethasone. In patients with equivocal results, second line tests, such as the dexamethasone-suppressed CRH test and desmopressin stimulation, usually enable the diagnosis to be confirmed. Measurement of plasma ACTH then allows the distinction between ACTH-dependent (e.g., pituitary or extrapituitary neuroendocrine tumors) and ACTH-independent causes (e.g., adrenal tumors). The last step in the diagnostic algorithm is often the most fraught with problems as the distinction between Cushing's disease and ectopic ACTH secretion relies on judicious interpretation of several diagnostic procedures. Positive responses to stimulation with CRH and inhibition by high doses of dexamethasone, if concurrent, enable a pituitary origin to be established whereas conflicting results call for inferior petrosal sinus sampling, the latter to be performed in experienced centres only. Visualisation of the tumor at pituitary imaging is helpful but not required for the diagnosis, as microadenomas often remain undectected by MRI and/or CT scan and, on the other hand, visualisation of a non-secreting incidentaloma may be misleading. Surgical removal of the pituitary tumor is the optimal treatment choice and should be attempted in every patient. Surgical failures as well as relapses can be treated by radiotherapy, medical therapy or, if necessary, bilateral adrenalectomy. Finally, patients cured of Cushing's disease require long-term monitoring given the risk of relapse and clinical burden of associated ailments."}, {"id": "pubmed23n0648_18952", "title": "The frequency of type 2 diabetic patients who meet the endocrinological screening criteria of subclinical Cushing's disease.", "score": 0.01205913334059787, "content": "Cushing's syndrome, including its mild form/state of adrenal-dependent subset (subclinical Cushing's syndrome; subCS), is known to enhance glucose intolerance, hypertension and obesity. Recently, subclinical Cushing's disease (subCD) has been identified, but its prevalence and the extent of consequent metabolic derangement are unclear. We screened 90 type 2 diabetic patients hospitalized in our department for subCD, according to the diagnostic guideline proposed by the working group of Japanese Ministry of Health, Welfare and Labor in 2006. Plasma ACTH and cortisol levels in the morning and at midnight were determined, and overnight 0.5 mg dexamethasone suppression test (DST) was performed. Those who showed poor cortisol suppression in DST underwent the desmopressin (DDAVP) test. Fifty-seven patients (63.3%) demonstrated abnormally high midnight cortisol levels (>or=2.5 microg/dL), while only nine of them failed to suppress plasma cortisol levels to <3 microg/dL after DST. Although none of the eight patients who underwent the DDAVP test demonstrated the anticipated paradoxical rise in plasma ACTH, these eight patients (8.9%) endocrinologically met the screening criteria of subCD. Since a considerable percentage of pituitary adenomas causing overt Cushing's disease are not identifiable in magnetic resonance imaging, many of those causing subCD may also be unidentifiable. Further follow-up studies including confirmatory testing and pituitary imaging are necessary."}, {"id": "article-20175_17", "title": "Cushing Disease -- Evaluation", "score": 0.01191016333938294, "content": "The most accurate test used to differentiate a pituitary adenoma from ectopic or adrenal Cushing syndrome is inferior petrosal sinus sampling. [12] [18] This invasive method measures the difference in ACTH level found in the inferior petrosal sinus (where the pituitary gland drains) compared to the periphery. [12] [18] A basal central to the peripheral ratio of over 3:1 when CRH is administered confirms the diagnosis of Cushing disease. [18] This test is considered the gold standard in diagnosing Cushing disease because it has a sensitivity and specificity of nearly 94%. Still, it is rarely used in clinical practice due to its high cost, invasiveness, rare but serious complications, and the required special expertise to perform."}, {"id": "Surgery_Schwartz_11118", "title": "Surgery_Schwartz", "score": 0.011906726062805916, "content": "is diagnostic 1) Overnight DST2) 24-hour urinary free cortisol3) 11:00 pm salivary cortisol1) Plasma ACTH2) High-dose DST and urinary cortisolConfirm the diagnosisACTH gradient?Determine source of hypercortisolismDecreased ACTHLack of suppressionCT scan adrenalsIncreased ACTHPositive Increased ACTHLack of suppressionEquivocalresultsFurther testingBilateral petrosalvein samplingAdrenalsourcePituitarysourceEctopic ACTHsourceSTEPS IN DIAGNOSISDIAGNOSTIC STUDIESYesNoFigure 38-44. Diagnosis of Cushing’s syndrome. ACTH = adrenocorticotropic hormone; CT = computed tomography; DST = dexamethasone suppression test.Brunicardi_Ch38_p1625-p1704.indd 169001/03/19 11:22 AM 1691THYROID, PARATHYROID, AND ADRENALCHAPTER 38of a pituitary tumor. In patients suspected of having ectopic ACTH production, CT or MRI scans of the chest and anterior mediastinum are performed first, followed by imaging of the neck, abdomen, and pelvis if the initial studies are negative.Treatment Laparoscopic"}, {"id": "article-20178_12", "title": "Cushing Syndrome -- Evaluation", "score": 0.011817688204669583, "content": "Serum ACTH levels can differentiate ACTH-dependent Cushing syndrome (elevated ACTH or inappropriately normal ACTH) from ACTH-independent (low ACTH level) Cushing syndrome. In patients with ACTH-dependent Cushing syndrome, the high-dose dexamethasone suppression test done by giving 8 mg dexamethasone by mouth at 2300 h and checking cortisol the next day at 0800 h, can differentiate pituitary ACTH from an ectopic ACTH source. A high-dose dexamethasone suppression test will decrease cortisol level by 50 % if the ACTH source is a pituitary adenoma, but not if ACTH is secreted by an ectopic tumor (e.g., oat cell lung carcinoma). Pituitary MRI, unenhanced CT scan of the adrenals, and chest X-ray and CT are also useful to localize the pathology."}, {"id": "wiki20220301en004_55212", "title": "Cushing's syndrome", "score": 0.011465005410780615, "content": "Cushing's syndrome is caused by either excessive cortisol-like medication, such as prednisone, or a tumor that either produces or results in the production of excessive cortisol by the adrenal glands. Cases due to a pituitary adenoma are known as Cushing's disease, which is the second most common cause of Cushing's syndrome after medication. A number of other tumors, often referred to as ectopic due to their placement outside the pituitary, may also cause Cushing's. Some of these are associated with inherited disorders such as multiple endocrine neoplasia type 1 and Carney complex. Diagnosis requires a number of steps. The first step is to check the medications a person takes. The second step is to measure levels of cortisol in the urine, saliva or in the blood after taking dexamethasone. If this test is abnormal, the cortisol may be measured late at night. If the cortisol remains high, a blood test for ACTH may be done."}, {"id": "pubmed23n0737_1554", "title": "Differential diagnosis of adrenocorticotropic hormone-independent Cushing syndrome: role of adrenal venous sampling.", "score": 0.011430208863983698, "content": "To outline the potential role for adrenal venous sampling in the diagnosis and management of adrenocorticotropic hormone (ACTH)-independent Cushing syndrome (CS). We present a case description and discuss the management of a 59-year-old woman with an 8-year history of weight gain, centripetal obesity, a round plethoric face, skin thinning, easy bruising, hirsutism, and progressive muscle weakness. The patient reported a prior personal history of asthma, type 2 diabetes mellitus, hypertension, dyslipidemia, and bilateral leg ulcers, but she denied having any personal or family history of endocrinopathy and was not taking any corticosteroid medication. Elevated midnight serum cortisol, failure to suppress cortisol levels with a low-dose dexamethasone suppression test, and undetectable plasma ACTH all indicated ACTH-independent CS. Additional investigations including dynamic tests and adrenal imaging were supported by adrenal venous sampling in order to make a diagnosis and formulate a management plan. She was ultimately noted to have bilateral functioning adrenal nodules (adenoma and adenolipoma) and underwent successful bilateral laparoscopic adrenalectomy, with postoperative glucocorticoid and mineralocorticoid replacement. Adrenal venous sampling may be an important step in the differential diagnosis of CS and localization of the source of cortisol excess. It may distinguish pheochromocytoma or benign nonfunctioning adrenal nodules from cortisol-secreting adenomas and may avoid unnecessary bilateral adrenalectomy. It can also ensure that the correct operation is completed, if required, and thus avoid the increased morbidity and mortality associated with repeated surgical interventions."}, {"id": "wiki20220301en004_55229", "title": "Cushing's syndrome", "score": 0.011201079622132255, "content": "Strictly, Cushing's syndrome refers to excess cortisol of any etiology (as syndrome means a group of symptoms). One of the causes of Cushing's syndrome is a cortisol-secreting adenoma in the cortex of the adrenal gland (primary hypercortisolism/hypercorticism). The adenoma causes cortisol levels in the blood to be very high, and negative feedback on the pituitary from the high cortisol levels causes ACTH levels to be very low. Cushing's disease refers only to hypercortisolism secondary to excess production of ACTH from a corticotroph pituitary adenoma (secondary hypercortisolism/hypercorticism) or due to excess production of hypothalamus CRH (Corticotropin releasing hormone) (tertiary hypercortisolism/hypercorticism). This causes the blood ACTH levels to be elevated along with cortisol from the adrenal gland. The ACTH levels remain high because the tumor is unresponsive to negative feedback from high cortisol levels."}, {"id": "InternalMed_Harrison_26973", "title": "InternalMed_Harrison", "score": 0.011113296616837135, "content": "Differential Diagnosis The evaluation of patients with confirmed Cushing’s should be carried out by an endocrinologist and begins with the differential diagnosis of ACTH-dependent and ACTH-independent cortisol excess (Fig. 406-10). Generally, plasma ACTH levels are suppressed in cases of autonomous adrenal cortisol excess, as a consequence of enhanced negative feedback to the hypothalamus and pituitary. By contrast, patients with ACTH-dependent Cushing’s have normal or increased plasma ACTH, with very high levels being found in some patients with ectopic ACTH syndrome. Importantly, imaging should only be used after it is established whether the cortisol excess is ACTH-dependent or ACTH-independent, because nodules in the pituitary or the adrenal are a common finding in the general population. In patients with confirmed ACTH-independent excess, adrenal imaging is indicated (Fig. 406-11), preferably using an unenhanced computed tomography (CT) scan. This allows assessment of adrenal"}, {"id": "Neurology_Adams_5297", "title": "Neurology_Adams", "score": 0.01058870207806378, "content": "Cushing disease Described in 1932 by Cushing, this condition is only about one-fourth as frequent as acromegaly. A distinction is made between Cushing disease and Cushing syndrome, as indicated in Chap. 26. The former term is reserved for cases that are caused by the excessive secretion of pituitary ACTH, which, in turn, causes adrenal hyperplasia; the usual basis is a pituitary adenoma. Cushing syndrome refers to the effects of cortisol excess from any one of several sources—excessive administration of steroids (the most common cause), adenoma of the adrenal cortex, ACTH-producing bronchial carcinoma, and, very rarely, other carcinomas that produce ACTH. The clinical effects are the same in all of these disorders and include truncal obesity, hypertension, proximal muscle weakness, amenorrhea, hirsutism, abdominal striae, hyperglycemia, osteoporosis, and in some cases a characteristic mental disorder (see “Cushing Disease and Corticosteroid Psychoses” in Chap. 49)."}, {"id": "wiki20220301en004_55226", "title": "Cushing's syndrome", "score": 0.010537654552266768, "content": "Endogenous Endogenous Cushing's syndrome results from some derangement of the body's own system of cortisol secretion. Normally, ACTH is released from the pituitary gland when necessary to stimulate the release of cortisol from the adrenal glands. In pituitary Cushing's, a benign pituitary adenoma secretes ACTH. This is also known as Cushing's disease and is responsible for 70% of endogenous Cushing's syndrome. In adrenal Cushing's, excess cortisol is produced by adrenal gland tumors, hyperplastic adrenal glands, or adrenal glands with nodular adrenal hyperplasia. Tumors outside the normal pituitary-adrenal system can produce ACTH (occasionally with CRH) that affects the adrenal glands. This etiology is called ectopic or paraneoplastic Cushing's disease and is seen in diseases such as small cell lung cancer. Finally, rare cases of CRH-secreting tumors (without ACTH secretion) have been reported, which stimulates pituitary ACTH production."}, {"id": "pubmed23n0274_14036", "title": "[Cushing syndrome in children].", "score": 0.010519683313700245, "content": "In pediatric patients, endogenous Cushing syndrome is an infrequent condition almost always due to one of two conditions. 1) Adrenal gland tumors account for 70% of Cushing syndromes in young pediatric patients. They cause rapidly progressive hypercorticism not due to increased ACTH production (elevated plasma and urine cortisol levels, very low ACTH and LPH levels unchanged by dexamethasone, metyrapone or CRH). Imaging techniques determine the side and spread of the tumor and look for metastases. Following surgical removal, patients with indicators of malignant disease (tumor weight above 30 g, extracapsular spread or metastases, independently from pathological data) are given op'DDD. 2) Cushing disease occurs in peripubertal patients and causes overweight with delayed statural gain. ACTH production is increased (positive dexamethasone suppression test and provocative metopirone and CRH tests) as a result of a pituitary adenoma which should be looked for by magnetic resonance imaging and whose removal ensures recovery in 50% of cases. Other therapeutic tools include op'DDD, radiation to the pituitary, and bilateral adrenalectomy as the last resort given the high risk of post-adrenalectomy pituitary tumor (50% of pediatric patients). Other causes are exceedingly rare: primary nodular hyperplasia of the adrenal glands and production of ACTH by a nonpituitary tumor. Corticosteroid treatment is the most common cause of Cushing syndrome in children."}, {"id": "InternalMed_Harrison_26975", "title": "InternalMed_Harrison", "score": 0.010276287020473068, "content": "For ACTH-dependent cortisol excess (Chap. 403), a magnetic resonance image (MRI) of the pituitary is the investigation of choice, but it may not show an abnormality in up to 40% of cases because of small tumors below the sensitivity of detection. Characteristically, pituitary corticotrope adenomas fail to enhance following gadolinium administration on T1-weighted MRI images. In all cases of confirmed ACTH-dependent Cushing’s, further tests are required for the differential diagnosis of pituitary Cushing’s disease and ectopic ACTH syndrome."}, {"id": "wiki20220301en015_42168", "title": "Cushing's disease", "score": 0.010272013314609665, "content": "ACTH stimulation test An ACTH stimulation test involving administration of corticotropin-releasing hormone (CRH) or another agent can differentiate this condition from ectopic ACTH secretion. In a patient with Cushing's disease, the tumor cells will be stimulated to release corticotropin and elevated plasma corticotropin levels will be detected. This rarely occurs with ectopic corticotropin syndrome and thus is quite useful for distinguishing between the two conditions. If ectopic, the plasma ACTH and cortisol levels should remain unchanged; if this is pituitary related, levels of both would rise. The CRH test uses recombinant human or bovine-sequence CRH, which is administered via a 100μg intravenous bolus dose. The sensitivity of the CRH test for detecting Cushing's disease is 93% when plasma levels are measured after fifteen and thirty minutes. However, this test is used only as a last resort due to its high cost and complexity."}, {"id": "wiki20220301en015_42170", "title": "Cushing's disease", "score": 0.01016631173308995, "content": "Inferior petrosal sinus sampling IPSS (inferior petrosal sinus sampling) or BIPSS (bilateral IPSS) is a more accurate but invasive test used to differentiate pituitary from ectopic or adrenal Cushing's syndrome. A corticotropin gradient sample via BIPSS is required to confirm diagnosis when pituitary MRI imaging and biochemical diagnostic tests have been inconclusive. A basal central:peripheral ratio of over 2:1, or a ratio over 3:1 when CRH is administered, is indicative of Cushing's disease. This test has been the gold standard for distinguishing between Cushing's disease and ectopic corticotropin syndrome. The BIPSS has a sensitivity and specificity of 94% for Cushing's disease but it is usually used as a last resort due to its invasiveness, rare but serious complications, and the expertise required to perform it."}, {"id": "wiki20220301en086_19076", "title": "Dexamethasone suppression test", "score": 0.010114164904862579, "content": "Test Procedures There are several types of DST procedures: Overnight DST - An oral dose of dexamethasone is given between 11pm and midnight, and the cortisol level is measured at 8 - 9am the next morning Two-day DST - This involves giving an oral dose of dexamethasone at six-hourly intervals for 2 days, with the cortisol level measured 6 hours after the final dose was given Intravenous DST Dexamethasone-CRT test Interpretation Low-dose and high-dose variations of the test exist. The test is given at low (usually 1–2 mg) and high (8 mg) doses of dexamethasone, and the levels of cortisol are measured to obtain the results. A low dose of dexamethasone suppresses cortisol in individuals with no pathology in endogenous cortisol production. A high dose of dexamethasone exerts negative feedback on pituitary neoplastic ACTH-producing cells (Cushing's disease), but not on ectopic ACTH-producing cells or adrenal adenoma (Cushing's syndrome)."}, {"id": "article-20175_14", "title": "Cushing Disease -- Evaluation", "score": 0.010087285902503294, "content": "Two or more positive initial screening tests in a patient with a high pretest probability of Cushing disease confirm the biochemical diagnosis of Cushing syndrome. [12] [17] Once Cushing syndrome has been diagnosed, the first step toward finding the cause is by measuring a baseline plasma ACTH level. [15] A level consistently greater than 3.3 pmol/L is classified as corticotropin-dependent. [15] To differentiate Cushing disease from ectopic corticotropin syndrome, a corticotropin-releasing hormone (CRH) test is needed. [15] In a patient with Cushing disease, the administered CRH stimulates additional corticotropin release, resulting in an elevated plasma corticotropin level. [15] The sensitivity of the CRH test for detecting Cushing disease is 93% when plasma levels are measured at fifteen and thirty minutes. [15] Alternatively, a high-dose 48-hour dexamethasone suppression test or pituitary magnetic resonance imaging (MRI) can be used. [15]"}]}}}} {"correct_option": 5, "explanations": {"1": {"exist": true, "char_ranges": [[54, 169]], "word_ranges": [[6, 26]], "text": "By rule 1 would be easy to eliminate, the treatment of colon cancer is surgical (with neoadjuvant in rectal cancer)."}, "2": {"exist": true, "char_ranges": [[171, 291]], "word_ranges": [[26, 46]], "text": "If biopsies confirm adenocarcinoma, there is no need to repeat them, since it is necessary to have the complete specimen."}, "3": {"exist": true, "char_ranges": [[293, 364]], "word_ranges": [[46, 58]], "text": "Most colon cancer develops on adenomas, so we could eliminate this one."}, "4": {"exist": true, "char_ranges": [[365, 607]], "word_ranges": [[58, 95]], "text": "Among the other 2, it is important to know that 5 is correct since they almost always present microsatellite instability even without fulfilling the Amsterdam criteria and have a better prognosis than other poorly differentiated colon cancer."}, "5": {"exist": true, "char_ranges": [[365, 607]], "word_ranges": [[58, 95]], "text": "Among the other 2, it is important to know that 5 is correct since they almost always present microsatellite instability even without fulfilling the Amsterdam criteria and have a better prognosis than other poorly differentiated colon cancer."}}, "full_answer": "Complicated question about an unusual adenocarcinoma. By rule 1 would be easy to eliminate, the treatment of colon cancer is surgical (with neoadjuvant in rectal cancer). If biopsies confirm adenocarcinoma, there is no need to repeat them, since it is necessary to have the complete specimen. Most colon cancer develops on adenomas, so we could eliminate this one. Among the other 2, it is important to know that 5 is correct since they almost always present microsatellite instability even without fulfilling the Amsterdam criteria and have a better prognosis than other poorly differentiated colon cancer.", "full_answer_no_ref": "Complicated question about an unusual adenocarcinoma. By rule 1 would be easy to eliminate, the treatment of colon cancer is surgical (with neoadjuvant in rectal cancer). If biopsies confirm adenocarcinoma, there is no need to repeat them, since it is necessary to have the complete specimen. Most colon cancer develops on adenomas, so we could eliminate this one. Among the other 2, it is important to know that [HIDDEN] since they almost always present microsatellite instability even without fulfilling the Amsterdam criteria and have a better prognosis than other poorly differentiated colon cancer.", "full_question": "During a colonoscopy, a 5-cm tumor located in the right colon is detected in a 48-year-old man. No other lesions were found. His maternal grandmother also suffered from colon cancer. The biopsies are superficial and show a poorly differentiated tumor with abundant inflammatory cells in the stroma that is diagnosed as a medullary type carcinoma.", "id": 7, "lang": "en", "options": {"1": "Chemotherapy is the treatment of choice.", "2": "Since the biopsy is superficial, it should be repeated before proceeding with treatment.", "3": "The prognosis of the tumor depends mainly on its high degree of anaplasia.", "4": "It is unlikely that this tumor has developed over a previous adenoma.", "5": "Microsatellite instability and DNA error repair genes should be studied."}, "question_id_specific": 208, "type": "DIGESTIVE", "year": 2011, "rag": {"clinical_case_options": {"MedCorp": {"RRF-2": [{"id": "wiki20220301en190_40247", "title": "Signet ring cell carcinoma", "score": 0.012132231404958678, "content": "Colorectal Primary signet ring cell carcinoma of the colon and rectum (PSRCCR) is rare, with a reported incidence of less than 1 percent. It has a poor prognosis because symptoms often develop late and it is usually diagnosed at an advanced stage. Five-year survival rates in previous studies ranged from nine to 30 percent. Average survival was between 20 and 45 months. It tends to affect younger adults with higher likelihood of lymphovascular invasion. It is worth noting that the overall survival rate of patients with SRCC was significantly poorer than that of patients with mucinous or poorly differentiated adenocarcinoma. In advanced gastric cancers, the prognosis for patients with the SRCCs was significantly worse than for the other histological types, which can be explained by the finding that advanced SRCC gastric cancers have a larger tumor size, more lymph node metastasis, a deeper invasive depth and more Borrmann type 4 lesions than other types. Stomach"}, {"id": "wiki20220301en010_160879", "title": "Colorectal cancer", "score": 0.011086802270577106, "content": "The U.S. National Comprehensive Cancer Network and American Society of Clinical Oncology provide guidelines for the follow-up of colon cancer. A medical history and physical examination are recommended every 3 to 6 months for 2 years, then every 6 months for 5 years. Carcinoembryonic antigen blood level measurements follow the same timing, but are only advised for people with T2 or greater lesions who are candidates for intervention. A CT-scan of the chest, abdomen and pelvis can be considered annually for the first 3 years for people who are at high risk of recurrence (for example, those who had poorly differentiated tumors or venous or lymphatic invasion) and are candidates for curative surgery (with the aim to cure). A colonoscopy can be done after 1 year, except if it could not be done during the initial staging because of an obstructing mass, in which case it should be performed after 3 to 6 months. If a villous polyp, a polyp >1 centimeter or high-grade dysplasia is found, it"}, {"id": "wiki20220301en589_7880", "title": "Histopathology of colorectal adenocarcinoma", "score": 0.01084844213808273, "content": "Microscopy Adenocarcinoma is a malignant epithelial tumor, originating from superficial glandular epithelial cells lining the colon and rectum. It invades the wall, infiltrating the muscularis mucosae layer, the submucosa, and then the muscularis propria. Tumor cells describe irregular tubular structures, harboring pluristratification, multiple lumens, reduced stroma (\"back to back\" aspect). Sometimes, tumor cells are discohesive and secrete mucus, which invades the interstitium producing large pools of mucus. This occurs in mucinous adenocarcinoma, in which cells are poorly differentiated. If the mucus remains inside the tumor cell, it pushes the nucleus at the periphery, this occurs in \"signet-ring cell.\" Depending on glandular architecture, cellular pleomorphism, and mucosecretion of the predominant pattern, adenocarcinoma may present three degrees of differentiation: well, moderately, and poorly differentiated. Micrographs (H&E stain) Microscopic criteria"}, {"id": "pubmed23n0933_23652", "title": "[Clinicopathological characteristics and prognosis analysis of colorectal synchronous multiple primary cancer].", "score": 0.010414163262036369, "content": "To investigate the clinicopathological features and prognosis of colorectal synchronous multiple primary cancer(SMPC). From January 2008 to June 2011, 51 patients diagnosed with colorectal SMPC underwent surgery at Department of General Surgery of Peking University First Hospital. Their clinicopathological features, diagnosis, treatment and prognosis were summarized and analyzed. SMPC was diagnosed according to the following criteria: each tumor must have a definite pathologic picture of malignancy; metastasis or recurrence from another colorectal cancer was excluded; tumors must be distinctly separated by at least 5 cm of all intact bowel wall from each other; SMPC has abnormal cells between tumor and normal mucosa and abnormal gland of transitional zone; each cancer is infiltrating carcinoma except the carcinoma in situ; all the cancers are detected at the same time or within 6 months. Multiple primary colorectal cancer originated from familial colonic polyposis or ulcerative colitis was excluded. These 51 colorectal SMPC patients accounted for 3.5% of 1 452 colorectal cancer patients in the same period at our hospital, with 32 males and 19 females, and mean age of (63±13)(29 to 82) years. Of 51 cases, 46(90.2%) had 2 original carcinoma, 3(5.9%) had 3 original carcinoma and 2(3.9%) had 4 carcinoma; 23(45.1%) complicated with colon polyps, 4(7.8%) complicated with malignancy outside the colorectum. In TNM staging, 7(13.7%), 15(29.4%), 24(47.1%) and 5(9.8%) patients were stage I(, II(, III( and IIII( respectively. Among 51 patients undergoing surgery by different procedures, 16 were subtotal colon resection, 8 were extended right colon resection, 5 were extended left hemicolon resection, 8 were right hemicolon resection plus Dixon procedure, 10 were Dixon, and 4 were right hemicolon resection plus sigmoid colon resection. Adjuvant chemotherapy and support treatment were given according to the condition after operation. A total of 105 tumors were found, including 25(23.8%) tumors in sigmoid colon, 24(22.9%) in rectum, 22(21.0%) in ascending colon and 4 in organs outside the colorectum. Tubular adenocarcinoma (86/105, 81.9%) was the main pathological type in these colorectal SMPC patients. During the follow-up of median 43.5 months, 10 cases presented local recurrence and 6 cases had liver metastasis. Multivariable analysis showed that ≤65 years old (OR=22.757, 95%CI: 1.562-331.543, P=0.002),undifferentiated carcinoma or mucous adenocarcinoma (OR=27.174, 95%CI: 2.834-260.512, P=0.004), stage III(-IIII( (OR=29.626, 95%CI: 3.216-272.884, P=0.003) were independent risk factors of postoperative 5-year recurrence and metastasis, but the number of SMPC lesions and the surgical method were not associated with postoperative 5-year recurrence and metastasis (P=0.564, P=0.513). The 3-year and 5-year survival rates of colorectal SMPC patients were 76.5% and 64.7%. Two-original carcinoma is the most common in colorectal SMPC patients, which mainly distributes in sigmoid colon and rectum. Postoperative monitoring should be strengthened for those patients with younger age, poor pathological types and advanced staging to prevent recurrence and metastasis."}, {"id": "wiki20220301en010_160872", "title": "Colorectal cancer", "score": 0.010253239559986603, "content": "In Stage I colon cancer, no chemotherapy is offered, and surgery is the definitive treatment. The role of chemotherapy in Stage II colon cancer is debatable, and is usually not offered unless risk factors such as T4 tumor, undifferentiated tumor, vascular and perineural invasion or inadequate lymph node sampling is identified. It is also known that the people who carry abnormalities of the mismatch repair genes do not benefit from chemotherapy. For stage III and Stage IV colon cancer, chemotherapy is an integral part of treatment."}, {"id": "wiki20220301en126_27013", "title": "Microsatellite instability", "score": 0.009900990099009901, "content": "Direct and indirect mechanisms contribute to chemotherapy resistance. Direct mechanisms include pathways that metabolize the drug, while indirect mechanisms include pathways that respond to the chemotherapy treatment. The NER DNA repair pathway plays a substantial role in reversing cell damage caused by chemotherapeutic agents such as 5-FU. Discoveries since 2010 In May 2017 the FDA approved an immunotherapeutic called Keytruda (pembrolizumab) (PD-1 inhibitor) for patients with unresectable or metastatic microsatellite instability-high (MSI-H) or mismatch repair deficient (dMMR) solid tumors that have progressed following prior treatment. This indication is independent of PD-L1 expression assessment, tissue type and tumor location. Researchers have found another MSI, called elevated microsatellite alterations at selected tetranucleotide repeats (EMAST). However, EMAST is unique in that it is not derived from MMR, and it is commonly associated with TP53 mutations."}, {"id": "pubmed23n0301_22311", "title": "[The role of colonoscopy in early diagnosis of intraluminal recurrences in patients already treated for colorectal cancer].", "score": 0.009900990099009901, "content": "It is a common opinion that the more often and the more rigorously the colon is examined, the more lesions will be discovered and diagnosed. However it has not been shown which methods of colonic examination and which regimen of surveillance should be used. Chart review was conducted on 481 patients who underwent curative resection for colorectal cancer between 1980 and 1990. Colonoscopy was performed preoperatively, after 12-15 months from surgical treatment, and then at an interval of 12-24 months, or when symptoms appeared. About ten percent of patients developed intraluminal recurrences, and more than 25% adenomatous polyps. More than one half of the metachronous lesions arise within the first 24 months. The median time to diagnosis was 25 months for intraluminal recurrences and 22 months for adenomatous polyps. Patients with left sited tumor at an advanced stage run a higher risk of developing recurrent intraluminal disease, and patients who presented associated polyps at the time of the operation for the index cancer have a higher risk of developing new polyps. About 50% of recurrences were detected when patients were asymptomatic. Colonoscopy must be performed within the first 12-15 months after operation, while an interval of 24 months between each examination seems sufficient to guarantee an early detection of metachronous lesion. Asymptomatic patients are more frequently reoperated for cure and thus have a better survival rate."}, {"id": "wiki20220301en010_160832", "title": "Colorectal cancer", "score": 0.009858336505900976, "content": "Bowel cancer may be diagnosed by obtaining a sample of the colon during a sigmoidoscopy or colonoscopy. This is then followed by medical imaging to determine whether the disease has spread. Screening is effective for preventing and decreasing deaths from colorectal cancer. Screening, by one of a number of methods, is recommended starting from the age of 50 to 75. During colonoscopy, small polyps may be removed if found. If a large polyp or tumor is found, a biopsy may be performed to check if it is cancerous. Aspirin and other non-steroidal anti-inflammatory drugs decrease the risk. Their general use is not recommended for this purpose, however, due to side effects."}, {"id": "wiki20220301en015_13653", "title": "Biopsy", "score": 0.00980392156862745, "content": "There are two types of liquid biopsy (which is not really a biopsy as they are blood tests that do not require a biopsy of tissue): circulating tumor cell assays or cell-free circulating tumor DNA tests. These methods provide a non-invasive alternative to repeat invasive biopsies to monitor cancer treatment, test available drugs against the circulating tumor cells, evaluate the mutations in cancer and plan individualized treatments. In addition, because cancer is a heterogeneous genetic disease, and excisional biopsies provide only a snapshot in time of some of the rapid, dynamic genetic changes occurring in tumors, liquid biopsies provide some advantages over tissue biopsy-based genomic testing. In addition, excisional biopsies are invasive, can’t be used repeatedly, and are ineffective in understanding the dynamics of tumor progression and metastasis. By detecting, quantifying and characterisation of vital circulating tumor cells or genomic alterations in CTCs and cell-free DNA in"}, {"id": "pubmed23n0565_13838", "title": "Localization, clinical and pathological characteristics and survival in sporadic colon cancer patients younger than 40 and over 65 years of age.", "score": 0.00980392156862745, "content": "Colorectal cancer is predominantly a disease of older population, but occasionally it affects younger patients, in whom very often diagnosis is overseen and treatment begins late. The aim of our study was to compare localization, clinical and pathological characteristics and survival of sporadic colorectal cancer patients aged up to 40 and over 65 years. The first group (group I) included 19 patients under 40 years and the second group (group II) 28 patients aged over 65 years, treated during 1997-2001. Patients with family history of colon cancer and inflammatory disease of the colon were not included. Arithmetic mean, standard deviation, Fisher's test, Student 's t test, x(2) test and the Kaplan-Meier method were used in the statistical analysis of the results. There was no difference among the tested groups regarding tumor localization. The most frequent localization was in the rectum and left colon. At presentation, in group I patients, besides the metastases in the liver and lymph nodes, colorectal cancer infiltrated also the duodenum, stomach, right kidney capsule in one patient, and adnexa in two patients. In group II patients we registered only liver and lymph node metastases. Pathologically, tubular and mucinous forms were present in all of the patients up to 40 years of age, while only one patient over 65 had tumor with mucinous component. In group I, Astler-Coller stage B was found in 1.5% of the patients, stage C in 72.5% and stage D in 26%; in group II, stage B was found in 1.5%, stage C in 84.5% and stage D in 14%. Grade III was 36.8% in group I and 17.8% in group II. No statistical differences were found in stage distribution (p=0.36) and grade (p=0.06) between group I and II. Five-year overall survival was 57.8% and 28.5% in younger and older patients, respectively (p=0.053). The results obtained showed no difference in clinical symptomatology and tumor localization in both groups. The incidence of more aggressive tumors was higher in younger persons. However, early detection combined with more aggressive therapeutic approach, could enable significant improvement of the 5-year survival of younger patients with colon cancer."}, {"id": "wiki20220301en126_27010", "title": "Microsatellite instability", "score": 0.009708737864077669, "content": "Microsatellite instability diagnostics MSI is a good marker for determining Lynch syndrome and determining a prognosis for cancer treatments. In 1996, the National Cancer Institute (NCI) hosted an international workshop on Lynch Syndrome, which led to the development of the “Bethesda Guidelines” and loci for MSI testing. During this first workshop the NCI has agreed on five microsatellite markers necessary to determine MSI presence: two mononucleotides, BAT25 and BAT26, and three dinucleotide repeats, D2S123, D5S346, and D17S250. MSI-H tumors result from MSI of greater than 30% of unstable MSI loci (>2 or more of the 5 loci). MSI-L tumors result from less than 30% of unstable MSI biomarkers. MSI-L tumors are classified as tumors of alternative etiologies. Several studies demonstrate that MSI-H patients respond best to surgery alone, rather than chemotherapy and surgery, thus preventing patients from needlessly experiencing chemotherapy."}, {"id": "pubmed23n0039_7596", "title": "Definitive treatment of \"malignant\" polyps of the colon.", "score": 0.009708737864077669, "content": "There has been an unremitting rise in incidence of colonic cancer in this country with no recent improvement in cure rate. As a result the evolution of colorectal cancer has been the focus of considerable attention with an enlarging body of evidence pointing to the common neoplastic polyp as a precursor to malignancy. \"Neoplastic\" polyps include \"adenomatous polyps,\" \"villous adenomas\" and, lately recognized, \"villo-glandular polyps.\" Experience with endoscopic removal of over 2,000 colonic polyps (with no mortality) has introduced two questions of prime concern to the surgeon: (1) What constitutes clinical malignancy in a polyp? AND, (2) When should laparatomy supplant or follow endoscopic removal? Eight hundred and ninety-two consecutive adenomatous (tubular), villous, villoglandular (villo-tubular) and \"polypoid cancer\" polyps are analyzed, 855 of which have been followed for 6 months to 4 years. Support is offered to the concept that villous and tubular growth patterns are merely variants of a similar base disturbance in cell renewal. Superficial cancer (carcinoma-in-situ) occurred in 6.6% of neoplastic polyps and represents no threat if the polyp is completely removed. Only when the cancer penetrates the muscularis mucosae should it be regarded as \"invasive.\" The term \"malignant polyp\" should be reserved for this form. Invasive cancer was found in 5.0% of neoplastic polyps in this series. Only in this group need the question of further surgical intervention be raised. Major considerations influencing a decision for subsequent laparotomy are polyp size and gross morphology (i.e. sessile or pedunculated), histologic type (of the polyp and of the cancer itself), adequacy of clearance between depth of invasion and plane of polyp resection, and the patient's age and general condition. These are analyzed. Twenty-five of 46 patients with \"malignant polyps\" were subjected to abdominal exploration: 17 showed no residual cancer, whereas 8 (5 with recognized incomplete endoscopic removal) had tumor in the bowel wall. Of the remaining 21 patients, for whom endoscopic polypectomy alone was deemed appropriate, none have shown residual or recurrent cancer on clinical and endoscopic followup. Colonoscopy appears to be a most promising approach in terms of the goals of cancer programs, offering both prophylaxis and opportunity for treatment at a favorable stage of disease."}, {"id": "wiki20220301en189_29036", "title": "Skin cancer in cats and dogs", "score": 0.009615384615384616, "content": "Treatment The specific treatment will depend on the tumor's type, location, size, and whether the cancer has spread to other organs. Surgical removal of the tumor remains the standard treatment of choice, but additional forms of therapy such as radiation therapy, chemotherapy, or immunotherapy exist. When detected early, skin cancer in cats and dogs can often be treated successfully. In many cases, a biopsy can remove the whole tumor, as long as the healthy tissues removed from just outside the tumor area do not contain any cancer cells. References External links Skin Cancer in Cats and Dogs from Pet Cancer Center Skin Cancer in Dogs from CanineCancer.com' Types of animal cancers Cancer in dogs Cancer in cats Integumentary neoplasia"}, {"id": "pubmed23n0387_9310", "title": "[Gastrocolic tumor progression--a possibility or mere supposition?].", "score": 0.009615384615384616, "content": "The authors examined the five-years postoperative survival rate of fifty patients who suffered from colorectal cancer along with the fact that the large bowel one of the neighbouring organs were resected. The subjects were divided into four groups: the colorectal resection was associated with (1) stomach resection (13 patients); (2) liver metastasectomy (14 patients); (3) small bowel resection (10 patients); (4) the resection of other organs (13 patients). In the first two years of the study they were examined once in every three months, in the next two years once in every six months and then yearly. The following tests were carried out: chest X ray, abdominal sonography, irrigography or colonoscopy and CEA. On condition that the colonoscope reached the caecum and the result was negative, the test was repeated only a year later. The patients were operated on between 1985 and 1997. The statistical analysis was made with the help of the Kaplan-Meier method. During this period fifty-six complex resections were performed. Out of fifty-six patients fifty were followed. Compliance 89%. In group 1, where the average age of patients was sixty-two years, one patient died in the forty-first and the other in the fifty-second month after the surgery. Survival rate: 11/13 (83%). The survival rates for the other groups were as follows: group 2 (average age 64) twelve patients died within five years. Survival rate 2/14 (14%). The difference between the survival rates in the first two groups in significant (P = 0.0001). Group 3 (average age 67) seven died and only three survived. Survival rate: 3/10 (30%). The difference between group 1 and group 3 is significant (P = 0.0022). Group 4 (average age 64) seven patients died. Survival rate 6/13 (46%). Comparing this rate to that of the group 1, the difference is not significant (P > 0.01). Having analysed the results of the four groups it can be concluded that the patients of group 1 lived the longest (stomach resection) and those of group 2 (liver metastasectomy) died the earliest after the operation. It is surprising that the patients of group 3 lived significantly shorter than the ones of group 1 in spite of the fact that they belong by far the greatest number to stage Dukes B (group 1: 12/13 = 92%; group 3: 7/10 = 70%). The authors assume that the partial or the total absence of the stomach keeps back the growth of the tumour (gastro-colic tumour growing dependency). They think that in case of colon cancer which infiltrates the stomach surgeons experienced in gastric and colorectal surgery should be encouraged to take the risk of the double resection providing the fact that the operation is accomplishable."}, {"id": "wiki20220301en015_13654", "title": "Biopsy", "score": 0.009523809523809525, "content": "in understanding the dynamics of tumor progression and metastasis. By detecting, quantifying and characterisation of vital circulating tumor cells or genomic alterations in CTCs and cell-free DNA in blood, liquid biopsy can provide real-time information on the stage of tumor progression, treatment effectiveness, and cancer metastasis risk. This technological development could make it possible to diagnose and manage cancer from repeated blood tests rather than from a traditional biopsy."}, {"id": "pubmed23n0239_1422", "title": "[Diagnosis and therapy of colorectal polyps with special reference to adenomas].", "score": 0.009523809523809525, "content": "Colo-rectal adenoms occur more frequently in the elderly and should be considered as precancerous. The structural changes of the glandular epithelium are known as dysplasia or atypia and are classified into three grades of severity; the \"severe epithelial dysplasia\" has all the histological characteristics of a malignant tumor which however has not infiltrated the muscularis mucosa and so has not gained access to the lymphatic system. Whenever these structural changes were present the terms focal carcinoma or carcinoma in situ were used. However in 1976 the WHO accepted to change the nomenclature to \"severe epithelial dysplasia\", as Morson had proposed. Their aim was to avoid superfluous radical surgical intervention. Whenever severe dysplasia is present in an adenoma, the necessary therapy is the local excision of the adenoma together with its pedick. An exact complete histological examination is necessary. Between 1976 and 1980 we saw 201 cases of adenoma of the colon or rectum at the Surgical Clinic, University of Düsseldorf. 27 of these cases showed severe epithelial dysplasia. As described in the literature there was a correlation between the size of the adenoma, the histological picture and the risk of malignancy. The reexamination of 105 patients showed that there was a significant percentage of recurrency at the site of excision or new polyps at a different site. Therefore, regular checkups are a must for all those patients in whom polyps of the large bowel have been removed."}, {"id": "wiki20220301en634_8498", "title": "Lipofibromatosis", "score": 0.009433962264150943, "content": "The diagnosis of LPF depends on its clinical presentation almost exclusively in newborn and young children and, most importantly, its histopathology as determined on biopsied intact tissue or fine-needle aspiration to obtain a sampling of the tumor's cells. Intact tissue samples typical show abundant mature-appearing adipose (i.e. fat) tissue mixed with a minor component of oval-shaped or spindle-shaped fibroblast-like cells some of which have a pseudolipoblast-like morphology. Needle biopsies should show these cells. However, LPF histopathology can vary widely between cases. The cited gene abnormalities in the above section are insufficient to support a diagnosis of LPF although further study of these and discoveries of other gene abnormalities may do so. The histopathology of lipofibromatosis-like neural tumors (LPF-NT) can closely resemble LPF tumors. Unlike LPF tumors, however, LPF-NT tumors have been diagnosed in adults in more than 27% of cases with the remaining cases diagnosed"}, {"id": "pubmed23n0271_12666", "title": "[Synchronous and metachronous tumors of the colon and rectum].", "score": 0.009433962264150943, "content": "Three hundred and seventy-two patients with colorectal tumours treated by curative resection between January 1982 and January 1992 were reviewed in order to determine the role of colonoscopy and the outcome of patients with multiple tumours. Thirty (8.1%) of them with a mean age of 57 (35-79) years (20 males, 10 females) had synchronous (19 cases) or metachronous (11 cases) lesions. Rectum and sigmoid colon were the most frequent site of multiple lesions, accounting for 73% of all lesions. Accurate pre-operative diagnosis was performed in 14 of the 19 patients with synchronous lesions, and in the remaining 5 cases failure to perform an intra-operative colonoscopy was the cause of missing the lesions. Three of them had over-looked lesions on the previous barium enema. Synchronous lesions has the tendency to be less invasive as compared to metachronous ones. Five-year survival rates (Kaplan-Meier method) were 45% and 58% for patients with multiple and single lesions respectively (not significant). For patients with colorectal carcinoma a thorough examination of the whole colon by intra-operative colonoscopy should be accomplished in order to rule out the possibility of associated lesions as well as to decrease the incidence of \"early\" metachronous lesions."}, {"id": "wiki20220301en040_16700", "title": "Benign tumor", "score": 0.009374557803877175, "content": "Tumors are formed by carcinogenesis, a process in which cellular alterations lead to the formation of cancer. Multistage carcinogenesis involves the sequential genetic or epigenetic changes to a cell's DNA, where each step produces a more advanced tumor. It is often broken down into three stages; initiation, promotion and progression, and several mutations may occur at each stage. Initiation is where the first genetic mutation occurs in a cell. Promotion is the clonal expansion (repeated division) of this transformed cell into a visible tumor that is usually benign. Following promotion, progression may take place where more genetic mutations are acquired in a sub-population of tumor cells. Progression changes the benign tumor into a malignant tumor. A prominent and well studied example of this phenomenon is the tubular adenoma, a common type of colon polyp which is an important precursor to colon cancer. The cells in tubular adenomas, like most tumors that frequently progress to"}, {"id": "wiki20220301en459_3656", "title": "Circulating tumor DNA", "score": 0.009345794392523364, "content": "The emergence of drug-resistant tumors due to intra- and inter-tumoral heterogeneity an issue in treatment efficacy. A minor genetic clone within the tumor can expand after treatment if it carries a drug-resistant mutation. Initial biopsies can miss these clones due to low frequency or spatial separation of cells within the tumor. For example, since a biopsy only samples a small part of the tumor, clones that resides in a different location may go unnoticed. This can mislead research that focuses on studying the role of tumor heterogeneity in cancer progression and relapse. The use of ctDNA in research can alleviate these concerns because it could provide a more representative 'screenshot' of the genetic diversity of cancer at both primary and metastatic sites. For example, ctDNA has been shown to be useful in studying the clonal evolution of a patient’s cancer before and after treatment regimens. Early detection of cancer is still challenging but recent progress in the analysis of"}, {"id": "pubmed23n0621_2234", "title": "[A thousand total colonoscopies: what is the relationship between distal and proximal findings?].", "score": 0.009345794392523364, "content": "Flexible sigmoidoscopy is indicated for colorectal cancer screening. The decision about who needs total colonoscopy based on distal findings is still controversial because of the uncertainty of the associations between distal and proximal findings. The purpose of the study was to characterize distal findings in patients with total colonoscopy, to investigate its importance as markers of advanced proximal lesions and to evaluate the usefulness of a clinical Predictive Index, already published in the literature, in the identification of these lesions. Retrospective analysis of the patients submitted to total colonoscopy between January 2006 and February 2007, with selection of 1000 consecutive cases with reference to polyps. We analysed demographic data, indication for the exam and morphological and histological characteristics of the polyps. Advanced lesion was defined as any adenoma larger than 10 mm or any polyp with villous characteristics, high grade dysplasia or cancer. The Predictive Index was obtained through the assignment of points to 3 categories: sex, age and distal findings, which result in 3 groups: low, intermediate and high risk. The mean age of patients was 64,69 years and 65,1% were male. Distal and proximal polyps were identified in 829 (82,9%) and 369 (36,9%) patients, respectively. Advanced distal lesion was found in 342 patients (34,2%) and advanced proximal lesion in 98 (9,8%). 587 patients (58,7%) were in the high risk group. In the group of patients with advanced proximal lesion, a third presented low and intermediate risk, 52% had no distal polyps, 88,7% had less than three distal polyps and 71,4% had no advanced distal lesion. Sensitivity values for these four categories ranged between 11,2% and 66,6%. If the decision to perform total colonoscopy is based on distal colonic findings or on the Predictive Index, the ability to identify advanced proximal lesion is markedly reduced, endangering the aim of a screening program."}, {"id": "wiki20220301en371_14817", "title": "Endoscopic mucosal resection", "score": 0.009259259259259259, "content": "For the esophagous Endoscopic mucosal resection has been advocated for early esophageal cancers (that is, those that are superficial and confined to the mucosa only) and has been shown to be a less invasive, safe, and effective therapy for early squamous cell carcinoma. It has also been shown to be safe and effective for early adenocarcinoma arising in Barrett’s esophagus. The prognosis after treatment with this method is comparable to surgical resection. This technique can be attempted in patients who have no evidence of nodal or distant metastases, with differentiated tumors that are slightly raised and less than 2 cm in diameter, or in differentiated tumors that are ulcerated and less than 1 cm in diameter. The most commonly employed modalities of endoscopic mucosal resection include strip biopsy, double-snare polypectomy, resection with combined use of highly concentrated saline and epinephrine, and resection using a cap."}, {"id": "pubmed23n0215_9020", "title": "A potentially brighter prognosis for colon carcinoma in the third and fourth decades.", "score": 0.009259259259259259, "content": "In contrast to earlier studies that suggested that colon carcinoma is unusually lethal in the young, 69 patients, ages 20 to 39 years, had a relatively good prognosis. Fifty-nine percent lived over 5 years after diagnosis, and 51% were cured. Furthermore, 67% were cured if they did not have distant spread of the carcinoma at the time of the initial operation. Neither age, sex, tumor size, location, mere presence of lymph node metastases, depth of tumor invasion, nor predisposing disease of the colon was a strong prognostic factor. Metastases to six or more lymph nodes and distant spread of the tumor at the time of initial surgery were ominous findings. Mucinous carcinoma was relatively frequent (28%) and was also an ominous feature (only 5 of 20 patients cured as opposed to 26 of 43 with classical adenocarcinoma)."}, {"id": "wiki20220301en189_29035", "title": "Skin cancer in cats and dogs", "score": 0.009174311926605505, "content": "Cytology is an important tool that can help the veterinarian distinguish a tumor from inflammatory lesions. The biopsy technique used will largely depend on the tumor's size and location. Small masses are usually completely excised and sent to the pathology lab to confirm that the surrounding healthy tissues that were excised along with the tumor do not contain any cancer cells. If the tumor is larger, a small sample is removed for analysis and depending on the results, appropriate treatment is chosen. Depending on the tumor type and its level of aggressiveness, additional diagnostic tests can include blood tests to assess the pet’s overall health, chest X-rays to check for lung metastasis, and abdominal ultrasound to check for metastasis to other internal organs."}, {"id": "pubmed23n0418_19398", "title": "[Clinical and macroscopic variables that influence the prognosis of colorectal carcinoma].", "score": 0.009174311926605505, "content": "The paradoxical evolution of approximately one third of patients with neoplasms cataloged in Dukes stages B and C demonstrates the desirability of utilizing other prognostic criteria that are capable of broadening the information provided by these two important variables. Only a small number of investigators have dedicated themselves to the study of the prognostic value of clinical and macroscopic parameters of colorectal neoplasms, and the results obtained have been shown to be controversial. The principal aim of this work was to evaluate the prognostic importance of these parameters. A study was made of 320 patients with colorectal cancer who underwent curative extirpation. They had a median age of 58 years, and there were 199 females (62.2%) and 121 males (37.8%). The patients were divided into three age groups: under 40 years old, between 40 and 60 years old and over 60 years old. The tumors were distributed in three intestinal segments: right colon, left colon and rectum. The neoplasms were classified as small (diameter less than or equal to 35 mm) and large (diameter greater than 35 mm). With regard to their form, they were classified as exophytic, when characterized by luminal growth, and endophytic, when there was intramural growth. The involvement of the intestinal circumference at the site of the neoplasm was considered as partial or total. Of the 320 patients, 22 (6.9%) were aged under 40 years, 159 (49.7%) from 40 to 60 years and 139 (43.4%) presented an age of over 60 years. Seventy-three (22.8%) of the neoplasms were located in the right colon, 130 (40.6%) in the left colon and 117 (36.6%) in the rectum. Regarding the size, 280 (87.5%) were large and 40 (12.5%) small; exophytic lesions predominated over endophytic ones - 173 (54.1%) vs 147 (45.9%). A greater number of tumors presented total involvement of the intestinal circumference - 216 (67.5%) - while 104 (32.5%) presented partial involvement. The 5-year survival of the patients was not influenced by their age and sex, or by the location and size of the neoplasms. Exophytic lesions conferred greater survival on their sufferers (65.9%), in comparison with endophytic lesions (49.0%). The survival of patients with lesions partially involving the intestinal circumference was greater than for those with total involvement - 72.1% vs. 51.4%. Clinical variables had no influence on the patients' prognosis. Among the macroscopic variables, the form of the neoplasia and its involvement in the intestinal circumference did influence the patients' prognosis. These last two variables are important data capable of contributing to the identification of patient subpopulations with greater or lesser prognostic risk."}, {"id": "wiki20220301en454_6872", "title": "Pancreatic neuroendocrine tumor", "score": 0.00909090909090909, "content": "In functioning PanNETs, octreotide is usually recommended prior to biopsy or surgery but is generally avoided in insulinomas to avoid profound hypoglycemia. PanNETs in Multiple endocrine neoplasia type 1 are often multiple, and thus require different treatment and surveillance strategies. Some PanNETs are more responsive to chemotherapy than are gastroenteric carcinoid tumors. Several agents have shown activity. In well differentiated PanNETs, chemotherapy is generally reserved for when there are no other treatment options. Combinations of several medicines have been used, such as doxorubicin with streptozocin and fluorouracil (5-FU) and capecitabine with temozolomide. Although marginally effective in well-differentiated PETs, cisplatin with etoposide has some activity in poorly differentiated neuroendocrine cancers (PDNECs), particularly if the PDNEC has an extremely high Ki-67 score of over 50%."}, {"id": "pubmed23n0220_18004", "title": "[Cancers of the colon. Results of surgical treatment. Presentation of a series of 234 patients].", "score": 0.00909090909090909, "content": "This report is a retrospective analysis of the results of surgical treatment in 234 consecutive cases of adenocarcinoma of the colon; 56.4 p. 100 of patients were male with a mean age of 66. Sixty per cent of the carcinoma were situated in the sigmoid. Carcinoma was complicated in 26.1 p. 100 of cases. The tumor was confined to the bowel wall in 14.3 p. 100 of cases (stage A), involved the serosa in 36.3 p. 100 of cases (stage B), lymph nodes in 25.5 p. 100 of cases (stage C), and distal organs in 23.8 p. 100 of cases (stage D). Global operative mortality was 10 p. 100. Obvious anastomotic leakages occurred in two patients with one death. The overall five year survival rate was 35 p. 100. The stage-by-stage 5-year survival rates depended mainly on the differentiation and on the extension of the tumor: 59.2 p. 100 in patients with stage A lesions, 54.8 p. 100 in those with stage B lesions, 30.2 p. 100 in those with stage C lesions, 3.9 p. 100 in those with stage D lesions. On the other hand, survival was not significantly related to the duration of symptoms. This suggests that early diagnosis of symptomatic disease does not guarantee a better prognosis. This can only be achieved by prevention of the disease."}, {"id": "pubmed23n0516_12328", "title": "[Association between microsatellite instability and clinico-pathological characteristics in sporadic colon cancer].", "score": 0.009057971014492754, "content": "Currently, colon cancer is a leading cause of cancer death world-wide. It progresses according to three molecular pathways, named suppressor, mutador and methylator. Microsatellite instability is a hallmark of the lack of reparation, of DNA mismatches and it characterizes a subset of colon tumors (unstable tumors, MSI). MSI-H patients (high degree of microsatellite instability) seem to share clinico-pathological differences with MSS (microsatellite stable) and MSI-L (low degree of microsatellite instability) patients. In this study, associations between high degree of microsatellite instability and pathological (location, mucinous content, differentiation grade, stages T3N0, stages II and III) and clinical features (response to chemotherapy, disease-free survival and overall survival) were evaluated. 117 patients with sporadic colon cancer were classified into two populations (MSS/MSI-L and MSI-H) by using PCR and electrophoresis of seven microsatellites, according to the National Cancer Institute recommendations. MSI-H tumors tended to be located in the right colon (p = 0.022) and were of mucinous histologic type (p = 0.04). No differences in disease-free survival and overall survival between group of stage II and III patients with MSS/ MSI-L and corresponding ones with MSI-H colon cancer were found (p = 0.54, p = 0.37, respectively). Conversely, MSI-H patients with stage II colon cancer had a favourable prognosis (p = 0.027). Nevertheless, response to 5-fluorouracil (5-FU) and leucovorin was similar in MSS/ MSI-L and MSI-H groups (p = 0.38). MSI-H patients are characterized by certain pathological features; those MSI-H patients with a stage II seem to have a better prognosis than MSS/ MSI-L patients."}, {"id": "wiki20220301en149_24224", "title": "Surgical pathology", "score": 0.009009009009009009, "content": "polyps are very common. The pathologist's interpretation of a biopsy is critical to establishing the diagnosis of a benign or malignant tumor, and can differentiate between different types and grades of cancer, as well as determining the activity of specific molecular pathways in the tumor. This information is important for estimating the patient's prognosis and for choosing the best treatment to administer. Biopsies are also used to diagnose diseases other than cancer, including inflammatory, infectious, or idiopathic diseases of the skin and gastrointestinal tract, to name only a few."}, {"id": "pubmed23n0261_15612", "title": "Synchronous colorectal carcinomas.", "score": 0.009009009009009009, "content": "Eighteen (5.0%) out of 358 patients who underwent resection of a colorectal carcinoma during the period 1978 through 1990 had synchronous colorectal carcinomas, and were 5.6 years younger on average than those with a single carcinoma. The distance between synchronous lesions was less than 10 cm in 69.6% of all the patients in the study. Among the synchronous carcinomas there was a higher incidence of ascending colon involvement, mucinous carcinoma, family history of malignant diseases, multiple malignant neoplasms associated with other organs and benign neoplastic polyps of the colorectum, and it is suggested that hereditary oncogenic factors influence these carcinomas. The synchronous lesions were detected pre-operatively in 14 of 18 patients with synchronous carcinomas, and the most common reason why synchronous lesions were missed was that the lesions on the anal side prevented the lesions on the proximal side from being examined. The prognosis in the synchronous lesion group was worse than in the solitary lesion group. Since it is difficult to predict synchronous colorectal carcinomas, careful pre-operative examination, including that of other organs, is necessary, and intra-operative colonoscopy should be carried out when pre-operative examination was insufficient."}, {"id": "wiki20220301en409_14583", "title": "Pembrolizumab", "score": 0.008928571428571428, "content": "In May 2017, pembrolizumab received an accelerated approval from the US FDA for use in any unresectable or metastatic solid tumor with DNA mismatch repair deficiencies or a microsatellite instability-high state (or, in the case of colon cancer, tumors that have progressed following chemotherapy). This approval marked the first instance in which the FDA approved marketing of a drug based only on the presence of a genetic mutation, with no limitation on the site of the cancer or the kind of tissue in which it originated. The approval was based on a clinical trial of 149 patients with microsatellite instability-high or mismatch repair deficient cancers who enrolled on one of five single-arm trials. Ninety patients had colorectal cancer, and 59 patients had one of 14 other cancer types. The objective response rate for all patients was 39.6%. Response rates were similar across all cancer types, including 36% in colorectal cancer and 46% across the other tumor types. Notably, there were 11"}, {"id": "pubmed23n0122_5726", "title": "Synchronous and 'early' metachronous carcinomas of the colon and rectum.", "score": 0.008928571428571428, "content": "In a review of cases of colorectal cancer presenting to St. Mark's Hospital over the 16-year period 1970-85, 59 patients were found to have a synchronous carcinoma (3.4 per cent). Although 82 per cent of these synchronous tumours were distal to the splenic flexure (and hence within reach of a 60 cm flexible sigmoidoscope) only 42 per cent were detected pre-operatively: the remaining tumours were noted at surgery (24 per cent) or found incidentally on pathological examination of the resected specimen (34 per cent). Histological examination of these synchronous lesions revealed a high proportion with favourable stage (Dukes' A-75 per cent) and grade (well or moderate differentiation-90 per cent). Over the same 16-year period, 10 patients presented with an 'early' metachronous lesion (less than 3 years from initial surgery). Review of these cases noted negative findings on the initial barium studies in four patients and a failure to conduct full examination of the colon at initial presentation in the remaining six. It is concluded that full examination of the colon in all patients presenting with primary colorectal cancer is mandatory and that, in the light of this experience and recent reports in the literature, this should be by pre- or peroperative colonoscopy."}]}}}} {"correct_option": 3, "explanations": {"1": {"exist": true, "char_ranges": [[240, 393]], "word_ranges": [[34, 59]], "text": "No changes should be made in the hypouricemic treatment during this period because the uric acid metabolic chain is altered and the situation may worsen."}, "2": {"exist": true, "char_ranges": [[240, 393]], "word_ranges": [[34, 59]], "text": "No changes should be made in the hypouricemic treatment during this period because the uric acid metabolic chain is altered and the situation may worsen."}, "3": {"exist": true, "char_ranges": [[0, 239]], "word_ranges": [[0, 34]], "text": "In the presence of acute gouty arthritis (the presence of intracellular crystals with negative birefringence confirms this) in a hyperuricemic patient previously treated with allopurinol, an NSAID should be added until the crisis subsides."}, "4": {"exist": true, "char_ranges": [[240, 393]], "word_ranges": [[34, 59]], "text": "No changes should be made in the hypouricemic treatment during this period because the uric acid metabolic chain is altered and the situation may worsen."}, "5": {"exist": true, "char_ranges": [[240, 393]], "word_ranges": [[34, 59]], "text": "No changes should be made in the hypouricemic treatment during this period because the uric acid metabolic chain is altered and the situation may worsen."}}, "full_answer": "In the presence of acute gouty arthritis (the presence of intracellular crystals with negative birefringence confirms this) in a hyperuricemic patient previously treated with allopurinol, an NSAID should be added until the crisis subsides. No changes should be made in the hypouricemic treatment during this period because the uric acid metabolic chain is altered and the situation may worsen.", "full_answer_no_ref": "In the presence of acute gouty arthritis (the presence of intracellular crystals with negative birefringence confirms this) in a hyperuricemic patient previously treated with allopurinol, an NSAID should be added until the crisis subsides. No changes should be made in the hypouricemic treatment during this period because the uric acid metabolic chain is altered and the situation may worsen.", "full_question": "A hyperuricemic patient who usually takes 100 mg of allopurinol daily comes to the ED with acute pain and inflammatory signs in the right knee. Arthrocentesis is performed and polarized light microscopy shows intracellular crystals with negative birefringence. Which of the following therapeutic approaches is the most appropriate in this case?", "id": 152, "lang": "en", "options": {"1": "Discontinue allopurinol and start colchicine treatment.", "2": "Discontinue allopurinol and start NSAIDs.", "3": "Add an NSAID until the crisis remits.", "4": "Increase the dose of allopurinol to 300 mg/day.", "5": "Substitute allopurinol for uricosuric acid."}, "question_id_specific": 72, "type": "RHEUMATOLOGY", "year": 2012, "rag": {"clinical_case_options": {"MedCorp": {"RRF-2": [{"id": "pubmed23n0824_19774", "title": "Does starting allopurinol prolong acute treated gout? A randomized clinical trial.", "score": 0.019327731092436976, "content": "Traditionally, allopurinol is not initiated during an acute gout attack to avoid prolonging the painful arthritis. The 2012 American College of Rheumatology Guidelines for the Management of Gout suggest that urate-lowering therapy can be started during an acute attack, based on \"consensus opinion of experts, case studies, or standard of care.\" The aim of this study was to determine whether initiating allopurinol will adversely affect the resolution of acute, treated gout. We conducted a 28-day, placebo-controlled, double-blind study of allopurinol initiation in patients with acute gout. Patients with crystal-proven gout by arthrocentesis were enrolled if they presented to the rheumatology clinic with an acute gout attack within 72 hours from initial therapy. The patients were also required to meet at least 1 additional criterion for urate-lowering therapy including (1) the presence of gouty tophi, (2) more than 1 acute gout attack per year, (3) a history of nephrolithiasis, or (4) urate overproduction (>1000 mg in 24-hour urine collection). Patients were excluded from the study if they had a glomerular filtration rate of less than 50 or liver function test of greater than 1.25 times the upper limit of normal. The treating physician determined therapy for the acute gout attack. Standard prophylaxis, with colchicine or nonsteroidal anti-inflammatory drugs, was prescribed. Allopurinol or placebo was initiated at 100 mg daily for the first 14 days and then increased to 200 mg daily for the next 14 days. The primary end point was protocol defined days to resolution of acute gout, incorporating patient-rated joint pain and physician examination. Secondary measures included Physician Global Assessment, patient-rated pain, adverse effects of therapy, and serum uric acid. Thirty-one patients (17 on placebo, 14 on allopurinol) completed the study. Both intent-to-treat and completer analyses showed only a statistically insignificant difference in days to resolution (15.4 days in the allopurinol group completers vs 13.4 days in the placebo group; P = 0.5). The secondary measures revealed that the acute phase of pain rapidly improved in both groups. We initiated allopurinol at low doses during an acute gout attack in patients who met criteria for starting urate-lowering therapy and did not have abnormal kidney or liver function. In this cohort, allopurinol did not prolong the acute, treated attack."}, {"id": "pubmed23n1047_3671", "title": "[ERRORS IN THE DIAGNOSTICS AND TREATMENT OF PATIENTS WITH GOUT AND THE ALGORITHM OF THERAPEUTIC TACTICS IN DIFFERENT PERIODS OF THE DISEASE (CLINICAL CASE)].", "score": 0.019324122479462285, "content": "Difficulties and errors in the treatment of patients with the gout arise, mainly, during urate-lowering therapy. The article discusses possible medical errors in acute gouty arthritis and during chronic tophaceous gout in the light of the updated international recommendations of the American College of Rheumatology (ACR) and the European Antirheumatic League (EULAR 2018). As an example of inadequate treatment, the authors describe a case of a patient with chronic tophaceous gout. Errors in the diagnosis and treatment of the patient caused various complications and unjustified surgical intervention - amputation of the right finger and removal of a large tophus in the left forearm. Based on the analysis of mistakes made in the diagnosis and treatment of gout, the authors propose an algorithm for therapeutic tactics in different periods of the disease. So, for the relief of exacerbation in acute gouty arthritis, it is recommended to take the following drugs at starting doses: colchicine at a dose of 1.8 mg/day (1.2 mg immediately followed by 0.6 mg 1 hour later during 7-10 days or until complete relief of the gout attack), non-steroidal anti-inflammatory drugs (nimesulide up to 200 mg/day) or glucocorticosteroids (prednisolone at a dose of 30 mg/day for 3-5 days with subsequent withdrawal). The first-line urate-lowering drugs for chronic tofaceous gout are xanthine oxidase inhibitors - allopurinol and febuxostat. Allopurinol is prescribed no earlier than 2 weeks after the arthritis attack has stopped at a starting dose of no more than 100 mg/day, the dose is gradually increased to the minimum effective. The starting dose of febuxostat is 40 mg/day. Also, together with allopurinol or febuxostat, it is recommended to take uricosuric drugs (probenecid 500 mg/day or benzbromarone 50-200 mg/day). At the same time, the authors draw attention to the inadmissibility of the combination of allopurinol and febuxostat. In case of gout that does not respond to the main methods of therapy, treatment with pegloticase is recommended. When prescribing urate-lowering therapy, dose titration is necessary, to avoid the development of toxic effects."}, {"id": "pubmed23n0740_13678", "title": "Initiation of allopurinol at first medical contact for acute attacks of gout: a randomized clinical trial.", "score": 0.01905453225660103, "content": "Streamlining the initiation of allopurinol could result in a cost benefit for a common medical problem and obviate the perception that no treatment is required once acute attacks have resolved. Our objective was to test the hypothesis that there is no difference in patient daily pain or subsequent attacks with early versus delayed initiation of allopurinol for an acute gout attack. A total of 57 men with crystal-proven gout were randomized to allopurinol 300 mg daily or matching placebo for 10 days. All subjects received indomethacin 50 mg 3 times per day for 10 days, a prophylactic dose of colchicine 0.6 mg 2 times per day for 90 days, and open-label allopurinol starting at day 11. Primary outcome measures were pain on visual analogue scale (VAS) for the primary joint on days 1 to 10 and self-reported flares in any joint through day 30. On the basis of 51 evaluable subjects (allopurinol in 26, placebo in 25), mean daily VAS pain scores did not differ significantly between study groups at any point between days 1 and 10. Initial VAS pain scores for allopurinol and placebo arms were 6.72 versus 6.28 (P=.37), declining to 0.18 versus 0.27 (P=.54) at day 10, with neither group consistently having more daily pain. Subsequent flares occurred in 2 subjects taking allopurinol and 3 subjects taking placebo (P=.60). Although urate levels decreased rapidly in the allopurinol group (from 7.8 mg/dL at baseline to 5.9 mg/dL at day 3), sedimentation rates and C-reactive protein levels did not differ between groups at any point. Allopurinol initiation during an acute gout attack caused no significant difference in daily pain, recurrent flares, or inflammatory markers."}, {"id": "pubmed23n0283_18062", "title": "Gout: modern management of an ancient malady.", "score": 0.017412370353546824, "content": "If Dr. Sydenham could have benefited from today's therapy, he likely would not have had to endure thirty years of \"violent ... torture\" that gave birth to his most elegant and classic description of acute gout. The five key points to remember in management of the gouty spectrum are: (1) Establish the diagnosis as clearly as possible or as clearly as seems necessary under the clinical circumstances (i.e. arthrocentesis with crystal analysis to establish diagnosis is not always necessary with reliable patients when septic joint seems highly unlikely). (2) Treat acute attacks with NSAIDs alone or perhaps steroids--or rarely IV colchicine under special circumstances. (3) DO NOT START ALLOPURINOL OR PROBENECID DURING AN ACUTE FLARE OF GOUT--IT MAY MAKE THE EPISODE WORSE. (4) The pattern of disease over time (frequency and severity of attacks) determines whether or not one decides to use an agent such as allopurinol, probenecid, or prophylactic colchicine chronically once a patient is over the acute attack--the mere presence of increased uric acid and a single or rare gouty attack would not usually require any other than the appropriate acute therapy. (5) The presence of visible tophi, uric acid renal calculi and destructive gouty arthritis nearly always warrant uric acid lowering therapy."}, {"id": "pubmed23n0318_11898", "title": "Managing problem gout.", "score": 0.016298946531504672, "content": "For the management of acute gouty arthritis, non-steroidal anti-inflammatory drugs (NSAIDs) are the drugs of choice. In recent years, the use of colchicine has declined because of its frequent adverse reactions, and its reduced efficacy when administered more than 24 hours after onset of an acute attack. Intra-articular corticosteroid therapy (e.g. methylprednisolone acetate) is indicated for the treatment of acute mono or oligoarticular gouty arthritis in aged patients, and in those with co-morbid conditions contraindicating therapy with either NSAIDs or colchicine. Oral corticosteroids (e.g. prednisone), and both parenteral corticotrophin (ACTH) and corticosteroids (e.g. intramuscular triamcinolone acetonide) are valuable, relatively safe alternate treatment modalities in those with polyarticular attacks. For the treatment of hyperuricaemia and chronic gouty arthritis, allopurinol is the preferred urate-lowering drug. Its toxicity in elderly individuals, those with renal impairment, and in cyclosporine-treated transplant patients can be minimised by adjusting the initial dose according to the patient's creatinine clearance. In those experiencing cutaneous reactions to allopurinol, cautious desensitisation to the drug can be achieved using a schedule of gradually increasing doses. The therapeutic usefulness of uricosuric drugs is limited by the presence of renal impairment, occurrence of intolerable side-effects, or concomitant intake of salicylates. They are particularly indicated in patients allergic to allopurinol and in those with massive tophi requiring combined therapy with both allopurinol and a uricosuric."}, {"id": "pubmed23n0897_25218", "title": "Crystal arthritides - gout and calcium pyrophosphate arthritis : Part 3: Treatment.", "score": 0.01571342564720048, "content": "The treatment of gout is based on several principles. Symptom control and termination of the inflammatory process are important early goals, whereas the urate level should be lowered in the long term to prevent further gout attacks and complications. The non-pharmacological approach is based on individually informing the patient on dietary measures and changes of life style. Besides physical measures, such as cold applications on the affected joint, various medications are available for treatment of an acute gout attack. The choice of drug depends on the individual risk profile. If non-steroidal anti-inflammatory drugs (NSAID) and coxibs are chosen it should be taken into account that the use is restricted in patients with renal insufficiency. Moreover, these drugs may have gastrointestinal side effects and are associated with increased cardiovascular morbidity and mortality. Colchicine has gastrointestinal side effects at high dosages but can also be used for differential diagnostics if there is a quick response to treatment. Steroids are an effective alternative and can be given orally or parenterally in patients with dysphagia. Moreover, steroids can be used in cases of renal insufficiency. After symptoms of the acute attack have subsided, urate lowering therapy should be initiated to prevent further attacks. Low-dose urate lowering therapy can be started during an acute gout attack when acute therapy is initiated. Allopurinol is still the medication of choice but its use is restricted in patients with renal insufficiency. A rare but serious side effect is allopurinol hypersensitivity syndrome. Febuxostat can be an alternative in patients who do not tolerate allopurinol. In February 2016, lesinurad, an URAT-1 and OAT-4 inhibitor, was approved in combination with allopurinol or febuxostat. Data on the effectiveness and safety of synthetic uricases and biologicals are still sparse for elderly patients. These substances are reserved for severe cases of gout."}, {"id": "pubmed23n0809_9520", "title": "Allopurinol for chronic gout.", "score": 0.014685756395410043, "content": "Allopurinol, a xanthine oxidase inhibitor, is considered one of the most effective urate-lowering drugs and is frequently used in the treatment of chronic gout. To assess the efficacy and safety of allopurinol compared with placebo and other urate-lowering therapies for treating chronic gout. We searched the Cochrane Central Register of Controlled Trials (CENTRAL), MEDLINE and EMBASE on 14 January 2014. We also handsearched the 2011 to 2012 American College of Rheumatology (ACR) and European League against Rheumatism (EULAR) abstracts, trial registers and regulatory agency drug safety databases. All randomised controlled trials (RCTs) or quasi-randomised controlled clinical trials (CCTs) that compared allopurinol with a placebo or an active therapy in adults with chronic gout. We extracted and analysed data using standard methods for Cochrane reviews. The major outcomes of interest were frequency of acute gout attacks, serum urate normalisation, pain, function, tophus regression, study participant withdrawal due to adverse events (AE) and serious adverse events (SAE). We assessed the quality of the body of evidence for these outcomes using the GRADE approach. We included 11 trials (4531 participants) that compared allopurinol (various doses) with placebo (two trials); febuxostat (four trials); benzbromarone (two trials); colchicine (one trial); probenecid (one trial); continuous versus intermittent allopurinol (one trial) and different doses of allopurinol (one trial). Only one trial was at low risk of bias in all domains. We deemed allopurinol versus placebo the main comparison, and allopurinol versus febuxostat and versus benzbromarone as the most clinically relevant active comparisons and restricted reporting to these comparisons here.Moderate-quality evidence from one trial (57 participants) indicated allopurinol 300 mg daily probably does not reduce the rate of gout attacks (2/26 with allopurinol versus 3/25 with placebo; risk ratio (RR) 0.64, 95% confidence interval (CI) 0.12 to 3.52) but increases the proportion of participants achieving a target serum urate over 30 days (25/26 with allopurinol versus 0/25 with placebo, RR 49.11, 95% CI 3.15 to 765.58; number needed to treat for an additional beneficial outcome (NNTB) 1). In two studies (453 participants), there was no significant increase in withdrawals due to AE (6% with allopurinol versus 4% with placebo, RR 1.36, 95% CI 0.61 to 3.08) or SAE (2% with allopurinol versus 1% with placebo, RR 1.93, 95% CI 0.48 to 7.80). One trial reported no difference in pain reduction or tophus regression, but did not report outcome data or measures of variance sufficiently and we could not calculate the differences between groups. Neither trial reported function.Low-quality evidence from three trials (1136 participants) indicated there may be no difference in the incidence of acute gout attacks with allopurinol up to 300 mg daily versus febuxostat 80 mg daily over eight to 24 weeks (21% with allopurinol versus 23% with febuxostat, RR 0.89, 95% CI 0.71 to 1.1); however more participants may achieve target serum urate level (four trials; 2618 participants) with febuxostat 80 mg daily versus allopurinol 300 mg daily (38% with allopurinol versus 70% with febuxostat, RR 0.56, 95% CI 0.48 to 0.65, NNTB with febuxostat 4). Two trials reported no difference in tophus regression between allopurinol and febuxostat over a 28- to 52-week period; but as the trialists did not provide variance, we could not calculate the mean difference between groups. The trials did not report pain reduction or function. Moderate-quality evidence from pooled data from three trials (2555 participants) comparing allopurinol up to 300 mg daily versus febuxostat 80 mg daily indicated no difference in the number of withdrawals due to AE (7% with allopurinol versus 8% with febuxostat, RR 0.89, 95% CI 0.62 to 1.26) or SAE (4% with allopurinol versus 4% with febuxostat, RR 1.13, 95% CI 0.71 to 1.82) over a 24- to 52-week period.Low-quality evidence from one trial (65 participants) indicated there may be no difference in the incidence of acute gout attacks with allopurinol up to 600 mg daily compared with benzbromarone up to 200 mg daily over a four-month period (0/30 with allopurinol versus 1/25 with benzbromarone, RR 0.28, 95% CI 0.01 to 6.58). Based on the pooled results of two trials (102 participants), there was moderate-quality evidence of no probable difference in the proportion of participants achieving a target serum urate level with allopurinol versus benzbromarone (58% with allopurinol versus 74% with benzbromarone, RR 0.79, 95% CI 0.56 to 1.11). Low-quality evidence from two studies indicated there may be no difference in the number of participants who withdrew due to AE with allopurinol versus benzbromarone over a four- to nine-month period (6% with allopurinol versus 7% with benzbromarone, pooled RR 0.80, 95% CI 0.18 to 3.58). There were no SAEs. They did not report tophi regression, pain and function.All other comparisons were supported by small, single studies only, limiting conclusions. Our review found low- to moderate-quality evidence indicating similar effects on withdrawals due to AEs and SAEs and incidence of acute gout attacks when allopurinol (100 to 600 mg daily) was compared with placebo, benzbromarone (100 to 200 mg daily) or febuxostat (80 mg daily). There was moderate-quality evidence of little or no difference in the proportion of participants achieving target serum urate when allopurinol was compared with benzbromarone. However, allopurinol seemed more successful than placebo and may be less successful than febuxostat (80 mg daily) in achieving a target serum urate level (6 mg/dL or less; 0.36 mmol/L or less) based on moderate- to low-quality evidence. Single studies reported no difference in pain reduction when allopurinol (300 mg daily) was compared with placebo over 10 days, and no difference in tophus regression when allopurinol (200 to 300 mg daily) was compared with febuxostat (80 mg daily). None of the trials reported on function, health-related quality of life or participant global assessment of treatment success, where further research would be useful."}, {"id": "pubmed23n0617_17753", "title": "Desensitization to allopurinol after allopurinol hypersensitivity syndrome with renal involvement in gout.", "score": 0.014516129032258063, "content": " This is a case report of a 36-year-old male with tophaceous gout for 16 years. He started therapy with 300 mg/day of allopurinol. He had received variable dexamethasone doses by self-prescription for 16 years. When allopurinol was initiated, he had hyperuricemia and normal renal function. Twenty days after starting allopurinol, he presented diffuse maculopapular rash, conjunctivitis, increase in serum creatinine values, leukocytosis and eosinophilia and the diagnosis of allopurinol hypersensitivity (AH) syndrome was made. He completely recovered from the AH and renal function normalized. However, the gout worsened over the following years in spite of treatment with benzobromarone, low doses of prednisone, and colchicine. Allopurinol desensitization was successful beginning with an oral low dose scheme (6.5 mug/day) until we reached 300mg/day. Today the patient receives allopurinol with no side effects. We believe that this is the first reported example of successful desensitization in full-blown AH with renal involvement. Our cautious regimen might be tried in other such patients."}, {"id": "wiki20220301en015_140922", "title": "Colchicine", "score": 0.014192835592572133, "content": "Medical uses Gout Colchicine is an alternative for those unable to tolerate NSAIDs when treating gout. Low doses appear to be well tolerated and may reduce gout symptoms and pain (1.2 mg in one hour, followed by 0.6 mg an hour later). This low dose may have a similar effectiveness to NSAIDS (low quality evidence). At high doses, side effects (primarily diarrhea, nausea, or vomiting) limit its use, however may be effective against pain. In addition, there is preliminary evidence that daily colchicine (0.6 mg twice daily) may be effective as a long-term prophylaxis when used with allopurinol to reduce the risk of increased uric acid levels and acute gout flares, although adverse gastrointestinal effects may occur. For treating gout symptoms, colchicine is used orally with or without food, as symptoms first appear. Subsequent doses may be needed if symptoms worsen."}, {"id": "wiki20220301en002_185653", "title": "Gout", "score": 0.014017017465293328, "content": "Treatment The initial aim of treatment is to settle the symptoms of an acute attack. Repeated attacks can be prevented by medications that reduce serum uric acid levels. Tentative evidence supports the application of ice for 20 to 30 minutes several times a day to decrease pain. Options for acute treatment include nonsteroidal anti-inflammatory drugs (NSAIDs), colchicine, and Glucocorticoids. While glucocorticoids and NSAIDs work equally well, glucocorticoids may be safer. Options for prevention include allopurinol, febuxostat, and probenecid. Lowering uric acid levels can cure the disease. Treatment of associated health problems is also important. Lifestyle interventions have been poorly studied. It is unclear whether dietary supplements have an effect in people with gout."}, {"id": "wiki20220301en002_185649", "title": "Gout", "score": 0.013615384615384616, "content": "While it has been recommended that urate-lowering measures should be increased until serum uric acid levels are below 300–360 µmol/l (5.0–6.0 mg/dl), there is little evidence to support this practice over simple putting people on a standard dose of allopurinol. If these medications are in chronic use at the time of an attack, it is recommended that they be continued. Levels that cannot be brought below 6.0 mg/dl while attacks continue indicates refractory gout. While historically it is not recommended to start allopurinol during an acute attack of gout, this practice appears acceptable. Allopurinol blocks uric acid production, and is the most commonly used agent. Long term therapy is safe and well-tolerated and can be used in people with renal impairment or urate stones, although hypersensitivity occurs in a small number of individuals."}, {"id": "pubmed23n0632_1646", "title": "Ultrasonography shows disappearance of monosodium urate crystal deposition on hyaline cartilage after sustained normouricemia is achieved.", "score": 0.013419101985175087, "content": "This study aimed at determining whether lowering serum urate (SU) to less than 6 mg/dl in patients with gout affects ultrasonographic findings. Seven joints in five patients with monosodium urate (MSU) crystal proven gout and hyperuricemia were examined over time with serial ultrasonography. Four of the five patients were treated with urate lowering drugs (ULDs) (allopurinol, n = 3; probenecid, n = 1). One patient was treated with colchicine alone. Attention was given to changes in a hyperechoic, irregular coating of the hyaline cartilage in the examined joints (double contour sign or \"urate icing\"). This coating was considered to represent precipitate of MSU crystals. Index joints included metacarpophalangeal (MCP) joints (n = 2), knee joints (n = 3), and first metatarsophalangeal (MTP) joints (n = 2). The interval between baseline and follow-up images ranged from 7 to 18 months. Serial SU levels were obtained during the follow-up period. During the follow-up period, three patients treated with ULD (allopurinol, n = 2; probenecid, n = 1) achieved a SU level of <6 mg/dl. In two patients, SU levels remained above 6 mg/dl (treated with allopurinol, n = 1; treated with colchicine, n = 1). At baseline, the double contour sign was seen in all patients. In those patients who achieved SU levels of <6 ml/dl, this sign had disappeared at follow-up. Disappearance of the double contour sign was seen in two knee joints, two first MTP joints, and one MCP joint. In contrast, disappearance of the double contour sign was not seen in patients who maintained a SU level > or =7 mg/dl. In one patient treated with allopurinol, SU levels improved from 13 to 7 mg/dl during the follow-up period. Decrease, but not resolution of the hyperechoic coating was seen in this patient. In the patient treated with colchicine alone, SU levels remained >8 mg/dl, and no sonographic change was observed. In our patients, sonographic signs of deposition of MSU crystals on the surface of hyaline cartilage disappeared completely if sustained normouricemia was achieved. This is the first report showing that characteristic sonographic changes are influenced by ULDs once SU levels remain < or =6 mg/dl for 7 months or more. Sonographic changes of gout correlate with SU levels and may be a non-invasive means to track changes in the uric acid pool. Larger prospective studies are needed to further assess these potentially important findings."}, {"id": "wiki20220301en534_23030", "title": "Lesinurad/allopurinol", "score": 0.013137254901960785, "content": "Lesinurad/allopurinol (trade name Duzallo) is a fixed-dose combination drug for the treatment of gout. It contains 200 mg of lesinurad and 300 mg of allopurinol. In August 2017, the US Food and Drug Administration approved it for the treatment of hyperuricemia associated with gout in patients for whom target serum uric acid levels have not been achieved with allopurinol alone. It was approved for medical use in the European Union in August 2018. In February 2019, it was discontinued by its manufacturer for business reasons and is no longer available. References Antigout agents Combination drugs AstraZeneca brands Withdrawn drugs"}, {"id": "pubmed23n0505_18289", "title": "Management of acute and chronic gouty arthritis: present state-of-the-art.", "score": 0.013131648936170214, "content": "There are three stages in the management of gout: (i) treating the acute attack; (ii) lowering excess stores of uric acid to prevent flares of gouty arthritis and to prevent tissue deposition of urate; and (iii) providing prophylaxis to prevent acute flares. It is important to distinguish between therapy to reduce acute inflammation in acute gout and therapy to manage hyperuricaemia in patients with chronic gouty arthritis. During the acute gouty attack nonpharmacological treatments such as topical ice and rest of the inflamed joint are useful. NSAIDs are the preferred treatment in acute gout. The most important determinant of therapeutic success is not which NSAID is chosen, but rather how soon NSAID therapy is initiated. Other treatments include oral and intravenous colchicine, intra-articular and systemic corticosteroids, and intramuscular corticotropin. Optimal treatment of chronic gout requires long-standing reduction in serum uric acid. The urate-lowering drugs used to treat chronic gout are the uricosuric drugs, the uricostatic drugs, which are xanthine oxidase inhibitors, and the uricolytic drugs. Xanthine oxidase inhibitors such as allopurinol, oxipurinol and febuxastat should be used as first-line treatment in patients with renal calculi, renal insufficiency, concomitant diuretic therapy and ciclosporin (cyclosporine) therapy, and urate overproduction. Uricosuric drugs include probenecid, benzbromarone, micronised fenofibrate and losartan. They are the urate-lowering drugs of choice in allopurinol-allergic patients and underexcretors with normal renal function and no history of urolithiasis. The use of recombinant urate oxidase in patients with chronic gout is limited by the need for parenteral administration, the potential antigenicity and production of anti-urate oxidase antibodies, and declining efficacy. The effectiveness of colchicine prophylaxis as an isolated therapy is still to be confirmed by placebo-controlled trials. Another issue is prophylaxis with NSAIDs. There are no comparative studies with colchicine."}, {"id": "pubmed23n0839_14448", "title": "Adherence to the 2012 American College of Rheumatology (ACR) Guidelines for Management of Gout: A Survey of Brazilian Rheumatologists.", "score": 0.012766474831806759, "content": "To describe the current pharmacological approach to gout treatment reported by rheumatologists in Brazil. We performed a cross-sectional survey study using an online questionnaire e-mailed to 395 rheumatologists, randomly selected, from among the members of the Brazilian Society of Rheumatology. Three hundred and nine rheumatologists (78.2%) responded to the survey. For acute gout attacks, combination therapy (NSAIDs or steroid + colchicine) was often used, even in monoarticular involvement, and colchicine was commonly started as monotherapy after 36 hours or more from onset of attack. During an acute attack, urate-lowering therapy (ULT) was withdrawn by approximately a third of rheumatologists. Anti-inflammatory prophylaxis (98% colchicine) was initiated when ULT was started in most cases (92.4%), but its duration was varied. Most (70%) respondents considered the target serum uric acid level to be less than 6 mg/dl. Approximately 50% of rheumatologists reported starting allopurinol at doses of 100 mg daily or less and 42% reported the initial dose to be 300 mg daily in patients with normal renal function. ULT was maintained indefinitely in 76% of gout patients with tophi whereas in gout patients without tophi its use was kept indefinitely in 39.6%. This is the first study evaluating gout treatment in a representative, random sample of Brazilian rheumatologists describing common treatment practices among these specialists. We identified several gaps in reported gout management, mainly concerning the use of colchicine and ULT and the duration of anti-inflammatory prophylaxis and ULT. Since rheumatologists are considered as opinion leaders in this disease, a program for improving quality of care for gout patients should focus on increasing their knowledge in this common disease."}, {"id": "pubmed23n0886_3759", "title": "Management of Gout: A Systematic Review in Support of an American College of Physicians Clinical Practice Guideline.", "score": 0.012727130325814535, "content": "Gout is a common type of inflammatory arthritis in patients seen by primary care physicians. To review evidence about treatment of acute gout attacks, management of hyperuricemia to prevent attacks, and discontinuation of medications for chronic gout in adults. Multiple electronic databases from January 2010 to March 2016, reference mining, and pharmaceutical manufacturers. Studies of drugs approved by the U.S. Food and Drug Administration and commonly prescribed by primary care physicians, randomized trials for effectiveness, and trials and observational studies for adverse events. Data extraction was performed by one reviewer and checked by a second reviewer. Study quality was assessed by 2 independent reviewers. Strength-of-evidence assessment was done by group discussion. High-strength evidence from 28 trials (only 3 of which were placebo-controlled) shows that colchicine, nonsteroidal anti-inflammatory drugs (NSAIDs), and corticosteroids reduce pain in patients with acute gout. Moderate-strength evidence suggests that low-dose colchicine is as effective as high-dose colchicine and causes fewer gastrointestinal adverse events. Moderate-strength evidence suggests that urate-lowering therapy (allopurinol or febuxostat) reduces long-term risk for acute gout attacks after 1 year or more. High-strength evidence shows that prophylaxis with daily colchicine or NSAIDs reduces the risk for acute gout attacks by at least half in patients starting urate-lowering therapy, and moderate-strength evidence indicates that duration of prophylaxis should be longer than 8 weeks. Although lower urate levels reduce risk for recurrent acute attacks, treatment to a specific target level has not been tested. Few studies of acute gout treatments, no placebo-controlled trials of management of hyperuricemia lasting longer than 6 months, and few studies in primary care populations. Colchicine, NSAIDs, and corticosteroids relieve pain in adults with acute gout. Urate-lowering therapy decreases serum urate levels and reduces risk for acute gout attacks. Agency for Healthcare Research and Quality. (Protocol registration: http://effectivehealth-care.ahrq.gov/ehc/products/564/1992/Gout-managment-protocol-141103.pdf)."}, {"id": "pubmed23n0629_4293", "title": "Febuxostat: a new treatment for hyperuricaemia in gout.", "score": 0.012517602879048663, "content": "Febuxostat is a new non-purine xanthine oxidase inhibitor that is more potent than allopurinol 300 mg daily. In two Phase III trials, significantly more febuxostat-treated gout patients met the primary endpoint [serum urate (sUA) <6 mg/dl (<360 mumol/l) at the last three visits] (48 and 53% with 80 mg; 65 and 62% with 120 mg), compared with those receiving allopurinol 300 mg (22 and 21%; P < 0.001 in both studies). Febuxostat was more effective than allopurinol in the subset with impaired renal function; no dose adjustment is required in mild-to-moderate renal impairment. Long-term extension studies confirmed the efficacy and tolerability of febuxostat. In patients who achieved the sUA target of 6 mg/dl (360 mumol/l), the incidence of gout flares fell steadily and tophi resolved in many patients. The incidence of adverse events such as dizziness, diarrhoea, headache and nausea with febuxostat was similar to allopurinol. The incidence of cardiovascular side-effects (Antiplatelet Trialists Collaboration events) was numerically higher with febuxostat than with allopurinol, but this was not statistically significant. Co-administration of febuxostat with AZA or 6-mercaptopurine is not recommended. Prophylaxis (colchicine and/or NSAIDs) against acute attacks should be used for at least the first 6 months, since early mobilization flares were observed in the clinical trials. In conclusion, febuxostat is more effective than allopurinol 300 mg daily in reducing sUA levels <6 mg/dl (360 mumol/l), the target recommended by EULAR, and offers a new option for the long-term treatment of gout."}, {"id": "wiki20220301en012_92531", "title": "Allopurinol", "score": 0.012499192558620243, "content": "It is also used to treat kidney stones caused by deficient activity of adenine phosphoribosyltransferase. Tumor lysis syndrome Allopurinol was also commonly used to treat tumor lysis syndrome in chemotherapeutic treatments, as these regimens can rapidly produce severe acute hyperuricemia; however, it has gradually been replaced by urate oxidase therapy. Intravenous formulations are used in this indication when people cannot take medicine by mouth. Inflammatory bowel disease Allopurinol cotherapy is used to improve outcomes for people with inflammatory bowel disease and Crohn's disease who do not respond to thiopurine monotherapy. Cotherapy has also been shown to greatly improve hepatoxicity side effects in treatment of IBD. Cotherapy invariably requires dose reduction of the thiopurine, usually to one-third of the standard dose depending upon the patient's genetic status for thiopurine methyltransferase."}, {"id": "pubmed23n0277_19348", "title": "Preventing acute gout when starting allopurinol therapy. Colchicine or NSAIDs?", "score": 0.012043010752688172, "content": "Acute gout is a well known complication of the commencement of allopurinol therapy. Prophylaxis is needed for some months, even after serum urate levels have returned to normal. Colchicine is usually preferable to NSAIDs for this purpose, being cheaper, and better tolerated, especially in patients with peptic ulcer, gastrointestinal bleeding or dyspepsia or who are taking anticoagulants."}, {"id": "wiki20220301en002_185648", "title": "Gout", "score": 0.0118939883645766, "content": "Medications As of 2020, allopurinol is generally the recommended preventative treatment if medications are used. A number of other medications may occasionally be considered to prevent further episodes of gout, including probenecid, febuxostat, benzbromarone, and colchicine. Long term medications are not recommended until a person has had two attacks of gout, unless destructive joint changes, tophi, or urate nephropathy exist. It is not until this point that medications are cost-effective. They are not usually started until one to two weeks after an acute flare has resolved, due to theoretical concerns of worsening the attack. They are often used in combination with either an NSAID or colchicine for the first three to six months."}, {"id": "wiki20220301en002_185633", "title": "Gout", "score": 0.01185693560986497, "content": "Treatment with nonsteroidal anti-inflammatory drugs (NSAIDs), glucocorticoids, or colchicine improves symptoms. Once the acute attack subsides, levels of uric acid can be lowered via lifestyle changes and in those with frequent attacks, allopurinol or probenecid provides long-term prevention. Taking vitamin C and eating a diet high in low-fat dairy products may be preventive. Gout affects about 1 to 2% of the Western population at some point in their lives. It has become more common in recent decades. This is believed to be due to increasing risk factors in the population, such as metabolic syndrome, longer life expectancy, and changes in diet. Older males are most commonly affected. Gout was historically known as \"the disease of kings\" or \"rich man's disease\". It has been recognized at least since the time of the ancient Egyptians. Signs and symptoms"}, {"id": "pubmed23n1063_18839", "title": "[Evaluation of a 12-week allopurinol-lowering therapy in combination with the non-steroidal anti-inflammatory drug meloxicam in patients with gout].", "score": 0.011739699149771092, "content": "To evaluate a 12-week course of combined alloturinol-lowering therapy with a prophylactic anti-inflammatory dose of movalis for the frequency of exacerbations and the quality of life of patients with gout. Allopurinol was administered orally, 1 time per day. Every 3 weeks, the dosage of the drug was increased by 50 mg to 300 mg per day under the control of the level of serum uric acid (sUA). The total daily dose of the drug movalis, used in the form of different dosage forms, was 7.515 mg. The clinical effectiveness of the treatment was evaluated after 3, 6, 9 and 12 weeks according to physical examination, the dynamics of joint pain at rest, during movement and palpation, according to the visual analogue scale (VAS) in millimeters, Likert scale, EuroQol-5D-5L questionnaire, care for oneself, habitual daily activities, the presence of anxiety and depression, assessment of satisfaction with treatment (on a scale of 1 to 5, where 1 is the complete absence of improvement or worsening, and 5 is a very good result); took into account the period of remission, as well as the time before the onset of relapse of gouty arthritis. An adverse event (AE) was recorded. On the background of treatment with movalis 7.5 mg per day more than two-thirds of patients showed no worsening of the articular syndrome with an increase in the dose of allopurinol to 300 mg per day. By the 12th week of observation, a significant difference was found between the severity of gouty arthritis characteristics in the direction of improving mobility, self-care, normal daily activities, reducing soreness, reducing anxiety and depression (p0.05). In addition, the ESR and sUA levels were significantly different initially and at the final observation point (p0.05), which indicates a positive effect on the inflammatory process. A 3-month course of combination therapy was not accompanied by significant increases in blood pressure, changes in creatinine clearance in blood serum. There were no adverse events from the gastrointestinal tract. 90.9% of patients rated the treatment result as very good. AE in the form of a skin allergic rash was observed in one patient; it did not require interruption of treatment and completely stopped without consequences after completion of the course. 12 a week-long combined therapy of the allopurinol-reducing drug with the anti-inflammatory dose movalis prevents the exacerbation of the articular syndrome and improves the quality of life of patients with gout."}, {"id": "wiki20220301en001_271444", "title": "Kidney stone disease", "score": 0.011532738095238094, "content": "Allopurinol For people with hyperuricosuria and calcium stones, allopurinol is one of the few treatments that have been shown to reduce kidney stone recurrences. Allopurinol interferes with the production of uric acid in the liver. The drug is also used in people with gout or hyperuricemia (high serum uric acid levels). Dosage is adjusted to maintain a reduced urinary excretion of uric acid. Serum uric acid level at or below 6 mg/100 ml) is often a therapeutic goal. Hyperuricemia is not necessary for the formation of uric acid stones; hyperuricosuria can occur in the presence of normal or even low serum uric acid. Some practitioners advocate adding allopurinol only in people in whom hyperuricosuria and hyperuricemia persist, despite the use of a urine-alkalinizing agent such as sodium bicarbonate or potassium citrate."}, {"id": "wiki20220301en049_51196", "title": "Mercaptopurine", "score": 0.011516595602374382, "content": "Mercaptopurine causes changes to chromosomes in animals and humans, though a study in 1990 found, \"while the carcinogenic potential of 6-MP cannot be precluded, it can be only very weak or marginal.\" Another study in 1999 noted an increased risk of developing leukemia when taking large doses of 6-MP with other cytotoxic drugs. Drug interactions Allopurinol inhibits xanthine oxidase, the enzyme that breaks down mercaptopurine. Those taking allopurinol (often used to prevent gout) are at risk for mercaptopurine toxicity. The dose should be reduced or allopurinol should be discontinued. Several published studies have demonstrated that the use of allopurinol in combination with low dose 6-MP helps reduce 6-MP levels, which are toxic to liver tissue, whilst increasing the therapeutic levels of 6-MP for some inflammatory conditions. Mechanisms of action"}, {"id": "First_Aid_Step1_515", "title": "First_Aid_Step1", "score": 0.011430477230004244, "content": "tREatmEnt Acute: NSAIDs (eg, indomethacin), glucocorticoids, colchicine. Chronic (preventive): xanthine oxidase inhibitors (eg, allopurinol, febuxostat). Previously called pseudogout. Deposition of The blue P’s—blue (when Parallel), Positive calcium pyrophosphate crystals within the birefringence, calcium Pyrophosphate, joint space. Occurs in patients > 50 years old; Pseudogout both sexes affected equally. Usually idiopathic, sometimes associated with hemochromatosis, hyperparathyroidism, joint trauma. Pain and swelling with acute inflammation (pseudogout) and/or chronic degeneration (pseudo-osteoarthritis). Most commonly affected joint is the knee. Chondrocalcinosis (cartilage calcification) on x-ray. Crystals are rhomboid and weakly ⊕ birefringent under polarized light (blue when parallel to light) A . Acute treatment: NSAIDs, colchicine, glucocorticoids. Prophylaxis: colchicine. MUSCULOSKELETAL, SKIN, AND CONNECTIVE TISSUE ` pathology SECTION III 468"}, {"id": "pubmed23n0651_7031", "title": "Management of hyperuricemia in gout: focus on febuxostat.", "score": 0.011278434362689032, "content": "Gout is the most common inflammatory arthritis in an elderly population, and can be diagnosed with absolute certainty by polarization microscopy. However, diagnosis may be challenging because atypical presentations are more common in the elderly. Management of hyperuricemia in the elderly with gout requires special consideration because of co-medication, contra-indications, and risk of adverse reactions. Urate-lowering agents include allopurinol and uricosuric agents. These also must be used sensibly in the elderly, especially when renal function impairment is present. However, if used at the lowest dose that maintains the serum urate level below 5.0 to 6.0 mg/dL (0.30 to 0.36 mmol/L), the excess urate in the body will eventually be eliminated, acute flares will no longer occur, and tophi will resolve. Febuxostat, a new xanthine oxidase inhibitor, is welcomed, as few alternatives for allopurinol are available. Its pharmacokinetics and pharmacodynamics are not significantly altered in patients with moderate renal function or hepatic impairment. Its antihyperuricemic efficacy at 80 to 120 mg/day is better than \"standard dosage\" allopurinol (300 mg/day). Long-term safety data and efficacy data on tophus diminishment and reduction of gout flares have recently become available. Febuxostat may provide an important option in patients unable to use allopurinol, or refractory to allopurinol."}, {"id": "pubmed23n0022_2964", "title": "Allopurinol-induced granulomatous hepatitis with cholangitis and a sarcoid-like reaction.", "score": 0.010764448537877044, "content": "A 36-year-old man had pain in both knees and an elevated uric acid concentration; his liver function was normal. Allopurinol therapy was started, 100 mg twice daily. After one month fever, lethargy, and severe polyarthralgia developed. On admission to our hospital liver function was abnormal, and a liver biopsy specimen showed granulomas with cholangitis and pericholangitis. He also had lymphopenia with a reduced number of T cells and granulomas in the bone marrow. One month after discontinuation of allopurinol therapy the patient was clinically well with normal liver function and a normal lymphocyte count. A repeated liver biopsy specimen showed normal liver tissue with no granulomas. The onset of the symptoms and findings shortly after the initiation of allopurinol therapy, and their disappearance after the discontinuation of therapy suggest a drug-induced hypersensitivity."}, {"id": "article-17382_13", "title": "Allopurinol -- Adverse Effects", "score": 0.010642931025096631, "content": "Due to the destabilization of intra-articular uric acid microtophi on initiating any urate-lowering therapy, there is an increased incidence of acute gouty flares, especially during the initial few months. To prevent this, patients should start an anti-inflammatory agent such as colchicine, nonsteroidal anti-inflammatory drug (NSAID), or low-dose prednisone (only in patients who cannot take colchicine or NSAIDs) before or at the same time as initiating allopurinol. [10]"}, {"id": "pubmed23n0361_16937", "title": "Clinical manifestations of gout and their management.", "score": 0.010396466278819221, "content": "Gout is an inflammatory response to deposition of monosodium urate crystals in and around joints. It is primarily a disease of adult men. In acute gout, treatment options include non-steroidal anti-inflammatory drugs (NSAIDs), colchicine and corticosteroids, administered either intra-articularly, orally or parenterally. Asymptomatic hyperuricaemia does not require specific treatment, but should prompt screening for atherosclerosis risk factors, and general lifestyle modification to reduce serum urate levels. Gout presents differently in the elderly. Both women and men are affected, attacks are frequently polyarticular and in the upper limbs, and the gout may be associated with diuretic use, hypertension and renal impairment. In patients with peptic ulcer disease, selective COX-2 inhibitors provide another treatment option. In the presence of renal impairment, allopurinol is the treatment of choice for urate lowering therapy, but doses of allopurinol and colchicine must be adjusted. Urate lowering therapy should only be used if recurrent episodes of gout occur despite aggressive attempts to reverse or control the underlying causes. It should not be introduced or discontinued during an acute episode of gout, and gout prophylaxis (NSAIDs or colchicine) should be prescribed during the introduction of urate lowering therapy."}, {"id": "pubmed23n0910_14966", "title": "Images in clinical medicine: Tophi.", "score": 0.009900990099009901, "content": "Tophi (plural of tophus, Latin for \"stone\") are stone-like deposits of monosodium urate in the soft tissues, synovial tissues, or in bones near the joints. They are pathognomonic for gout, the most common inflammatory arthritis in the United States, with an estimated lifetime prevalence of 4%. It is usually the end result of loss of the balance between uric acid production and excretion. It can be found anywhere in the body especially in areas of friction or trauma. It is usually painless and rarely to present as the initial manifestation of gout. It is diagnosed mainly clinically. Imaging is mainly used to assess the complication like bony erosions. The American College of Rheumatology (ACR) guidelines currently indicate that urate-lowering therapy should be initiated in patients with the presence of tophi visible on examination or imaging (ACR Evidence A). First-line therapy for urate lowering remains the xanthine oxidase inhibitor allopurinol. The ACR currently recommends colchicine, 0.6 mg (or 0.5 mg) once or twice daily, or low dose NSAIDs should be continued to reduce gout flare incidence for six months after resolution of the tophus. Daily prednisone ≤10 mg has been endorsed as an acceptable second-line prophylactic agent. 99.5%. Most diagnostic laboratories use commercial kits that contain antigens from both HIV-1 and HIV-2 and thus are able to detect antibodies to either. These kits use both natural and recombinant antigens and are continuously updated to increase their sensitivity to newly discovered species, such as group O viruses (Fig. 226-1). The fourth-generation EIA tests combine detection of antibodies to HIV with detection of the p24 antigen of HIV. EIA tests are generally scored as positive (highly reactive), negative (nonreactive), or indeterminate (partially reactive). While the EIA is an extremely sensitive test, it is not optimal with regard to specificity. This is particularly true in studies of low-risk individuals, such as"}, {"id": "pubmed23n0619_4406", "title": "RT-PCR detection of HIV in Republic of Macedonia.", "score": 0.010552011969446412, "content": "The aim of the study was to detect HIV RNA in seropositive patients using RT-PCR method and thus, to establish PCR methodology in the routine laboratory works. The total of 33 examined persons were divided in two groups: 1) 13 persons seropositive for HIV; and 2) 20 healthy persons - randomly selected blood donors that made the case control group. The subjects age was between 25 and 52 years (average 38,5). ELFA test for combined detection of HIV p24 antigen and anti HIV-1+2 IgG and ELISA test for detection of antibodies against HIV-1 and HIV-2, were performed for each examined person. RNA from the whole blood was extracted using a commercial kit based on salt precipitation. Detection of HIV RNA was performed using RT-PCR kit. Following nested PCR, the product was separated by electrophoresis in 1,5 % agarose gel. The result was scored positive if the band of 210bp was visible regardless of intensity. Measures of precaution were taken during all the steps of the work and HIV infected materials were disposed of accordingly. In the group of blood donors ELFA, ELISA and RT-PCR were negative. Assuming that prevalence of HIV infection is zero, the clinical specificity of RT-PCR is 100 %. The analytical specificity of RT-PCR method was tested against Hepatitis C and B, Human Papiloma Virus, Cytomegalovirus, Herpes Simplex Virus, Rubella Virus, Mycobacterium tuberculosis, Chlamydia trachomatis. None of these templates yielded amplicon. In the group of 13 seropositive persons, 33 samples were analyzed. HIV RNA was detected in 15 samples. ELISA and ELFA test were positive in all samples. Different aliquots of the samples were tested independently and showed the same results. After different periods of storing the RNA samples at -70 masculineC, RT-PCR reaction was identical to the one performed initially. The obtained amplicons were maintained frozen at -20 masculineC for a week and the subsequently performed electrophoresis was identical to the previous one. The reaction is fast, simple for manipulation; with low detection level of 60 IU/ml. RT-PCR needs a small amount of RNA, as well as a small volume of sample. HIV RNA was detected in different periods of time with different clinical presentations in patients, with or without antiretroviral therapy. RT-PCR method gives the opportunity for reliable determination of HIV-1 RNA with border of detection of 60 IU/ml. The test is reproducible and has high analytical and clinical specificity."}, {"id": "wiki20220301en015_23214", "title": "Diagnosis of HIV/AIDS", "score": 0.010297600659046443, "content": "a period of six months suggests the results are not due to HIV infection. In a generally healthy low-risk population, indeterminate results on western blot occur on the order of 1 in 5,000 patients. However, for those individuals who have had high-risk exposures to individuals where HIV-2 is most prevalent, Western Africa, an inconclusive western blot test may prove infection with HIV-2."}, {"id": "wiki20220301en015_23235", "title": "Diagnosis of HIV/AIDS", "score": 0.010006780295059737, "content": "AIDS denialism HIV tests have been criticized by AIDS denialists (a fringe group whose members believe that HIV either does not exist or is harmless). The accuracy of serologic testing has been verified by isolation and culture of HIV and by detection of HIV RNA by PCR, which are widely accepted \"gold standards\" in microbiology. While AIDS denialists focus on individual components of HIV testing, the combination of ELISA and western blot used for the diagnosis of HIV is remarkably accurate, with very low false-positive and -negative rates as described above. The views of AIDS denialists are based on highly selective analysis of mostly outdated scientific papers; there is broad scientific consensus that HIV is the cause of AIDS."}, {"id": "pubmed23n0606_5568", "title": "Evaluation of Determine HIV-1/2 rapid diagnostic test by 4th generation ELISA using blood donors' serum at Felege Hiwot Referral Hospital, northwest Ethiopia.", "score": 0.009900990099009901, "content": "The study aims to evaluate the HIV-1/2 rapid diagnostic test kit is routinely used to screen HIV infection for safe blood transfusion and VCT services in many parts of Ethiopia. A total of 324 sera were collected from consecutive blood donors from February to May 2006. All samples were screened for HIV infection using Determine HIV-1/2 (Abbott Japan) at hospital blood bank laboratory. Blindly, all serums were retested at Regional Health Research Laboratory using 4th generation ELISA (Vironostika HIV Uni-Form II AG/Ab) and Determine HIV-1/2 (Abbott lab). Discordant samples were repeatedly retested using the same ELISA and Determine HIV-1/2 to avoid technical errors. Finally, discordant results were resolved using Western Blot at the National HIV/AIDS Laboratory. Determine HIV-1/2 and ELISA showed 94.4% concordance in HIV antibody testing with fair Cohen's Kappa statistic value (0.68) among blood donors. The sensitivity, specificity, positive and negative predictive values of Determine HIV-1/2 were 60.5%, 98.9%, 88.5% and 94.9% respectively. As a rapid HIV screening test for blood donors, Determine HIV-1/2 showed poor sensitivity. Further evaluation at multiple centres is recommended to test its validity as a routine HIV screening test in blood donors. Use of a combination of rapid assays is also recommended for screening of HIV infection among the donor population."}]}}}} {"correct_option": 3, "explanations": {"1": {"exist": false, "char_ranges": [], "word_ranges": [], "text": ""}, "2": {"exist": false, "char_ranges": [], "word_ranges": [], "text": ""}, "3": {"exist": true, "char_ranges": [[251, 440]], "word_ranges": [[36, 66]], "text": "In our environment the most frequent cause is surgery of the split gland with a percentage of occurrence between 10 and 80%. Conservative treatment with botulinum toxin offers good results."}, "4": {"exist": false, "char_ranges": [], "word_ranges": [], "text": ""}, "5": {"exist": false, "char_ranges": [], "word_ranges": [], "text": ""}}, "full_answer": "You are asking about auriculotemporal syndrome or Frey's syndrome. It is the clinical expression of a vegetative neuropathy caused by mechanical or irritative lesion of the vegetative fibers of the auriculotemporal nerve in its infratemporal pathway. In our environment the most frequent cause is surgery of the split gland with a percentage of occurrence between 10 and 80%. Conservative treatment with botulinum toxin offers good results.", "full_answer_no_ref": "You are asking about auriculotemporal syndrome or Frey's syndrome. It is the clinical expression of a vegetative neuropathy caused by mechanical or irritative lesion of the vegetative fibers of the auriculotemporal nerve in its infratemporal pathway. In our environment the most frequent cause is surgery of the split gland with a percentage of occurrence between 10 and 80%. Conservative treatment with botulinum toxin offers good results.", "full_question": "A 47-year-old man, with a history of a right parotid pleomorphic adenoma, treated with surgery (extrafacial parotidectomy) 6 months ago, who comes to our office for presenting pain with sweating and reddening of the skin in the preauricular region during mastication. What treatment would be the treatment of choice?", "id": 458, "lang": "en", "options": {"1": "Extended total parotidectomy on suspicion of tumor recurrence.", "2": "Pregabalin.", "3": "Intradermal botulinum toxin injection.", "4": "Broad-spectrum antibiotherapy.", "5": NaN}, "question_id_specific": 126, "type": "OTOLARYNGOLOGY AND MAXILLOFACIAL SURGERY", "year": 2018, "rag": {"clinical_case_options": {"MedCorp": {"RRF-2": [{"id": "pubmed23n0680_6897", "title": "[Thirteen years' experience with superficial partial parotidectomy as treatment for benign parotid tumours].", "score": 0.0177750058470414, "content": "Most authors agree that surgery is the treatment of choice for benign tumours of the parotid gland. However, the best surgical technique and the extent of surgery remain controversial. This study attempts to establish whether the implementation of a partial superficial parotidectomy (PSP) is appropriate for the treatment of benign parotid gland tumours. We selected 63 patients with benign parotid gland surgery, of whom 43 had a pleomorphic adenoma and 20, a Warthin tumour. Of this group of 63 patients, 6 could not be included. We consequently studied 57 patients, 41 of them diagnosed as pleomorphic adenoma and 16, as Warthin tumours. In all of them, a PSP was performed without intraoperative monitoring of the facial nerve. Transient facial nerve paralysis, 14 patients (24.5%). Ten cases were resolved within the first month after surgery and 4 before the third month, after indicating facial physiotherapy. One patient (1.7%) had a permanent difficulty in keeping one side of the lower lip aligned under pressure-mobility, without altering commissure mobility. None of the patients studied had a recurrence (control, 3-13 years). Although PSP is a technique with a few complications, it has a recurrence rate comparable to or lower than other techniques used for the treatment of pleomorphic adenomas or Warthin parotid tumours. Intraoperative facial nerve monitoring can be helpful during surgery. The lack of monitoring would not be considered a contraindication for surgery."}, {"id": "pubmed23n0347_17139", "title": "Botulinum toxoid in the management of gustatory sweating (Frey's syndrome) after superficial parotidectomy.", "score": 0.01709901351845086, "content": "Botulinum toxin has been successfully used to treat Frey's syndrome occurring in a 31-year-old patient following superficial parotidectomy for pleomorphic adenoma. An initial injection of 7.5 U (0.3 ml over 6 cm2 of cheek) resulted in 3 months' resolution of gustatory sweating and flushing and a second injection 12 months' symptomatic improvement. The symptoms recurred after further facial surgery."}, {"id": "pubmed23n0585_13388", "title": "[Frey syndrome secondary to submaxillectomy and botulinic treatment].", "score": 0.01661220043572985, "content": "A case of Frey syndrome (FS) secondary to submaxillar gland exeresis is presented and the results of the treatment with botulinum toxin (BTX) type A. FS is a condition of sweating cheek and preauricular area during realtime as a sequel detected in about 20-60% of patients after parotidectomy. The clinical symptoms include swelling, flushing and hyperhidrosis. The treatment choice for this condition is intracutaneous injection of BTX type A which blocks acetylcholine release at the sweat glands. A 30-year-old man, with thyroid medullar carcinoma diagnosed in 2002 received 6 cicles of cisplatin plus doxorubicin previous to the thyroidectomy with anterolateral neck dissection. During surgery the left ramus marginalis mandibulae was damaged. Two years later the patient referred sweating in submaxillar region during meals. CT scan demonstrated the absence of left submaxillar gland. Minor's test disclosed the affected area and BTX type A was injected (2.5 U/cm2/17 points). A twenty-one-day control showed a 95% reduction of the affected skin area. Persistent efficacy was observed up to one year follow-up time when he was reinjected. The FS, also known as \"gustatory hyperhidrosis\", was probably first reported by M. Duphenix in 1757. Lucja Frey considered its physiopathology as a disorder of both sympathetic and parasympathetic innervation. In our case the FS was caused by a misdirected regeneration of postganglionic parasympathetic nerve fibers that arrised from the nervus lingualis rami ganglionares of the nervus trigeminus. After nerve injury the colinergic parasympathetic fibers seek out colinergic receptors--sympathetic receptors of the skin--innervating sweat glands and small skin vessels. All previous cases were located at masseter region post-parotidectomy. We have not found any description of FS in the submaxillary region. The self-assessed efficacy of the treatment with a hyperhidrosis disease severity scale revealed a very satisfied patient at 20 months follow-up after being injected twice with BTX type A."}, {"id": "pubmed23n0055_18124", "title": "[Treatment of recurrence of pleomorphic adenomas of the parotid gland].", "score": 0.016504329004329004, "content": "This is a retrospective study of 10 patients who underwent surgery for a first or multiple recurrence of pleomorphic adenoma (P.A.). Recurrence may come more than 10 years after an initial episode. During initial surgery, the risk of recurrence is related to pre-operative dissemination and to failure to identify tumoral prolongations in the parotid gland. Recurrence is generally multifocal. In 2 cases, carcinoma developed in association with P.A. Treatment of benign recurrence is surgical: it consists of totalizing the previous parotidectomy. If the previous parotidectomy has been total, tumorectomy is carried out. In all cases, surgery preserves the facial nerve trunk."}, {"id": "pubmed23n0376_14673", "title": "[Frey syndrome after lateral parotidectomy. Follow-up and therapeutic outlook].", "score": 0.01647457627118644, "content": "Gustatory sweating is a common complication of parotid surgery. In order to evaluate the incidence of Frey's syndrome following superficial parotidectomy, 69 patients who underwent surgery due to adenoma were studied. Forty-three patients (62%) suffered from gustatory sweating following superficial parotidectomy, and 33 of them requested treatment. Nineteen patients felt that their quality of life had been decreased by the symptoms. Minor's starch iodine test proved that 85% of the patients who did not notice Frey's syndrome after surgery actually had a subclinical manifestation. Eight patients were successfully treated with intracutaneous injections of botulinum toxin A. Within 1 week gustatory sweating disappeared. Frey's syndrome is present in almost all patients following superficial parotidectomy and there is a strong need for treatment. Intracutaneous injection of botulinum toxin A is an effective treatment in severe cases of the syndrome."}, {"id": "wiki20220301en421_8393", "title": "Parotidectomy", "score": 0.01601510169285105, "content": "Complications Complications that may occur due to parotidectomy involve nerve dysfunction, Frey’s syndrome (uncharacteristic sweating near glands), salivation from wound, numbness, facial asymmetry, necrosis (death of skin) near incision, and tumor reappearance. Prognosis There is a 25-50% risk of facial weakness directly after parotidectomy and a 1-2% risk of permanent weakness. Frey’s syndrome may occur in up to 90% of patients. Risk of mortality is very low in reference to the surgery. In a case of benign tumor, such as pleomorphic adenoma, a significant outcome is also the rate of tumor recurrence. Pleomorphic adenomas may recur after a very long time from primary surgery, on average over 7–10 years and up to 24 years. Survival rates due to malignancy depends on the patient and extent of disease. A 10-year survival ranges from 32-83%. Of all cancers, salivary gland tumors account for only 1%. Parotid tumors account for 7% of all head and neck cancers. Etymology"}, {"id": "pubmed23n0297_7224", "title": "[Development and results of surgical treatment of pleomorphic adenoma of the parotid gland in 245 patients, 1974-1994].", "score": 0.015899949723479134, "content": "To describe the results of parotidectomy for pleomorphic adenoma of the parotid gland in 245 patients, 1974-1994. Descriptive. Academic Medical Hospital, Free University, Amsterdam, the Netherlands. Of all patients follow-up data were obtained by yearly physical diagnostic examination during 10 years, and by a questionnaire (1995) from the general practitioners involved. In the period 1974-1994, 246 primary surgical parotid procedures were performed on 245 patients for pleomorphic adenoma. The surgical procedures included: 131 'partial' and 61 'complete' superficial parotidectomies, 30 partial superficial/ deep lobe parotidectomies, 8 total parotidectomies, and 16 'selective' deep lobe parotidectomies. Eleven patients received postoperative radiotherapy for different reasons. The median follow-up was 95 months. Fourteen patients died without recurrent tumour. Two patients (0.8%) developed a local recurrence, both after total parotidectomy for a deep lobe tumour. None of the patients experienced permanent facial nerve paresis or paralysis. The incidence of auriculotemporal sweating for partial superficial parotidectomy proved to be 6.9% (9/I31) as compared with 13.1% (8/61) for complete superficial parotidectomy. In the later years, in the majority of patients, the posterior branch of the greater auricular nerve was preserved. In the hands of the experienced head and neck surgeon partial parotidectomy is an effective treatment for the great majority of pleomorphic adenomas: local recurrence is rare, while in general morbidity is minimal. Rarely there is a need for prolonged follow-up."}, {"id": "pubmed23n0299_15861", "title": "[Reoperation and recurrence of pleomorphic adenoma of the parotid. A propos of 62 cases].", "score": 0.015194244604316547, "content": "Between 1967 and 1994, 344 patients were treated with total conservative parotidectomy for benign pleomorphic adenoma of the parotid gland. Our retrospective study focuses on a sixty-two patients group treated for recurrence after biopsy, enucleation or total parotidectomy. Twenty-two patients underwent a systematic total parotidectomy after biopsy (n = 7) or enucleation (n = 15). Twenty-nine patients were treated with total parotidectomy for local recurrence after enucleation. The mean time before this treatment was 8 years-9 months. In the third group, 11 patients, (7 patients from our institution), were surgically treated for recurrence after total parotidectomy. After enucleation, the recurrence rate was high and insufficient margins were found in 27% of the cases. In this group, a multicentric recurrence was found in 45% of the cases. In our own experience, recurrence after total parotidectomy was noted in 2.4%. The surgical salvage was performed with enucleation after identification of the branches of the facial nerve. The operative microscope was usefull. In 1 case, a second recurrence occured, and in 1 case iterative recurrence was noted. The local control rate after total parotidectomy was 99.6% (292/293). Total conservative parotidectomy is, for us, the treatment of choice for pleomorphic adenoma of the parotid gland."}, {"id": "pubmed23n0416_22248", "title": "[Successful use of botulinum toxin injection in the treatment of salivary fistula following parotidectomy].", "score": 0.015060080106809079, "content": "A twenty-year-old woman underwent right superficial parotidectomy for pleomorphic adenoma. On the 10th postoperative day she presented with a salivary fistula, for which repeated aspirations with pressure dressings were applied for a month. Despite decreases in the salivary fluid volume, reaccumulation persisted. Following aspiration of the salivary fluid, 40 units of botulinum toxin was injected into the pouch. On the second day of injection, the discharge ceased and the pouch disappeared. No side effects were observed and the patient remained symptom-free during four-month follow-up."}, {"id": "pubmed23n0589_14957", "title": "Treatment of Frey's syndrome with botulinum toxin.", "score": 0.014991728701406122, "content": "Frey s syndrome or Gustatory sweating was first described by Baillarger in 1853. Lucie Frey had described a patient as \"auriculotemporal syndrome\" in 1923. The explanation for this symptom has been an aberrant regeneration of postganglionic parasympathetic fibers feeding the parotid gland that are severed during parotidectomy. After parotidectomy, these cholinergic parasympathetic fibers regenerate and anastomosis with postganglionic sympathetic fibers that supply vessel and sweat gland of the skin. According to a recent study, the treatment of Frey's syndrome has no treatment of choice. The authors investigated the effectiveness of botulinum toxin type A in the treatment of Frey's syndrome for the first time in Thai patients. The present study was a prospective non-randomized, exploratory study. Nine patients with a median involvement skin area of 4.2 cm2 (1-16.3) were injected intradermal with botulinum toxin type A 2 unit in every 1 cm2 of involved skin. The mean total dose was 10.6 units (range 2-32 unit). All of the patients showed improvement after 4-7 days. Five patients have no Gustatory sweating. In the same way, four patients present with a dramatic decrease in Gustatory sweating. When comparing the skin involvement area, indicated by Minor's iodine starch test and calculated by program ImageJ 1.34s, between before and after injection of botulinum toxin type A using sign test, the result is statistically significant with p = 0.0039. The result lasted for 9.2 months (7-10 months). Intradermal injection of botulinum toxin type A for patients with Frey's syndrome is not only effective with no side effect but also minimally invasive. The present report supports that intradermal injection of botulinum toxin type A should be the treatment of choice for Frey's syndrome."}, {"id": "pubmed23n0369_8023", "title": "[Post-parotidectomy Frey's syndrome. Treatment with botulinum toxin type A].", "score": 0.014760904105376456, "content": "The Frey's syndrome, manifest after parotid trauma, is characterized by head and neck hyperemia and abundant sweating of the hyperemic skin in response to gustatory stimuli. The use of the botulin toxin to treat the symptoms in patients with Frey's syndrome has been described in numerous studies. For some time up until now our Center has achieved excellent results using the group A botulin toxin to overcome the hypertonus of the cricopharyngeal muscle in patients who had undergone laryngectomy and were rehabilitated with voice button. We have sought to extend the use of this toxin to Frey's syndrome, a relatively frequent complication of parotidectomy. A total of 86 patients participated in the study: 41 males (47.6%) and 45 females (52.4%) ranging in age from 25 to 77 years (average age 51 years). Of these patients 7 (8.1%) had undergone post-operative radiotherapy. Of the 86 patients studied, 18 referred significant symptoms in terms of abundance and frequency. The syndrome was considered severe if the symptoms were present at each meal and if the patient indicated a significant worsening of his quality of life. Intermittent episodes were indicated by 22 patients. The remaining 46 (43.5%) did not complain of any symptoms. The exact extension of the cervicofacial gustatory sweating was evaluated using the Minor test and the involved region was divided into 1 square centimeters sections. The amount of skin surface involved ranged from 10 to 80 square centimeters. The type A neurotoxin was frozen and was reconstituted with a sterile saline solution at a final concentration of 2.5 UI/0.1 ml. The intracutaneous infiltration was performed without anesthesia, infiltrating 0.1 ml of solution, containing 2.5 UI of toxin into the center of each 1 square centimeters section. Statistical analysis was performed to evaluate the potential relationship between how long the treatment was effective, incidence of recurrence, seriousness of the crises and the following variables: age, sex, histology, cutaneous surface involved, injected dose of botulin toxin and post-operative radiotherapy. In the group of 18 patients with severe symptoms (20.9%) the benefit was immediate in all cases although the recurrence rate was 50%. The Frey's syndrome symptoms disappeared within 7 days of infiltration. In the group of 22 patients with less severe involvement (25.5%), the treatment gave positive, definitive results in 16 patients (72.7%). Those patients whose symptoms persisted were treated a second time with an infiltration of 2.5 UI per square centimeters. We feel that the use of the type A botulin toxin is the most appropriate treatment for the Frey's syndrome. In fact, such treatment offers the following advantages: it is effective within 7 days, has limited side effects, can be applied on an outpatient basis, is inexpensive and is positively considered by the patients."}, {"id": "wiki20220301en073_31938", "title": "Pleomorphic adenoma", "score": 0.014067656765676569, "content": "There have been several approaches for surgery of parotid pleomorphic adenoma in the course of time. Enucleation of the tumor (i.e. intracapsular dissection), a procedure that was common in the early 20th century, is nowadays obsolete due to very high incidence of recurrence. After the time of enucleations, pleomorphic adenomas of parotid gland were recommended to be routinely treated with superficial or total parotidectomy. These procedures combine complete tumor removal and identification of the main trunk of facial nerve during surgery to avoid any lesions to the nerve. However, extensive surgery may cause significant morbidity, such as Frey´s syndrome (excessive sweat while eating) and salivary fistula. Also, aesthetic outcome may be compromised. Therefore, less invasive procedures have been preferred in selected cases during the recent years, and introduction of perioperative neuromonitoring enabled the evolution of several different surgical techniques some twenty years ago."}, {"id": "pubmed23n0546_18065", "title": "Treatment of gustatory sweating with low-dose botulinum toxin A: a case report.", "score": 0.013776834158999765, "content": "Frey's syndrome, gustatory sweating in the preauricular area, is an unpleasant phenomenon occurring during meals after surgery on the parotid gland. Recently, botulinum toxin A (BTX) has been shown to reduce the symptoms, but the variation in the reported doses is large. To quantify the effect of treatment with low-dose BTX in a case of Frey's syndrome over a period of 6 months. A 56-year-old woman was treated with 10 U Botox given as 20 single, intracutaneous injections of 0.5 U, one for each cm(2), 3 years after resection of the parotid gland. Before treatment and repeatedly during the 6-month period, the sweating was rated subjectively on a 100-mm visual analog scale (VAS) and by a severity index, and objectively by assessment of the extent of the involved skin area using Minor's iodine-starch test, staining the area of sweating dark. The treatment decreased the involved area from 20 to 5 cm(2) and the VAS ratings from 98 to 8 mm. The index showed that treatment affected the sweating intensity, not the frequency. After the 6-month period the patient was still satisfied, but the involved skin area had increased; however, not entirely to pretreatment values. The effect of BTX injections for gustatory sweating obtained in this case was comparable to results reported using higher doses. Low doses of BTX can therefore be used in the treatment of Frey's syndrome, but studies to clarify the dose-response relationship, in terms of both time-course and obtained effect, are needed."}, {"id": "pubmed23n0559_20248", "title": "Management of Frey syndrome.", "score": 0.013661202185792351, "content": "Almost all patients who undergo parotidectomy will to some extent develop Frey syndrome (auriculotemporal syndrome or gustatory sweating) after surgery, because of aberrant regeneration of cut parasympathetic fibers between otic ganglion and subcutaneous vessels. However, only the minority of these patients needs treatment. The syndrome consists of gustatory sweating, flushing, and warming over the preauricular and temporal areas. Thick skin flap and partial superficial parotidectomy are the most important techniques to minimize the risk of developing symptomatic Frey syndrome. Intracutaneous injection of botulinum toxin A is an effective, long-lasting, and well-tolerated treatment of Frey syndrome. If recurrence occurs, the treatment can be repeated."}, {"id": "article-73061_35", "title": "Parotidectomy -- Complications", "score": 0.013440285204991086, "content": "Frey’s syndrome: more accurately referred to as gustatory sweating. Patients report facial swelling and sweating at the site of the parotidectomy in occurrence with meals. Etiology is believed to be aberrant innervation of the sweat glands with branches emerging from the auriculotemporal nerve after their division during surgery. This provides parasympathetic innervation to the normally sympathetic-innervated sweat glands [38] . Diagnosis is usually based on patient history, however if there is any doubt an iodine-starch test (Minor test) will confirm the diagnosis, where iodine starch placed on the affected area turns blue signaling sweat secretion. The incidence historically has been reported as high as 50 to 100%, though, with modern techniques and the use of SMAS flaps and thicker skin flaps at the time of initial elevation, this is greatly reduced and is now quite rare. Should this develop, surgical treatment options can be disappointing, with the best results obtained using SMAS and superficial temporal artery flaps as a barrier between the surgical site and the skin. Gold standard treatment now is botulinum toxin injection. Relief of symptoms is obtained for 6 to 36 months. It works at the pre-synaptic level of the neuromuscular and neuroglandular junction by blocking the release of acetylcholine. [37] [39]"}, {"id": "wiki20220301en073_31939", "title": "Pleomorphic adenoma", "score": 0.013385208452312964, "content": "Currently, the choice of surgical approach for parotid pleomorphic adenoma is mainly based on the size, location, and mobility of the tumor. The recommended main techniques include extracapsular dissection, partial superficial parotidectomy, and lateral or total parotidectomy. Nevertheless, the experience of surgeon plays a key role in the results of these distinct procedures. An important point of view is that recurrent pleomorphic adenomas may occur after a very long time from primary surgery, on average over 7–10 years but up to 24 years afterwards. Thus, it is of utmost importance to evaluate the ultimate results of these different surgical techniques in the future. The benign tumors of the submandibular gland is treated by simple excision with preservation of mandibular branch of the facial nerve, the hypoglossal nerve, and the lingual nerve. Other benign tumors of minor salivary glands are treated similarly."}, {"id": "pubmed23n0320_7935", "title": "[Surgery in benign parotid tumors: individually adapted or standardized radical interventions?].", "score": 0.013159937888198758, "content": "Several authors demand emphatically that the minimal operative procedure in benign parotid gland tumors has to be a superficial parotidectomy. Of a consecutive series of 372 patients with benign parotid tumors treated in our department between 1973-1996 81% of the patients could be followed up 1-24 years. in 10.9% a total parotidectomy was performed, in 16% a lateral parotidectomy and in 73.1% a simple extirpation of the tumor (often taking away a small margin of surrounding parotid parenchyma). The operating microscope and microsurgical techniques were used in all of these operations. Of all the followed-up patients 2.3% developed a recurrence. There were no recurrences of cystadenolymphomas or of rare types of adenomas. Recurrences of primary treated pleomorphic adenomas occurred in 3.0%. In recurrent pleomorphic adenomas a further recurrence could be seen in 7.4% of the cases. The over-all incidence of permanent facial nerve weakness was 2.1%: 0.7% after extirpation, 3.3% after lateral parotidectomy and 9.7% after total parotidectomy. we observed in 6.3% a gustatory sweating. Our data prove that with simple extirpation similar results compared to lateral parotidectomy can be achieved concerning recurrence, function of the facial nerve and the Frey's syndrome. We suggest a surgical management adapted to the extent, the size and the location of the parotid gland tumors. In our opinion lateral or total parotidectomy should be reserved for tumors of larger amount or deep located tumors."}, {"id": "wiki20220301en421_8390", "title": "Parotidectomy", "score": 0.01309090909090909, "content": "Post-Operation After completion of a parotidectomy, patients can expect postoperative hospitalization ranging from one-to-three days, to help ensure the safest and most effective postoperative management. At this time, patients will be administered antibiotics to minimize risk of infection as well as an assessment of pain management throughout their stay. Duration of hospitalization is subject to change from patient to patient, with most patients being discharged within 24 hours after surgery. If a tumor was malignant, many patients are referred to radiation therapy. For benign tumors and slow growing cancers, surgery typically provides a complete cure or remission (no evidence for disease). Patient Care after Discharge"}, {"id": "wiki20220301en421_8385", "title": "Parotidectomy", "score": 0.01266025641025641, "content": "Throughout history, many different types and techniques have been developed in order to complete a parotidectomy and consequently, many different names have been associated with each type. However, there are really only two main distinctions to be made in parotidectomies: The specific nerve(s) to be dissected or not dissected The amount of gland excised It is important to note that the specific surgery chosen is based on preservation of the facial nerve in order to avoid significant morbidities (diseases). Furthermore, there are still many controversies regarding the choice of surgery and incidence of cancer recurrence. Below indicates the various and main techniques typically associated with a parotidectomy: Extracapsular dissection - excision of the parotid tumor surrounded by some millimetres of healthy tissue, without searching and exposing the main truck of the facial nerve."}, {"id": "pubmed23n0792_11512", "title": "Pleomorphic adenoma and benign parotid tumors: extracapsular dissection vs superficial parotidectomy--review of literature and meta-analysis.", "score": 0.01253930817610063, "content": "This study compared extracapsular dissection (ED) vs superficial parotidectomy (SP) in the treatment of pleomorphic adenoma and benign parotid tumors. The research covered the years 1950-2011 in PubMed, Ovid MEDLINE, the Cochrane Database of Systematic Reviews, and Scopus. Of 1152 articles screened, 123 studies met the inclusion criteria. A review of the nomenclature of the different parotid surgery techniques was done. Recurrence rate, permanent facial nerve paralysis, and Frey syndrome of patients who underwent ED vs those who underwent SP were compared by meta-analysis. Our meta-analysis data comparing ED and SP found that: (1) the recurrence rate is higher in patients treated with SP; (2) SP has a higher incidence of cranial nerve VII paralysis; and (3) Frey syndrome is more common after SP. ED may be a viable option in the treatment of unilateral benign parotid tumors of the superficial lobe, sized less than 4 cm, without involvement of the facial nerve."}, {"id": "wiki20220301en110_5731", "title": "Frey's syndrome", "score": 0.012330648685771366, "content": "Treatments Injection of botulinum toxin A Surgical transection of the nerve fibers (a temporary treatment) Application of an ointment containing an anticholinergic drug such as scopolamine Cochrane reviews of interventions to either prevent or treat Frey’s syndrome have found little or no evidence to support their effectiveness or safety, and conclude that further clinical trials are needed. Epidemiology The condition is rare, although the exact incidence is unknown. The disorder most often occurs as a complication of the surgical removal of a parotid gland (parotidectomy). The percentage of individuals who develop Frey syndrome after a parotidectomy is controversial and reported estimates range from 30–50 percent. In follow-up examinations, approximately 15 percent of affected individuals rated their symptoms as severe. Frey syndrome affects males and females in equal numbers."}, {"id": "pubmed23n0266_6450", "title": "[Pleomorphic adenoma of the parotid gland. Results of surgical treatment].", "score": 0.012303485987696514, "content": "Twenty years experience of lateral parotidectomy as suspical treatment for pleomorphic adenoma are reviewed. All cases were managed at the ORL Clinic of the University of Zürich. 167 patients were followed for the frequency of possible recurrent tumors. Three patients (3/123) operated primarily developed a recurrences. Recurrences appeared after an average of 10 years, ranging from 1-30 years. The follow-up time varied from 1 to 21 years (average, 8 years). 39% (13 of 33) of the patients, who were re-operated for a recurrent tumor, developed another recurrence. The second recurrence appeared after an average of 10 years, ranging from 1-22 years. A persistent partial paresis of the facial nerve was found in 1% of the patients operated primarily and in 9% of the patients operated more than once. No paralysis was seen. We now choose \"en-bloc\" resections of pleomorphic adenomas without intra-operative opening of the tumor capsule as the treatment of choice. This treatment was possible in 83% of all cases, using a lateral parotidectomy. If tumor extends into the medial parotid lobe, total parotidectomy is required."}, {"id": "pubmed23n0598_787", "title": "[Parotid gland's tumors in children].", "score": 0.01148989898989899, "content": "The tumors of the salivary glands are infrequent in children, and parotid gland is involved in 80% of them. When a salivary gland tumor is present, the chance of malignancy is greater in the child than in the adult. We reviewed 8 cases identified in patients aged 14 years and younger in our hospital, analyzing its antecedents, signs and symptoms, histological features, diagnosis, treatment and evolution. All the patients displayed preauricular painless, non-inflammatory and slow-growing masses to an age between 10 months and 14 years. Four or them were pleomorphic adenomas, two haemangiomas, one epidermal cysts and one myoepithelial carcinoma. We emphasize the exceptional nature of the carcinoma for its rareness and for the high degree of malignancy expressed. We made a fine needle aspiration biopsy in four cases but they were conclusive only in three. All were treated by surgical resection of the tumour except for the myoepithelial carcinoma and the recurrent pleomorphic adenoma that were treated by total parotidectomy. The malignant tumours of the parotid gland are clinically indistinguishable of the benign ones, thus when any palpable mass appears in the zone of the parotid gland, an accurate diagnosis should be made without delay. The treatment of choice is the surgical excision with wide margins, being other adjuvant treatments less useful to this age than in the adult age."}, {"id": "pubmed23n1091_11060", "title": "Botulinum toxin for chronic parotid sialadenitis: A case series and systematic review.", "score": 0.011457609999368727, "content": "To evaluate salivary gland chemodenervation with botulinum toxin in chronic parotid sialadenitis. Patients who underwent parotid gland chemodenervation for chronic sialadenitis due to duct stenosis refractory to siaendoscopy were reviewed (case series). Additionally, a systematic review of the literature on botulinum toxin injection for chronic parotid sialadenitis was performed. Inclusion criteria included studies containing original data on botulinum toxin injections in patients with chronic sialadenitis symptoms. Sialadenitis symptoms from 10 patients with 13 affected parotid glands were examined. All had duct stenosis diagnosed on sialendoscopy, refractory sialadenitis symptoms, and received parotid onabotulinum toxin injection(s) (median dose 65U). Of patients with 3-month follow-up, 78% reported significant improvement in symptoms. Mean Chronic Obstructive Sialadenitis Symptoms (COSS) Score improved at 3 months post-injection (47-25.9, 10 WBCs/high power field (hpf) 2. Dipstick-Positive leukocyte esterase (due to pyuria) and nitrites (bacteria convert nitrates to nitrites) 3. Culture-greater than 100,000 colony forming units (gold standard) D. Etiology 1. E coli (80%) 2. Staphylococcus saprophyticus-increased incidence in young, sexually active women (but E coli is still more common in this population) 3. Klebsiella pneumoniae 4. Proteus mirabilis-alkaline urine with ammonia scent Fig. 12.15 Linear IF, Goodpasture syndrome. Fig. 12.16 lgA nephropathy. (Courtesy ofTony (Courtesy ofTony Chang, MD) Chang, MD) 5. Enterococcus faecalis E. Sterile pyuria is the presence of pyuria (> 10 WBCs/hpf and leukocyte esterase) with a negative urine culture."}, {"id": "pubmed23n0778_14939", "title": "Antibiotic sensitivity pattern of uropathogens from pregnant women with urinary tract infection in Abakaliki, Nigeria.", "score": 0.014916435298600905, "content": "Urinary tract infection (UTI) is a common bacterial infection during pregnancy and a significant cause of perinatal and maternal morbidity and mortality. The causative bacteria have remained virtually the same although with variations in individual prevalence. There has been an increasing resistance by these bacteria to the commonly available antibiotics. To determine the prevalence of UTI, the common causative bacteria, and their antibiotic sensitivity pattern among pregnant women with UTI. This is a descriptive study that was carried out at the Obstetrics Department of two tertiary institutions in Abakaliki, Ebonyi State, Nigeria (Federal Medical Center and Ebonyi State University Teaching Hospital) over a period of 12 months. Midstream urine specimens from selected pregnant women with clinical features of UTI were collected for microscopy, culture, and sensitivity. The results were analyzed with the 2008 Epi Info™ software. A total of 542 pregnant women presented with symptoms of UTI and were recruited for the study over the study period. Of the 542 pregnant women, 252 (46.5%) had significant bacteriuria with positive urine culture and varying antibiotic sensitivity pattern. The prevalence of symptomatic UTI was 3%. Escherichia coli was the most common bacteria isolated with a percentage of 50.8%. Other isolated micro organisms included Stapylococcus aereus (52 cultures, 20.6%), Proteus mirabilis (24 cultures, 9.5%), S. saprophyticus (18 cultures, 7.1%), Streptococcus spp. (14 cultures, 5.6%), Citrobacter spp. (5 cultures, 2.0%), Klebsiella spp. (4 cultures, 1.6%), Enterobacter spp. (4 cultures, 1.6%), and Pseudomonas spp. (3 cultures, 1.2%). Levofloxacin had the highest overall antibiotic sensitivity of 92.5%. Others with overall antibiotic sensitivity pattern greater than 50% included cefpodoxime (87.3%), ofloxacin (77.4%), ciprofloxacin (66.7%), ceftriaxone (66.7%), and gentamicin (50.8%). E. coli was the most common etiological agent of UTI in pregnancy with Enterococcus (Staphylococcus) gaining prominence. Cephalosporin and quinolones were shown to be very effective against the organisms causing UTI in these pregnant women."}, {"id": "pubmed23n0601_2316", "title": "Contemporary management of uncomplicated urinary tract infections.", "score": 0.014780577001219016, "content": "Uncomplicated urinary tract infections (uUTIs) are common in adult women across the entire age spectrum, with mean annual incidences of approximately 15% and 10% in those aged 15-39 and 40-79 years, respectively. By definition, UTIs in males or pregnant females and those associated with risk factors known to increase the risk of infection or treatment failure (e.g. acquisition in a hospital setting, presence of an indwelling urinary catheter, urinary tract instrumentation/interventions, diabetes mellitus or immunosuppression) are not considered herein. The majority of uUTIs are caused by Escherichia coli (70-95%), with Proteus mirabilis, Klebsiella spp. and Staphylococcus saprophyticus accounting for 1-2%, 1-2% and 5-10% of infections, respectively. If clinical signs and symptoms consistent with uUTI are present (e.g. dysuria, frequency, back pain or costovertebral angle tenderness) and there is no vaginal discharge or irritation present, the likelihood of uUTI is >90-95%. Laboratory testing (i.e. urinary nitrites, leukocyte esterase, culture) is not necessary in this circumstance and empirical treatment can be initiated. The ever-increasing incidence of antimicrobial resistance of the common uropathogens in uUTI has been and is a continuing focus of intensive study. Resistance to cotrimoxazole (trimethoprim/sulfamethoxazole) has made the empirical use of this drug problematic in many geographical areas. If local uropathogen resistance rates to cotrimoxazole exceed 10-25%, empirical cotrimoxazole therapy should not be utilized (fluoroquinolones become the new first-line agents). In a few countries, uropathogen resistance rates to the fluoroquinolones now exceed 10-25%, rendering empirical use of fluoroquinolones problematic. With the exception of fosfomycin (a second-line therapy), single-dose therapy is not recommended because of suboptimal cure rates and high relapse rates. Cotrimoxazole and the fluoroquinolones can be administered in 3-day regimens. Nitrofurantoin, a second-line therapy, should be given for 7 days. beta-Lactams are not recommended because of suboptimal clinical and bacteriological results compared with those of non-beta-lactams. If a beta-lactam is chosen, it should be given for 7 days. Management of uUTIs can frequently be triaged to non-physician healthcare personnel without adverse clinical consequences, resulting in substantial cost savings. It can be anticipated that the optimal approach to the management of uUTIs will change substantially in the future as a consequence of antimicrobial resistance."}, {"id": "pubmed23n0759_24868", "title": "Symptomatic Shigella sonnei urinary tract infection in pregnancy.", "score": 0.014674719220173766, "content": "This report describes a case of urinary tract infection (UTI) due to Shigella sonnei during pregnancy. A 31-year-old pregnant woman was admitted complaining of left-flank tenderness, dysuria, and fever. Following examination, significant laboratory data were collected including increased leukocyte count (10,800/ul with 86% neutrophils) and C-reactive protein (9.6 mg/dl). Urinalysis revealed 30 to 50 leukocytes per high power field while from the quantitative urine culture Shigella sonnei was recovered after 24 h incubation at 37 degrees C. After a two-week course with 750 mg cefuroxime every 8 h, the patient experienced gradual resolution of all symptoms and urinary cultures were negative two weeks and one month, respectively, after completing the therapy. The gestational course was uneventful and the patient delivered a healthy baby girl at term. Shigella sonnei can be responsible for UTI during pregnancy even when no predisposing factors or an apparent source of infection can be identified."}, {"id": "pubmed23n0771_21526", "title": "Urinary tract infection in outpatient febrile infants younger than 30 days of age: a 10-year evaluation.", "score": 0.014210128495842782, "content": "To determine the prevalence of outpatient-diagnosed urinary tract infection (UTI) in consecutive febrile neonates ≤ 30 days of age and correlate demographic, laboratory and radiographic imaging results with infectious etiology. Review of medical records of consecutive febrile infants ≤ 30 days of age presenting to an urban pediatric emergency department during a 10-year period, whose policy is to perform a sepsis evaluation (urine culture obtained by bladder catheterization) and hospitalize for parenteral antibiotic therapy pending culture results. Of 670 febrile neonates ≤ 30 days of age evaluated for sepsis, urine culture was obtained in 651 cases (97%). Of 100 patients with UTI (15.4%), 73% were male; the most common uropathogens were Escherichia coli (71%), Enterococcus (10%) and Klebsiella sp. (10%). In all, 39% had a maximum documented fever ≥ 102 °F, and 40% had CBC total white blood cells count ≥ 15,000/mm(3). Urine dipstick test was positive for leukocyte esterase or nitrite in 79%. Renal ultrasound performed in 95 patients (95%) showed anatomic abnormalities in 47%; 5/26 (24%) with hydronephrosis had vesicoureteral reflux on voiding cystourethrogram. Four patients had urosepsis; none had bacterial meningitis and no patients died. UTI affects approximately 1 in 6 febrile neonates ≤ 30 days of age. Males are affected 2.5-times greater than females. E. coli continues to be the predominant uropathogen. Clinical parameters like height of fever, CBC total white blood cell count and urine dipstick test lack sensitivity in identifying UTI risk in the outpatient setting. Only 4 infants had urosepsis (4%). Nearly half of neonates with UTI have a radiographically identified anatomic abnormality. All febrile young infants should receive performance of a urine culture; those with UTI require imaging."}, {"id": "Obstentrics_Williams_7288", "title": "Obstentrics_Williams", "score": 0.013614987864077669, "content": "If this infection is suspected, a urine sample obtained by catheterization may be preferred to avoid obscuring contamination from the lower genital tract. he urinary sediment contains many leukocytes, frequently in clumps, and numerous bacteria. Bacteremia is demonstrated in 15 to 20 percent of these women. E coli is isolated from urine or blood in 70 to 80 percent of infections, Klebsiela pneumoniae in 3 to 5 percent, Enterobacter or Proteus species in 3 to 5 percent, and gram-positive organisms, including group B Streptococcus and Staphylococcus aureus, in up to 10 percent of cases (Hill, 2005; Wing, 2000)."}, {"id": "pubmed23n0788_19788", "title": "Can a simple urinalysis predict the causative agent and the antibiotic sensitivities?", "score": 0.013450710519259987, "content": "The objective of this study was (1) to determine the reliability of urinalysis (UA) for predicting urinary tract infection (UTI) in febrile children, (2) to determine whether UA findings can predict Escherichia coli versus non-E. coli urinary tract infection, and (3) to determine if empiric antibiotics should be selected based on E. coli versus non-E. coli infection predictions. This was a retrospective chart review of children from 2 months to 2 years of age who presented to the emergency department with fever (rectal temperature >100.4°F) and had a positive urine culture. This study was conducted between January 2004 and December 2007. Negative UA was defined as urine white blood cell count less than 5 per high-power field, negative leukocyte esterase, and negative nitrites. Urine cultures were classified into E. coli and non-E. coli groups. These groups were compared for sex, race, and UA findings. Multivariate forward logistic regression, using the Wald test, was performed to calculate the likelihood ratio (LR) of each variable (eg, sex, race, UA parameters) in predicting UTI. In addition, antibiotic sensitivities between both groups were compared. Of 749 medical records reviewed, 608 were included; negative UA(-) was present in 183 cases, and positive UA(+) was observed in 425 cases. Furthermore, 424 cases were caused by E. coli, and 184 were due to non-E. coli organisms. Among 425 UA(+) cases, E. coli was identified in 349 (82.1%), whereas non-E. coli organisms were present in 76 (17.9%); in contrast, in 183 UA(-) cases, 108 (59%) were due to non-E. coli organisms versus 75 (41%), which were caused by E. coli. Urinalysis results were shown to be associated with organism group (P < 0.001). Positive leukocytes esterase had an LR of 2.5 (95% confidence interval [CI], 1.5-4.2), positive nitrites had an LR of 2.8 (95% CI, 1.4-5.5), and urine white blood cell count had an LR of 1.8 (95% CI, 1.3-2.4) in predicting E. coli versus non-E. coli infections. Antibiotic sensitivity compared between UA groups demonstrated equivalent superiority of cefazolin (94.7% sensitive in UA(+) vs 84.0% in UA(-) group; P < 0.0001), cefuroxime (98.2% vs 91.7%; P < 0.001), and nitrofurantoin (96.1% vs 82.2%; P < 0.0001) in the UA(+) group. In contrast, the UA(-) group showed significant sensitivity to trimethoprim-sulfamethoxazole (82.2% vs 71.3% in UA(+); P = 0.008). Urinalysis is not an accurate predictor of UTI. A positive urine culture in the presence of negative UA most likely grew non-E. coli organisms, whereas most UA(+) results were associated with E. coli. This study also highlighted local patterns of antibiotic resistance between E. coli and non-E. coli groups. Negative UA results in the presence of strong suspicion of a UTI suggest a non-E. coli organism, which may be best treated with trimethoprim-sulfamethoxazole. Conversely, UA(+) results suggest E. coli, which calls for treatment with cefazolin or cefuroxime."}, {"id": "pubmed23n0948_2044", "title": "Clinical and laboratory features of urinary tract infections in young infants.", "score": 0.013385262780911452, "content": "Urinary tract infection (UTI) is the most common serious bacterial infection in young infants. Signs and symptoms are often nonspecific. To describe clinical, demographic and laboratory features of UTI in infants ≤ 3 months old. Cross-sectional study of infants ≤ 3 months old with UTI diagnosed in a pediatric emergency department, for the period 2010-2012. UTI was defined as ≥ 50,000 colony-forming units per milliliter of a single uropathogen isolated from bladder catheterization. Paired urinalysis and urine culture from group culture-positive and group culture-negative were used to determine the sensitivity and specificity of pyuria and nitrite tests in detecting UTI. Of 519 urine cultures collected, UTI was diagnosed in 65 cases (prevalence: 12.5%); with male predominance (77%). The most common etiologies were Escherichia coli (56.9%), Klebsiella pneumoniae (18.5%) and Enterococcus faecalis (7.7%). Frequent clinical manifestations were fever (77.8%), irritability (41.4%) and vomiting (25.4%). The median temperature was 38.7°C. The sensitivity of the nitrite test was 30.8% (95%CI:19.9-43.4%), specificity of 100% (95%CI:99.2-100%). Pyuria ≥ 10,000/mL had a sensitivity of 87.7% (95%CI:77.2-94.5%), specificity of 74.9% (95%CI:70.6 -78.8%). The median peripheral white blood cell count was 13,150/mm3; C-reactive protein levels were normal in 30.5% of cases. The male: female ratio for urinary tract infection was 3.3:1. Non-Escherichia coli etiologies should be considered in empirical treatment. Fever was the main symptom. Positive nitrite is highly suggestive of UTI but has low sensitivity; whereas pyuria ≥ 10,000/mL revealed good sensitivity, but low specificity. Peripheral white blood cell count and C-reactive protein concentration have limited usefulness to suggest UTI."}, {"id": "wiki20220301en001_192883", "title": "Urinary tract infection", "score": 0.012920489296636086, "content": "Diagnosis In straightforward cases, a diagnosis may be made and treatment given based on symptoms alone without further laboratory confirmation. In complicated or questionable cases, it may be useful to confirm the diagnosis via urinalysis, looking for the presence of urinary nitrites, white blood cells (leukocytes), or leukocyte esterase. Another test, urine microscopy, looks for the presence of red blood cells, white blood cells, or bacteria. Urine culture is deemed positive if it shows a bacterial colony count of greater than or equal to 103 colony-forming units per mL of a typical urinary tract organism. Antibiotic sensitivity can also be tested with these cultures, making them useful in the selection of antibiotic treatment. However, women with negative cultures may still improve with antibiotic treatment. As symptoms can be vague and without reliable tests for urinary tract infections, diagnosis can be difficult in the elderly."}, {"id": "pubmed23n0679_18816", "title": "Escherichia coli septic arthritis of a lumbar facet joint following urinary tract infection.", "score": 0.012839506172839507, "content": "Septic arthritis of a lumbar facet joint is a rare condition. We report the case of a 77-year-old diabetic woman who developed fever and back pain 15 days after she had been diagnosed with a genitourinary infection for which she had received ciprofloxacin. Physical examination showed fever (38°C) and pain on pressure over the lower lumbar spinous vertebral apophyses and over the lower left paraspinal musculature. Investigations showed a white cell count of 8.4×10⁹/l, neutrophils 85.3%, erythrocyte sedimentation rate of 125 mm/h, and C-reactive protein of ≥9 mg/dl. Two blood cultures were both positive for Escherichia coli resistant to ciprofloxacin. There was no growth of pathogens from the urine cultures. Scintigraphy with gallium citrate Ga⁶⁷ showed vertical lower lumbar (L4-L5) radionuclide uptake lateralized to the left. Magnetic resonance imaging of the lumbar spine demonstrated signal changes and alteration of the structure at the left interapophyseal L4-L5 joint, an adjacent small collection of 1cm in diameter, and infiltration of the surrounding soft tissues, which extended to the epidural area, left conjunction hole, and paraspinal muscles. The patient was treated with intravenous cefotaxime and gentamicin and bed rest for 21 days, and recovered. This is the first report of interapophyseal arthritis caused by E. coli."}, {"id": "wiki20220301en098_56853", "title": "Nitrite test", "score": 0.01182412358882947, "content": "Urinary nitrite test A nitrite test is a standard component of a urinary test strip. A positive test for nitrites in the urine is called nitrituria. This test is commonly used in diagnosing urinary tract infections (UTIs). A positive nitrite test indicates that the cause of the UTI is a gram negative organism, most commonly Escherichia coli. The reason for nitrites' existence in the presence of a UTI is due to a bacterial conversion of endogenous nitrates to nitrites. This may be a sign of infection. However, other parameters, such as leukocyte esterase, urine white blood cell count, and symptoms such as dysuria, urinary urgency, fevers, and chills must be correlated to diagnose an infection."}, {"id": "article-30856_8", "title": "Urinary Tract Infection in Pregnancy -- Pathophysiology", "score": 0.01145320197044335, "content": "Organisms causing UTI in pregnancy are the same uropathogens which commonly cause UTI in non-pregnant patients. Escherichia coli is the most common organism isolated. An 18-year retrospective analysis found E. coli to be the causative agent in 82.5% of cases of pyelonephritis in pregnant patients. [3] Other bacteria which may be seen include Klebsiella pneumoniae, Staphylococcus, Streptococcus, Proteus, and Enterococcus species."}, {"id": "wiki20220301en254_18243", "title": "Urine test strip", "score": 0.01132948434039861, "content": "It is normal to find up to 3 (occasionally 5) leukocytes per high power field (40X) in a urine sample, with women having slightly higher results owing to vaginal contamination. Higher numbers indicate urinary infection. The urine test strip test for white blood cells detects leukocyte esterase, which is present in azurophilic granules of monocytes and granulocytes (neutrophilic, eosinophilic and basophilic). Bacteria, lymphocytes and epithelial cells from the genitourinary tract do not contain esterases. Neutrophil granulocytes are the leukocytes most commonly associated with urinary infections. A positive test for leukocyte esterase normally indicates the presence of bacteria and a positive nitrite test (although it is not always the case). Infections caused by Trichomonas, Chlamydia and yeasts produce leukocyturia without bacteriuria. The inflammation of the renal tissues (interstitial nephritis) can produce leukocyturia, in particular toxic interstitial nephritis with predominant"}, {"id": "wiki20220301en001_192882", "title": "Urinary tract infection", "score": 0.01091644204851752, "content": "Pathogenesis The bacteria that cause urinary tract infections typically enter the bladder via the urethra. However, infection may also occur via the blood or lymph. It is believed that the bacteria are usually transmitted to the urethra from the bowel, with females at greater risk due to their anatomy. After gaining entry to the bladder, E. Coli are able to attach to the bladder wall and form a biofilm that resists the body's immune response. Escherichia coli is the single most common microorganism, followed by Klebsiella and Proteus spp., to cause urinary tract infection. Klebsiella and Proteus spp., are frequently associated with stone disease. The presence of Gram positive bacteria such as Enterococcus and Staphylococcus increased. The increased resistance of urinary pathogens to quinolone antibiotics has been reported worldwide and might be the consequence of overuse and misuse of quinolones. Diagnosis"}, {"id": "pubmed23n0386_10564", "title": "Comparison of test characteristics of urine dipstick and urinalysis at various test cutoff points.", "score": 0.010905741114941163, "content": "We compare the test characteristics of urine dipstick and urinalysis at various test cutoff points in women presenting to emergency departments and an intermediate care center with symptoms of urinary tract infection. This was a prospective, observational study of adult women presenting to 1 of 2 community hospital EDs or an intermediate care center with dysuria, urgency, or urinary frequency on history, or suprapubic or costovertebral angle tenderness on examination. Patients who had taken antibiotics in the past 72 hours, had indwelling Foley catheters, symptomatic vaginal discharge, diabetes mellitus, immunodeficiency disorders, or were unable to provide a reliable history were excluded. The patient's clean-catch or catheterized urine specimen was tested immediately by a nurse using a Multistix 9 SG reagent strip. A second aliquot was sent within 1 hour of collection to the hospital laboratory, where a semiautomated microscopic urinalysis and a urine culture were performed. A positive urine culture was defined as more than 100,000 colonies of 1 or 2 uropathogenic bacteria per mL of urine at 48 hours. Dipstick and urinalysis data were compared with urine culture results. Sensitivity, specificity, and predictive values were calculated at various definitions of a positive test, or \"test cutoff points,\" for combinations of leukocyte esterase, nitrite, and blood on dipstick and for RBCs and WBCs on urinalyses. The probability of an erroneous decision to withhold treatment on the basis of a negative test result was defined as \"undertreatment,\" or 1 minus the negative predictive value. \"Overtreatment\" was defined as 1 minus the positive predictive value. Three hundred forty-three patients were enrolled in this study. Twelve patients were withdrawn because of missing laboratory results. Forty-six percent (152/331) of patients had positive urine cultures. If urine dipstick results are defined as positive when leukocyte esterase or nitrite is positive or blood is more than trace, the overtreatment rate is 47% (156/331) and the undertreatment rate is 13% (43/331). If urinalysis results are defined as positive when WBCs are more than 3 per high-power field or RBCs are more than 5 per high-power field, the overtreatment rate is 44% (146/331) and the undertreatment rate is 11% (36/331). Matched pairs of test characteristics were identified when the analysis was repeated using more than 10,000 colonies per mL as a positive culture. In this patient population, similar overtreatment and undertreatment rates were identified for various test cutoff points for urine dipstick tests and urinalysis. Although a urine dipstick may be equivalent to a urinalysis for the diagnosis of urinary tract infection, the limitations in the diagnostic accuracy of both tests should be incorporated into medical decisionmaking."}, {"id": "pubmed23n1048_11618", "title": "Antimicrobial susceptibility patterns of uropathogens isolated from pregnant women in KwaZulu-Natal Province: 2011 - 2016.", "score": 0.010443583118001722, "content": "Urinary tract infection (UTI) is one of the most common infections during pregnancy, which can lead to significant maternal and perinatal morbidity and mortality if left untreated. Challenges when treating UTIs in pregnancy include fetal protection and resistance development of uropathogens. Currently, the Essential Medicines List recommends nitrofurantoin to treat cystitis and ceftriaxone to treat pyelonephritis in pregnant women. To determine common pathogens causing UTI in pregnancy and their antibiotic susceptibility patterns. A retrospective analysis was performed of laboratory data for positive urine specimens from obstetric departments of 6 KwaZulu- Natal Province hospitals during 2011 - 2016. Identification and susceptibility testing were performed using the VITEK 2 system. Results were interpreted according to the breakpoints of the Clinical and Laboratory Standards Institute, USA. From 5 971 positive urine specimens, the most common isolate was Escherichia coli (n=3 236; 54.2%), followed by Klebsiella pneumoniae (n=770; 12.9%). Group B streptococcus (GBS) (n=239; 4.0%) and Enterococcus faecalis (n=251; 4.2%) were the most common Gram-positive pathogens. E. coli displayed significant resistance to trimethoprim-sulfamethoxazole (65.1%), cephalothin (38.3%), cefuroxime (27.3%), ciprofloxacin (16.9%) and amoxicillin-clavulanic acid (17.1%). Resistance to ceftriaxone and nitrofurantoin remained low ‒ 9.1% and 7.7%, respectively. Among Gram-positive pathogens, GBS displayed 100% penicillin susceptibility and E. faecalis showed 92.9% susceptibility to ampicillin. E. coli is unsurprisingly the most common cause of UTI in pregnancy in KwaZulu-Natal. Susceptibility to ceftriaxone and nitrofurantoin remains good. Among Gram positives, GBS is prevalent and susceptible to penicillin, while E. faecalis is susceptible to ampicillin. As antimicrobial resistance evolves, routine surveillance is necessary to modify recommended empirical antibiotic use."}, {"id": "pubmed23n1007_14583", "title": "A successful management after preterm delivery in a patient with severe sepsis during third-trimester pregnancy.", "score": 0.009900990099009901, "content": "A 33-year-old woman visited the emergency department presenting with fever and dyspnea. She was pregnant with gestational age of 31 weeks and 6 days. She had dysuria for 7 days, and fever and dyspnea for 1 day. The vital signs were as follows: blood pressure 110/70 mmHg, heart rate 118 beats/minute, respiratory rate 28/minute, body temperature 38.7℃, and oxygen saturation by pulse oximetry 84% during inhalation of 5 liters of oxygen by nasal prongs. Crackles were heard over both lung fields. There were no signs of uterine contractions. Chest X-ray and chest computed tomography scan showed multiple consolidations and air bronchograms in both lungs. According to urinalysis, there was pyuria and microscopic hematuria. She was diagnosed with community-acquired pneumonia and urinary tract infection (UTI) that progressed to severe sepsis and acute respiratory failure. We found extended-spectrum beta-lactamase producing 5 WBCs/hpf) and bacteriuria (1 organism/hpf = 106 organisms/mL) are suggestive. Urine culture: The gold standard is > 105 CFU/mL."}]}}}} {"correct_option": 2, "explanations": {"1": {"exist": true, "char_ranges": [[60, 268]], "word_ranges": [[9, 41]], "text": "Regarding weight, metformin to a lesser extent, SGLT-2 inhibitors and, above all, GLP-1 agonists have been associated with a significant decrease in weight in patients with DM-2 (answers 1,3 and 4 incorrect)."}, "2": {"exist": true, "char_ranges": [[269, 450]], "word_ranges": [[41, 70]], "text": "On the other hand, pioglitazone, as reflected in its data sheet, can produce dose-dependent weight gain, mainly due to fat accumulation and added, in some cases, to water retention."}, "3": {"exist": true, "char_ranges": [[60, 268]], "word_ranges": [[9, 41]], "text": "Regarding weight, metformin to a lesser extent, SGLT-2 inhibitors and, above all, GLP-1 agonists have been associated with a significant decrease in weight in patients with DM-2 (answers 1,3 and 4 incorrect)."}, "4": {"exist": true, "char_ranges": [[60, 268]], "word_ranges": [[9, 41]], "text": "Regarding weight, metformin to a lesser extent, SGLT-2 inhibitors and, above all, GLP-1 agonists have been associated with a significant decrease in weight in patients with DM-2 (answers 1,3 and 4 incorrect)."}, "5": {"exist": false, "char_ranges": [], "word_ranges": [], "text": ""}}, "full_answer": "A somewhat more complicated question than the previous one. Regarding weight, metformin to a lesser extent, SGLT-2 inhibitors and, above all, GLP-1 agonists have been associated with a significant decrease in weight in patients with DM-2 (answers 1,3 and 4 incorrect). On the other hand, pioglitazone, as reflected in its data sheet, can produce dose-dependent weight gain, mainly due to fat accumulation and added, in some cases, to water retention.", "full_answer_no_ref": "A somewhat more complicated question than the previous one. Regarding weight, metformin to a lesser extent, SGLT-2 inhibitors and, above all, GLP-1 agonists have been associated with a significant decrease in weight in patients with DM-2 ([HIDDEN]). On the other hand, pioglitazone, as reflected in its data sheet, can produce dose-dependent weight gain, mainly due to fat accumulation and added, in some cases, to water retention.", "full_question": "A 66-year-old woman diagnosed with type 2 diabetes mellitus since three months ago. She has a BMI of 31 kg/m2 and presents poor glycemic control despite a program of non-pharmacological measures (healthy diet, exercise). Which of the following hypoglycemic drugs is associated with weight gain and should we avoid in this patient?:", "id": 522, "lang": "en", "options": {"1": "Metformin (biguanide).", "2": "Pioglitazone (thiazolidinedione).", "3": "Canagliflozin (sodium-glucose cotransporter 2 inhibitor- iSGLT2).", "4": "Liraglutide (GLP-1 receptor agonist).", "5": NaN}, "question_id_specific": 165, "type": "ENDOCRINOLOGY", "year": 2021, "rag": {"clinical_case_options": {"MedCorp": {"RRF-2": [{"id": "pubmed23n1011_13119", "title": "Role of Metformin, Sodium-Glucose Cotransporter-2 (SGLT2) Inhibitors, Glucagon-Like Peptide-1 (GLP-1) Receptor Agonists, and Orlistat based Multidrug Therapy in Glycemic Control, Weight Loss, and Euglycemia in Diabesity: A Real-World Experience.", "score": 0.015575757575757576, "content": "This study evaluated the real-world weight loss and glycemic outcomes of multidrug therapy (MDT) according to various combinations of metformin, sodium-glucose cotransporter -2 inhibitor (SGLT2i), glucagon-like peptide-1 receptor analogs (GLP1a), and orlistat in diabesity. Data retrospectively captured from medical records of 2 different centers in New Delhi for patients >35 years-age having prediabetes/diabetes and on at least any one of the 4 above medications with >6-months follow-up was analyzed. In total, 5,336 patient records were screened; 2,442 with prediabetes/diabetes were considered; 1,509 patients who fulfilled all criteria were analyzed. Use of metformin, SGLT2i, sulfonylureas, DPP4i, pioglitazone, orlistat, and GLP1a was 85.35%, 74.95%, 68.32%, 60%, 39.16%, 9.08%, and 4.17%, respectively. However, 365, 970, and 104 patients were on one of 4 concerned medications (Group-1; 24.18%), dual MDT (Group-2; 64.28%), and triple/quadruple MDT (Group-3; 6.89%). Metformin with SGLT2i was most commonly used dual MDT (94.12%). Analysis according to weight-loss quartiles from 558 patients showed 6.9 kg weight-loss in the highest quartile. People losing maximum weight were significantly younger; had higher use of metformin, SGLT2i, GLP1, orlistat, and lower pioglitazone use; greatest HbA1c reduction (-1.3 vs. -0.3; quartile-1 vs. quartile -4; 50 percent even after five losses (Table 18-4). TABLE 18-4. Predicted Success Rate of Subsequent Pregnancy According to Age and Number of Previous Miscarriages"}, {"id": "pubmed23n0960_18312", "title": "Early Accreta and Uterine Rupture in the Second Trimester.", "score": 0.010648417269988903, "content": "The differential diagnosis of third trimester bleeding can range from placenta abruptia to placenta previa to uterine rupture and the placenta accreta spectrum (PAS). However, patients with risk factors such as multiple cesarean sections (c-sections), advanced maternal age (AMA), grand multiparity, and single-layer uterine closure are at greater risk of developing these complications earlier than we would traditionally expect. This case recounts a 38-year-old gravida 6 preterm 3 term 1 abortus 1 live 4 (G6P3114) at 23 weeks and five days gestational age (GA) with a past medical history of preterm pregnancy, pre-eclampsia, chronic abruptia, three previous c-sections, and low-lying placenta who presented to the emergency department (ED) with vaginal bleeding. Initial workup revealed placenta accreta and possible percreta. The patient was placed on intramuscular (IM) corticosteroids in anticipation of preterm delivery. As soon as the patient was stable, she was discharged home. She presented to a different hospital the next day with the same complaints. Imaging was consistent with accreta and her presentation with abruption. During the hospital stay, the patient went into threatened preterm labor (PTL). At first, we suspected preterm premature rupture of membranes (PPROM) due to apparent pooling of amniotic fluid in the vaginal canal. Upon further work up, the diagnosis was consistent with chronic abruption oligohydramnios sequence (CAOS). Before this could be investigated, her hospital course was complicated by acute abruption and Category III/nonreassuring fetal heart rate (FHR) tracing. The patient underwent an emergency c-section at 26 weeks GA as well as a planned supracervical hysterectomy for desired permanent sterilization. During the operation, the patient suffered a postpartum hemorrhage (PPH) of 4500 mL. She was later discharged home on postoperative day (POD) eight."}, {"id": "pubmed23n0511_555", "title": "Standard term of pregnancy.", "score": 0.009900990099009901, "content": "What is it? This question arose early this year during a discussion with my doula partner, who lived in Great Britain for 20 years, and one of the midwives I work with, who attends only homebirths, which is very rare in France. The mom whose case we were discussing was late going into labor but not postdated according to the official pregnancy term here in France (37-42 WA--\"weeks of amenorrhea\"). The midwife expressed her discomfort with waiting. My doula partner and I felt differently, but we knew we were influenced by American and British midwives' practices. I had been shocked the previous December by the position of the chief of the maternity department in a private hospital (a small unit with no residents, in which nurse-midwives attend \"normal\" births and obstetricians are called in case of complications only). This OB explained to my client and me that if she didn't go into labor naturally, she would be called at 41 + 1 for a vaginal exam to check her cervix and would be induced at 41 + 2. Waiting until 42 weeks requires daily checks, for which he has neither the room nor personnel. He clearly stated that it was a matter of management of time and finances. A month later one of our clients reported the story of her brother and sister-in-law's planned homebirth in London. Their doctors had put a lot of pressure on the mother during her pregnancy with gestational diabetes regarding her length of term. They started to talk about induction. The parents didn't feel comfortable with this, and at that point our client had asked me to refer them to someone who could help them there. We referred them to the National Childbirth Trust and to the sweetest doula we know there. This doula (Liliana Lammers) and her famous doctor partner (Dr. Michel Odent) were a good match. The doctor advised waiting, on the condition that the health of the baby and the amount of fluid be checked daily at the local hospital. The mother had already been waiting several weeks past what was supposed to be her term. Finally, she went naturally into labor at home. The doctor and the doula came and, after some hours of observation, decided it would be wiser for the mother to deliver in the hospital. The doctor and doula were not comfortable with the prolonged prelabor, when, at nearly 44 weeks, the health of the baby and the amount of fluid had not been checked for five days. The mother finally had a vaginal birth without drugs at the hospital. After hearing this story, I suggested it would be interesting to collect the official lengths of term and the different routines in other countries as a learning tool and in order to give us something other than French protocol on which to base our practice. So I sent the question to every midwife for whom I had an e-mail address from the last Midwifery Today conference in Paris. Beyond this motivation was my own curiosity regarding the relationship between the official term in each country and its uses and routines. The most significant (because the most unique) answer, in my opinion, is from The Netherlands, where physiology is a priority."}, {"id": "pubmed23n0536_12712", "title": "[What should you tell a patient with a history of cervical incompetence in the first trimester?].", "score": 0.009900990099009901, "content": "Data is now more than ever available to inform couples at risk of second trimester miscarriage or preterm delivery. We are able to give customized information according to the obstetrical history and to the evolution of the cervix during the second trimester although the level of scientific evidence is limited or poor. Elective cerclage can be proposed to patients with a history of at least 3 second trimester miscarriages or preterm deliveries. There is no clear consensus on which patients could benefit from therapeutic cerclage. Indications would have to be motivated by a short cervix on ultrasound measurements and the cerclage performed before 24 weeks of gestation."}, {"id": "pubmed23n0689_15606", "title": "First case of vaginal radical trachelectomy in a pregnant Japanese woman.", "score": 0.00980392156862745, "content": "A diagnosis of cervical cancer during pregnancy poses difficult management and ethical problems. Survival of the patient is the foremost concern, but fetal viability and well-being must also be addressed. Radical trachelectomy (RT) has recently begun to be performed as a possible treatment modality for early stage invasive uterine cervical cancer in pregnant patients who would like to continue their pregnancy. A 32-year-old Japanese woman visited a local hospital for prenatal care, and was diagnosed with a FIGO I B1 adenocarcinoma of the uterine cervix. She had a strong desire to avoid pregnancy termination, so she was admitted to our hospital for fertility-preserving surgery. After extensive counseling, vaginal radical trachelectomy with abdominal pelvic lymphadenectomy was performed in the 16th gestational week. The excised uterine cervix and lymph nodes were pathologically negative for cancer. To maintain her pregnancy, daily vaginal disinfection with povidone iodine, bed rest, and administration of ritodrine and an ulinastatin vaginal suppository were continued until the delivery. At 34 weeks' gestation, an emergency cesarean section was performed because of sudden premature rupture of the membranes. A baby girl was born weighing 2112 g, with Apgar score of 8/9. The mother remains without evidence of recurrence at the time of this report. This is the first case of successful pregnancy and delivery in Japan after vaginal RT."}, {"id": "Obstentrics_Williams_5684", "title": "Obstentrics_Williams", "score": 0.00980392156862745, "content": "Prevention of preterm birth remains an elusive goal. Still, may be achievable. Of options, cerclage placement may be used to prevent pre term birth in at least three circumstances. First, the procedure may beneit women who have a history of recurrent midtrimester losses and who are diagnosed with cervical insuiciency. A second instance is the woman identiied during sonographic examination to have a short cervix. The third indication is a \"rescue\" cerclage, done emergently when cervical incompetence is recognized in women with threatened preterm labor."}, {"id": "pubmed23n1097_18310", "title": "Successful cervical cerclage in a dichorionic diamniotic twin pregnancy: A case report.", "score": 0.009708737864077669, "content": "A 26-year-old primigravid woman presented with a dichorionic diamniotic twin pregnancy after 7 years of infertility. No formal ultrasound was performed until a morphology check at 19 weeks and 4 days of gestation, at which time a shortened cervix was identified. The patient was already on vaginal progesterone pessaries from conception, as per her infertility specialist, and was advised to change to a rectal route of administration. At 20 weeks and 5 days, progesterone pessaries were increased to twice daily. A repeat scan at 21 weeks and 4 days showed a funnelled cervix 29 mm in length, a closed portion of 4-6 mm and bulging membranes. A speculum examination at this time showed a shortened cervix, 5 mm open, with visible membranes. A cervical cerclage was placed at 21 weeks and 5 days. The patient was given oral antibiotics for 1 week and was continued on progesterone pessaries. The patient was managed through the twins clinic and had serial ultrasound scans throughout the pregnancy. She went on to develop gestational diabetes and pre-eclampsia. She had a caesarean section at 33 weeks and 4 days due to pre-eclampsia, with abnormal doppler scans. Cervical cerclage was removed at the time of the caesarean section. Both twins were admitted to the nursery for prematurity and progressed well. This case report illustrates how a cervical cerclage can be utilised successfully in a primigravid dichorionic diamniotic twin pregnancy."}, {"id": "pubmed23n0800_14851", "title": "Breathlessness with pulmonary metastases: a multimodal approach.", "score": 0.009615384615384616, "content": "Case Study  Sarah is a 58-year-old breast cancer survivor, social worker, and health-care administrator at a long-term care facility. She lives with her husband and enjoys gardening and reading. She has two grown children and three grandchildren who live approximately 180 miles away. SECOND CANCER DIAGNOSIS  One morning while showering, Sarah detected a painless quarter-sized lump on her inner thigh. While she thought it was unusual, she felt it would probably go away. One month later, she felt the lump again; she thought that it had grown, so she scheduled a visit with her primary care physician. A CT scan revealed a 6.2-cm soft-tissue mass in the left groin. She was referred to an oncologic surgeon and underwent an excision of the groin mass. Pathology revealed a grade 3 malignant melanoma. She was later tested and found to have BRAF-negative status. Following her recovery from surgery, Sarah was further evaluated with an MRI scan of the brain, which was negative, and a PET scan, which revealed two nodules in the left lung. As Sarah had attended a cancer support group during her breast cancer treatment in the past, she decided to go back to the group when she learned of her melanoma diagnosis. While the treatment options for her lung lesions included interleukin-2, ipilimumab (Yervoy), temozolomide, dacarbazine, a clinical trial, or radiosurgery, Sarah's oncologist felt that ipilimumab or radiosurgery would be the best course of action. She shared with her support group that she was ambivalent about this decision, as she had experienced profound fatigue and nausea with chemotherapy during her past treatment for breast cancer. She eventually opted to undergo stereotactic radiosurgery. DISEASE RECURRENCE  After the radiosurgery, Sarah was followed every 2 months. She complained of shortness of breath about 2 weeks prior to each follow-up visit. Each time her chest x-ray was normal, and she eventually believed that her breathlessness was anxiety-related. Unfortunately, Sarah's 1-year follow-up exam revealed a 2 cm × 3 cm mass in her left lung, for which she had a surgical wedge resection. Her complaints of shortness of breath increased following the surgery and occurred most often with anxiety, heat, and gardening activities, especially when she needed to bend over. Sarah also complained of a burning \"pins and needles\" sensation at the surgical chest wall site that was bothersome and would wake her up at night. Sarah met with the nurse practitioner in the symptom management clinic to discuss her concerns. Upon physical examination, observable signs of breathlessness were lacking, and oxygen saturation remained stable at 94%, but Sarah rated her breathlessness as 7 on the 0 to 10 Borg scale. The nurse practitioner prescribed duloxetine to help manage the surgical site neuropathic pain and to assist with anxiety, which in turn could possibly improve Sarah's breathlessness. Several nonpharmacologic modalities for breathlessness were also recommended: using a fan directed toward her face, working in the garden in the early morning when the weather is cooler, gardening in containers that are at eye level to avoid the need to bend down, and performing relaxation exercises with pursed lip breathing to relieve anxiety-provoked breathlessness. One month later, Sarah reported relief of her anxiety; she stated that the fan directed toward her face helped most when she started to feel \"air hungry.\" She rated her breathlessness at 4/10 on the Borg scale. SECOND RECURRENCE: MULTIPLE PULMONARY NODULES  Sarah's chest x-rays remained clear for 6 months, but she developed a chronic cough shortly before the 9-month exam. An x-ray revealed several bilateral lung lesions and growth in the area of the previously resected lung nodule. Systemic therapy was recommended, and she underwent two cycles of ipilimumab. Sarah's cough and breathlessness worsened, she developed colitis, and she decided to stop therapy after the third cycle. In addition, her coughing spells triggered bronchospasms that resulted in severe anxiety, panic attacks, and air hunger. She rated her breathlessness at 10/10 on the Borg scale during these episodes. She found communication difficult due to the cough and began to isolate herself. She continued to attend the support group weekly but had difficulty participating in conversation due to her cough. Sarah was seen in the symptom management clinic every 2 weeks or more often as needed. No acute distress was present at the beginning of each visit, but when Sarah began to talk about her symptoms and fear of dying, her shortness of breath and anxiety increased. The symptom management nurse practitioner treated the suspected underlying cause of the breathlessness and prescribed oral lorazepam (0.5 to 1 mg every 6 hours) for anxiety and codeine cough syrup for the cough. Opioids were initiated for chest wall pain and to control the breathlessness. Controlled-release oxycodone was started at 10 mg every 12 hours with a breakthrough pain (BTP) dose of 5 mg every 2 hours as needed for breathlessness or pain. Sarah noted improvement in her symptoms and reported a Borg scale rating of 5/10. Oxygen therapy was attempted, but subjective improvement in Sarah's breathlessness was lacking. END OF LIFE  Sarah's disease progressed to the liver, and she began experiencing more notable signs of breathlessness: nasal flaring, tachycardia, and restlessness. Opioid doses were titrated over the course of 3 months to oxycodone (40 mg every 12 hours) with a BTP dose of 10 to 15 mg every 2 hours as needed, but her breathlessness caused significant distress, which she rated 8/10. The oxycodone was rotated to IV morphine continuous infusion with patient-controlled analgesia (PCA) that was delivered through her implantable port. This combination allowed Sarah to depress the PCA as needed and achieve immediate control of her dyspneic episodes. Oral lorazepam was also continued as needed. Sarah's daughter moved home to take care of her mother, and hospice became involved for end-of-life care. As Sarah became less responsive, nurses maintained doses of morphine for control of pain and breathlessness and used a respiratory distress observation scale to assess for breathlessness since Sarah could no longer self-report. A bolus PCA dose of morphine was administered by Sarah's daughter if her mother appeared to be in distress. Sarah died peacefully in her home without signs of distress. "}, {"id": "pubmed23n0568_8656", "title": "[Cervical cerclage and evidence-based medicine: if, how and when].", "score": 0.009615384615384616, "content": "Cervical cerclage has always been the main treatment option in cases of so-called cervical insufficiency, a condition that is notoriously associated with a high risk of second trimester abortion and/or preterm delivery. We can distinguish between a prophylactic cerclage, to be performed electively, usually at 13-16 weeks gestation, only when the woman has a history extremely suggestive for cervical incompetence (3 or more mid-trimester abortions or preterm deliveries) and a therapeutic cerclage. This last cerclage is recommended either for women who have ultrasonographic changes consistent with a short cervix or the presence of funneling after the 16-20 weeks gestation (urgent cerclage) and for women who present the asymptomatic dilation of the uterine cervix of at least 2 cm and/or a prolapse of the amniochorial membranes (emergent cerclage). So far there is still a lack of controlled and randomized trials that can unquestionably demonstrate the advantages of the cervical cerclage in comparison with a ''wait and see'' aptitude. The cerclage can be performed either transvaginally, usually according to the McDonald technique, or transabdominally. This last approach is recommended when a transvaginal cerclage has to be avoided because of technical difficulties depending on the conditions of the cervix or when the pregnant woman has a history of one or more failed transvaginal cerclages. Interesting perspectives are currently offered by the laparoscopic cerclage, a method that has been effective and unexpectedly safe till now."}, {"id": "pubmed23n1095_2800", "title": "Chorioamnionitis caused by Serratia marcescens in a healthcare worker: A case report.", "score": 0.009523809523809525, "content": "Healthcare workers (HCWs) are at an increased risk for exposure to infections. Peptoniphilus indolicus infection in a pregnant woman: a case report.", "score": 0.008849557522123894, "content": "Microsporum audouinii: a severe form in an immunocompetent girl.", "score": 0.009708737864077669, "content": "A 9-year-old girl presented a large inflammatory cup-shaped scalp lesion with alopecia surrounded by pustules, dander, and suppuration associated with an occipital inflammatory lymphadenopathy for 1 month. Wood's light exam was positive as well as KOH mount showing ectothrix type hair involvement. Hair and pus culture on Sabouraud dextrose agar (SDA) added with chloramphenicol and supplemented with cycloheximide isolated a dermatophyte species identified as 50 years of age, immunocompromised, or with preexisting malignancy. If SAH is suspected, obtain a head CT without contrast. If CT is , LP is mandatory. Obtain a CBC."}, {"id": "pubmed23n0995_14786", "title": "Ultrasound-Assisted Diagnosis of Optic Neuritis in the Emergency Department: A Case Report.", "score": 0.009708737864077669, "content": "Optic neuritis is a common cause of subacute unilateral vision loss, occurring in 1-5 per 100,000 persons per year. It is more common in Caucasians, women, and those from countries with northern latitudes. Those aged 20-49 years are at greatest risk. The condition arises due to inflammation of the optic nerve. Inflammation may occur due to systemic inflammatory disorders, most commonly multiple sclerosis. A 21-year-old African-American male presented to our emergency department with a complaint of painful unilateral vision loss. On examination he was found to have a relative afferent pupillary defect and red desaturation. A bedside ultrasound suggested pseudopapilledema suggestive of optic neuritis. He was admitted to Neurology for confirmation of and treatment for optic neuritis. Magnetic resonance imaging confirmed optic neuritis. The patient was treated with i.v. steroids and discharged after improvement in visual function. WHY SHOULD AN EMERGENCY PHYSICIAN BE AWARE OF THIS?: Optic neuritis is a clinical diagnosis. The subtle historical components and examination findings make it a diagnostic challenge for the busy emergency physician. Early diagnosis may improve visual outcomes. Discovery of pseudopapilledema on bedside ultrasound may be seen in optic neuritis, and is another finding that emergency physicians may assess for in patient presenting with unilateral vision loss."}, {"id": "pubmed23n0321_12634", "title": "Masticatory muscle pain: an important indicator of giant cell arteritis.", "score": 0.009708737864077669, "content": "Giant cell arteritis (GCA) is a polysymptomatic disease which constitutes an ophthalmic emergency because early recognition and management can prevent blindness. There is conflicting information in the literature on the validity, sensitivity, and specificity of various systemic symptoms and signs of GCA. This paper presents a review of our prospective studies on the subject, and our findings are particularly relevant to dentists. We investigated 363 patients in a prospective study. Positive temporal artery biopsy was seen in 106 patients and negative in 257 referred for diagnosis of GCA. Systemic symptoms and signs of GCA and erythrocyte sedimentation rate (Westergren-ESR) and C-reactive protein (CRP) levels were compared in these two groups of patients. The odds of having a positive temporal artery biopsy (i.e., GCA) were 9.1 times greater with jaw claudication (pain in masticatory muscles on eating), 3.4 times with neck pain, 3.2 times with CRP > 2.45 mg/dL, 2.0 times with ESR 47.107 mm/hr, 2.7 times with ESR > 107 mm/hr, and 2.0 times when the patients were aged > or = 75 years. Other signs and symptoms did not show a significant association with a positive biopsy. Our study showed that \"normal\" ESR values do not rule out GCA but that CRP is a more useful test than ESR. Since jaw claudication is one of the most important symptoms of GCA, dentists should keep this possibility in mind when older patients come complaining of jaw pain while eating."}, {"id": "pubmed23n1145_24843", "title": "Paracentral Acute Middle Maculopathy After COVID-19 Disease: Multimodal Evaluation.", "score": 0.009615384615384616, "content": "To report the case and multimodal imaging findings of a healthy young woman who developed paracentral acute middle maculopathy (PAMM) 9 weeks after COVID-19 disease. Case report. Ultra-widefield fundus photography, macular spectral-domain optical coherence tomography (SD-OCT), fluorescein angiography (FA), and OCT-angiography (OCT-A) were performed. A 36-year-old woman who developed PAMM 9 weeks after SARS-CoV-2 infection. A 36-year-old woman went to the emergency department (ED) with sudden, painless, left eye (LE) vision loss. The only relevant past medical history was COVID-19 disease 9 weeks before. Best corrected visual acuity (BCVA) was 20/200, a LE relative afferent pupillary defect (RAPD) was present and superficial hemorrhages throughout the macular area and peripheral retina were found. Nearly four hours after admission, LE BCVA recovered to 20/20 without RAPD. Five days after presentation in the ED, the patient returned with recurrent LE vision loss, with spontaneous recovery within 12 hours. Macular SD-OCT revealed hyperreflectivity of the inner plexiform and inner nuclear layers and the diagnosis of PAMM was established. The patient started oral acetylsalicylic acid and oral prednisolone. The patient did not report any new episodes of vision loss and there was a progressive resolution of abnormal fundus findings. SARS-CoV-2 infection increases the risk of vascular thrombotic events with possible involvement of the retinal circulation, and PAMM may present as a possible complication. Ophthalmologists should be able to recognize it promptly through multimodal imaging findings."}, {"id": "pubmed23n0596_4516", "title": "Anterior ischemic optic neuropathy due to giant cell arteritis with normal inflammatory markers.", "score": 0.009615384615384616, "content": "In anterior ischemic optic neuropathy (AION), it is important not to miss the diagnosis of giant cell arteritis (GCA) because this requires immediate steroid treatment to prevent involvement of the second eye and possible blindness. A missed diagnosis also might lead to fatal systemic complications. Observational case report. A 79-year-old woman noticed decreased visual and visual field loss in the right eye. At presentation, right visual acuity was 10/20 (ETDRS chart 2000). There was a right relative afferent pupillary defect of 0.6 log units. Asked for symptoms of GCA she complained about temporal and occipital headache, jaw claudication combined with malaise, and myalgia of the upper limbs. Laboratory tests showed normal inflammatory markers. Repeated tests confirmed ESR and CRP to be within the normal range. GCA being suspected, ultrasound of the superficial temporal arteries and temporal artery biopsy were performed unilaterally on the right side. Histology showed a chronic inflammatory cell infiltrate consistent with active GCA. The patient was treated with high-dose corticosteroids (250 mg methylprednisolone, three times/day, initially) and symptoms rapidly resolved, but visual loss remained unchanged. The case presented here proves that GCA with typical related visual loss (AION) is possible even when both ESR and CRP are in the normal range. Therefore, in the presence of typical symptoms, the clinician must not rely solely on laboratory testing, but start steroid therapy immediately and order a temporal artery biopsy."}, {"id": "wiki20220301en015_138436", "title": "Sciatica", "score": 0.009523809523809525, "content": "Cancer should be suspected if there is previous history of it, unexplained weight loss, or unremitting pain. Spinal epidural abscess is more common among those with diabetes mellitus or immunocompromised or who had spinal surgery, injection or catheter; it typically causes fever, leukocytosis and increased erythrocyte sedimentation rate. If cancer or spinal epidural abscess are suspected, urgent magnetic resonance imaging is recommended for confirmation. Proximal diabetic neuropathy typically affects middle aged and older people with well-controlled type-2 diabetes mellitus; onset is sudden causing pain usually in multiple dermatomes quickly followed by weakness. Diagnosis typically involves electromyography and lumbar puncture. Shingles is more common among the elderly and immunocompromised; usually (but not always) pain is followed by appearance of a rash with small blisters along a single dermatome. Acute Lyme radiculopathy may follow a history of outdoor activities during warmer"}, {"id": "pubmed23n0525_4392", "title": "[Blindness in both eyes due to late diagnosis of giant cell arteritis].", "score": 0.009523809523809525, "content": "Giant cell arteritis (GCA) is often diagnosed very late, variable \"facets\" of the disease exist render the diagnosis more difficult. Follow-up observations of five very old patients are reported in whom diagnosis was made too late, resulting in blindness of both eyes. Five patients (age 76-84 years, 4 women, one man) with GCA became blind in both eyes because diagnosis had been delayed (two patients) or onset of therapy was too late (three patients). In two patients who also had arterial hypertension, the symptom \"headache\" had been misleading. Symptoms of accompanying general diseases masked the real diagnosis, particularly in the second patient who had renal insufficiency, coronary artery disease, and unilateral obstruction of the internal carotid artery. Symptoms that failed to lead to the correct diagnosis were: muscle or chewing pain (three patients), circumscribed numbness around the mouth (second patient), and persistent headache despite normalization of blood pressure. Normal findings from cranial CTAs (two patients) led to the wrong reassurance of the patient. Swelling of the optic disk (two patients) was misdiagnosed by ophthalmologists, as was a retinal branch arterial occlusion (first patient). Three patients, afraid of possible side effects caused by glucocorticoids, took ineffective alternative medications. Poor vigilance led to blindness of the fifth patient with long-standing polymyalgia rheumatica. Targeted examinations at the onset of symptoms are necessary. GCA-symptoms were mis-constructed by additional diseases that disguised the correct diagnosis. The danger of bilateral blindness is particularly great in patients of great age."}, {"id": "pubmed23n1037_7823", "title": "A tearfully painful darkness.", "score": 0.009433962264150943, "content": "A 70-year-old woman presented with new onset of left eye and facial pain. Ophthalmic and neurological examinations, magnetic resonance imaging brain, erythrocyte sedimentation rate, and C-reactive protein were unrevealing. A few days later, she developed vision loss in her left eye. Examination revealed decreased visual acuity with a relative afferent pupillary defect in the left eye and a diffuse mild swelling of the left optic nerve head. Repeat magnetic resonance imaging showed T2 hyperintensity and enhancement of the intraorbital optic nerve and surrounding tissues with no other intracranial abnormalities. Serum studies showed elevated myelin oligodendrocyte glycoprotein IgG titer. She was treated with IV methylprednisolone 1000 mg daily for 3 days and was discharged on prolonged prednisone taper with return of vision to baseline."}, {"id": "pubmed23n0314_1466", "title": "[A 62-year-old man with an acute onset of consciousness disturbances].", "score": 0.009433962264150943, "content": "We report a 62-year-old man who developed coma and died in a fulminant course. The patient was well until May 1, 1996 when he noted chillness, tenderness in his shoulders, and he went to bed without having his lunch and dinner. In the early morning of May 2, his families found him unresponsive and snoring; he was brought into the ER of our hospital. He had histories of hypertension, gout, and hyperlipidemia since 42 years of the age. On admission, his blood pressure was 120/70, heart rate 102 and regular, and body temperature 36.3 degrees C. His respiration was regular and he was not cyanotic. Low pitch rhonchi was heard in his right lower lung field. Otherwise general physical examination was unremarkable. Neurologic examination revealed that he was somnolent and he was only able to respond to simple questions such as opening eyes and grasping the examiner's hand, but he was unable to respond verbally. The optic discs were flat; the right pupil was slightly larger than the left, but both reacted to light. He showed ptosis on the left side, conjugate deviation of eyes to the left, and right facial paresis. The oculocephalic response and the corneal reflex were present. His right extremities were paralyzed and did not respond to pain Deep tendon reflexes were exaggerated on the right side and the plantar response was extensor on the right. No meningeal signs were present. Laboratory examination revealed the following abnormalities; WBC 18,400/ml, GOT 131 IU/l GPT 50 IU/l, CK616 IU/l, BUN 30 mg/dl, Cr 2.1 mg/ dl, glucose 339 mg/dl, and CRP 27.4 mg/dl. ECG showed sinus tachycardia and ST elevation in II, III and a VF leads and abnormal q waves in I, V5, and V6 leads. Chest X-ray revealed cardiac enlargement but the lung fields were clear. Cranial CT scan revealed low density areas in the left middle cerebral and left posterior cerebral artery territories. The patient was treated with intravenous glycerol infusion and other supportive measures. At 2: 10 AM on May 3, he developed sudden hypotension and cardiopulmonary arrest. He was pronounced dead at 3:45 AM. The patient was discussed in a neurological CPC, and the chief discussant arrived at the conclusion that the patient had acute myocardial infarction involving the inferior and the true posterior walls and left internal carotid embolism from a mural thrombus. Post mortem examination revealed occlusion of the circumflex branch of the left coronary artery due to atherom plaque rupture and myocardial infarction involving the posterior and the lateral wall with a rupture in the postero-lateral wall. Marked atheromatous changes were seen in the left internal carotid, the middle cerebral and the basilar arteries; the left internal carotid and the middle cerebral arteries were almost occluded by thrombi and blood coagulate. The territories of the left middle cerebral and the occipital arteries were infarcted; but the left thalamic area was spared. The neuropathologist concluded that the infarction was thrombotic origin not an embolic one as the atherosclerotic changes were severe. Cardiac rupture appeared to be the cause of terminal sudden hypotension and cardiopulmonary arrest. It appears likely that a vegetation which had been attached to the aortic valve induced thromboembolic occlusion of the left internal carotid artery which had already been markedly sclerotic by atherosclerosis. It is also possible that the vegetations in the aortic valve came from mural thrombi at the site of acute myocardial infarction, as no bacteria were found in those vegetations."}, {"id": "pubmed23n1076_12839", "title": "Childhood-Onset Leber Hereditary Optic Neuropathy: Particular Features.", "score": 0.009345794392523364, "content": "Leber hereditary optic neuropathy (LHON) is an optic neuropathy of mitochondrial inheritance. Childhood-onset disease is relatively rare and there are limited data on this important patient subgroup. We present 3 particular presentations of LHON. Patient 1 was an 8-year-old boy admitted to the emergency department reporting a progressive bilateral visual loss and intermittent headaches. Neuro-ophthalmological examination revealed a bilateral pseudopapilledema. Lumbar puncture identified intracranial hypertension and the brain and orbits magnetic resonance imaging showed T2 hyperintensity in the posterior region of the left optic nerve and the optic chiasm. Patient 2 was a 12-year-old boy admitted to the emergency department reporting painless, progressive central vision loss in the right eye. Fundus examination revealed a hyperemic disc and vascular network papillary and peripapillary vascular microdilations. Three months later, the left eye presented visual loss. Patient 3 was a 6-year-old female child referred to the neuro-ophthalmology specialist due to painless central visual loss in both eyes. Her BCVA was 1/10 and counting fingers in right and left eye, respectively, and fundus examination revealed a pallor optic disc in the temporal sector. The phenotype of childhood-onset disease may present itself distinct from classical adult-onset LHON. The absence of classical clinical features could lead to initial misdiagnosis. There should exist a high index of suspicion in children presenting unexplained subnormal vision in order to avoid potential diagnostic delays."}, {"id": "pubmed23n0244_2773", "title": "[The Tolosa-Hunt syndrome: report of a case with recurrent (9 times) painful ophthalmoplegia (author's transl)].", "score": 0.009345794392523364, "content": "A 48-year-old woman was referred to the First Dept. of Int. Med., Nagasaki Univ. Sch. Med., in August, 1979, with a six-month history of recurrent episodes of right-sided painful ophthalmoplegia and diplopia. An epidode affected the right eye, lasted one to two weeks, and relapsed every month. On examination she had a complete ptosis on the right side and pain on the right eye. All extraocular muscle supplied by the 3rd nerve were paralysed. The pupils were equal in size both sides, reacting to light completely. Visual acuity was normal except myopia. All the other cranial nerves and the remainder of central nervous system was normal. Results of thyroid function tests and of lumbar puncture were normal. The glucose tolerance test showed a mild diabetic pattern. Blood and CSF cultures for bacteria, fungi, and acid-fast bacillus were negative. The skull, brain CT scan, and carotid angiogram were within normal limits. A tentative diagnosis of Tolosa-Hunt syndrome was made after an unproductive search for a cause for this woman's painful ophthalmoplegia and unsuccessful treatment of ophthalmoplegia with antibiotics or diet therapy for mild hyperglycemia. The patient was given prednisolone 30 mg daily orally when she had the 9th attack of painful ophthalmoplegia Pain, ptosis, and diplopia disappeared in 5 days and she did not show any recurrence of symptoms over the next 7 months."}, {"id": "wiki20220301en053_41040", "title": "Posterior ischemic optic neuropathy", "score": 0.009259259259259259, "content": "Defective light perception in one eye causes an asymmetrical pupillary constriction reflex called the afferent pupillary defect (APD). Arteritic PION A-PION most commonly affects Caucasian women, with an average age of 73. At onset vision loss is unilateral, but without treatment it rapidly progresses to involve both eyes. Vision loss is usually severe, ranging from counting fingers to no light perception. Associated symptoms are jaw pain exacerbated by chewing, scalp tenderness, shoulder and hip pain, headache and fatigue. Perioperative PION Vision loss is usually apparent upon waking from general anesthesia. Signs observable to a bystander include long surgery duration and facial swelling. Vision loss is usually bilateral and severe, ranging from counting fingers to no light perception. Cause"}, {"id": "pubmed23n0334_10172", "title": "Lessons to be learned: a case study approach--a case of temporal arteritis.", "score": 0.009259259259259259, "content": "A 71-year-old male presented with a history of sudden partial visual loss in the right eye with an inferior visual field defect over the past 3-4 days. He had no history of headache or of facial pain. Clinical examination confirmed that vision on the right side was reduced to 6/18 and on the left to 6/12. The right eye showed a relative afferent pupillary defect. There was no other abnormality of the anterior segment of either eye. The right retina showed a pale swollen optic disc and a provisional diagnosis of anterior ischaemic optic neuropathy (AION) was made. An urgent erythrocyte sedimentation rate (ESR) was ordered and the patient was asked to return to the eye clinic in one month. However, 16 days later--when it was first recognised that his ESR was elevated to 75 mm in the first hour--the patient was recalled immediately in order to commence systemic steroid treatment; but regrettably, by this time, his right eye had become totally blind. In this case, although the attending doctor made a correct clinical diagnosis on presentation, he failed to act upon the result of the blood test."}, {"id": "wiki20220301en019_114604", "title": "Ankylosing spondylitis", "score": 0.009174311926605505, "content": "These diagnostic criteria include: Inflammatory back pain:Chronic, inflammatory back pain is defined when at least four out of five of the following parameters are present: (1) Age of onset below 40 years old, (2) insidious onset, (3) improvement with exercise, (4) no improvement with rest, and (5) pain at night (with improvement upon getting up) Past history of inflammation in the joints, heels, or tendon-bone attachments Family history for axial spondyloarthritis or other associated rheumatic/autoimmune conditions Positive for the biomarker HLA-B27 Good response to treatment with nonsteroidal anti-inflammatory drugs (NSAIDs) Signs of elevated inflammation (C-reactive protein and erythrocyte sedimentation rate) Manifestation of psoriasis, inflammatory bowel disease, or inflammation of the eye (uveitis) If these criteria still do not give a compelling diagnosis magnetic resonance imaging (MRI) may be useful. MRI can show inflammation of the sacroiliac joint. Imaging"}, {"id": "pubmed23n0479_17655", "title": "Progression of preexisting trigeminalgia to Tolose-Hunt-like syndrome. The importance of neuroimaging for early differential diagnosis.", "score": 0.009174311926605505, "content": "Recurrent unbearable, paroxysmal, unilateral facial pain in the distribution of one or more branches of the trigeminal nerve often provoked by sensory stimuli is typical for idiopathic trigeminal neuralgia. The less frequent localization in the area of ophthalmic branch (5%) is particularly controversial and should be distinguished from pathological lesions in the brainstem and middle and posterior cranial fossa and from diseases of the orbit and eye. This case study presents a 79-year-old woman with typical clinical features of 1st division trigeminalgia without any neurological loss and with normal results of laryngological, ophthalmological, and stomatological examinations as well as neuroimaging CT, and MR /MRA evaluation. Only the evoked potential blink and masseter reflexes demonstrated the pathological values in the early phase of illness. After 1 year of pharmacological treatment no improvement was achieved and the pain became neuropathic and paresis of 3rd, 4th and 6th nerves developed, as observed in Tolose-Hunt syndrome. MRI of the orbit revealed a pathological mass in its apex with a connection to the superior orbital fissure. However, treatment with steroids was completely ineffective. Surgical resection of the tumor (leiomyosarcoma) only partially reversed oculomotor palsy and diminished aching. In differential diagnosis of idiopathic and symptomatic trigeminalgia, early MR and MRA imaging is the most essential and sometimes may be the best single test to evaluate lesions even in distant areas of the nervous system branches."}, {"id": "pubmed23n0945_12479", "title": "Man with a Swollen Eye: Nonspecific Orbital Inflammation in an Adult in the Emergency Department.", "score": 0.00909090909090909, "content": "Nonspecific orbital inflammation (NSOI) is a rare idiopathic ocular pathology characterized by unilateral, painful orbital swelling without identifiable infectious or systemic disorders, which can be complicated by optic nerve compromise. A 50-year-old man presented to the Emergency Department with recurring, progressive painless left eye swelling, decreased visual acuity, and binocular diplopia in the absence of trauma, infection, or known malignancy. His physical examination was notable for left-sided decreased visual acuity, an afferent pupillary defect, severe left eye proptosis and chemosis, and restricted extraocular movements; his dilatated funduscopic examination was notable for ipsilateral retinal folds within the macula, concerning for a disruption between the sclera and the retina. Ocular examination of the right eye was unremarkable. Laboratory data were unrevealing. Gadolinium-enhanced magnetic resonance imaging showed marked thickening of the left extraocular muscles associated with proptosis, dense inflammatory infiltration of the orbital fat, and characteristics consistent with perineuritis. The patient was diagnosed with NSOI with optic neuritis and admitted for systemic steroid therapy; he was discharged on hospital day 2 after receiving high-dose intravenous (i.v.) methylprednisolone with significant improvement. WHY SHOULD AN EMERGENCY PHYSICIAN BE AWARE OF THIS?: NSOI is a rare and idiopathic ocular emergency, with clinical mimicry resembling a broad spectrum of systemic diseases such as malignancy, autoimmune diseases, endocrine disorders, and infection. Initial work-up for new-onset ocular proptosis should include comprehensive laboratory testing and gadolinium-enhanced magnetic resonance imaging. Timely evaluation by an ophthalmologist is crucial to assess for optic nerve involvement. Signs of optic nerve compromise include decreased visual acuity, afferent pupillary defect, or decreased color saturation. Patients with optic nerve compromise require admission for aggressive anti-inflammatory therapy with i.v. steroids in an attempt to reduce risk of long-term visual sequelae. Our case demonstrates a severe presentation of this disorder and exhibits remarkable visual recovery after 48 h of systemic i.v. steroid treatment."}, {"id": "First_Aid_Step2_571", "title": "First_Aid_Step2", "score": 0.00909090909090909, "content": "Also called giant cell arteritis; due to subacute granulomatous infl ammation of the large vessels, including the aorta, external carotid (especially the temporal branch), and vertebral arteries. The most feared manifestation is blindness 2° to occlusion of the central retinal artery (a branch of the internal carotid artery). Risk factors include polymyalgia rheumatica (affects almost half of TA patients), age > 50, and female gender. Presents with new headache (unilateral or bilateral); scalp pain and temporal tenderness; and jaw claudication. Fever, permanent monocular blindness, weight loss, and myalgias/arthralgias (especially of the shoulders and hips) are also seen. ESR > 50 (usually > 100). Ophthalmologic evaluation. Temporal artery biopsy: Look for thrombosis; necrosis of the media; and lymphocytes, plasma cells, and giant cells."}, {"id": "pubmed23n1033_13514", "title": "Leber's hereditary optic neuropathy following unilateral painful optic neuritis: a case report.", "score": 0.009009009009009009, "content": "Leber's hereditary optic neuropathy (LHON) is a maternally inherited mitochondrial disease, characterized by acute or subacute, painless, bilateral visual loss. LHON is often misdiagnosed as optic neuritis at an early stage because of the similarity of their clinical presentation. To date, there has been no reported case of actual optic neuritis and LHON in one patient. A 40-year-old, healthy man was referred to our clinic with acute painful visual loss in the right eye for 2 weeks. In the right eye, visual acuity decreased to 20/40, and the Ishihara colour test score was 8/14 with a relative afferent pupillary defect. Optic disc swelling was found only in the right eye, and magnetic resonance imaging revealed enhancement of the the right optic nerve, consistent with optic neuritis. After receiving 1 g of intravenous methylprednisolone daily for three days, his ocular pain resolved, and visual acuity improved to 20/20 within 2 weeks. Seven months later, the patient developed acute painless visual loss in the right eye. Visual acuity decreased to 20/200 in the right eye. There was no response to the intravenous methylprednisolone therapy at that time. Eight months later, he developed subacute painless visual loss in the left eye. Genetic testing for LHON was performed and revealed the pathologic mtDNA 11778 point mutation. We report a case with painful unilateral optic neuritis preceding the onset of LHON. Even if a typical optic neuritis patient has completely recovered from steroid treatment once in the past, it is advisable to keep in mind the possibility of LHON if acute or subacute loss of vision subsequently or simultaneously occurs in both eyes and does not respond to steroids."}, {"id": "pubmed23n0019_2125", "title": "[Arteriitis temporalis--a major disease developing in advance age (author's transl)].", "score": 0.009009009009009009, "content": "Arteriitis temporalis, a disease that was largely unknown in the early fifties, has gained importance in recent years. In the past ten years 32 patients suffering from arteriitis temporalis were diagnosed and treated both in the eye hospital and in the rheumatic-cardiological hospital of the Berlin-Buch Municipal Hospital. The following clinical findings were essential for the diagnosis: advanced or old age of the patient, massive headache in the temporal and/or occipital regions, myalgia primarily in the shoulders and the neck that responded relatively poorly to treatment, reduced eyesight, loss of weight and decrease in vitality. Extremely high BSR, an increased amount of alpha-2 and an increased level of alkaline phosphatase were most important among the laboratory findings. Corticosteroids have proved to be the medicine of choice for treating this condition."}, {"id": "pubmed23n0536_7105", "title": "Three presentations of monocular vision loss.", "score": 0.008928571428571428, "content": "Carotid artery disease is estimated to affect 30% of persons older than 50. Risk factors include hypertension, cigarette smoking, hyperlipidemia and diabetes mellitus. Symptoms ascribed to carotid artery lesions with stenosis of the artery or plaque formation include monocular vision loss and transient ischemic attacks. Patients can present with transient monocular vision loss as their initial symptom. Three patients from a geriatric clinic in Wilmington, Delaware presented with different complaints of vision loss with similar overall outcomes. Patient A was an 87-year-old woman who presented with dimming of vision on extreme left head turn. Dilated fundus examination found a retinal arterial emboli in the left eye (O.S.). Carotid duplex examination found 50% to 79% left internal carotid stenosis with no hemodynamic stenosis of the right internal carotid. Patient B was a 78-year-old woman who presented with a right superior altitudinal defect and transient vision loss in the right eye. Dilated fundus examination found retinal arterial emboli in the right eye (O.D.). Carotid duplex examination found 50% to 79% carotid stenosis in both the left and right internal carotids. Patient C was an 84-year-old man who complained of a superior altitudinal visual field defect O.D. Dilated fundus examination found a retinal arterial emboli O.D. Carotid duplex results showed calcified atherosclerotic plaques present at the level of the common carotid artery bifurcations bilaterally, with 50% to 70% narrowing of the right internal carotid artery with no significant narrowing of the left internal carotid artery. These 3 presentations show that in patients older than 50 who present with chief complaints of monocular vision loss, a differential diagnosis of carotid artery disease must be considered. Patients who exhibit retinal arterial emboli are at increased risk for stroke and vascular death. Appropriate measures for confirming a diagnosis include duplex ultrasound imaging, magnetic resonance angiography (MRA), and carotid angiography. Surgical techniques such as carotid angioplasty and carotid endarterectomy may be recommended."}, {"id": "pubmed23n0491_3108", "title": "Sometimes (what seems to be) a heart attack is (really) a pain in the neck.", "score": 0.008928571428571428, "content": "A 31-year-old patient complained of severe crushing chest pain that radiated to his left arm and jaw. After admission to the hospital, tests revealed a normal electrocardiogram, normal treadmill, normal coronary arteriogram, and normal cardiac enzymes. However, the patient continued to have pain, which was relieved by sublingual and intravenous nitroglycerine. He was discharged from the hospital with a diagnosis of \"musculoskeletal\" chest pain, taking nonsteroidal anti-inflammatory drugs, muscle relaxants, and narcotics. Two weeks later, the patient returned with worsening symptoms. Cardiac work-up was again negative. Thoracic and cervical spine radiographs were ordered for possible discogenic pain. After abnormalities were found on cervical radiographs, magnetic resonance imaging (MRI) was ordered, and the patient was referred to an orthopedic surgeon. Further work-up revealed a herniated disk at C6-C7, with radicular pain. Surgery on the suspect disk totally relieved the patient's pain."}, {"id": "pubmed23n1030_6059", "title": "Central retinal artery occlusion as initial presentation of Moyamoya disease in a middle-aged woman.", "score": 0.008849557522123894, "content": "To present a case of central retinal artery occlusion as the first symptomatic manifestation of Moyamoya disease in a middle-aged patient. Case report of a 48-year-old female Chinese-American patient who presented with sudden onset painless unilateral vision loss. Fundus photos, optical coherence tomography, fluorescein angiography, magnetic resonance angiography, computed tomography angiography, and catheter cerebral angiogram were performed. The patient's dilated fundus examination showed classic findings of a central retinal artery occlusion. Diagnostic brain imaging demonstrated extensive stenosis of the cerebrovascular network, with almost complete unilateral occlusion of the internal carotid artery along with compensatory collateral vessels. This led to a new diagnosis of Moyamoya disease. The patient was treated with extracranial-intracranial bypass surgery. Arterial abnormalities in patients with Moyamoya disease are uncommon and have previously only been reported in younger patients in their teens and 20s. Young and middle-aged patients presenting with central retinal artery occlusions should undergo complete neurologic workup including stroke evaluation; in this case, revealing Moyamoya disease, a rare yet life-threatening condition, as the underlying etiology."}, {"id": "pubmed23n1025_22054", "title": "Lessons of the month 4: Giant cell arteritis with normal inflammatory markers and isolated oculomotor nerve palsy.", "score": 0.008849557522123894, "content": "Giant cell arteritis (GCA) is an important condition to suspect and treat early, as failure to do so can result in anterior ischaemic optic neuropathy and subsequent permanent visual loss.A 71-year-old woman presented to her local emergency department with a 1-week history of constant, moderate-severe global headache associated with intermittent periorbital pain. Two weeks later she developed sudden horizontal diplopia. Examination demonstrated right oculomotor nerve palsy. Her erythrocyte sedimentation rate (ESR) was 9 mm/hr. Repeat blood tests 1 month later showed an ESR of 67 mm/hr. Temporal artery biopsy was positive.A review from a cohort of 764 patients with suspected GCA who underwent biopsy found the sensitivity of an elevated ESR and c-reactive protein was 84% and 86%, respectively, but the specificity was only 30%. Therefore, inflammatory markers should only act as a guide, and caution should be taken in their interpretation especially with respect to the time of sampling in the disease evolution.Isolated oculomotor nerve palsy in association with GCA is rare. The first case series was described by miller fisher in 1959 who observed two patients presenting with diplopia, ptosis and ocular palsies. In anyone over the age of 50 who develops a new, refractory headache and cranial neuropathy, GCA should be the first consideration."}, {"id": "pubmed23n1143_24145", "title": "Isolated Infiltrative Optic Neuropathy in an Acute Lymphoblastic Leukemia Relapse.", "score": 0.008771929824561403, "content": "Optic nerve infiltration as the first sign of isolated central nervous system relapse of acute lymphoblastic leukemia (ALL) is rare. A seven-year-old girl with standard-risk B-cell ALL who was in remission presented with sudden onset of left eye pain and loss of vision. Examination revealed no perception to light in the left eye with positive relative afferent pupillary defect. The optic disc was hyperemic and swollen with total obscuration of the disc margin associated with central retinal artery and vein occlusion. Magnetic resonance imaging of the brain and optic nerve showed left intraorbital optic nerve thickening associated with perineural enhancement and intraconal fat involvement. Lumbar puncture revealed leukemic infiltration with blast cells after a week of eye symptoms, while bone marrow aspiration was negative for malignant cells. A diagnosis of left leukemic optic nerve infiltration with central retinal artery and vein occlusion was made. A high index of suspicion with repeat cerebrospinal fluid sampling is crucial to confirm the diagnosis as vitreous biopsy may fail to reveal infiltrative cells."}, {"id": "wiki20220301en053_41039", "title": "Posterior ischemic optic neuropathy", "score": 0.008695652173913044, "content": "Signs and symptoms PION is characterized by moderate to severe painless vision loss of abrupt onset. One or both eyes may be affected and color vision is typically impaired. Ophthalmoscopic exam Looking inside the person's eyes at the time of onset, ophthalmoscope exam reveals no visible changes to the optic nerve head. Weeks after ischemic insult, nerve atrophy originating from the damaged posterior optic nerve progresses to involve the anterior optic nerve head. Four to eight weeks after onset, atrophy of the optic nerve head is observable upon ophthalmoscope exam. Pupils If both eyes are affected by PION, the pupils may look symmetrical. However, if the eyes are asymmetrically affected, i.e. one eye's optic nerve is more damaged than the other, it will produce an important sign called an afferent pupillary defect. Defective light perception in one eye causes an asymmetrical pupillary constriction reflex called the afferent pupillary defect (APD)."}, {"id": "pubmed23n0524_20048", "title": "[Transient trochlear nerve palsy as the presenting neurological sign of panarteritis nodosa].", "score": 0.008695652173913044, "content": "Panarteritis nodosa (PAN) is a systemic vasculitis affecting small and medium-sized arteries. Neuro-ophthalmological complications of PAN are rare but numerous, and may affect the eye, the visual and the oculomotor pathways. Such complications occur mainly in patients previously diagnosed with PAN. A 51-year-old woman presented with an isolated right trochlear (IV) palsy, in the setting of headaches and fluctuating fever of unknown etiology. Erythrocyte sedimentation rate was 13 mm and full blood cell count was normal. Previous chest X-ray and blood studies were negative for an infection or inflammation. Orbital and cerebral CT scan was normal. Spontaneous recovery of diplopia ensued over four days. Two days later, paresthesia and sensory paresis of the dorsal portion of the left foot were present. Lumbar puncture revealed 14 leucocytes (76 percent lymphocytes) with elevated proteins, but blood studies and serologies were negative. A diagnosis of undetermined meningo-myelo-radiculoneuritis was made. Because of a possible tick bite six weeks previously the patient was empirically treated with 2 g intravenous ceftriaxone for 3 weeks. Fever rapidly dropped. Six weeks after the onset of diplopia, acute onset of blindness in her right eye, diffuse arthralgias and fever motivated a new hospitalization. There was a central retinal artery occlusion of the right eye. Blood studies now revealed signs of systemic inflammation (ESR 30 mm, CRP 12 mg/L, ANA 1/80, pANCA 1/40, leucocytosis 12.4 G/L, Hb 111 g/L, Ht 33 percent). Biopsy of the left sural nerve revealed arterial fibrinoid necrosis. A diagnosis of PAN was made. Transient diplopia can be the heralding symptom of a systemic vasculitis such as PAN, giant cell arteritis and Wegener granulomatosis. In this patient the presence of accompanying systemic symptoms raised a suspicion of systemic inflammation, but the absence of serologic and imaging abnormalities precluded a specific diagnosis initially. A few weeks later, the presence of a second ischemic event (retinal) and positive blood studies led to a further diagnostic procedure. Oculomotor and abducens palsies have rarely been reported in association with PAN. We report the first case of trochlear nerve paresis as the inaugural neurological sign of PAN. This case highlights the importance of considering inflammatory systemic disorders in patients with acute diplopia particularly when they are young, lack vascular risk factors or cause, and complain of associated systemic symptoms."}]}}}} {"correct_option": 1, "explanations": {"1": {"exist": true, "char_ranges": [[18, 175]], "word_ranges": [[3, 24]], "text": "The picture described is very suggestive of pseudocrisis with asynchronous limb movements, pelvic movements, crying and poor response to antiepileptic drugs."}, "2": {"exist": false, "char_ranges": [], "word_ranges": [], "text": ""}, "3": {"exist": false, "char_ranges": [], "word_ranges": [], "text": ""}, "4": {"exist": false, "char_ranges": [], "word_ranges": [], "text": ""}, "5": {"exist": false, "char_ranges": [], "word_ranges": [], "text": ""}}, "full_answer": "Correct answer 1: The picture described is very suggestive of pseudocrisis with asynchronous limb movements, pelvic movements, crying and poor response to antiepileptic drugs.", "full_answer_no_ref": "Correct answer 1: The picture described is very suggestive of pseudocrisis with asynchronous limb movements, pelvic movements, crying and poor response to antiepileptic drugs.", "full_question": "In a patient diagnosed with epilepsy who presents with episodes of unresponsiveness to external stimuli, irregular movements of all four limbs, closed eyes, crying and pelvic movements, lasting five to twenty seconds and unresponsive to treatment with antiepileptic drugs, which complementary study is most likely to clarify the diagnosis?", "id": 32, "lang": "en", "options": {"1": "Video-EEG monitoring for diagnosis of pseudocrisis (psychogenic seizures).", "2": "Holter ECG for diagnosis of arrhythmic heart disease.", "3": "Routine EEG to diagnose the type of epilepsy (generalized or foc).", "4": "Brain MRI to detect epileptogenic lesions (cortical dysplasia, tumor, medial temporal sclerosis).", "5": "Determine capillary blood glucose for diagnosis of hypoglycemia."}, "question_id_specific": 64, "type": "NEUROLOGY AND NEUROSURGERY", "year": 2011, "rag": {"clinical_case_options": {"MedCorp": {"RRF-2": [{"id": "wiki20220301en323_32903", "title": "Vertiginous epilepsy", "score": 0.018162393162393164, "content": "Other means used in diagnosis of vertiginous epilepsy include: Electroencephalography (EEG) Magnetic resonance imaging (MRI) Positron emission tomography (PET) Neuropsychological testing The EEG measures electrical activity in the brain, allowing a physician to identify any unusual patterns. While EEGs are good for identifying abnormal brain activity is it not helpful in localizing where the seizure originates because they spread so quickly across the brain. MRIs are used to look for masses or lesions in the temporal lobe of the brain, indicating possible tumors or cancer as the cause of the seizures. When using a PET scan, a physician is looking to detect abnormal blood flow and glucose metabolism in the brain, which is visible between seizures, to indicate the region of origin. Management"}, {"id": "wiki20220301en063_48102", "title": "Psychogenic non-epileptic seizure", "score": 0.012961578178969483, "content": "The most definitive test to distinguish epilepsy from PNES is long term video-EEG monitoring, with the aim of capturing one or two episodes on both video recording and EEG simultaneously (some clinicians may use suggestion to attempt to trigger an episode). Additional clinical criteria are usually considered in addition to video-EEG monitoring when diagnosing PNES. By recording the event in question on video and EEG simultaneously, a clear diagnosis can usually be obtained. Laboratory testing can detect rising blood levels of serum prolactin if samples are taken in the right time window after most tonic-clonic or complex partial epileptic seizures. However, due to false positives and variability in results, this test is relied upon less frequently."}, {"id": "wiki20220301en228_24736", "title": "Paroxysmal dyskinesia", "score": 0.012637008381689232, "content": "The pathogenesis of PED has also been linked to mutations in the GLUT1 glucose transporter which can result in transient energy deficits in the basal ganglia. Diagnosis Diagnosis is similar, but slightly different for each type of PD. Some types are more understood than others, and therefore have more criteria for diagnosis. PKD The guidelines for diagnosing PKD were reviewed and confirmed by Unterberger and Trinka. PKD consists of unexpected forms of involuntary movements of the body. The patient is usually diagnosed sometime before their 20s, and is more likely diagnosed during childhood than early adulthood. Almost all PKD's are idiopathic, but there have been examples of autosomal dominant inheritance as well. Physical examination and brain imaging examinations show normal results, and an EEG shows no specific abnormalities as well. However, the negative synchronous EEG results can be used to prove that PKD is not a sort of reflex epilepsy, but a different disease."}, {"id": "wiki20220301en056_68359", "title": "Lennox–Gastaut syndrome", "score": 0.011817387505460901, "content": "To confirm diagnosis, awake and asleep EEG and magnetic resonance imaging (MRI) are performed. MRI is used to detect focal brain lesions. Ruling out other diagnoses Certain diagnoses must be ruled out before diagnosing LGS. These diagnoses are: Doose syndrome Dravet syndrome pseudo-Lennox Gastaut syndrome (atypical benign partial epilepsy) LGS is more easily distinguished from Doose syndrome by seizure type after the syndrome has progressed. Doose syndrome has more myoclonic seizures and LGS has more tonic seizures. The Doose syndromes is less likely to have cognitive disabilities. Pseudo-Lennox–Gastaut syndrome can be distinguished from LGS because pseudo-LGS has different spike-and-wave patterns on EEG. Treatment There are several treatment options, including medications, surgery, and diet. Medications In most patients with LGS, the treatment does not end seizure recurrence."}, {"id": "pubmed23n0371_17131", "title": "[Neuroimaging and electrophysiological study in epilepsy].", "score": 0.011666666666666665, "content": "There exist various morphological and biochemical changes closely associated with electrophysiological phenomena which cause epileptic seizures in the brains of epilepsy patients. Recent developments in investigation methods, not only electrophysiological(EEG and MEG), but also neuroimaging involving morphological imaging(CT and conventional MRI) and functional imaging(SPECT, PET, functional MRI and MRS) is able to demonstrate these changes. SPECT and PET can particularly clarify the changes of cerebral blood flow and glucose metabolism between interictal and ictal periods. In our experience of 423 patients who underwent epilepsy surgery for intractable seizures, these interventions provide important information to identify the epileptogenic foci. However, in practice, discordance in the results of these presurgical evaluations is recognized, and invasive intracranial recordings are needed in such cases. These problems in diagnosis were shown especially in patients with mesial temporal sclerosis and focal cortical dysplasia. To detect an epileptogenic focus more clearly, a combination of morphological and functional findings, new functional imaging such as neurotransmitter receptor imaging, EEG-triggered or neuropharmacological functional MRI, as well as, statistical parametric analysis may be needed."}, {"id": "wiki20220301en172_19823", "title": "Progressive myoclonus epilepsy", "score": 0.011447039199332779, "content": "Diagnosis Diagnosis of PME is based on the individual’s signs and symptoms as well as failure to respond to antiepileptic drugs and therapy. Further diagnosis support includes EEG results, genetic testing, enzyme testing, and skin and muscle biopsies. Gaucher’s disease can be diagnosed through enzyme testing as it is a metabolic disease. Lafora’s disease can be diagnosed using skin biopsies. While Action myoclonus renal failure (AMRF) syndrome can only be diagnosed using genetic test. Using EEG’s as a form of diagnosis can prove difficult as patients differ in their neurophysiology. In Lafora’s disease EEGs can show slowing background activity or focal discharges as well as epileptiform discharges. In ULD EEGs show generalized epileptiform discharges and in MERRF patients show background slowing. Therefore, diagnosis is best made using a combination of different tools like signs and symptoms, age of onset, EEG, gene testing, enzyme measurements, and biopsy of skin and muscle."}, {"id": "wiki20220301en000_158053", "title": "Epilepsy", "score": 0.011042253521126762, "content": "For adults, the testing of electrolyte, blood glucose and calcium levels is important to rule out problems with these as causes. An electrocardiogram can rule out problems with the rhythm of the heart. A lumbar puncture may be useful to diagnose a central nervous system infection but is not routinely needed. In children additional tests may be required such as urine biochemistry and blood testing looking for metabolic disorders. Together with EEG and neuroimaging, genetic testing is becoming one of the most important diagnostic technique for epilepsy, as a diagnosis might be achieved in a relevant proportion of cases with severe epilepsies, both in children and adults. For those with negative genetic testing, in some it might be important to repeat or re-analyze previous genetic studies after 2–3 years."}, {"id": "wiki20220301en000_158038", "title": "Epilepsy", "score": 0.01063034188034188, "content": "Diagnosis The diagnosis of epilepsy is typically made based on observation of the seizure onset and the underlying cause. An electroencephalogram (EEG) to look for abnormal patterns of brain waves and neuroimaging (CT scan or MRI) to look at the structure of the brain are also usually part of the initial investigations. While figuring out a specific epileptic syndrome is often attempted, it is not always possible. Video and EEG monitoring may be useful in difficult cases. Definition Epilepsy is a disorder of the brain defined by any of the following conditions: {| cellpadding=5 style=\"border:1px solid #ccc\" |- bgcolor=\"#fafafa\" | At least two unprovoked (or reflex) seizures occurring more than 24 hours apart One unprovoked (or reflex) seizure and a probability of further seizures similar to the general recurrence risk (at least 60%) after two unprovoked seizures, occurring over the next 10 years Diagnosis of an epilepsy syndrome |}"}, {"id": "pubmed23n0543_2570", "title": "Psychogenic pseudosyncope: an underestimated and provable diagnosis.", "score": 0.009900990099009901, "content": "The goal of this study was to estimate the frequency of psychogenic pseudosyncope in patients with \"syncope of unknown origin.\" Twenty to thirty percent of patients referred to epilepsy centers for refractory seizures have psychogenic seizures. With syncope, about 20-30% of the cases remain unexplained after a complete evaluation, but, unlike in seizures, a psychogenic etiology is not usually investigated. We prospectively evaluated patients referred to our epilepsy center for evaluation of recurrent syncope-like episodes, that is, limp, motionless fainting. All patients had a negative syncope workup. We performed EEG-video monitoring with activation by suggestion (\"induction\"), similar to what is used for diagnosis of psychogenic seizures. Activation was performed with patients standing or sitting up. The diagnosis of psychogenic pseudosyncope required: (1) an activation procedure that triggered the habitual event; (2) a clinical event of loss of postural tone and limp, motionless unresponsiveness with eyes closed; (3) normal EEG before, during, and after the clinical event, that is, no epileptiform abnormalities, a normal alpha rhythm during unresponsiveness, and no suppression of background or slowing as is typically seen in syncope. Ten patients were recruited over an 18-month period. Habitual syncope-like episodes were triggered in 9 of 10 (90%) patients, and all 9 were shown to have psychogenic pseudosyncope (eyes closed, motionless, unresponsive with normal EEG including normal alpha rhythm). In one patient, no episode was triggered, so a diagnosis could not be made. Among the 9 patients for whom episodes were recorded, age ranged from 21 to 60 (mean=36). Five were women. Duration of symptoms ranged from 6 months to 15 years (mean=4.2 years). Event frequency ranged from four per day to two per month. Prior evaluations for syncope included ECG in all patients, two-dimensional echocardiogram in three, Holter monitoring in two, and tilt-table test in five. Four patients had undergone cardiac catheterization, and one had received a pacemaker. Neurologic tests included CT of the head in seven and MRI of the brain in eight. Many patients with \"syncope of unknown origin\" may have psychogenic pseudosyncope, but most such patients do not undergo EEG-video monitoring, which is the only way to demonstrate a psychogenic etiology. Psychogenic pseudosyncope is not simply a diagnosis of exclusion, and can be firmly diagnosed. As is usually recommended for seizure-like events, patients with syncope-like events and a negative evaluation should undergo EEG-video monitoring with induction, specifically looking for a possible psychogenic etiology."}, {"id": "pubmed23n0206_20604", "title": "Hypoglycemic activation of focal abnormalities in the EEG of patients considered for temporal lobectomy.", "score": 0.009900990099009901, "content": "EEGs were recorded in 22 patients with medically refractory complex partial epilepsy undergoing presurgical evaluation and 11 age-matched controls while subjected to moderate levels of hypoglycemia to determine if changes activated were predictive of underlying pathology. Five patients had fasting EEGs showing focal abnormalities not seen in the non-fasting state. With hypoglycemia, EEG tracings in normal individuals showed diffuse background slowing, whereas 7 of 22 patients developed focal temporal changes, including focal spike and focal slow wave activation. The development of focal changes correlated well with clinical data concerning underlying focal pathology; focal abnormalities were not evoked in patients with multifocal disease. Hypoglycemic activation of the EEG may be a useful technique for predicting the presence of pathology in patients considered for anterior temporal lobectomy."}, {"id": "wiki20220301en583_4018", "title": "Diagnosis (American TV series)", "score": 0.009890205646566747, "content": "Kamiyah Morgan is a 6 year old little girl who suffers from a very unusual set of fainting episodes that will leave her unresponsive and immobile, and they can happen up to 300 times a day. When she experiences a fainting episode she will become completely paralyzed affecting everything in her body including her lungs, her mother states that every day that passes her ability to breathe diminishes. Kamiyah’s mother said that these fainting episodes started when she was about 8 months old as she would be crawling and suddenly tip over and go limp. At first their pediatric physician said the episodes looked like she was having a seizure, but after running an EEG there was no seizure being detected during the episodes. They then tried testing with MRI for any brain tumors or malignancies but again there was nothing. They were then referred over to the NIH or the National Institute of Health where their entire purpose is to be able to research and hopefully diagnose very strange cases."}, {"id": "wiki20220301en203_16065", "title": "Post-traumatic epilepsy", "score": 0.00980392156862745, "content": "Diagnosis To be diagnosed with PTE, a person must have a history of head trauma and no history of seizures prior to the injury. Witnessing a seizure is the most effective way to diagnose PTE. Electroencephalography (EEG) is a tool used to diagnose a seizure disorder, but a large portion of people with PTE may not have the abnormal \"epileptiform\" EEG findings indicative of epilepsy. In one study, about a fifth of people who had normal EEGs three months after an injury later developed PTE. However, while EEG is not useful for predicting who will develop PTE, it can be useful to localize the epileptic focus, to determine severity, and to predict whether a person will suffer more seizures if they stop taking antiepileptic medications. Magnetic resonance imaging (MRI) is performed in people with PTE, and CT scanning can be used to detect brain lesions if MRI is unavailable. However, it is frequently not possible to detect the epileptic focus using neuroimaging."}, {"id": "pubmed23n0094_19906", "title": "[Recently experienced ten cases of insulinoma--preoperative diagnosis of localization and intraoperative simultaneous monitoring of glucose and insulin].", "score": 0.00980392156862745, "content": "We have experienced 10 cases of insulinoma during the last 10 years from 1977 to 1986. All cases had strong hypoglycemic symptoms such as disturbance of consciousness, and insulinoma still tended to be misdiagnosed as epilepsy. The diagnosis of insulinoma was easily available from serum IRI (immunoreactive insulin)/plasma glucose ratio in all of the ten cases. As preoperative procedures for the diagnosis of localization, arteriography, computed tomography and portal blood sampling were positive in 6 of 8, 4 of 6 and 2 of 2 patients, respectively. At operation, all insulinomas could be identified by digital palpation. We performed simple excision of the tumor in 6 patients and distal pancreatectomy in 4 patients. The tumors were solitary and benign in all patients, ranging in size from 1.0 cm to 4.5 cm. Three cases were presented as case reports. In these cases, portal blood sampling and/or intraoperative monitoring of plasma glucose and serum IRI were performed. Portal blood sampling was effective even for a case which was negative in image diagnostic procedures. Furthermore, simultaneous monitoring of plasma glucose and serum IRI by quick radioimmunoassay seemed to be a good guide to the completeness of resection of insulin producing tumors."}, {"id": "wiki20220301en253_30183", "title": "Electroencephalography", "score": 0.009708737864077669, "content": "Epilepsy monitoring is typically done to distinguish epileptic seizures from other types of spells, such as psychogenic non-epileptic seizures, syncope (fainting), sub-cortical movement disorders and migraine variants, to characterize seizures for the purposes of treatment, and to localize the region of brain from which a seizure originates for work-up of possible seizure surgery. Hospitals use an EEG monitor to help diagnose a seizure. They use that information to help with the treatment process as well as discovering risks. \"Many professionals have stated the importance of EEG’s when it comes to suspected seizures, for diagnosis and evaluation\". Doctors will be able to use the EEG monitoring system to help look at some treatment options as well as some risk factors. As technology advances, researchers are finding new monitors that are more accurate in regards to seizures. \"Advanced techniques with continuous EEG and simplified technique with aEEG allows clinicians to detect more"}, {"id": "pubmed23n0046_18662", "title": "Regional brain glucose metabolism in patients with complex partial seizures investigated by intracranial EEG.", "score": 0.009708737864077669, "content": "We performed interictal 18F-2-fluoro-2-deoxy-D-glucose positron emission tomography (18FDG-PET) studies in 57 patients with complex partial epilepsy (CPE), not controlled by medical treatment and considered for surgical resection of their epileptic focus. A precise localization of the epileptic focus was obtained in 37 of these patients with a combination of subdural and depth electrodes. We visually inspected the metabolic images; we also measured glucose consumption in a number of brain regions and compared the values with those obtained in 17 normal controls. Eighty-two percent of the 57 patients had an area of glucose hypometabolism on the 18FDG-PET images. Six patients had a frontal epileptic focus, 3 of them had a frontal lobe hypometabolism. Twenty-six patients had a unilateral temporal lobe focus and all of them displayed a temporal lobe hypometabolism. The asymmetry was more pronounced in the lateral temporal cortex (-20%) than in the mesial part of the temporal lobe (-9.6%). In each cortical brain region on the side of the epileptic focus (except the sensorimotor cortex), glucose consumption rate was lower than in the contralateral region or than in controls. No differences could be found between patients with a seizure onset restricted to the hippocampus and patients with a seizure onset involving the hippocampus and the adjacent neocortex. Divergent metabolic patterns were obtained in 5 patients with bilateral temporal seizure foci. Combined with other non invasive techniques (EEG, neuroradiology), PET contributes increasingly to the selection of patients with CPE who could benefit from surgical treatment.(ABSTRACT TRUNCATED AT 250 WORDS)"}, {"id": "pubmed23n0603_20146", "title": "[Insulinoma misdiagnosed and treated as epilepsy].", "score": 0.009615384615384616, "content": "Although insulinoma constitutes almost 90% of neuroendocrine tumors localized in the pancreas, it is a rare disease. Quite commonly prior to the diagnosis there is a history of several years and misdiagnosis as neurological or cardiological disease is not infrequent. Patient, 22 years old, since 7 years experiencing multiple incidents of neuroglycopenia with concurrent hyperadrenergic reaction. Consciousness disturbances and muscle tremor together with feeling of hunger and tachycardia occurred mainly in the morning hours, or after physical exercise, and subsided after glucose intake. Increase in body weight, typical for insulinoma, was also observed. The patient was hospitalized twice in Pediatric Department and although hypoglycemia was observed, no additional testing was performed to exclude insulinoma; reported symptoms and abnormalities in EEG recording after provocation resulted in diagnosis and treatment of epilepsy. During hospitalization in the Department of Endocrinology fasting test was performed, which revealed inadequately high insulin level with glucose level of 41 mg% and signs of neuroglycopenia. The image of pancreas was normal in the acquired abdominal ultrasound and in CT a tumor was found in the tail of pancreas. The patient underwent laparoscopic operation and the clinical diagnosis was confirmed by histopathology. Antiepileptic drugs were discontinued. Total remission of symptoms was achieved. The presented case demonstrates the difficulties in correct interpretation of reported symptoms, while the results of biochemical tests and imaging studies point precisely to the diagnosis. Focal neurological signs resulting from multiple episodes of hypoglycemia may lead to misdiagnosis and treatment of epilepsia."}, {"id": "pubmed23n0213_13491", "title": "Hypometabolic cortical lesions in tuberous sclerosis with epilepsy: demonstration by positron emission tomography.", "score": 0.009523809523809525, "content": "Four patients with a well-established diagnosis of tuberous sclerosis and grand mal type epileptic seizures as their principal clinical symptom were examined by conventional surface electroencephalography (EEG), X-ray computed tomography, and positron emission tomography (PET) using the [18F]-2-fluoro-2-deoxyglucose method. The interictal EEG showed various abnormalities of poor localizing value, but no focal epileptic discharges. X-ray computed tomography demonstrated subependymal calcifications in all cases, although cortical lesions were found only twice. However, in the PET images of each patient one or two localized cortical foci with a metabolic rate for glucose more than 40% lower than in the respective contralateral region were clearly delineated. It may be assumed that those hypometabolic areas represent the epileptogenic cortical tubers, which are characteristic of the disease but usually cannot be detected in vivo by other methods."}, {"id": "wiki20220301en001_111077", "title": "Seizure", "score": 0.00950943056087613, "content": "In adults, testing electrolytes, blood glucose and calcium levels is important to rule these out as causes, as is an electrocardiogram. A lumbar puncture may be useful to diagnose a central nervous system infection but is not routinely needed. Routine antiseizure medical levels in the blood are not required in adults or children. In children additional tests may be required. A high blood prolactin level within the first 20 minutes following a seizure may be useful to confirm an epileptic seizure as opposed to psychogenic non-epileptic seizure. Serum prolactin level is less useful for detecting partial seizures. If it is normal an epileptic seizure is still possible and a serum prolactin does not separate epileptic seizures from syncope. It is not recommended as a routine part of diagnosis epilepsy."}, {"id": "wiki20220301en010_49780", "title": "Macropsia", "score": 0.009433962264150943, "content": "Epilepsy Macropsia may present itself as a symptom of both frontal lobe epilepsy and temporal lobe epilepsy, which may actually help in the diagnosis of those diseases. Children who experience nocturnal hallucinations accompanied by macropsia may seek medical care for panic attack disorders and instead are diagnosed with forms of epilepsy. Epilepsy patients may have no memory of the seizure, but can remember the hallucinations and aura which proceed the attack. Electroencephalography, or EEG imaging, can then be utilized while the patient experiences the episode. It may be subsequently concluded that the EEG is congruent with temporal or frontal lobe seizure. Anxiety and headaches accompany the episodes of visual distortion associated with epilepsy. While Valproic acid has been used to treat this type of seizure, anti-seizure medications appropriate for focal-onset seizures, like oxcarbazapine, have also been used successfully in the treatment of epilepsy-related macropsia."}, {"id": "pubmed23n0130_13895", "title": "Local cerebral metabolic rate for glucose during petit mal absences.", "score": 0.009433962264150943, "content": "Four patients with primary generalized or true petit mal epilepsy were studied with positron emission tomography using [18F]fluorodeoxyglucose (FDG). FDG studies were carried out during 10 minutes of hyperventilation before and again after medical control of spontaneous absences. Before seizures were controlled all 4 patients demonstrated frequent bilaterally synchronous three-per-second spike-and-wave discharges associated with altered consciousness. After spontaneous seizures were controlled, hyperventilation produced only electroencephalographic slowing without clinical symptoms in 3; the fourth patient had absences less frequently. Patterns of local cerebral metabolic rate for glucose (CMRGlc) were normal and identical for ictal and interictal scans; there was, however, a 2.5- to 3.5-fold diffuse ictal increase in global CMRGlc evident when ictal studies were compared with hyperventilation control studies in which no seizures occurred. The CMRGlc was similar in the two scans obtained from the patient who had absences during both studies. No anatomical substrate of petit mal epilepsy was identified. The CMRGlc in these patients during petit mal absences was higher than that recorded in other patients during partial or generalized convulsive seizures. This difference may reflect the fact that petit mal absences are not associated with postictal depression."}, {"id": "wiki20220301en025_34869", "title": "Ictal headache", "score": 0.009345794392523364, "content": "For the diagnosis it is necessary to perform an EEG during the headache that shows epilepsy-compatible discharges coinciding with the onset and cessation of the headache. The so-called hemicrania epileptica is a variant of EH characterized by the fact that head pain and EEG paroxysms are located on the same side. MRI is necessary to establish the cause, which, as in all focal epilepsies, can be varied: malformations/dysplasia, neoplasms, encephalopathies, traumatic brain injury, vasculopathies. Therapy. It depends on the etiology. During the headache, like most seizures, i.v. benzodiazepines are usually effective. Antiepileptic drugs can be used as preventive. 2. Ictal non-epileptic headache. Rare cases are reported. It is a condition that can be differentiated with certainty from the previous one if the headache episode is also present outside the seizure, that is, before and/or after, without specific EEG abnormalities. References External links"}, {"id": "pubmed23n0227_4297", "title": "Interictal cerebral glucose metabolism in partial epilepsy and its relation to EEG changes.", "score": 0.009345794392523364, "content": "Interictal positron computed tomography (PCT) with 18F-fluorodeoxyglucose was performed on 50 patients with partial seizures disorders. Electroencephalographic (EEG) monitoring was carried out during the metabolic studies using scalp and sphenoidal electrodes in 33 patients and stereotaxically implanted depth electrodes in 17. Four patients in this series had focal abnormalities on x-ray computed tomographic scans, but these were at the site of the presumed epileptogenic lesion in only 2. One or more discrete zones of hypometabolism were identified in 35 patients, and only 1 patient appeared to show focal interictal hypermetabolism. No quantitative relationship could be demonstrated between the degree of focal hypometabolism and either the frequency of interictal EEG spikes of the presence of focal nonepileptiform EEG changes. It was concluded that metabolic and electrophysiological techniques measure different aspects of cerebral dysfunction in seizure disorders. Although interictal PCT in patients with partial epilepsy usually demonstrates zones of hypometabolism this finding, per se, does not reveal the epileptic nature of the abnormality."}, {"id": "wiki20220301en128_19212", "title": "Unverricht–Lundborg disease", "score": 0.009259259259259259, "content": "Other methods to diagnose Unverricht–Lundborg disease are currently being explored. While electroencephalogram (EEG) is useful in identifying or diagnosing other forms of epilepsy, the location of seizures in ULD is currently known to be generalized across the entire brain. Without a specific region to pinpoint, it is difficult to accurately distinguish an EEG reading from an individual with ULD from an individual with another type of epilepsy characterized by generalized brain seizures. However, with recent research linking ULD brain damage to the hippocampus, the usefulness of EEG as a diagnostic tool may increase. Magnetic Resonance Imaging (MRI) is also often used during diagnosis of patients with epilepsy. While MRIs taken during the onset of the disease are generally similar to those of individuals without ULD, MRIs taken once the disease has progressed show characteristic damage"}, {"id": "pubmed23n0498_8099", "title": "Symptomatic occipital lobe epilepsy following neonatal hypoglycemia.", "score": 0.009259259259259259, "content": "This study reports on the clinical, electrophysiologic, and neuroradiologic aspects of patients with epilepsy secondary to neonatal hypoglycemia. Fifteen patients with epilepsy and/or posterior cerebral lesions, and neonatal hypoglycemia were studied in the epilepsy clinic between February 1990 and March 2003. The mean age was 12 years. The different types of neonatal hypoglycemia were as follows: four patients had transitional-adaptive, seven classic transient, two secondary-associated, and two severe recurrent hypoglycemia. As to epilepsy, we recognized a larger group of 12 patients characterized by focal seizures and posterior abnormalities on the electroencephalogram, the majority of whom had a good outcome, and a second group of two patients presenting electroclinical features of encephalopathy with refractory seizures. All patients except two manifested parieto-occipital lesions on neuroradiologic images. Neurologic examination was normal in one patient. Six patients had microcephaly; eight manifested visual disturbances. Fourteen patients were mentally retarded. One had a pervasive developmental disorder. This study indicates neonatal hypoglycemia may cause posterior cerebral lesions, abnormal findings at neurologic examination, and symptomatic epilepsy, most frequently occipital lobe epilepsy, usually with a good prognosis, and occasionally epileptic encephalopathy with refractory seizures. MRI studies are essential to define the characteristics of cerebral lesions after neonatal hypoglycemia."}, {"id": "pubmed23n0071_4019", "title": "Positron emission tomography findings relevant to neurosurgery for epilepsy.", "score": 0.009174311926605505, "content": "Using the 2-[F-18]fluorodeoxyglucose method, 213 positron emission tomographic (PET) studies of local brain glucose metabolism (CMRglu) were performed in 124 patients with various forms of epilepsy. Interictal PET scans of primary epileptics typically showed some global metabolic depression and decreased functional activity of insular, basal and anterior temporal cortex. Epilepsia partialis continua Kozevnikov was characterized by hypo- or hyper-metabolism of perirolandic cortex. Tuberous sclerosis was distinguished by neocortical foci of significantly decreased glucose consumption. Even in the interictal resting state, with regard to sensitivity (greater than 90%) and accuracy of focus localization. PET was superior to other diagnostic methods in typical temporal lobe epilepsy. Averaging 23% below normal CMRglu, the majority of hypometabolic foci were found in mesial temporal structures. Improved distinction between the epileptogenic area and the surrounding tissue showing comparatively normal functional responsiveness, was achieved by psychophysical activation using emotional speech or continuous visual recognition during PET scanning. In patients who had undergone total cerebral hemispherectomy because of uncontrolled epilepsy, remarkable recruitment of association areas was observed on both motor and speech activation."}, {"id": "pubmed23n0262_1928", "title": "[Continuous partial epilepsy disclosing diabetes mellitus].", "score": 0.00909090909090909, "content": "Continuous partial epilepsy (CPE) is characterized by isolated, subintrant clonus focalized to a limited territory with critical focal electroencephalography in a concordant territory. CPE is observed in various cortical lesions but also in disorders of metabolism and notably decompensated diabetes mellitus. We report a case of CPE without focal lesion at MRI which revealed hyperglycaemia without ketosis. The 54-year old female patient was hospitalised for C.P.E.. Early CT and later MRI gave normal results. Biochemistry showed hyperglycaemia without kenoturia, acidosis or hyperosmolality. Insulin therapy rapidly brought glycaemia down to its normal level and the clonsism disappeared. Five months later, the patient had no other seizure and the EEG was normal. Epileptic seizures are frequent in hyperglycaemia without ketosis (25% of the cases) where they are mainly partial and motor (75 to 86% of the cases), rarely associated with a focal lesion (15% of the cases with CT scan). They are rare in patients with ketoacidosis. This apparent protective effect of ketoacidosis may be attributed to an increase of GABA bioavailability consecutive to acidosis. CPE is resistant to antiepileptic treatments. In CPE induced by hyperglycaemia without ketosis normalization of blood glucose level with insulin therapy is concomitant with a rapid cure of epilepsy. Thus glycaemia should be measured in all patients presenting with CPE, the aim being to diagnose hyperglycaemia without ketosis rapidly to avoid hyperosmolality and to prescribe an adequate treatment based exclusively on insulin and rehydration."}, {"id": "pubmed23n0408_11448", "title": "[Acute repetitive giratory seizures as a manifestation of nonketotic hyperglycemia].", "score": 0.009009009009009009, "content": "This 71 years old women without any history of epilepsy had diabetes mellitus. She was admitted for repetitive giratory seizures in relation with non-ketotic hyperglycaemia. The EEG showed right centro-parietal paroxysmal slow activity. Symptomatology disappeared within 48 hours after insulin therapy. One month later, she presented with a left hemiplegia in relation with a right sylvian infraction. The role of focal transitory ischaemia in connection with hyperglycaemia is discussed."}, {"id": "wiki20220301en253_30173", "title": "Electroencephalography", "score": 0.008928571428571428, "content": "EEG is most often used to diagnose epilepsy, which causes abnormalities in EEG readings. It is also used to diagnose sleep disorders, depth of anesthesia, coma, encephalopathies, and brain death. EEG used to be a first-line method of diagnosis for tumors, stroke and other focal brain disorders, but this use has decreased with the advent of high-resolution anatomical imaging techniques such as magnetic resonance imaging (MRI) and computed tomography (CT). Despite limited spatial resolution, EEG continues to be a valuable tool for research and diagnosis. It is one of the few mobile techniques available and offers millisecond-range temporal resolution which is not possible with CT, PET or MRI."}, {"id": "pubmed23n0398_10811", "title": "Focal and global cortical hypometabolism in patients with newly diagnosed infantile spasms.", "score": 0.008928571428571428, "content": "To evaluate the occurrence and prognostic importance of focal defects in cerebral cortical glucose metabolism in infants with newly diagnosed symptomatic and cryptogenic infantile spasms. Ten children with symptomatic and seven with cryptogenic infantile spasms underwent MRI, video-EEG, and PET using fluorodeoxyglucose as a tracer within 2 weeks of diagnosis. PET was repeated at 1 year of age in 12 patients. Cortical hypometabolic foci were found in 13 children (77%) with newly diagnosed spasms (six cryptogenic and seven symptomatic). The hypometabolic foci disappeared in seven of nine reexamined at age 1. The occipital foci disappeared in all (n = 6). Focal findings on PET correlated well with focal findings on video-EEG. There was no difference in quantitative cortical or subcortical glucose metabolic rate at the onset of infantile spasms between children with cryptogenic and symptomatic etiology of spasms. The glucose metabolic rate at the onset of spasms or focal lesions in glucose metabolism did not have prognostic value for seizure outcome. Infantile spasms are often associated with transient cortical, especially occipital, hypometabolic foci that are not necessarily associated with structural lesions and do not indicate a poor prognosis."}, {"id": "wiki20220301en518_5803", "title": "Occipital epilepsy", "score": 0.008866461398106968, "content": "Diagnosis Procedures for diagnosis of occipital epilepsy include hematology, biochemistry, screenings for metabolic disorders, DNA analysis, and most commonly, MRI. Electroencephalogram (EEG) is also used to detect abnormal brain waves and activity that is reflected as slow waves, or spikes on the recordings. For occipital epilepsy, commonly identified abnormalities on the EEG when a seizure is not occurring (inter-ictal) includes posterior lateralized slow waves, asymmetrical alpha and photic following, and unilateral occipital spikes. Idiopathic cases may appear mostly normal, with occipital spikes or paroxysms. Ictal EEG’s show occipital paroxysmal fast activity, spiking, or both, as well as brief occipital flattening. About one-third of occipital seizures do not show any obvious changes."}, {"id": "wiki20220301en075_60927", "title": "Reflex seizure", "score": 0.008849557522123894, "content": "The activation of the hyper-excitable areas of the brain are additionally regulated by facilitating factors that may increase the likelihood of eliciting a seizure. Most commonly these include fatigue, sleep deprivation, or stress. Facilitating factors are different for each individual. Due to the large variance between the different kinds of reflex epilepsies, the specific mechanism causing reflex seizures may vary. Diagnosis The diagnosis of reflex epilepsy usually includes a comprehensive medical and family history as well as a variety of tests. These tests may include a electroencephalography (EEG), magnetic resonance imaging (MRI), as well as genetic testing. The procedure for diagnosing epilepsy generally follows three steps: Determining if the seizure or seizure like event is truly an epileptic seizure. Determining what kind of seizure that someone has suffered from. Determining if this seizure or seizures are a part of a specific epilepsy syndrome or disease."}, {"id": "pubmed23n0136_4722", "title": "[Hypoglycemic states in the clinical picture of emergency neuropathology].", "score": 0.008849557522123894, "content": "The authors made a clinico-electrophysiological analysis of hypoglycemic conditions in 20 patients admitted to the clinic with acute cerebral symptomatology. They describe the following neurological symptoms developing in hypoglycemia: paroxysmal disturbances of consciousness, including epileptic seizures; pseudostrokes; pseudotumours; comatose states. The paper presents criteria of the differentiation between primary cerebral disorders and neuroglycopenic symptoms of hypoglycemic conditions. These criteria include the development of consciousness disturbances in morning and after long intervals between meals, excessive weakness for sweets, fluctuations of the degree of focal and general cerebral symptoms from mild lipothymic states to the development of pronounced focal neurological symptomatology, the presence of high amplitude slow wave activity at all EEG leads and the efficacy of the intravenous administration of glucose. It has been shown that hypoglycemia may manifest itself by various cerebral disorders and that the development of recurrent hypoglycemic states may be responsible for secondary metabolic encephalopathy with various focal neurological symptomatology."}]}}}} {"correct_option": 4, "explanations": {"1": {"exist": true, "char_ranges": [[596, 858]], "word_ranges": [[94, 129]], "text": "Among the X-linked immunodeficiencies is Wiskott-Aldrich syndrome, an entity described with an initial triad of symptoms consisting of bleeding (typical BUT absent in the case: heavy bleeding after circumcision, bloody diarrhea), recurrent infections and eczema."}, "2": {"exist": true, "char_ranges": [[968, 1187]], "word_ranges": [[144, 177]], "text": "The Hyper-IgE option lacks very characteristic clinical data such as bone alterations and skin lesions, which are not atopic dermatitis, since they follow a different pattern (papulopustular rash on the face and scalp)."}, "3": {"exist": true, "char_ranges": [[1188, 1432]], "word_ranges": [[177, 212]], "text": "The option of transient hypogammaglobulinemia of infancy and the common severe and variable combined immunodeficiency fails, among other features, the determination of immunoglobulins G and M, which are at the lower limit, but within normality."}, "4": {"exist": true, "char_ranges": [[367, 595]], "word_ranges": [[58, 94]], "text": "are describing an immunodeficiency that by the family-maternal history, seems to be X-linked, as several males have died of a similar clinical condition (the father of the patient contributed the Y chromosome, the mother the X)."}, "5": {"exist": false, "char_ranges": [], "word_ranges": [], "text": ""}}, "full_answer": "To correctly answer this question it should be emphasized that among the symptomatology that is exposed we find: - 2-year-old child. - ENT infections. - Pulmonary infections. - Hospital admissions. - PTI. - Family history of infections with deaths due to infections in males (maternal family). - Atopic Dermatitis. All these data, in addition to the analytical data, are describing an immunodeficiency that by the family-maternal history, seems to be X-linked, as several males have died of a similar clinical condition (the father of the patient contributed the Y chromosome, the mother the X). Among the X-linked immunodeficiencies is Wiskott-Aldrich syndrome, an entity described with an initial triad of symptoms consisting of bleeding (typical BUT absent in the case: heavy bleeding after circumcision, bloody diarrhea), recurrent infections and eczema. In addition to thrombocytopenia, they are at increased risk for autoimmune phenomena and lymphoid neoplasms. The Hyper-IgE option lacks very characteristic clinical data such as bone alterations and skin lesions, which are not atopic dermatitis, since they follow a different pattern (papulopustular rash on the face and scalp). The option of transient hypogammaglobulinemia of infancy and the common severe and variable combined immunodeficiency fails, among other features, the determination of immunoglobulins G and M, which are at the lower limit, but within normality.", "full_answer_no_ref": "To correctly answer this question it should be emphasized that among the symptomatology that is exposed we find: - 2-year-old child. - ENT infections. - Pulmonary infections. - Hospital admissions. - PTI. - Family history of infections with deaths due to infections in males (maternal family). - Atopic Dermatitis. All these data, in addition to the analytical data, are describing an immunodeficiency that by the family-maternal history, seems to be X-linked, as several males have died of a similar clinical condition (the father of the patient contributed the Y chromosome, the mother the X). Among the X-linked immunodeficiencies is Wiskott-Aldrich syndrome, an entity described with an initial triad of symptoms consisting of bleeding (typical BUT absent in the case: heavy bleeding after circumcision, bloody diarrhea), recurrent infections and eczema. In addition to thrombocytopenia, they are at increased risk for autoimmune phenomena and lymphoid neoplasms. The Hyper-IgE option lacks very characteristic clinical data such as bone alterations and skin lesions, which are not atopic dermatitis, since they follow a different pattern (papulopustular rash on the face and scalp). [HIDDEN] [HIDDEN]", "full_question": "2-year-old boy. His personal history includes 3 episodes of acute otitis media, 1 meningococcal meningitis and 2 pneumonias (one middle lobe and one left upper lobe). She has been admitted on 3 occasions for thrombopenic purpura (on three occasions antiplatelet antibodies were negative and bone marrow showed normal megakaryocytes). Several males of the maternal family had died in childhood due to infectious processes. Physical examination showed lesions typical of atopic dermatitis. The immunological study showed a slight decrease in T-lymphocyte subpopulations; elevated IgA and IgE; decreased IgM and IgG at the lower limit of normal. What is the most likely diagnosis?", "id": 106, "lang": "en", "options": {"1": "Wiskott-Aldrich syndrome.", "2": "Hyper IgE syndrome.", "3": "Transient hypogammaglobulinemia of childhood.", "4": "X-linked severe combined immunodeficiency.", "5": "Common variable immunodeficiency."}, "question_id_specific": 135, "type": "GENETICS AND IMMUNOLOGY", "year": 2012, "rag": {"clinical_case_options": {"MedCorp": {"RRF-2": [{"id": "pubmed23n0703_8952", "title": "[Wiskott-Aldrich syndrome].", "score": 0.01846494213750851, "content": "The Wiskott-Aldrich syndrome is a primary immunodeficiency characterized by congenital microthrombocytopenia, eczema and recurrent infections. This paper reports the case of a 3-year-6-month male patient, whose maternal uncle died at the age of 3 months due to fulminant sepsis from a pulmonary infection. The patient was a product of the first pregnancy, he was born at 27 weeks' gestation and weighed 1,400 g. As a neonate he was hospitalized during the first 2 months of life because of a low gastrointestinal bleeding, thrombocytopenia and severe infections. In the next 4 months and before coming to our hospital the infant was hospitalized 54 times. On admission he presented disseminated dermatosis, enlarged neck lymph nodes and psychomotor retardation. Laboratory studies revealed hemoglobin 8.1 g/dL, platelets 31,000/uL, mean platelet volume 5.6 fL, IgM 39.3 mg/dL, IgA 67 mg/dL, IgG 1,380 mg/dL. On several occasions he received globular packages and platelet concentrates. The infusion of immunoglobulin G was started every 21 days. Bone marrow transplantation was delayed due to the complications that merited 13 hospitalizations and severe thrombocytopenia, low gastrointestinal bleeding, septic arthritis, infectious gastroenteritis, chronic suppurative otitis media and severe folliculitis. At the age of 4 years BMT of cord was performed, and 26 days after transplantation he presented septic shock and died. The prognosis of bone marrow transplantation in Wiskott-Aldrich syndrome and in other primary immunodeficiencies depends on the promptness of its performance at early stages in life. It is important that the first contact physicians be aware of the primary immunodeficiency signs and symptoms."}, {"id": "pubmed23n0423_22855", "title": "A case of IgG subclass deficiency with the initial presentation of transient hypogammaimmuno-globulinemia of infancy and a review of IgG subclass deficiencies.", "score": 0.018255746411086217, "content": "Primary immunodeficiency diseases are not common in children. The possibility of an immunological defect should be considered in any individual with repeated infections. A definite diagnosis for immodeficiency is sometimes difficult to achieve because of overlapping clinical manifestations. Immunoglobulin subclass deficiency is an immunological deficiency disease with which, one or more IgG subclasses are deficient. T cell immunity is normal. Patients may develop recurrent bacterial and respiratory infections or could remain asymptomatic. The authors report a case of immunoglobulin G subclass deficiency presenting initially as transient hypogammaglobulinemia of infancy. A 2 month-old boy presented to Siriraj Hospital with a history of chronic protracted diarrhea, disseminated scabies and sepsis. On presentation, he had generalized scaly and maculopapular rash with no palpable lymph nodes. CBC revealed WBC 22,100 cells/cm3 with PMN 42 per cent, lymphocytes 38 per cent, Eosinophils 4 per cent, Basophil 2 per cent and platelets 254,000/cm3. The immunoglobulin levels were as follows: IgG 181 mg/dl, IgA < 6.6 mg/dl, IgM 26.3 mg/dl. Lymphocyte enumerations revealed CD4 of 2,433 cells/cm3 (N 1,460-5,160); CD8 4,682 cells/cm3 (N 650-2,450); CD19 1,588 cell/cm3 (N 500-1,500); CD16 230 cell/cm3 (N 573 +/- 264). The initial diagnosis was X-linked agammaglobulinemia vs common variable immunodeficiency disease. His diarrhea and five courses of sepsis responded well to antibiotics administration and courses of intravenous immunoglobulin (IVIG) replacement. His through IgG became normal at 2 years of age (after 12 months of IVIG). IVIG was stopped and the diagnosis was changed to transient hypogammaglobulinemia of infancy (THI). Nevertheless, during his 4 month follow-up he developed recurrent sinopulmonary infections (i.e, otitis media and pneumonia). Repeated immunoglobulin profile showed IgG 1,200 mg/dl, IgA 135 mg/dl, IgM 26 mg/dl, IgG subclass were IgG, 1,030 mg/dl (N 280-830), IgG2 30 mg/dl (N 40-2,400), IgG3 22 mg/dl (N 6-130), IgG4 3 mg/dl (N 3-120). A diagnosis of IgG2 subclass deficiency presenting early as transient hypogammaglobulinemia of infancy was then made. Treatment with monthly IVIG was reinitiated and the patient is currently doing well. The authors present a case of IgG subclass deficiency presenting as transient hypogammaglbulinemia of infancy. Follow-up of the immune profile and clinical manifestation is necessary for a definite diagnosis."}, {"id": "pubmed23n0521_19925", "title": "De novo mutation causing X-linked hyper-IgM syndrome: a family study in Taiwan.", "score": 0.017767295597484276, "content": "X-linked hyper-IgM syndrome (XHIM) is a rare primary immunodeficiency disorder caused by mutations of the gene encoding the CD40 ligand (CD40L). It is characterized by recurrent infections with markedly decreased serum IgG, IgA and IgE levels but normal or elevated IgM levels. We report the clinical manifestations and complete immune studies in the first family with molecularly proven XHIM in Taiwan. A 5-month-old boy presented with rapidly progressive pneumonia which responded poorly to antibiotics. High levels of IgM and very low levels of IgG, IgA, and IgE were noted in his plasma specimen: IgM, 128 mg/dl; IgG, 18 mg/dl; IgA, 4 mg/dl); IgE, 1 IU/ml. Whole blood flow cytometry when he was 21 months old showed that only a small percentage (0.48%) of his in vitro-activated CD4+ T cells expressed CD40L. When he was 3 years old, repeated flow cytometry showed essentially the same result (0.4%), compared with his father's CD40L expression of over 85%. The patient's mother had moderately decreased CD40L expression (74.4%). Hyper-IgM syndrome was confirmed by CD40L mutation analysis in the boy, which revealed a Lys 96 stop (nucleotide A307T) in exon 2 of CD40L, with a truncated protein resulting in the loss of the entire TNF domain. His mother was a carrier and apparently the individual in whom the mutation originated. Eleven other family members, including the patient's father, sister, and grandmother, and the mother's sisters and their children, all had normal results on CD40L mutation analysis. The patient has remained without significant bacterial infection on a regimen of monthly IVIG infusion and oral trimethoprim-sulfamethoxazole for Pneumocystis carinii pneumonia (PCP) prophylaxis, although he has had recurrent oral ulcers and neutropenia. Bone marrow transplantation is planned."}, {"id": "pubmed23n0525_4354", "title": "Wiskott-Aldrich syndrome complicated by an atypical lymphoproliferative disorder: a case report.", "score": 0.01755601755601756, "content": "Wiskott-Aldrich syndrome (WAS) is an X-linked syndrome consisting of eczema, recurrent pyogenic infection, and thrombocytopenia with decreased platelet volume. Immunologic studies reveal normal immunoglobulin G (IgG), decreased IgM, elevated IgA and IgE levels, and decreased T-cell function. Patients with WAS often have increased susceptibility to lymphoproliferative disorders (LPDs). We report a 3-year-old boy who had persistent thrombocytopenia with bleeding, recurrent infections, and chronic eczema with frequent skin infections since birth. A blood smear revealed small platelets (50% of normal size). Immunologic studies showed normal IgG (1880 mg/dL), decreased IgM (76 mg/dL) and increased IgA (228 mg/dL) and IgE (14,282 IU/mL) levels. The relative proportions of immune cells were CD2 52.2%, CD3 41.1%, CD4 23.4%, CD8 16.8%, CD19 8.0%, CD57 7.7% and active T cells 14.6%. T-cell dysfunction was detected on the multitest for cell-mediated immunity. The WAS diagnosis was confirmed by mutation analysis which demonstrated a 4-base pair deletion in WAS protein gene exon 1. His thrombocytopenia was uncontrolled despite intravenous immunoglobulin infusions, so splenectomy was performed. The platelet count then rose to about 60,000 to 80,000/microL. However, about 2 weeks after splenectomy, he developed generalized lymphadenopathy and lymphoma was misdiagnosed based on lymph node biopsy at another hospital where he was admitted for urgent care. However, our analysis of his lymph node pathology led to the diagnosis of atypical LPD (ALPD). The lymphadenopathy regressed spontaneously 1 month later without chemotherapy. Early and correct diagnosis of WAS complicated with ALPD is important to avoid unnecessary chemotherapy."}, {"id": "pubmed23n0703_8951", "title": "[Satisfactory evolution of a patient diagnosed in childhood with Bruton's disease].", "score": 0.017043847241867045, "content": "Bruton's agammaglobulinemia is a primary immunodeficiency with a disease onset during the first months of age, when the maternal serum immunoglobulin levels decrease. It is characterized by recurrent infections and agammaglobulinemia. We report the case of a 6-year-old male patient with third-degree consanguinity, product of a third pregnancy and complete immunization scheme. He had a history of oral candidiasis at the age of 3 months, chicken pox at the age of 7 months, and two episodes of complicated bronchopneumonia at the age of 1 year and 6 years. He was admitted to the hospital because of fever and cough. Examination of the chest showed rales and right basilar hypoventilation, and a blood cell count revealed leukocytosis and neutrophilia. The diagnosis of pneumonia was made. He was treated with IV antibiotics. Serum immunoglobulins were reported to be low (IgM 55 mg/dL, IgA 0.9 mg/dL, and IgG 199 mg/dL). With these findings the clinical diagnosis of X-linked agammaglobulinemia (ALX) was concluded. A molecular test was performed fining a BTK gene confirming the diagnosis of Bruton's disease. Therapy with intravenous IgG was started every 21 days. During his evolution, he presented three episodes of rhinosinusitis, one of suppurative otitis media, and four events of pneumonia that required 37 days of hospitalization. After hospital discharge, the patient was free of infections and he returned to his daily activities. In cases of recurrent and severe respiratory infections in children, we must consider primary immunodeficiency disease in the differential diagnosis, mainly antibiotic deficiency. Early diagnosis and treatment improves the survival and quality of life in these patients."}, {"id": "pubmed23n0393_18251", "title": "Recurrent pneumonia as warning manifestation for suspecting primary immunodeficiencies in children.", "score": 0.017009719839908517, "content": "Two hundred and eight children with recurrent pneumonia were studied over a 5-year period. Among these patients we found 10 cases with primary immunodeficiency disease: 6 cases of IgA deficiency, 1 case of X-linked agammaglobulinemia, 1 case of common variable immunodeficiency, 1 case of hyper IgM syndrome, and 1 case of Wiskott-Aldrich syndrome. This study describes the clinical features of these cases and assesses the usefulness of our immunodeficiency screening protocol. In this group 6 were males; the mean age at first episode of pneumonia was 3 years (range 3 months to 18 years), and the age of diagnosis ranged between 10 months and 19 years. The average number of episodes of pneumonia in each patient was 5 (range 2 to 12), and the number of hospitalizations ranged up to 13. The etiologic agents isolated from this recurrent pneumonia were S. pneumoniae, Moraxella, adenovirus, respiratory syncytial virus, and influenza B virus. Intravenous immunoglobulin was used in four cases. Two patients had chronic pulmonary damage with bronchiectasis and interstitial pneumonia. Only one patient died (Wiskott-Aldrich syndrome) during the follow-up from an intracranial hemorrhage. We found that the screening protocol applied to patients with recurrent pneumonia is a useful tool for ruling out the primary immunodeficiency disorders."}, {"id": "pubmed23n0027_424", "title": "Serum IgD and IgE concentrations in immunodeficiency diseases.", "score": 0.015323891339679591, "content": "Concentrations of IgD and IgE were measured in sera from 165 patients with well-defined immunodeficiency in an effort to find information possibly relevant to the roles of antibodies of these classes in host defense. Values for both immunoglobulins were generally quite low in patients who had marked deficiencies of all three major immunoglobulins, although occasional normal or high normal values for IgD were seen in hypogammaglobulinemic patients. Group mean IgD concentrations were also depressed in patients with Wiskott-Aldrich syndrome and in those with selective IgA deficiency; IgE concentrations were depressed in patients with X-linked immunodeficiency with hyper-IgM and in those with ataxia telangiectasia. IgD and IgE were both significantly elevated in patients with extreme hyperimmunoglobulinemia E and undue susceptibility to infection and in a patient with the Nezelof syndrome; none of these patients had histories suggestive of atopy. In addition, the mean IgE concentration was significantly elevated in patients with selective IgA deficiency, many of whom were atopic, and in those with the Wiskott-Aldrich syndrome. The highest IgD concentration (163 mg/100 ml) was found in serum from a boy with variable immunodeficiency who had a lifelong history of severe recurrent pharyngeal infections, primarily streptococcal in etiology. Recurrent staphylococcal infection was a feature common to many but not all patients with elevated serum IgE concentration. These data may prove useful in the future delineation of biologic roles for antibodies in these two immunoglobulin classes."}, {"id": "pubmed23n0079_3796", "title": "[Hypogammaglobulinemia G and A with hypergammaglobulinemia M. Apropos of 12 cases].", "score": 0.015151515151515152, "content": "Hyper IgM with low IgG and IgA is a rare humoral immunodeficiency. We presently report 12 new observations which have been clinically and immunologically studied. On one occasion the syndrome was found to be associated with congenital rubella. Since 10/12 children were male, X-linked inheritance is suggested which has been confirmed in 2 cases. In most cases (9/12), the first infections occurred within the first year of life. The syndrome is causing upper and lower respiratory tract infections due to bacteria, as well as gut infections. Lymphoid organ hyperplasia has been noted in 11/12 patients. Polyclonal hyper IgM serum contrasts with low or absent IgG, IgA and IgE. In some instances, some IgM antibody response was detected. A dysfunction of cellular immunity was not detected. Autoimmunity was detected in 3 patients. Finally, transient neutropenia occurred in 50% of the patients. Intravenous immunoglobulin G substitution treatment resulted in a significant reduction in the occurrence of infections as well as in normalization of growth rate. Immunoglobulin infusion also frequently induced correction of hyper IgM and neutropenia."}, {"id": "pubmed23n0517_20443", "title": "Hyper-IgM syndrome: report of one case.", "score": 0.01498652896273013, "content": "The hyper-IgM syndrome (HIM) is a rare primary immunodeficiency disorder caused by defects in the CD40 ligand (CD40L)/CD40-signaling pathway. It is characterized by recurrent infections with markedly decreased IgG, IgA and IgE levels but normal or elevated serum IgM levels. A 5-month-old boy presented with rapidly progressive pneumonia which responded poorly to antibiotics. High levels of IgM and very low levels of IgG, IgE and IgA were noted in his plasma specimen (IgM, 128 mg/dl; IgG, 18 mg/dl; IgE, 1 IU/ml; IgA, 4 mg/dl). The relative proportions of immune cells were CD3 24.6%, CD4 10.3%, CD8 2.2%, CD19 30.2%, CD57 1.0% and active T cells 1.1%. After IVIG treatment, the pneumonia improved. Repeat assessment at the age of 15 months showed IgM decreased to the normal range (32 mg/dl). Whole blood flow cytometry assay for CD40L expression confirmed the diagnosis of hyper-lgM syndrome when he was 21 months old. Only a small percentage (0.48%) of the patient's in vitro activated CD4+ T cells expressed CD40L, compared with 33.54% from a healthy control. The patient's father, mother and sister all had a normal CD40L expression activation patterns (43.52%, 40.78%, 34.11%, respectively). On a regimen of monthly IVIG infusion and oral trimethoprim-sulfamethoxazole for Pneumocystis carinii pneumonia (PCP) prophylaxis, the patient has had no recurrent infections."}, {"id": "pubmed23n0501_10269", "title": "[Clinical features of X-linked agammaglobulinemia: analysis of 8 cases].", "score": 0.014412216499503028, "content": "X-linked agammaglobulinemia (XLA), caused by mutations in Bruton's tyrosine kinase (BTK), is a common form of inherited antibody deficiency. There were very few case reports of this disease that were diagnosed only based on clinical findings in China. The purpose of this study was to evaluate the clinical features of 8 Chinese cases with XLA with BTK defect which were confirmed by flow cytometry and/or gene analysis. Based on clinical findings, 8 suspected XLA patients were confirmed by detecting the expression of BTK by flow cytometry and/or gene analysis of BTK. The history and thorough physical examination and routine immunological evaluation of 8 cases were collected and reviewed. The age of onset of all the 8 male patients were from 3 months to 3 years. The mean age at diagnosis was 6 years. Recurrent upper respiratory infection and pneumonia with fever were seen in all the patients. Nasopharynx infection was mainly contributed to upper respiratory infection. Very few or no otitis (1/8) and sinusitis (0/8) were involved. Polyarthritis without evidence of infection was common (3/8). Chronic diarrhea was documented during the first 2 years after the onset of the disease in 2 cases. Two of the patients suffered from meningitis one time each. Skin infection was not serious in two patients. Osteomyelitis occurred in one case, which occurred secondary to a trauma. One case had poliomyelitis-like disease that was considered to be related to polio vaccine. Only two cases had unconfirmed maternal family history of XLA. The prominent signs at diagnosis were dystrophia, growth and developmental retardation and markedly decreased or absent tonsils and lymph nodes. Concentration of all classes of serum immunoglobulins (Igs) and the number of B cells in the peripheral circulation were dramatically decreased. The ratio of CD4/CD8 in most of the patients (6/8) was markedly inverse. The age at diagnosis of this reported group was older. Clinical symptoms displayed recurrent upper respiratory infection (nasopharynx infection but rare or no otitis or sinusitis) and pneumonia; polyarthritis was common. There were no confirmed family history of XLA. Most of the patients showed inverse ratios of CD4/CD8, the reason and potential significance are unclear."}, {"id": "wiki20220301en037_31604", "title": "Wiskott–Aldrich syndrome", "score": 0.014056026829749458, "content": "Signs and symptoms WAS occurs most often in males due to its X-linked recessive pattern of inheritance, affecting between 1 to 10 males per million. The first signs are usually petechiae and bruising, resulting from a low platelet count (i.e. thrombocytopenia). Spontaneous nose bleeds and bloody diarrhea are also common and eczema typically develops within the first month of life. Recurrent bacterial infections typically develop by three months of age. The majority of children with WAS develop at least one autoimmune disorder, and cancers (mainly lymphoma and leukemia) develop in up to a third of patients. Immunoglobulin M (IgM) levels are reduced, IgA and IgE are elevated, and IgG levels can be normal, reduced, or elevated. In addition to thrombocytopenia, WAS patients have abnormally small platelets (i.e. microthrombocytes) and ~30% also have elevated eosinophil counts (i.e. eosinophilia)."}, {"id": "wiki20220301en037_31610", "title": "Wiskott–Aldrich syndrome", "score": 0.01361875637104995, "content": "Diagnosis The diagnosis can be made on the basis of clinical findings, the peripheral blood smear, and low immunoglobulin levels. Typically, IgM levels are low, IgA levels are elevated, and IgE levels may be elevated; paraproteins are occasionally observed. Skin immunologic testing (allergy testing) may reveal hyposensitivity. Individuals with Wiskott–Aldrich syndrome however are at higher risk for severe food allergies. Not all patients have a positive family history of the disorder; new mutations do occur. Often, leukemia may be suspected on the basis of low platelets and infections, and bone marrow biopsy may be performed. Decreased levels of WASp are typically observed. The current gold standard for diagnosis is DNA sequence analysis, which can detect WAS and the related disorders XLT and XLN in 95% of patients and carriers."}, {"id": "wiki20220301en559_4909", "title": "List of primary immunodeficiencies", "score": 0.013536953242835596, "content": "Normal numbers of B cells with decreased IgG and IgA and increased IgM: Hyper-IgM syndromes Normal numbers of B cells with isotype or light chain deficiencies: heavy chain deletions, kappa chain deficiency, isolated IgG subclass deficiency, IgA with IgG subclass deficiency, selective immunoglobulin A deficiency Specific antibody deficiency to specific antigens with normal B cell and normal Ig concentrations Transient hypogammaglobulinemia of infancy (THI)"}, {"id": "pubmed23n0059_21268", "title": "Early bone marrow transplantation in an infant with Wiskott-Aldrich syndrome.", "score": 0.013450710519259987, "content": "The Wiskott-Aldrich Syndrome (WAS) is a rare X-linked immunohematological disorder characterized by eczema, profound thrombocytopenia, and progressive immunodeficiency. Severe hemorrhage, overwhelming sepsis, or lymphoreticular malignancy usually cause death in childhood. Recently, bone marrow transplantation (BMT) has been curative in some well-established cases, but there is no general agreement about the place of BMT in infants with WAS before the development of significant immunological abnormalities. We describe the successful use of early histocompatible BMT in a 10-month-old infant in whom WAS was diagnosed on the basis of eczema, thrombocytopenia, small platelets, and raised serum immunoglobulin A (Ig) and IgE, but before the development of immunodeficiency as evidenced clinically by recurrent infections, or immunologically by low serum IgM or consistently abnormal lymphocyte responses to mitogens. After an unstable period for several weeks posttransplantation when he developed marked hepatomegaly and severe interstitial pneumonitis, he made a good recovery. His eczema and thrombocytopenia resolved and he has shown no clinical or laboratory evidence of immunodeficiency. It is now over 2 years since his BMT. Because of the poor prognosis of WAS, where a histocompatible donor is available, BMT at the earliest opportunity, despite the inherent risks of such a procedure, may be the best option for an infant with WAS."}, {"id": "pubmed23n0858_16600", "title": "[Clinical features and genotype analysis of 132 patients with Wiskott-Aldrich syndrome].", "score": 0.012908061292471684, "content": "To investigate the clinical and immunological laboratory features, gene mutations, treatment and prognosis in children with Wiskott-Aldrich syndrome (WAS). The clinical, laboratory characteristics, treatment and prognosis of 132 children with WAS, who visited Children's Hospital of Chongqing Medical University from April 2000 to June 2015, were analyzed retrospectively. All patients were male. The median age of disease onset was 15 days and the median age at diagnosis was 10 months. Of the 132 cases, 112 had classic WAS, 20 had X-linked thrombocytopenia (XLT). The median platelet count was 23×10(9)/L. All cases had the clinical characteristics of WAS including bleeding, eczema, and being susceptible to infection. The initial symptoms include hemorrhage (75.0%) and eczema (16.7%). Twenty-one cases had autoimmune diseases and one patient had leukemia. WAS protein (WASP) expression in 115 cases were measured by flow cytometry, 88 cases were negative, in 12 cases WASP decreased, in 5 cases it was normal, 10 cases had bimodal distribution. Eighty-one kinds of mutations were found in 122 families, including eight kinds of hot-spot mutations, which were 290 C> N / 291G> N (R86C / H / L), 665 C> T (R211X), 155 C> T (R41X), 168 C> T (T45 M), IVS1+ 1 g> t/ a, IVS6 + 5 g> a, IVS8 + 1 g> a and IVS8 + 1to + 6del gtga. Meantime, 29 kinds of novel mutations were found, which were 321T>C, 415C>A, 471C>T, 102-105delC, 521 del C, 1330 del A, IVS2-2 a>c, 168 C>A/1412 C> T, exon1-2 del/1412 C>T, and so on. The proportion of CD3(+) T cells (31.3%), helper T cells (37.3%) and cytotoxic T cells (38.6%) in the peripheral blood declined. The serum levels of IgG (51.1%), IgA (43.3%) and IgE (40.0%) increased, IgM (25.6%) decreased. Of the 132 cases, 72 remain survived, of whom 36 cases received hematopoietic stem cell transplantation (HSCT), 14 patients with classic WAS received intravenous immunoglobulin (IVIG) therapy. With regular IVIG therapy, the frequency of infections was reduced and the patients' symptoms were improved. The clinical characteristics of Wiskott-Aldrich syndrome were early age of onset, microthrombocytopenia, eczema and recurrent infections. The proportion of T lymphocyte declined, the serum levels of IgG, IgA, and IgE increased, and level of IgM decreased in a part of patients. The detection of WAS gene mutation and WAS protein detection was the key diagnostic methods. Regular IVIG can gain more time for children who will receive HSCT and improve their quality of life."}, {"id": "article-17681_26", "title": "Antibody Deficiency Disorder -- Differential Diagnosis", "score": 0.012896825396825396, "content": "The most important differential diagnosis includes the following: X-linked agammaglobulinemia characterizes by recurrent bacterial infections in boys, and genetic studies may reveal the presence of Bruton tyrosine kinase (BTK) mutations. Transient hypogammaglobulinemia of newborns presents in newborns above the age of 4 months and characterizes by recurrent pneumonia, meningitis, otitis media, and other problems that resemble Bruton disease. It is a physiological defect in the immune system caused by maternal IgG disappearance and corrected soon but requires treatment. In super-IgM syndrome, recurrent bacterial infections occur, but the cause of this illness is a mutation in the gene encoding for CD40 on T lymphocytes that causes a failure in T and B lymphocyte cooperation. Common variable immunodeficiency presents with recurrent bacterial infections, including sinopulmonary problems but later in life (second-fourth decade), and the diagnosis is made once all causes of immunodeficiency have been ruled out. [6] [7] [21] [33]"}, {"id": "wiki20220301en067_45376", "title": "Hypogammaglobulinemia", "score": 0.012797933544675354, "content": "of primary immunodeficiency (PID). These different forms can affect different parts of the immune system, including immunoglobulin production. Primary immunodeficiencies usually have a delay of several years between initial clinical presentation and diagnosis. Some primary immune deficiencies include ataxia-telangiectasia (A-T), autosomal recessive agammaglobulinemia (ARA), common variable immunodeficiency (CVID), hyper-IgM syndromes, IgG subclass deficiency, isolated non-IgG immunoglobulin deficiencies, severe combined immunodeficiency (SCID), specific antibody deficiency (SAD), Wiskott-Aldrich syndrome, or X-linked agammaglobulinemia. CVID is the most common form of primary immunodeficiency. SCID is considered a medical emergency and suspected cases require immediate specialist center referral for diagnosis and treatment. It is more often that hypogammaglobulinemia develops as a result of another condition, which are called secondary or acquired immune deficiencies. These"}, {"id": "wiki20220301en062_59354", "title": "X-linked agammaglobulinemia", "score": 0.012641165755919854, "content": "Signs and symptoms Affects males 50% of the time if mother is a carrier for the gene. Children are generally asymptomatic until 6–9 months of age when maternal IgG decreases. Present with recurrent infections with Streptococcus pneumoniae, Haemophilus influenzae, Mycoplasma pneumoniae, hepatitis virus, and enterovirus CNS infections. Examination shows lymphoid hypoplasia (tonsils and adenoids, no splenomegaly or lymphadenopathy). There is significant decrease in all immunoglobulins. Genetics Most antibodies are gamma globulins. Antibodies are made mainly by plasma cells, which are daughter cells of the B cell line. The Btk enzyme plays an essential role in the maturation of B cells in the bone marrow, and when mutated, immature pro-B lymphocytes are unable to develop into pre-B lymphocytes, which normally develop into mature (naive) B cells that leave the bone marrow into the blood stream."}, {"id": "pubmed23n0626_19418", "title": "[Hyper-IgM syndrome in a boy with recurrent pneumonia and hepatosplenomegaly].", "score": 0.012531036623215395, "content": "We present a boy diagnosed at age 14 years with hyper-immunoglobulin (Ig) M syndrome, a congenital immunodeficiency characterized by reduced plasma concentrations of IgA, IgE and IgG, with normal or elevated concentrations of IgM. This syndrome is caused by a defect of CD40 ligand (CD40L) on T-helper lymphocytes, impeding the \"second signal\" during activation of B lymphocytes and interactions of T cells with dendritic cells and macrophages, resulting in the absence of secondary immune response (class switching, affinity maturation, immune memory), as well as responses to T-dependent antigens, with an impairment of cellular immunity. The history of the presented patient was dominated by frequent lower respiratory infections and failure to thrive. Physical examination demonstrated severe hepatosplenomegaly. The suspicion of hyper-IgM syndrome was raised by low plasma IgA (0.36 g/l) with high plasma IgM (35.5 g/l), while the concentration of IgG was within the normal range (12.1 g/l). The diagnosis was confirmed by flow cytometry, which demonstrated the absence of expression of CD40L on lymphocytes following stimulation by phorbolmyristylacetate and calcium ionophore. Since the time of diagnosis, intravenous immunoglobulin therapy has led to catch-up growth, recession of hepatosplenomegaly and reduction in the frequency of respiratory infections. Our report emphasizes the importance for the primary healthcare paediatrician to be well informed about the clinical presentation and pathogenesis of hyper-IgM syndrome, in order to provide early detection and increase the likelihood of success in treating this rare immunodeficiency. To the best of our knowledge, this is the first case of hyper-IgM syndrome reported in the Republic of Serbia."}, {"id": "wiki20220301en218_4190", "title": "List of skin conditions", "score": 0.012372812372812373, "content": "Bare lymphocyte syndrome Chronic granulomatous disease (Bridges–Good syndrome, chronic granulomatous disorder, Quie syndrome) Common variable immunodeficiency (acquired hypogammaglobulinemia) Complement deficiency DiGeorge syndrome (DiGeorge anomaly, thymic hypoplasia) Graft-versus-host disease Griscelli syndrome Hyper-IgE syndrome (Buckley syndrome, Job syndrome) Immunodeficiency with hyper-IgM Immunodeficiency–centromeric instability–facial anomalies syndrome (ICF syndrome) Isolated IgA deficiency Isolated primary IgM deficiency Janus kinase 3 deficiency Leukocyte adhesion molecule deficiency LIG4 syndrome Myeloperoxidase deficiency Neutrophil immunodeficiency syndrome Nezelof syndrome (thymic dysplasia with normal immunoglobulins) Omenn syndrome Purine nucleoside phosphorylase deficiency Severe combined immunodeficiency (alymphocytosis, Glanzmann–Riniker syndrome, severe mixed immunodeficiency syndrome, thymic alymphoplasia) Shwachman–Bodian–Diamond syndrome"}, {"id": "wiki20220301en559_4908", "title": "List of primary immunodeficiencies", "score": 0.012202503450112583, "content": "Predominantly antibody deficiencies In primary antibody deficiencies, one or more isotypes of immunoglobulin are decreased or don't function properly. These proteins, generated by plasma cells, normally bind to pathogens, targeting them for destruction. Absent B cells with a resultant severe reduction of all types of antibody: X-linked agammaglobulinemia (btk deficiency, or Bruton's agammaglobulinemia), μ-Heavy chain deficiency, l 5 deficiency, Igα deficiency, BLNK deficiency, thymoma with immunodeficiency B cells low but present or normal, but with reduction in 2 or more isotypes (usually IgG & IgA, sometimes IgM): common variable immunodeficiency (CVID), CD19 deficiency, TACI (TNFRSF13B) deficiency, BAFF receptor deficiency. Normal numbers of B cells with decreased IgG and IgA and increased IgM: Hyper-IgM syndromes"}, {"id": "wiki20220301en071_14447", "title": "Common variable immunodeficiency", "score": 0.012178655582325063, "content": "Diagnosis According to a European registry study, the mean age at onset of symptoms was 26.3 years old. As per the criteria laid out by ESID (European Society for Immunodeficiencies) and PAGID (Pan-American Group for Immunodeficiency), CVID is diagnosed if: the person presents with a marked decrease of serum IgG levels (<4.5 g/L) and a marked decrease below the lower limit of normal for age in at least one of the isotypes IgM or IgA; the person is four years of age or older; the person lacks antibody immune response to protein antigens or immunization. Diagnosis is chiefly by exclusion, i.e. alternative causes of hypogammaglobulinemia, such as X-linked agammaglobulinemia, must be excluded before a diagnosis of CVID can be made."}, {"id": "wiki20220301en225_1951", "title": "Humoral immune deficiency", "score": 0.012158326442125025, "content": "Absent B cells with a resultant severe reduction of all types of antibody: X-linked agammaglobulinemia (btk deficiency, or Bruton's agammaglobulinemia), μ-Heavy chain deficiency, l 5 deficiency, Igα deficiency, BLNK deficiency, thymoma with immunodeficiency B cells low but present, but with reduction in 2 or more isotypes (usually IgG & IgA, sometimes IgM): common variable immunodeficiency (CVID), ICOS deficiency, CD19 deficiency, TACI (TNFRSF13B) deficiency, BAFF receptor deficiency. Normal numbers of B cells with decreased IgG and IgA and increased IgM: Hyper-IgM syndromes Normal numbers of B cells with isotype or light chain deficiencies: heavy chain deletions, kappa chain deficiency, isolated IgG subclass deficiency, IgA with IgG subsclass deficiency, selective immunoglobulin A deficiency Transient hypogammaglobulinemia of infancy (THI)"}, {"id": "article-23400_15", "title": "Biochemistry, Immunoglobulin M -- Clinical Significance -- Selective IgM Deficiency", "score": 0.012094188960774029, "content": "Selective IgM deficiency (SIGMD) is a rare disorder with fewer than 300 cases reported. SIGMD is associated with an isolated deficiency in IgM in the presence of normal levels of other immunoglobulins such as IgG and IgA and normal levels of T cells and other leukocytes. [12] Individuals with SIGMD may be asymptomatic, or they may suffer from recurring infections from encapsulated bacteria (e.g., S. pneumoniae and H. influenzae ) in addition to viral infections. Additionally, SIGMD can be associated with malignancy, autoimmunity, or allergy. SIGMD may occur as a secondary effect of another disease, such as malignancy or bacteremia. Yet, primary causes of SIGMD have also been described, as some are associated with deletions on chromosome 22, for example. [12] The diagnosis of SIGMD is one of exclusion. Other diseases that result in low levels of multiple isotypes must be excluded, such as common variable immunodeficiency or X-linked agammaglobulinemia, which will likely result in reduced levels of several antibody isotypes. Conversely, Wiskott-Aldrich syndrome is often associated with low IgM yet elevated levels of IgG and IgA. Cold Agglutinin Disease"}, {"id": "wiki20220301en218_4191", "title": "List of skin conditions", "score": 0.011810415808974887, "content": "Severe combined immunodeficiency (alymphocytosis, Glanzmann–Riniker syndrome, severe mixed immunodeficiency syndrome, thymic alymphoplasia) Shwachman–Bodian–Diamond syndrome Thymoma with immunodeficiency (Good syndrome) Transient hypogammaglobulinemia of infancy Warts–hypogammaglobulinemia–infections–myelokathexis syndrome (WHIM syndrome) Wiskott–Aldrich syndrome X-linked agammaglobulinemia (Bruton syndrome, sex-linked agammaglobulinemia) X-linked hyper-IgM syndrome X-linked hypogammaglobulinemia X-linked lymphoproliferative disease (Duncan's disease) X-linked neutropenia"}, {"id": "pubmed23n0329_9114", "title": "Severe combined immunodeficiency with B-lymphocytes (T-B+SCID): report of two cases.", "score": 0.011628300037183518, "content": "Severe combined immunodeficiency (SCID) is a rare pediatric medical emergency in Taiwan. The early diagnosis of infants with SCID is very important because it can save the life of these critical infants. The essential clues important for early diagnosis of SCID patients include positive family history of early infant death, paucity of tonsil and lymphoid tissue, cutaneous fungal infection and lymphopenia. Severe combined immunodeficiency is a heterogeneous group of inherited disorders characterized by the failure of both cellular and humoral immunity. It can be categorized into SCID with B-lymphocytes predominant (T-B+SCID) and SCID with paucity of B-lymphocytes (T-B-SCID), according to the number of B-lymphocytes in the patient's peripheral circulation. We report two male infants with T-B+SCID who had been suffering from severe pulmonary distress with persistent O2 desaturation when they were transferred to our pediatric intensive care unit. Tracing back these infant's family histories, it was discovered that both of them had an elder brother who had died to overwhelming infection within the first year of life, and Pneumocystis carinii pneumonitis (PCP) was confirmed in the elder brother of case 2. After hospitalization, the immune condition of these two infants were evaluated which showed a decrease in T-cell and NK cell number, an increase in B-cell number, and decreased serum levels of all the Igs except IgM, which was elevated in case 1. These were the diagnostic immunological findings for T-B+SCID, which included X-linked SCID and Jak-3-deficient SCID. During hospitalization, severe mucocutaneous candidiasis and PCP were noted and confirmed in case 1 and PCP was highly suspected in case 2. Bone marrow transplantation, the only curable treatment for T-B+SCID at present, could not be performed in these two patients because of their grave clinical condition. Both of them expired due to their progressively downhill pulmonary conditions."}, {"id": "pubmed23n0886_14692", "title": "[Recurrent fever, hepatosplenomegaly and eosinophilia in a boy].", "score": 0.011404639175257733, "content": "A 2-year-old boy was admitted into the hospital because of cough and fever. Lymph node tuberculosis was noted when he was 2 months old and he was subsequently hospitalized several times because of cough and fever. After hospitalization the laboratory examination showed an increased eosinophia level in blood. The immune function tests shows decreased levels of IgG, IgA, and IgM. The patient had no response to anti-tuberculosis, anti-bacterial, and anti-fungal treatment, resulting in recurrent fever and progressive enlargement of the liver and spleen. Jam-like stools were noted 35 days after admission. B ultrasonography showed suspected intussusception. Laparotomy, reduction of intussusception and ileocecum angioplasty, biopsies of intestinal wall nodules and lymphoglandulae mesentericae, and hepatic biopsy were then performed under general anesthesia. The patient eventually died because of postoperative severe liver damage, disseminated intravascular coagulation and electrolyte disorder. Both the blood culture and hepatic biopsy tests showed Penicillium marneffei infecton. Immunodeficiency gene test was performed on the patient, his bother and their parents. T→G base substitution mutation (IVS1-3 T→G) in the CD40L gene was found in the patient. X-linked hyper-IgM syndrome was thus diagnosed in the patient. His mother was a carrier of the mutated CD40L gene, but his father was normal in the gene test. Hemizygous mutation in the CD40L gene was found in both the patient and his bother."}, {"id": "pubmed23n0533_4200", "title": "Bone and joint disease associated with primary immune deficiencies.", "score": 0.01138164063763055, "content": "Primary immune deficiencies (PIDs) are characterized by functional and/or quantitative abnormalities of one or more immune system components. Several bone and joint abnormalities can occur in patients with PID, with arthritis being the most common. Joint manifestations, of which arthritis is the most common, occur chiefly in humoral PIDs (agammaglobulinemia, common variable immunodeficiency, hyper-IgM syndromes, and IgA deficiency) and occasionally in other PIDs (chronic granulomatous disease and Wiskott-Aldrich syndrome). Monoarthritis or oligoarthritis is the usual pattern, although polyarthritis may occur, occasionally with nodules suggesting rheumatoid arthritis. Arthritis in patients with PID is usually infectious in nature, the most common causative organism being Mycoplasma, followed by Staphylococcus, Streptococcus, and Haemophilus. These bacteria can induce not only synovial infections, but also aseptic arthritogenic inflammatory responses. Arthritis having no demonstrable relation to chronic infection has been reported also and ascribed to dysimmunity-driven mechanisms that exhibit a number of specific features. Bone lesions are far less common and usually due to infections complicating humoral PID. Distinctive bone manifestations occur in a number of rare PIDs (e.g., hyper-IgE syndrome and Di George syndrome) and in syndromes characterized by spondyloepiphyseal dysplasia. Familiarity with PID syndromes both enhances the diagnostic capabilities of physicians and provides insight into the pathophysiology of bone and joint abnormalities associated with immune dysfunction. In children and occasionally in adults, a combination of bone and/or joint manifestations and hypogammaglobulinemia may indicate PID. When there is no evidence of lymphoproliferative disease, infection, or iatrogenic complications, investigations for PID should be obtained. PID-related arthritis is a unique model for studying the pathogenesis of presumably postinfectious arthritis and of inflammatory joint diseases including rheumatoid arthritis."}, {"id": "wiki20220301en186_26784", "title": "Immune disorder", "score": 0.011329652634000459, "content": "Primary immune deficiencies Severe combined immunodeficiency (SCID) DiGeorge syndrome Hyperimmunoglobulin E syndrome (also known as Job's Syndrome) Common variable immunodeficiency (CVID): B-cell levels are normal in circulation but with decreased production of IgG throughout the years, so it is the only primary immune disorder that presents onset in the late teens years. Chronic granulomatous disease (CGD): a deficiency in NADPH oxidase enzyme, which causes failure to generate oxygen radicals. Classical recurrent infection from catalase positive bacteria and fungi. Wiskott–Aldrich syndrome (WAS) Autoimmune lymphoproliferative syndrome (ALPS) Hyper IgM syndrome: X-linked disorder that causes a deficiency in the production of CD40 ligand on activated T-cells. This increases the production and release of IgM into circulation. The B-cell and T-cell numbers are within normal limits. Increased susceptibility to extracellular bacteria and opportunistic infections."}, {"id": "wiki20220301en121_8056", "title": "Hyper-IgM syndrome type 2", "score": 0.011214543861570475, "content": "Hyper IgM Syndrome Type 2 is a rare disease. Unlike other hyper-IgM syndromes, Type 2 patients identified thus far did not present with a history of opportunistic infections. One would expect opportunistic infections in any immunodeficiency syndrome. The responsible genetic lesion is in the AICDA gene found at 12p13. Hyper IgM syndromes Hyper IgM syndromes is a group of primary immune deficiency disorders characterized by defective CD40 signaling; via B cells affecting class switch recombination (CSR) and somatic hypermutation. Immunoglobulin (Ig) class switch recombination deficiencies are characterized by elevated serum IgM levels and a considerable deficiency in Immunoglobulins G (IgG), A (IgA) and E (IgE). As a consequence, people with HIGM have an increased susceptibility to infections."}, {"id": "First_Aid_Step2_928", "title": "First_Aid_Step2", "score": 0.010503705801822217, "content": "Combined (continued) Wiskott-Aldrich syndrome An X-linked disorder with less severe Band T-cell dysfunction. Patients have eczema, ↑ IgE/IgA, ↓ IgM, and thrombocytopenia. The classic presentation involves bleeding, eczema, and recurrent otitis media. ↑↑ risk of atopic disorders, lymphoma/leukemia, and infection from S. pneumoniae, S. aureus, and H. infl uenzae type b. Phagocytic Chronic granulomatous disease (CGD) Leukocyte adhesion def ciency Chédiak-Higashi syndrome An X-linked (2/3) or autosomal-recessive (1/3) disease with def cient superoxide production by PMNs and macrophages. Anemia, lymphadenopathy, and hypergamma-globulinemia may be present. A defect in the chemotaxis of leukocytes. An autosomal-recessive disorder that leads to Chronic skin, pulmonary, GI, and urinary tract infections; osteomyelitis and hepatitis. Infecting organisms are catalase . ↑ risk of infection with Aspergillus. May have granulomas of the skin and GI/GU tracts. Recurrent skin, mucosal, and pulmonary"}, {"id": "wiki20220301en477_12562", "title": "DOCK8 deficiency", "score": 0.010483870967741934, "content": "Diagnosis A diagnosis can only be definitively made after genetic testing to look for a mutation in the DOCK8 gene. However, it can be suspected with a high IgE level and eosinophilia. Other suggestive laboratory findings include decreased numbers of B cells, T cells, and NK cells; and hypergammaglobulinemia. It can be distinguished from autosomal dominant hyper-IgE (STAT3 deficiency) because people with DOCK8 deficiency have low levels of IgM and an impaired secondary immune response. IgG and IgA levels are usually normal to high. It can be distinguished from the similar X-linked Wiskott–Aldrich syndrome by the presence of thrombocytopenia and the consequent bloody diarrhea, as well as its pattern of inheritance. WHIM syndrome, caused by a mutation in CXCR4, is associated with similar chronic cutaneous viral infections."}]}}}} {"correct_option": 4, "explanations": {"1": {"exist": true, "char_ranges": [[0, 165]], "word_ranges": [[0, 28]], "text": "The confusional picture is determined by the dementia that the patient already suffers, so no matter how much we delay surgery, we are not going to achieve anything."}, "2": {"exist": false, "char_ranges": [], "word_ranges": [], "text": ""}, "3": {"exist": true, "char_ranges": [[166, 404]], "word_ranges": [[28, 68]], "text": "The elevation of blood pressure is due, in principle, to pain (so the first option is an analgesic) and then to the stressful situation that leads an already hypertensive patient to increase her blood pressure, so labetalol could help us."}, "4": {"exist": false, "char_ranges": [], "word_ranges": [], "text": ""}, "5": {"exist": true, "char_ranges": [[405, 637]], "word_ranges": [[68, 104]], "text": "The last answer is not correct because it depends on the type of fracture, which is not specified in the statement: a pertrochanteric fracture would require closed osteosynthesis and a subcapital fracture would require a prosthesis."}}, "full_answer": "The confusional picture is determined by the dementia that the patient already suffers, so no matter how much we delay surgery, we are not going to achieve anything. The elevation of blood pressure is due, in principle, to pain (so the first option is an analgesic) and then to the stressful situation that leads an already hypertensive patient to increase her blood pressure, so labetalol could help us. The last answer is not correct because it depends on the type of fracture, which is not specified in the statement: a pertrochanteric fracture would require closed osteosynthesis and a subcapital fracture would require a prosthesis.", "full_answer_no_ref": "The confusional picture is determined by the dementia that the patient already suffers, so no matter how much we delay surgery, we are not going to achieve anything. The elevation of blood pressure is due, in principle, to pain (so the first option is an analgesic) and then to the stressful situation that leads an already hypertensive patient to increase her blood pressure, so labetalol could help us. [HIDDEN] because it depends on the type of fracture, which is not specified in the statement: a pertrochanteric fracture would require closed osteosynthesis and a subcapital fracture would require a prosthesis.", "full_question": "We are consulted to assess an 83-year-old woman admitted to the Trauma service for a hip fracture 6 hours ago. She has AP of hypertension, LBP, moderate dementia and lives in a nursing home. Her usual treatment is thiazide, atorvastatin, donepezil, Calcium and vitamin D. EF: Confused patient, pulse 90 bpm, respiratory rate 20 rpm, T art 170/88, jugular venous pressure normal. The CBC and chest X-ray are normal and the ECG shows sinus rhythm without ischemic alterations. Which of the following is the most correct therapeutic approach?", "id": 62, "lang": "en", "options": {"1": "Delay surgery until the confusional picture has disappeared.", "2": "Delay surgery and perform an echocardiogram.", "3": "Delay surgery until good blood pressure control.", "4": "Start a beta-blocker and initiate surgery.", "5": "Perform closed osteosynthesis, avoiding in any case the implantation of prosthesis."}, "question_id_specific": 119, "type": "ANESTHESIOLOGY AND CRITICAL CARE", "year": 2011, "rag": {"clinical_case_options": {"MedCorp": {"RRF-2": [{"id": "Surgery_Schwartz_2833", "title": "Surgery_Schwartz", "score": 0.01129745417713629, "content": "an injection of 1% lidocaine solution around this structure should attenuate this reflexive response.The most common delayed complication following carotid endarterectomy remains myocardial infarction. The possibility of a postoperative myocardial infarction should be considered as a cause of labile blood pressure and arrhythmias in high-risk patients.Thyroid and Parathyroid Glands. Surgery of the thyroid and parathyroid glands can result in hypocalcemia in the immedi-ate postoperative period. Manifestations include ECG changes (shortened P-R interval), muscle spasm (tetany, Chvostek’s sign, and Trousseau’s sign), paresthesias, and laryngospasm. Treatment includes calcium gluconate infusion and, if tetany ensues, chemical paralysis with intubation. Maintenance treat-ment is thyroid hormone replacement (after thyroidectomy) in addition to calcium carbonate and vitamin D.Recurrent laryngeal nerve (RLN) injury occurs in less than 5% of patients. Of those with injury, approximately 10%"}, {"id": "pubmed23n0274_22316", "title": "[Anesthesia and intensive therapy for a patient with mitochondrial myopathy].", "score": 0.009900990099009901, "content": "Since 1983 we have been involved in the diagnostic work-up and emergency treatment of a female patient now 48 years old who has a mitochondrial myopathy resembling Luft's disease. The syndrome was first described in 1959, and in more detail in 1962, by Luft and et al., who reported a picture of hypermetabolism with high temperature, extreme sweating, tachycardia, dyspnoea at rest, polydipsia, polyphagia and irritability but normal thyroid function. In 1971 and 1976 Haydar and Di Mauro presented a second case and proposed treatment with chloramphenicol. Our patient has the third case of the syndrome reported so far: her case was initially published in 1987. CASE REPORT. Since her 17th year of life the patient had suffered from episodes of fever, tachycardia and sweating. At the age of 32 these attacks worsened, leading to unconsciousness and apnoea. The patient then had to be intubated, ventilated and sometimes resuscitated. The diagnosis of MH susceptibility and Luft's disease was made on biochemical grounds after the first muscle biopsy in 1983. Therapy with chloramphenicol failed. Therapy with beta blockers, vitamin C and K or E, coenzyme Q10 and a high-caloric diet was started in 1985. The patient was registered with an emergency service, which flew her to our ICU whenever she had a severe crisis. For milder episodes she was supplied with an oxygen breathing mask at home. Myalgia increased with the episodes starting in 1988, and the patient needed dantrolene infusions and analgesics at home. To facilitate venepuncture a Port-A-Cath system was implanted in 1987, which had to be removed four times due to infection and sepsis. A muscle biopsy was taken in Rotterdam, which revealed differences in mitochondrial function from the biochemical findings recorded in 1983 and not in keeping with Luft's disease. Unfortunately, the patient was not able to undergo further metabolic investigations or therapeutic trials. ANAESTHESIA. The patient received three local and six general anaesthetics in our clinic. The muscle biopsies, two in 1983 and one in 1985, were performed under local infiltration with procaine and were uneventful. The general anaesthetics were carried out without MH trigger substances following pretreatment with dantrolene for the following surgical procedures: the repair of an extensive arterio-venous fistula between the brachiocephalicus trunk and the right jugular and subclavian vein, revision of the sternum cerclage, implantations and explanations of infectious Port-A-Cath systems. We used etomidate, propofol and fentanyl or alfentanil with nitrous oxide and oxygen for induction and maintenance of anaesthesia. Muscle relaxation was induced with vecuronium or atracurium. All cardiovascular, respiratory, metabolic and temperature measurements stayed in normal ranges. After the extensive vascular repair (av fistula) the patient had to be mechanically ventilated for some hours until normal body temperature was restored. At the end of all other periods of anaesthesia she was extubated in the operating theatre. In five cases the postoperative period was uneventful. Only once she developed a crisis with hyperthermia, tachycardia, sweating and dyspnoea. INTENSIVE CARE. From 1985 to 1992 the patient was treated in our ICU 21 times. On 11 occasions she was already intubated and being ventilated by the emergency service on arrival. Extubation was usually possible within 2-20 h. During the crisis, heart rate was about 160-190 per minute and temperature above 40 degrees C. Serum values of CK, glucose, BUN, electrolytes, lactate and thyroid hormones were always in the normal ranges. Blood gas controls showed a constant respiratory alkalosis, arterial pCO2 values decreasing to 20 mm Hg or less. In addition to mechanical ventilation, treatment consisted in dantrolene infusions and droperidol injections, supplemented from 1989 onward with piritramide injections because of the increased severity of myalgia. In 1991 we gave propofol by"}, {"id": "pubmed23n0652_3528", "title": "[What's new in geriatric medicine].", "score": 0.009900990099009901, "content": "Several studies clarified the role of different interventions such as vitamine D replacement, denosumab treatment, and vertebroplasty in the prevention and management of falls and fractures. A trial tested the effectiveness of pharmaceutical assistance at the time of discharge, emphasizing the potential benefits for the patients and the health care system. Syncopal episodes frequently lead to hospital admission. A retrospective study evaluated the diagnostic yield of different tests and emphasized the importance to actively seek orthostatic hypotension in older patients. Finally, advances remain modest in the field of dementias."}, {"id": "pubmed23n0929_14420", "title": "Extracorporeal membrane oxygenation system as a bridge to reparative surgery in ventricular septal defect complicating acute inferoposterior myocardial infarction.", "score": 0.00980392156862745, "content": "Post-infarction ventricular septal defect (VSD) is a rare but potentially lethal complication of acute myocardial infarction. Medical management is usually futile, so definitive surgery remains the treatment of choice but the risk surgery is very high and the optimal timing for surgery is still under debate. A 55-year-old man with no previous medical history attended the emergency-room for 12 h evolution of oppressive chest pain and strong anginal pain 7 days ago. On physical examination, blood pressure was 96/70 mmHg, pansystolic murmur over left sternal border without pulmonary crackles. An electrocardiogram revealed sinus rhythm 110 bpm, elevation ST and Q in inferior-posterior leads. Transthoracic echocardiogram showed inferoposterior akinesia, posterior-basal septal rupture (2 cm × 2 cm) with left-right shunt. Suspecting VSD in inferior-posterior acute myocardial infarction evolved, we performed emergency coronarography with 3-vessels disease and complete subacute occlusion of the mid segment of the right coronary artery. Left ventriculography demonstrated shunting of contrast from the left ventricule to the right ventricule. He was rejected for heart transplantation because of his age. Considering the high surgical risk to early surgery and his hemodynamic and clinical stability, delayed surgical treatment is decided, and 4 days after admission the patient suffered hemodynamic instability so venoarterial extracorporeal membrane oxygenation system (ECMO) is implanted as a bridge to reparative surgery. The 9th day after admission double bypass, interventricular defect repair with pericardial two-patch exclusion technique, and ECMO decannulation were performed. The patient's postoperative course was free of complications and was discharged 10 days post VSD repair surgery. Follow-up 3-month later revealed the patient to be in good functional status and good image outcome with intact interventricular septal patch without shunt. ECMO as a bridge to reparative surgery in postinfarction VSD is an adequate option to stabilize patients until surgery."}, {"id": "pubmed23n0698_23113", "title": "[Patient whose surgery was postponed due to complete atrioventricular block on arrival at operating theater].", "score": 0.00980392156862745, "content": "An 86-year-old woman with low cardiac function was scheduled to undergo hip fracture surgery. Preoperative electrocardiogram showed complete left bundle brunch block, first degree atrioventricular block, left axis deviation and bigeminy. However, her electrocardiogram had changed to complete atrioventricular block on arrival at operating theater. ACC/AHA guideline on perioperative cardiovascular evaluation and care for non cardiac surgery indicates the assessment of both the urgency of the surgery and cardiac complications. Because complete atrioventricular block is classified to \"active cardiac conditions\", we decided to postpone the surgery for more detailed evaluation and treatment of cardiac conditions. In spite of the discontinuation of digoxin and carvegilol, complete atrioventricular block continued for a week, and the permanent pacemaker was inserted. The surgery was performed 2 weeks following the insertion of the pacemaker without any problems under combined general and lumbar epidural anesthesia."}, {"id": "wiki20220301en027_68001", "title": "Aneurysm of sinus of Valsalva", "score": 0.009708737864077669, "content": "Treatment Medical therapy of aneurysm of the aortic sinus includes blood pressure control through the use of drugs, such as beta blockers. Another approach is surgical repair. The determination to perform surgery is usually based upon the diameter of the aortic root (with 5 centimeters being a rule of thumb - a normal size is 2-3 centimeters) and the rate of increase in its size (as determined through repeated echocardiography). An alternative to surgical repair or a ruptured aneurysm is percutaneous closure. In this technique, a wire is introduced via a small incision in the groin and advanced through the vascular system to the aneurysm. A closure device is advanced along the wire before being expanded to straddle the site of rupture."}, {"id": "pubmed23n0945_6227", "title": "[Severe hypercalcemia of unusual cause, looking for the culprit: Case report and review of the literature].", "score": 0.009708737864077669, "content": "Hypercalcemia is not a rare event and can lead to severe consequences. Its main etiologies are primary hyperparathyroidism and neoplasic conditions. The iatrogenic etiology by vitamin D intoxication is more rarely found. A 76-year-old finish woman comes to the emergency room for chest pain. Her medical history is impossible to specify due to the language barrier and initial confusion. She has severe hypercalcaemia (4.14mmol/L), renal insufficiency, cardiac arrhythmia later complicated by an ischemic cardiac episode. Clinic and biologic examinations initially guided the research towards a hematological and neoplasic pathology. The iatrogenic etiology will be permitted by the contribution of details on its medical history and treatment learnt secondly. She was treated for post-surgical hypoparathyroidism by dihydrotachysterol, a vitamin D derivative. The cessation of substitution, treatment with hydration and biphosphonates allowed the rapid correction of hypercalcemia. Dihydrotachysterol intoxication is a rare etiology of hypercalcemia. Because of the longer half-life of this molecule, the risk of hypercalcemia seems to be greater than with other vitamin D derivatives. This molecule, withdrawn from the French market in 1982, is not detected by the dosage of 25 and 1.25 OH vitamin D. We report an original case of intoxication by dihydrotachysterol. The risk of hypercalcemia encountered with this molecule must be known. The close medical follow-up recommended in case of hypoparathyroidism seems to be particularly necessary in case of supplementation by this molecule."}, {"id": "pubmed23n0609_14977", "title": "Cardiotoxicity after massive amantadine overdose.", "score": 0.009615384615384616, "content": "Amantadine hydrochloride is an antiviral medication used as therapy for parkinsonism and as a cognitive enhancer. We report 2 cases of massive, acute ingestion of amantadine hydrochloride confirmed with serial serum levels. A 47-year-old woman presented to the emergency department (ED) 30 minutes after ingesting 10 g of amantadine (150 mg/kg) by her report. Initial ECG revealed a sinus rhythm with rate of 93 bpm, and a QRS of 84 msec. While in the ED, the patient sustained a pulseless cardiac arrest and the monitor revealed ventricular tachycardia. She was successfully defibrillated. Postdefibrillation ECG showed a sinus rhythm (rate = 82 bpm), QRS of 236 msec, and QTc of 567 msec. The serum potassium was 1.0 mEq/L (1.0 mmol/L). The patient was given 300 ml (300 cc) 3% sodium chloride IV over 10 minutes. Ten minutes after completion of the hypertonic saline infusion, the patient's ECG abnormalities resolved and the QRS was 88 msec. Her potassium was repleted over the next 11 hours postpresentation, and she also received an IV bolus of 4 g of magnesium sulfate immediately after the cardiac arrest. No further hypotension, dysrhythmia, conduction delay, or ectopy was noted during the patient's hospital stay. The second case involved a 33-year-old female patient who presented 1 hour after ingesting 100 tablets of amantadine hydrochloride (100 mg/tab). Initial ECG revealed sinus tachycardia with a QRS of 113 msec, an R wave in lead aVR of 4-5 mm and a QTc of 526 msec. Her serum potassium was 3.0 mEq/L (3.0 mmol/L), her serum calcium was 9.4 mg/dl (2.35 mmol/L), and serum magnesium was 2.1 mg/dl (0.86 mmol/L) on labs drawn at initial presentation. The patient was intubated for airway protection, and her potassium was repleted and corrected over the next 9 hours. Her ECG abnormalities improved 8 hours after initial presentation and normalized at approximately 14 hours postingestion. The patient was discharged home 11 days after her ingestion. Acute amantadine toxicity manifests with life-threatening cardiotoxicity. Concurrent, often profound, hypokalemia may complicate the administration of sodium bicarbonate in the management of cardiac dysrhythmias."}, {"id": "pubmed23n0597_23565", "title": "[Interaction of calcium drug and vitamin D3 with some medicines used in coronary heart disease therapy].", "score": 0.009615384615384616, "content": "130 young and middle age patients of both sexes with chronic form of coronary heart disease: functional class II-III stable exertional angina pectoris including functional class I-III chronic cardiac insufficiency were studied. In protocol 1 cured 70 patients (48 (68.6%) males and 22 (31.4%) females) 32-59 years of age (medium age was 48.4 +/- 3.25 years) with coronary heart disease. In protocol 2 (with prescription of calcium-D3) cured 60 patients (40 (66.7%) males and 20 (33.3%) females) 34-58 years of age (medium age was 47.8 +/- 3.12 years) with coronary heart disease. The groups were comparable on key parameters of disease. All patients had alimentary calcium deficit and (or) risk factors of osteoporosis, instrumental signs (X-ray filming and densitometry) of initial or evident osteoporosis. Correction of alimentary calcium deficit was realized by prescription of 1-3 tablets of calcium- Ds in different food intakes. Positive dynamics in decrease of functional class of angina pectoris and nitroglycerin requirement in both groups was noticed. Negative influence of calcium- D3 on studied indices of coronary heart disease severity was absent. The thirst and dry mouth in patients, who took furosemide, in group 1 were noticed against the background of body weight decrease (p < 0.05) and increase of diuresis. Decrease of the therapy antiarrhythmic action (p < 0.05) in patients, who took hydrochlorothiazide, was noticed too. It leaded to needs of furosemide and hydrochlorothiatide dose correction in protocol 1. In whole use of calcium- D3 together with anti-ischemic drugs in patients with chronic forms of coronary heart disease did not impair clinical course of angina pectoris and did not decrease efficiency of coronary heart disease therapy."}, {"id": "wiki20220301en047_45308", "title": "Hip fracture", "score": 0.009523809523809525, "content": "Most hip fractures are treated surgically by implanting a prosthesis. Surgical treatment outweighs the risks of nonsurgical treatment which requires extensive bedrest. Prolonged immobilization increases risk of thromboembolism, pneumonia, deconditioning, and decubitus ulcers. Regardless, the surgery is a major stress, particularly in the elderly. Pain is also significant, and can also result in immobilization, so patients are encouraged to become mobile as soon as possible, often with the assistance of physical therapy. Skeletal traction pending surgery is not supported by the evidence. Regional nerve blocks are useful for pain management in hip fractures. Peripheral nerve blocks may reduce pain on movement and acute confusional state, may improve time to first mobilisation, and may reduce the risk of postoperative lower respiratory tract infection. Surgery can be performed under general anaesthesia or with neuraxial techniques – choice is based on surgical and patient factors, as"}, {"id": "pubmed23n0037_938", "title": "[Hypo and hypercalcemia as an emergency].", "score": 0.009523809523809525, "content": "1. Hypo- and hypercalcemia can be explained as derangements of the calcium homeostasis. Hypocalcemic tetany usually alarming the patient tremendously is, at least in adults, rarely life-threatening. Hypercalcemia leads in 30% of the cases to clinical symptoms which may inadvertedly pass into a state of hypercalcemic crisis. This latter requires an often difficult emergency treatment. 2. Hypocalcemic tetany may be reversed by administering calcium i.v. or, in severe cases, by a calcium infusion. Only rarely are magnesium supplements necessary to let the tetany disappear. Vitamin D or dihydrotachysterol (DHT) do not correct hypocalcemia immediately, since their effects may be delayed up to 15-25 days. In order to normalize the serum calcium permanently, vitamin D or DHT treatment should be instituted as rarely as possible. 3. Initially, hypercalcemic crisis is best treated by forced intravenous fluid administration with normal saline (and furosemide) in combination with high doses of prednisone. Fluid-, sodium- and potassium balances ought to be checked during this type of treatment. A first evaluation of the effectiveness of these measures is recommended after 24 hours: treatment is continued in patients who respond favorably, while subjects who do not show a significant decrease of the serum calcium may either be given a phosphate infusion or mithramycine as a bolus. Calcitonin appears to be useful only to start treatment before institution of a phosphate infusion."}, {"id": "pubmed23n0651_7761", "title": "[Case of ischemic heart disease resulting from persistent diuresis after giant ovarian tumor resection].", "score": 0.009433962264150943, "content": "A patient with a giant ovarian tumor weighing about 7 kg was successfully removed by operation. However, her ECG demonstrated ischemic changes after the operation. We report a case of ischemic heart disease due to persistent diuresis after giant ovarian tumor resection. A 75-year-old, 56.5 kg, 143.5 cm woman was admitted to our hospital for ovarian tumor resection. The preoperative ECG showed normal sinus rhythm and no ischemic changes. Both general anesthesia and epidural anesthesia were planed. An epidural catheter was inserted at T12-L1. Anesthesia was induced with propofol 100 mg, fentanyl 100 microg and vecuronium 8 mg under 100% oxygen inhalation. General anesthesia was maintained with sevoflurane while epidural anesthesia was achieved using 0.375% ropivacaine 6 ml. During the operation, blood pressure was 90-110/70-80 mmHg, with SaO2, 100% and heart rate, 70-80 beats x min(-1). The content of tumor was suctioned for 30 minutes. Surgery was successfully finished without any other incidence. After extubation, her ECG changed to atrial fibrillation from normal sinus rhythm and showed ST-T depression. And then her systolic blood pressure became 80 mmHg or below, but we found continued diuresis at about 10 ml x kg(-1) x hr(-1) for over 2 hr. The total of 7 unit vasopressin was intermittently given for vasoconstriction and antidiuresis. Her hemodynamic was immediately restored, and ECG turned to normal ST-T. The patient had uneventful postoperative recovery."}, {"id": "pubmed23n0933_8774", "title": "[Geriatric medicine].", "score": 0.009433962264150943, "content": "2017 highlights benefits of prevention. Better control of cardiovascular risk reduces the incidence of dementia and monthly high-dose vitamin D the incidence of respiratory infections in nursing home. Pre-operative geriatric assessment lowers by 20% the rate of delirium after hip-fracture surgery and complications in vascular surgery. Deleterious effects are also reported. High-dose vitamin D triples the rate of falls in supplemented residents and doesn't improve gait speed in sedentary men. Widely used in cardiovascular prevention, antithrombotic therapy is associated with an astonishing risk of subdural bleeding that further increases with the number of drugs combined together. Finally, the non-pharmacological management of behavioral and psychotic symptoms in advanced dementia, although effective, doesn't reduce the associated burden for proxies."}, {"id": "pubmed23n1094_2769", "title": "The Young Heart Tears Easily Apart: A Case Report of Spontaneous Coronary Artery Dissection.", "score": 0.009345794392523364, "content": "Spontaneous coronary artery dissection (SCAD) is a rare cause of acute coronary syndrome (ACS), seen mostly in young females. The rarity and limited knowledge of the disease make its management challenging. Prompt diagnosis of the condition is extremely important to decrease both long- and short-term complications. Treatment options depend on hemodynamic stability and the location of the dissection- with more distal lesions treated more conservatively as opposed to proximal lesions which are treated with percutaneous coronary intervention (PCI) or coronary artery bypass graft (CABG). The following are the two cases with different presentation, management and outcomes. Our first patient was a 35-year-old woman with no medical history who presented with acute, anginal pain, diaphoresis and palpitations. She was hemodynamically stable on presentation, with work-up significant for electrocardiogram (ECG) with sinus bradycardia, ST elevation in leads V1-V6, and elevated troponin level of 4 ng/ml. There was no evidence of a pulmonary embolism on computed tomography (CT) of the chest. A coronary angiogram showed 100% dissection of the proximal to mid-left anterior descending (LAD) artery. Attempts to place a stent in the proximal to mid LAD were unsuccessful as the true lumen of the LAD was not accessible. The patient became hemodynamically unstable, and an emergent CABG was done, restoring blood flow. The patient recovered during her hospital stay and was discharged with dual antiplatelet therapy (DAPT), beta-blockers, and atorvastatin. The second patient was a 28-year-old woman, with a history of hypertension who presented with anginal chest pain. Workup showed ECG with minimal ST elevations in anteroseptal leads, with elevated troponin level to 0.71 ng/ml. Coronary angiogram showed 40-50% stenosis of the mid LAD with an aneurysmal segment. An echocardiogram showed no evidence of wall motion abnormalities, and she had a normal left ventricular ejection fraction (LVEF). She was discharged home the next day, on medical management. After two days, she returned to the hospital with similar complaints, with work-up significant for ECG with non-specific ST-T abnormality, and troponin level which peaked at 2.22 ng/ml. She was started on a heparin drip, and a repeat left heart catheterization revealed type 2 dissection of the mid to distal LAD, with intravascular ultrasound showing a fractional flow reserve of 0.76. She was discharged home on DAPT, beta-blocker, calcium channel blocker (CCB), and atorvastatin, with close cardiology follow up. These two cases highlight the importance of keeping in mind the possibility of SCAD, especially when relatively healthy young women present with anginal symptoms. Early diagnosis of the condition and prompt management are extremely important to ensure favourable outcomes. The two cases also describe the coronary angiogram findings in SCAD, and the different strategies employed in the management of this condition."}, {"id": "pubmed23n0070_2240", "title": "[The parathyroid risk in thyroid surgery. Argument against the early postoperative prescription of vitamin D. Experience with 729 thyroidectomies in 1988].", "score": 0.009345794392523364, "content": "729 consecutive patients underwent thyroidectomy in 1988 in the same institution, including 477 (68%) bilateral resections and 242 (33%) total thyroidectomies. An effort was made to see and save all 4 parathyroids and their blood supply. Early post-operative hypoparathyroidism was defined at day 5, by serum calcium less than 8 mg/dl. and serum phosphate less than 4 mg/dl or by serum calcium only if greater than 7.5 mg/dl. Patients afflicted with early hypoparathyroidism were given calcium tablets without any vit D for 1 year at most. Follow-up, checking serum Ca, P and i PIH was done on a 3 months basis during 1 year. Permanent hypoparathyroidism was defined by persistence of the above-mentioned criteria after 1 year, and eventually vit D was started. 27 patients (5.6% our of 477 bilateral thyroid resections) experienced early post-op hypoparathyroidism. Inciting factors were previous thyroid surgery (4), radioiodine treatment (2), modified neck dissection (2), sternal split with mediastinal node clearance (1), visualization of 1 parathyroid gland only (3 redo cases) and autotransplantation of more than 1 parathyroid (1 case). 1 patient was lost for follow-up. 25 others recovered a normal parathyroid function. 1 is permanently hypoparathyroid (1 redo case with other risk factors). Painstaking parathyroid dissection allows a 0% rate of permanent hypoparathyroidism after primary surgery, if vit D is not given in the early post-operative period. We suggest that avoidance of early vit D prescription in cases of early post-operative hypoparathyroidism, leading to mild sustained hypocalcemia, stimulates the spared parathyroid glands (including a possible 5th) and therefore allows full recovery of the parathyroid function."}, {"id": "pubmed23n0800_14851", "title": "Breathlessness with pulmonary metastases: a multimodal approach.", "score": 0.009259259259259259, "content": "Case Study  Sarah is a 58-year-old breast cancer survivor, social worker, and health-care administrator at a long-term care facility. She lives with her husband and enjoys gardening and reading. She has two grown children and three grandchildren who live approximately 180 miles away. SECOND CANCER DIAGNOSIS  One morning while showering, Sarah detected a painless quarter-sized lump on her inner thigh. While she thought it was unusual, she felt it would probably go away. One month later, she felt the lump again; she thought that it had grown, so she scheduled a visit with her primary care physician. A CT scan revealed a 6.2-cm soft-tissue mass in the left groin. She was referred to an oncologic surgeon and underwent an excision of the groin mass. Pathology revealed a grade 3 malignant melanoma. She was later tested and found to have BRAF-negative status. Following her recovery from surgery, Sarah was further evaluated with an MRI scan of the brain, which was negative, and a PET scan, which revealed two nodules in the left lung. As Sarah had attended a cancer support group during her breast cancer treatment in the past, she decided to go back to the group when she learned of her melanoma diagnosis. While the treatment options for her lung lesions included interleukin-2, ipilimumab (Yervoy), temozolomide, dacarbazine, a clinical trial, or radiosurgery, Sarah's oncologist felt that ipilimumab or radiosurgery would be the best course of action. She shared with her support group that she was ambivalent about this decision, as she had experienced profound fatigue and nausea with chemotherapy during her past treatment for breast cancer. She eventually opted to undergo stereotactic radiosurgery. DISEASE RECURRENCE  After the radiosurgery, Sarah was followed every 2 months. She complained of shortness of breath about 2 weeks prior to each follow-up visit. Each time her chest x-ray was normal, and she eventually believed that her breathlessness was anxiety-related. Unfortunately, Sarah's 1-year follow-up exam revealed a 2 cm × 3 cm mass in her left lung, for which she had a surgical wedge resection. Her complaints of shortness of breath increased following the surgery and occurred most often with anxiety, heat, and gardening activities, especially when she needed to bend over. Sarah also complained of a burning \"pins and needles\" sensation at the surgical chest wall site that was bothersome and would wake her up at night. Sarah met with the nurse practitioner in the symptom management clinic to discuss her concerns. Upon physical examination, observable signs of breathlessness were lacking, and oxygen saturation remained stable at 94%, but Sarah rated her breathlessness as 7 on the 0 to 10 Borg scale. The nurse practitioner prescribed duloxetine to help manage the surgical site neuropathic pain and to assist with anxiety, which in turn could possibly improve Sarah's breathlessness. Several nonpharmacologic modalities for breathlessness were also recommended: using a fan directed toward her face, working in the garden in the early morning when the weather is cooler, gardening in containers that are at eye level to avoid the need to bend down, and performing relaxation exercises with pursed lip breathing to relieve anxiety-provoked breathlessness. One month later, Sarah reported relief of her anxiety; she stated that the fan directed toward her face helped most when she started to feel \"air hungry.\" She rated her breathlessness at 4/10 on the Borg scale. SECOND RECURRENCE: MULTIPLE PULMONARY NODULES  Sarah's chest x-rays remained clear for 6 months, but she developed a chronic cough shortly before the 9-month exam. An x-ray revealed several bilateral lung lesions and growth in the area of the previously resected lung nodule. Systemic therapy was recommended, and she underwent two cycles of ipilimumab. Sarah's cough and breathlessness worsened, she developed colitis, and she decided to stop therapy after the third cycle. In addition, her coughing spells triggered bronchospasms that resulted in severe anxiety, panic attacks, and air hunger. She rated her breathlessness at 10/10 on the Borg scale during these episodes. She found communication difficult due to the cough and began to isolate herself. She continued to attend the support group weekly but had difficulty participating in conversation due to her cough. Sarah was seen in the symptom management clinic every 2 weeks or more often as needed. No acute distress was present at the beginning of each visit, but when Sarah began to talk about her symptoms and fear of dying, her shortness of breath and anxiety increased. The symptom management nurse practitioner treated the suspected underlying cause of the breathlessness and prescribed oral lorazepam (0.5 to 1 mg every 6 hours) for anxiety and codeine cough syrup for the cough. Opioids were initiated for chest wall pain and to control the breathlessness. Controlled-release oxycodone was started at 10 mg every 12 hours with a breakthrough pain (BTP) dose of 5 mg every 2 hours as needed for breathlessness or pain. Sarah noted improvement in her symptoms and reported a Borg scale rating of 5/10. Oxygen therapy was attempted, but subjective improvement in Sarah's breathlessness was lacking. END OF LIFE  Sarah's disease progressed to the liver, and she began experiencing more notable signs of breathlessness: nasal flaring, tachycardia, and restlessness. Opioid doses were titrated over the course of 3 months to oxycodone (40 mg every 12 hours) with a BTP dose of 10 to 15 mg every 2 hours as needed, but her breathlessness caused significant distress, which she rated 8/10. The oxycodone was rotated to IV morphine continuous infusion with patient-controlled analgesia (PCA) that was delivered through her implantable port. This combination allowed Sarah to depress the PCA as needed and achieve immediate control of her dyspneic episodes. Oral lorazepam was also continued as needed. Sarah's daughter moved home to take care of her mother, and hospice became involved for end-of-life care. As Sarah became less responsive, nurses maintained doses of morphine for control of pain and breathlessness and used a respiratory distress observation scale to assess for breathlessness since Sarah could no longer self-report. A bolus PCA dose of morphine was administered by Sarah's daughter if her mother appeared to be in distress. Sarah died peacefully in her home without signs of distress. "}, {"id": "InternalMed_Harrison_28527", "title": "InternalMed_Harrison", "score": 0.009259259259259259, "content": "use calcitriol (doses of 0.5–1 μg/d) because of the rapidity of onset of effect and prompt cessation of action when stopped, in comparison to other forms of vitamin D. A rise in blood calcium after several months of vitamin D replacement may indicate restoration of parathyroid function to normal. It is also appropriate to monitor serum PTH serially to estimate gland function in such patients."}, {"id": "pubmed23n1140_22727", "title": "Atypical Presentation of Interval Colorectal Cancer/Post-Colonoscopy Colorectal Cancer in a Nursing Home Patient.", "score": 0.009174311926605505, "content": "The Centers for Disease Control and Prevention estimates that there are around 1.7 million beds in certified nursing homes across the United States and approximately 1.3 million residents in long-term and end-of-life care. There could be several factors causing a delayed recovery in such patients, such as decreased ambulation, multiple comorbidities, and polypharmacy. An 83-year-old Caucasian woman sustained a fall resulting in compression fractures of the thoracic and lumbar spine. She had multiple comorbidities, including anemia of chronic disease, malnutrition, and a significant weight loss of 30 lbs over the four months prior to hospitalization. She was on antihypertensives, antidepressants, vitamin D, and calcium supplementation. Her medical history was significant for constipation with the passage of stools once in three days. Her family history was significant for colorectal cancer (CRC) and her screening colonoscopy three years ago was normal. Physical examination revealed no abdominal tenderness or distention. Subsequently, she developed edema in the left lower extremity. She underwent a venous Doppler/ultrasound study, which showed an occlusive thrombus from the common femoral vein to the popliteal vein. She was started on anticoagulants and supportive therapy. Four months later, while at the nursing home, she developed bloating and flatulence, in addition to pre-existing constipation. Examination revealed a 6 x 7 cm mass in the right lower quadrant without peritoneal signs. Bowel sounds were significantly decreased. CT imaging showed a 6-cm diameter cecal mass. The tumor was a low-grade 4 x 9 cm T4N0M0 cecal cancer, and she underwent placement of a Greenfield filter and subsequent hemicolectomy. She had methicillin-resistant 2–3 days, therapy with a vitamin D analogue and/or oral calcium (2–4 g/d) should be started (see below). It is cost-effective to use calcitriol (doses of 0.5–1 μg/d) because of the rapidity of onset of effect and prompt cessation of action when stopped, in comparison to other forms of vitamin D. A rise in blood calcium after"}, {"id": "pubmed23n0827_6236", "title": "Iatrogenic aortic insufficiency following mitral valve replacement: case report and review of the literature.", "score": 0.008928571428571428, "content": "We report a 28-year-old white female who suffered significant aortic insufficiency (AI) following mitral valve (MV) replacement for endocarditis. The patient had history of rheumatoid arthritis and presented to our emergency department with a 3-month history of dyspnea, orthopnea, fevers and weight loss, worsening over 2 weeks, for which she took intermittent acetaminophen. On admission, vital signs revealed blood pressure of 99/70 mm Hg, heart rate of 120 beats/minute, and temperature of 98.8 °F; her weight was 100 lbs. Physical exam revealed a thin and pale female. Cardiac auscultation revealed regular tachycardic rhythm with a third heart sound, and a short early systolic murmur at the left lower sternal border without radiation. Lungs revealed right lower lobe rhonchi. Initial pertinent laboratory evaluation revealed hemoglobin 9.6 g/dL and white blood cell count 17,500/μL. Renal function was normal, and hepatic enzymes were mildly elevated. Chest radiogram revealed right lower lobe infiltrate. Blood cultures revealed Enterococcus faecalis. Two-dimensional echocardiogram revealed large multilobed vegetation attached to the anterior MV leaflet with severe mitral regurgitation (MR), otherwise normal left ventricular systolic function. She was started on appropriate antibiotics and underwent MV replacement with 25-mm On-X prosthesis. She was noted post-operatively to have prominent systolic and diastolic murmurs. Repeat echocardiogram revealed normal mitral prosthesis function, with new moderately severe AI. Transesophageal echocardiogram revealed AI originating from a tethered non-coronary cusp, due to a suture preventing proper cusp mobility. The patient declined further surgery. She recovered slowly and was discharged to inpatient rehabilitation 4 weeks later. This case highlights the importance of vigilance to this potential serious complication of valve surgery with regard to diagnosis and treatment to prevent long-term adverse consequences. "}, {"id": "InternalMed_Harrison_17969", "title": "InternalMed_Harrison", "score": 0.008928571428571428, "content": "TREATMEnT ManageMent of aV conduction Block"}, {"id": "pubmed23n0618_7426", "title": "Delayed vascular injury and severe respiratory distress as a rare complication of a central venous catheter and total parenteral nutrition.", "score": 0.008849557522123894, "content": "Complications related to central venous catheters (CVCs) in the postoperative period can be fatal. We recently had a case of bilateral pleural effusion and respiratory distress caused by delayed vascular injury. A 79-y-old Japanese woman was admitted to our hospital because of advanced gastric carcinoma. A multiple-lumen CVC was placed through the left subclavian vein 1 d before surgery for postoperative nutritional management. The patient suddenly complained of dyspnea, and the chest X-ray film revealed right massive pleural effusion. Although the patient's symptoms soon disappeared after the thoracentesis, she again developed severe respiratory distress, and an endotracheal intubation was performed and her respiration was managed by mechanical ventilation. Computed tomographic scan of the chest revealed a displacement of the tip of the CVC out of the wall of the superior vena cava, mediastinitis, and leakage of intravenous fluid, which may have been caused by delayed vascular injury due to the CVC. The CVC was removed immediately after the diagnosis of delayed vascular injury at 10 d after surgery. The patient soon recovered with conservative treatment and was discharged from the hospital 43 d after surgery. This case highlights an extremely rare presenting complication of CVC placement and total parenteral nutrition."}, {"id": "pubmed23n0210_9490", "title": "Differential diagnosis of hypercalcaemia and indications for treatment.", "score": 0.008849557522123894, "content": "Primary HPT is a common medical problem of the middle-aged and elderly. The diagnosis is generally simple and seldom requires elaborate studies. The precise consequences of mild-to-moderate virtually asymptomatic disease are not clarified and provided there is regular follow-up attendance expectancy may be justified. In most patients, however, the experienced surgeon will have a more than 90% chance of success at the first exploration, which for the patient is a fairly safe procedure."}, {"id": "pubmed23n0721_1962", "title": "Indications and results for the Exogen™ ultrasound system in the management of non-union: a 59-case pilot study.", "score": 0.008771929824561403, "content": "This pilot series sought to assess the use of external ultrasound stimulation (Exogen™) in the treatment of femoral or tibial non-union. A continuous retrospective study was conducted from 2004 to 2009. It included patients with a non-united fracture or osteotomy at 6 months or more post-surgery, with less than 10mm inter-fragment gap. Daily 20-min ultrasound sessions were continued until bone healing was achieved or for a maximum 6-month duration. Radio-clinical control was performed at months 3 and 6; treatment compliance and transmitter positioning were checked at each follow-up visit. Sixty non-unions were included in the series. One patient was excluded for early material breakage. Mean fracture-to-surgery interval was 271 days. The 6-month consolidation rate was 88%. There was no loss to follow-up. Mean ultrasound treatment duration was 151 days (range, 90-240 days). Bone healing correlated significantly with stability of the internal fixation assembly (P=0.01). The seven cases of failure included four fixations,considered unstable at inclusion, one femoral non-union associated with BMI 45 and one inadequate subchondral roughening (at the time of arthrodesis). There was a significant difference in delay to non-union treatment start between the groups with (251 days) and without (420 days) bone healing. The present results are in line with the literature. The main prognostic factors were fracture fixation stability, short time to treatment, and inter-fragment gap less than 10mm. Bone healing rates in the literature are around 80% for non-union treated at around 6 months, versus 60% for more than 12 months' delay. Factors such as gender, bone site, smoking, numbers of previous operations or type of osteosynthesis do not impact consolidation. External treatment offers an alternative to traditional surgery (graft, or bone-marrow concentrate or bone morphogenetic protein injection), provided that the fracture fixation is stable. Bone healing rates are better, and the procedure is non-invasive. External treatment results using ultrasound are similar to those using electromagnetic fields; the main difference lies in treatment session duration, which is 20 min for ultrasound, versus 3 hours for electromagnetic fields. Active patient commitment is vital, as the treatment is delivered at home, although the machine is equipped with a monitor to count treatment cycles. The 88% bone healing rate supports advocating first-line implementation in non-union of less than 10mm with stable osteosynthesis. This rate is higher than in traditional surgery, with a unit cost at least 60% lower: €1772 for external therapy, versus €4480 for decortication with or without fracture fixation exchange (itemized 08c50 under the French healthcare treatment coding system). Level IV. Retrospective therapeutic study."}, {"id": "pubmed23n0746_21359", "title": "Reducing cardiovascular mortality in chronic kidney disease: something borrowed, something new.", "score": 0.008771929824561403, "content": "CLINICAL VIGNETTE: A 48-year-old man with chronic kidney disease stage five due to type II diabetes mellitus and hypertension was referred for hemodialysis initiation. His physical exam showed a blood pressure of 150/80, normal fundi, a positive fourth heart sound (S4), and trace pedal edema. Moderate aortic calcification was present on prior chest X-ray. The ECG showed left ventricle hypertrophy by voltage and slight prolongation of the QT interval. Medications included chlorthalidone, amlodipine, carvedilol, cholecalciferol, erythropoietin, and a phosphate binder. What additional therapy should be initiated to reduce vascular calcifications and cardiovascular mortality?"}, {"id": "wiki20220301en200_11479", "title": "Meniscus tear", "score": 0.008695652173913044, "content": "There are three phases that follow meniscal surgery. Each phase consists of rehabilitation goals, exercises, and criteria to move on to the next phase. Phase I starts immediately following surgery to 4–6 weeks or until the patient is able meet progression criteria. The goals are to restore normal knee extension, reduce and eliminate swelling, regain leg control, and protect the knee (Fowler, PJ and D. Pompan, 1993). During the first 5 days following the surgery, a passive continuous motion machine is used to prevent a prolonged period of immobilization which leads to muscular atrophy and delays functional recovery. During the 4–6 weeks post-surgical, active and passive non-weight bearing motions which flex the knee up to 90° are recommended. For patients with meniscal transplantation, further knee flexion can damage the allograft because of the increased shear forces and stresses. If any weight-bearing exercises are applied, a controlled brace should be worn on the knee to keep the"}, {"id": "InternalMed_Harrison_21076", "title": "InternalMed_Harrison", "score": 0.008695652173913044, "content": "500–800 mg, followed by continuous infusion at 2–5 mg/min) is now rarely used in this setting but may be tried for persisting, hemodynamically stable arrhythmias. Intravenous calcium gluconate is no longer considered safe or necessary for routine administration. It is used only in patients in whom acute hyperkalemia is known to be the triggering event for resistant VF, in the presence of known hypocalcemia, or in patients who have received toxic doses of calcium channel antagonists. Cardiac arrest due to bradyarrhythmias or asystole (B/A cardiac arrest) is managed differently (Fig. 327-3B). The patient is promptly intubated, CPR is continued, and an attempt is made to control hypoxemia and acidosis and identify other reversible causes. Epinephrine may be given intravenously or by an intraosseous route. Atropine is no longer considered effective for asystole or PEA, but can be used for bradyarrhythmias. External pacing devices are used to attempt to establish a regular rhythm when"}, {"id": "pubmed23n0277_19971", "title": "[A case of spontaneous rupture of the ascending aorta].", "score": 0.008620689655172414, "content": "We report a rare case of spontaneous rupture of the ascending aorta without any evidence of aneurysm formation or aortic dissection. A woman aged 64 was admitted to our cardiac care unit as an emergency patient with severe chest pain. Her face was pale and systolic blood pressure was 70 mmHg in spite of intravenous administration of dopamine (10 micrograms/kg/min). She had a history of hypertension for two years under good medical control. No trace of the chest trauma was noted before her admission. Physical examination revealed neck vein engorgement and distant heart sounds. Chest X-ray film showed enlargement of the cardiac silhouette. ECG showed no evidence of acute coronary syndrome. Pericardial effusion with a floating hematoma-like mass was detected by 2-dimensional echocardiogram. Pericardiocentesis revealed bloody pericardial fluid (Ht: 26%). Aortagraphy was performed resulting in a clinical diagnosis of acute aortic dissection, but there were no signs of a false lumen, aneurysm formation or extravasation of the contrast medium. Although continuous pericardial drainage was performed, she suddenly lost consciousness, collapsed and died. A longitudinal intimal laceration 5 cm long was observed in the ascending aorta. Pathological examination revealed cystic medial necrosis and irregularity of the elastic fibers in the media. No atheromatous plaque was noted in the intima. Spontaneous rupture of the aorta is a life-threatening condition that requires urgent surgery.(ABSTRACT TRUNCATED AT 250 WORDS)"}, {"id": "pubmed23n0755_2271", "title": "Vitamin D toxicity presenting as hypercalcemia and complete heart block: An interesting case report.", "score": 0.008620689655172414, "content": "Vitamin D deficiency is widely prevalent across the globe. This has lead to widespread use of vitamin D supplements in populations. We present our experience of vitamin D toxicity in a subject resulting in hypercalcemia and CHB (Complete Heart Block). A 70-year-old female, known hypertensive for thirty five years and diabetic for seven years underwent total knee replacement (TKR) for osteoarthritis left knee in December 2010. For perioperative glycemic control, multiple subcutaneous injections of insulin were advised. Patient later presented with poor glycemic control, decreased appetite and constipation for last 1 month with history of episodes of transient loss of consciousness for 15 days and recurrent vomiting. Biochemical work-up showed hypercalcemia (Serum calcium 12.4 mg/dL). Sr. albumin, ALP, Sr. phosphorus and PTH levels were normal, thus suggesting PTH independent hypercalcemia. Strong suspicion led us to check vitamin D levels in dilution which were 2016 ng/mL, thus confirming vitamin D toxicity. Retrospective analysis of treatment history revealed patient receiving 4 injections of Architol (6 Lac units im) prior to presentation. Work-up for malignancy was negative, brain imaging and EEG were normal. Holter was suggestive of intermittent CHB. Patient was given hydration, injection calcitonin 100 I.U. subcutaneously, injection pamidronate 60 mg infusion, with serum calcium levels normalizing, with relief in constipation, vomiting and behavioral improvement. However, persistence of rhythm disturbances led to permanent pacemaker placement. The present case highlights the dangers of indiscriminate vitamin D usage, exposing patients to potentially life threatening complications."}]}}}} {"correct_option": 3, "explanations": {"1": {"exist": false, "char_ranges": [], "word_ranges": [], "text": ""}, "2": {"exist": false, "char_ranges": [], "word_ranges": [], "text": ""}, "3": {"exist": true, "char_ranges": [[0, 83]], "word_ranges": [[0, 14]], "text": "BI-RADS Breast Imaging Reporting and Data System. BI-RADS 3 is defined by answer 3."}, "4": {"exist": false, "char_ranges": [], "word_ranges": [], "text": ""}, "5": {"exist": false, "char_ranges": [], "word_ranges": [], "text": ""}}, "full_answer": "BI-RADS Breast Imaging Reporting and Data System. BI-RADS 3 is defined by answer 3.", "full_answer_no_ref": "BI-RADS Breast Imaging Reporting and Data System. BI-RADS 3 is defined by answer [HIDDEN].", "full_question": "A 40-year-old woman consults because she has noticed a lump in the superoexternal quadrant of the right breast for the past month. She provides a mammography report describing a BIRADS 3 lesion. What is the best course of action?", "id": 592, "lang": "en", "options": {"1": "Reassure him, since an imaging test has already been done and malignancy has been ruled out.", "2": "This classification probably implies surgery since the probability of cancer is greater than 10%. He explains it to you and refers you preferentially to the Breast Unit.", "3": "This is a probably benign finding, since there is less than a 2% chance of cancer. He explains that it requires follow-up every 6-12 months until 24 months or a biopsy.", "4": "The findings are of low suspicion of cancer (between 2 and 10 %) but a biopsy is necessary.", "5": NaN}, "question_id_specific": 76, "type": "ONCOLOGY", "year": 2022, "rag": {"clinical_case_options": {"MedCorp": {"RRF-2": [{"id": "pubmed23n0424_10969", "title": "Evaluation of abnormal mammography results and palpable breast abnormalities.", "score": 0.01905453225660103, "content": "Because approximately 1 in 10 women with a breast lump or abnormal mammography result will have breast cancer, a series of decisions must be taken by a primary care practitioner to exclude or establish a diagnosis of breast cancer among these women. To determine the most accurate and least invasive means to evaluate an abnormal mammography result and a palpable breast abnormality. MEDLINE search (January 1966 to March 2003) for articles and reviews describing the accuracy of clinical examination, biopsy procedures, and radiographic examination for patients with abnormal mammography results or palpable breast abnormalities. The authors reviewed abstracts and selected articles that provided relevant primary data. Studies were included if 1) mammography, fine-needle aspiration biopsy, or core-needle biopsy was performed before a definitive diagnosis was obtained; 2) the study sample included 100 or more women; and 3) breast cancer status was determined from histopathology review of excisional biopsy specimens, from linkage with a state cancer registry or the Surveillance, Epidemiology, and End Results program, or from clinical follow-up of 95% or more of the study sample. One investigator abstracted results. Methods were evaluated for major potential biases, but methodologic scoring was not performed. Likelihood ratios for first screening mammography were 0.1 for the Breast Imaging Reporting and Data System (BI-RADS) assessment category \"negative or benign finding,\" 1.2 for \"probably benign finding,\" 7 for \"need additional imaging evaluation,\" 125 for \"suspicious abnormality,\" and 2200 for \"highly suggestive of malignancy.\" For fine-needle aspiration biopsy of a palpable lump performed by formally trained physicians, the likelihood ratio was infinity for an assessment of \"malignant,\" 2.6 for \"atypical/suspicious,\" and 0.02 for \"benign.\" When diagnostic mammography was used to evaluate a palpable lump or nonpalpable breast abnormality, the positive likelihood ratios were 5.6 and 9.4, and the negative likelihood ratios were 0.15 and 0.19, respectively. Women whose screening mammography results are interpreted as \"suspicious abnormality\" or \"highly suggestive of malignancy\" have a high risk for breast cancer and should undergo core-needle biopsy or needle localization with surgical biopsy. Women whose screening mammography results are interpreted as \"need additional imaging evaluation\" have a moderate risk for breast cancer and should undergo diagnostic mammography or ultrasonography to decide whether a nonpalpable breast lesion should be biopsied. Women whose screening mammography results are interpreted as \"probably benign finding\" have a low risk for breast cancer and can undergo follow-up mammography in 6 months. Either fine-needle aspiration biopsy or ultrasonography is recommended as the first diagnostic test of a palpable breast abnormality to distinguish simple cysts from solid masses. Fine-needle aspiration biopsy also allows characterization of a solid mass. Diagnostic mammography does not help determine whether a palpable breast mass should be biopsied and should not affect the decision to perform a biopsy."}, {"id": "wiki20220301en012_128008", "title": "Mammography", "score": 0.015033105394551178, "content": "The importance of these missed cancers is not clear, particularly if the woman is getting yearly mammograms. Research on a closely related situation has shown that small cancers that are not acted upon immediately, but are observed over periods of several years, will have good outcomes. A group of 3,184 women had mammograms that were formally classified as \"probably benign\". This classification is for patients who are not clearly normal but have some area of minor concern. This results not in the patient being biopsied, but rather in having early follow up mammography every six months for three years to determine whether there has been any change in status. Of these 3,184 women, 17 (0.5%) did have cancers. Most importantly, when the diagnosis was finally made, they were all still stage 0 or 1, the earliest stages. Five years after treatment, none of these 17 women had evidence of re-occurrence. Thus, small early cancers, even though not acted on immediately, were still reliably"}, {"id": "wiki20220301en185_5771", "title": "Breast cancer screening", "score": 0.012698412698412698, "content": "Mammography Mammography is a common screening method, since it is relatively fast and widely available in developed countries. Mammography is a type of radiography used on the breasts. It is typically used for two purposes: to aid in the diagnosis of a woman who is experiencing symptoms or has been called back for follow-up views (called diagnostic mammography), and for medical screening of apparently healthy women (called screening mammography). Mammography is not very useful in finding breast tumors in dense breast tissue characteristic of women under 40 years. In women over 50 without dense breasts, breast cancers detected by screening mammography are usually smaller and less aggressive than those detected by patients or doctors as a breast lump. This is because the most aggressive breast cancers are found in dense breast tissue, which mammograms perform poorly on."}, {"id": "pubmed23n0551_10377", "title": "Follow-up recommendations for benign breast biopsies.", "score": 0.011915706102821583, "content": "Histologically proven benign breast disease increases a woman's relative risk for subsequent cancer development. Yet follow-up guidelines for mammogram and clinical breast examination after a benign breast biopsy are lacking. Our objective was to determine if increased surveillance is indicated following a benign breast biopsy. Following institutional review board approval, a retrospective database review was conducted of prospectively gathered patients who had a benign breast biopsy (core or excisional) for an abnormality detected on mammogram, ultrasound, or clinical breast examination. Follow-up, for all subjects, was a clinical breast examination and mammogram or ultrasound at 6 months, 1 year, and 2 years after benign breast biopsy by a breast surgeon. End points were the need for additional biopsies or cancer detection. Statistical analysis was performed using chi-squared analysis. From January 2000 to July 2003, 156 patients age 18-86 years had a benign breast biopsy. During the 2 year follow-up, 20 patients (13%) required a subsequent biopsy. No significant difference was observed in mean age, race, menarche, menopause, parity, age at first live birth, use of oral contraceptives, history of prior biopsy, or the pathology of the initial lesion between those who needed a subsequent biopsy and those who did not. Seven excisional biopsies were performed (one at 6 months, four at 1 year, and two at 2 years follow-up) for growth of the benign breast biopsy lesion, and pathology remained concordant with the original diagnosis. Thirteen biopsies were done for new findings on mammogram or ultrasound. Three of these (1.9%) yielded a cancer diagnosis (one at 6 months, one at 1 year, and one at 2 years follow-up). No new lesions were identified on follow-up by clinical breast examination alone. Increased surveillance following a benign breast biopsy is necessary because of the increased need for subsequent biopsy or risk of cancer development. This should include imaging (mammography or ultrasound) and a clinical breast examination 6 months, 1 year, and 2 years after a benign breast biopsy."}, {"id": "article-18585_3", "title": "Breast Imaging Reporting and Data System -- Issues of Concern", "score": 0.011123136123136123, "content": "The final assessment includes the BI-RADS 0 to 6 categorization. A category assessment of BI-RADS 0 refers to an incomplete evaluation with further imaging required including additional mammographic views including spot compression or magnification and or ultrasound. BI-RADS 1 refers to a negative examination, meaning that there are no masses, suspicious calcifications or areas of architectural distortion. There can be no description of a finding in the report if it is categorized as a BI-RADS 1.  BI-RADS 2 is consistent with benign findings. Benign findings include secretory calcifications, simple cysts, fat-containing lesions, calcified fibroadenomas, implants and intramammary lymph nodes. BI-RADS 3 is probably benign and should have shortened interval follow-up to determine stability. The risk of malignancy is below 2%. There are very strict classifications to qualify a finding in the BI-RADS 3 category: a non-palpable, circumscribed mass on a baseline mammogram; a focal asymmetry, which becomes less dense on spot compression images, or a solitary group of punctate calcifications. Any findings other than this cannot be placed in the category 3. BI-RADS 4 is a suspicious abnormality, which can represent the chance of being malignant (in percent). The BI-RADS category 4 is subdivided into a, b, and c. The subcategory of (a) has a low probability of malignancy with a 2% to 10% chance of malignancy. The subcategory of (b) has an intermediate change of malignancy ranging from 10% to 50%. The subcategory of (c) has a high probability of malignancy ranging from 50% to 95%. BI-RADS 5 is highly suggestive of malignancy more than 95%. If something is placed in this classification and the pathology comes back as benign, the recommendation is still surgical consultation, because the pathology is discordant with the radiographic findings. The last category that was recently added is the BI-RADS 6, used for pathology proven malignancy."}, {"id": "wiki20220301en185_5777", "title": "Breast cancer screening", "score": 0.011083451872925556, "content": "The accidental harm from screening mammography has been underestimated. Women who have mammograms end up with increased surgeries, chemotherapy, radiotherapy and other potentially procedures resulting from the over-detection of harmless lumps. Many women will experience important psychological distress for many months because of false positive findings. Half of suspicious findings will not become dangerous or will disappear over time. Consequently, the value of routine mammography in women at low or average risk is controversial. With unnecessary treatment of ten women for every one woman whose life was prolonged, the authors concluded that routine mammography may do more harm than good. If 1,000 women in their 50s are screened every year for ten years, the following outcomes are considered typical in the developed world:"}, {"id": "pubmed23n0315_1784", "title": "Investigation of lesions detected by mammography. The Steering Committee on Clinical Practice Guidelines for the Care and Treatment of Breast Cancer. Canadian Association of Radiation Oncologists.", "score": 0.009900990099009901, "content": "To provide information and recommendations to facilitate decision-making when a mammographic abnormality is detected by screening. References identified by use of MEDLINE, AIDSLINE, CANCERLIT and reference lists of review articles to December 1996. Where experimental evidence is lacking, recommendations are based on expert opinion. The evidence is graded accordingly in \"levels\" (page S2). Exclusion or confirmation of the presence of cancer with minimum intervention and delay. When an abnormality is detected on screening mammography, clinical evaluation and thorough radiologic work-up are needed to determine its significance. Clinical evaluation should include a history and a thorough examination of the breast, axilla and supraclavicular areas. In the radiologic work-up, diagnostic mammograms should be obtained with additional views, spot compression and magnification views as appropriate. Current mammograms should be compared with previous mammograms whenever possible. The mammographic report should include a precise description of the abnormal features visualized and an estimate of the level of suspicion of cancer they imply. Whenever there is any doubt in the interpretation of mammograms, the interpretation of 2 experienced readers should be obtained. (The following radiologic classification into 4 categories is suggested: 1--benign, not due to cancer; 2--low risk, probability of cancer under 2%; 3--intermediate risk, probability of cancer 2% to 10%; 4--high risk, probability of cancer over 10%.) Ultrasonography can be used to clarify the nature of noncalcified nodular lesions. Management decisions require close communication between the woman and her physicians. Throughout, a clinician in charge should be identified who will coordinate and transmit all decisions. Management will depend on the estimated level of risk Category 1 abnormalities require no further investigation. Category 2 abnormalities may be followed up by periodic mammographic and clinical examinations. Follow-up examination of category 2 abnormalities should be carried out at approximately 6 and 12 months. If the abnormality is stable, examination should be repeated annually for 2 to 3 years thereafter. The rationale of follow-up should be explained, and women should be made aware that it is not possible to provide complete assurance that an abnormality is benign. Category 3 abnormalities usually require image-guided fine-needle or core biopsy. Every image-guided needle biopsy should be accompanied by a full report. Category 4 abnormalities should usually be excised. This may be preceded by image-guided needle biopsy. When surgical biopsy is carried out, the margins of the resected specimen must be free of tumour. The intact pathology specimen should be examined radiographically to confirm that all mammographic abnormalities have been removed. The patient should be kept fully informed as to the reason for each test and the meaning of its results. The process, from initial detection of the mammographic abnormality to the final management decision, should be completed as rapidly as possible. Guidelines were reviewed and revised by the Writing Committee, expert primary reviewers, secondary reviewers selected from all regions of Canada and by the Steering Committee. The final document reflects a consensus of all these contributors."}, {"id": "pubmed23n0247_4207", "title": "[The importance of mammography in relationship to the number of treated carcinomas of the breast. Partial results from 1976--1978 of a long term study (author's transl)].", "score": 0.009900990099009901, "content": "From 1976 to 1978 11, 197 women were examined clinically and mammographically. Biopsy material from 1,673 breasts were examined microscopically. In 536 cases, or almost every third case (32%), a carcinoma of the breast was detected. The cancer was bilateral in 19 cases and the total number of women was therefore 517. A clinically occult tumour was only found in 7.7% (40 of 517) of the cases. 5% of these patients were high risk patients and 2.7% preventive examinations. 5 women with occult carcinoma of the breast were under age 40 and 14 under age 50. Benign changes of the glandular tissue were found in 59.5% of the cases. Marked proliferative changes were found in 4.6% of the cases and carcinoma in situ was found in 3.8% of the patients. In the age group 45--54 benign and proliferative changes of the parenchyma occured almost twice as often as cancers. The ratio between benign and malignant findings was 1:1 in the age group 55--59 and was less than 1:2 in the age group over 70. A sophistication of the mammograhic technique must be obtained. A thorough microscopic examination of tissue from subcutaneous mastectomies and tissue obtained at the time of reduction mammoplasties showed occasionally unexpected malignant tissue in an unexpected location. Especially these cases are suitable for later comparison to the mammographies."}, {"id": "pubmed23n0357_10958", "title": "Atypical medullary carcinoma of the breast with cartilaginous metaplasia in a patient with a BRCA1 germline mutation.", "score": 0.00980392156862745, "content": "We examined a 34-year-old premenopausal woman who had noticed a left-breast lump a month previously. She had no past history of malignancies but had a family history of breast and ovarian cancers. Her mother had suffered from ovarian cancer when aged 47 years and had died of the disease at age 52. The younger two of the patient's four aunts had developed breast cancer when they were 37 and 48 years old. A physical examination showed an ill-defined mass, 1.5 cm in diameter, located in the upper outer quadrant of the patient's left breast. Mammography revealed diffuse microcalcification in both breasts but ultrasonography revealed an irregular tumorous lesion only in the left breast. Aspiration breast cytology revealed adenocarcinoma of the left breast. Modified radical mastectomy of the left breast and excision of a biopsy specimen from the right breast were carried out simultaneously. Histopathologically the left-breast tumor was an atypical medullary carcinoma with cartilaginous metaplasia, of histological grade 3, and the right-breast specimen showed fibrocystic changes with atypical ductal hyperplasia. Estrogen receptors were positive, but progesterone receptor was not detected on the tumor cells, which were immunopositive for nuclear p53 although c-erbB-2 overexpression was not observed. A nonsense germline mutation of the BRCA1 gene (exon5) was detected. The patient has been well since the operation (10 months). These findings may provide useful information about the carcinogenesis and biological behavior of BRCA1-associated breast cancers."}, {"id": "pubmed23n0900_19235", "title": "How Many of the Biopsy Decisions Taken at Inexperienced Breast Radiology Units Were Correct?", "score": 0.00980392156862745, "content": "In this study, we aimed to determine the need for biopsy in patients referred from other clinics for the performance of biopsy with the suspicion of breast cancer. 112 patients were included in the study. It was decided that their biopsies be performed following examinations in other clinics and they presented to the breast radiology unit of our hospital for a second opinion. The demographic characteristics, diagnostic studies completed in the other centers, properties of lesions, decision made as a result of examinations and BI-RADS (Breast Imaging Reporting and Data Systems) categorizations were recorded on the registration forms of the study patients. In addition, the quality of examinations, reasons of repeat tests, additional tests features and the last decision of our clinic were documented. The obtained data were analyzed in terms of re-examination, additional tests and change in the biopsy decision. Changes in the biopsy decisions for patients were specifically inquired. The biopsy decisions were cancelled in our breast radiology unit for 63 out of 112 patients (56.3%) whose biopsy decisions were made at an external institute. For 42 patients, examinations made by the other clinics were deemed adequate, yet there was no need for biopsy in 22 of them. The biopsy decisions were cancelled for 27 out of 47 patients (57.4%) with repeat examination and 18 out of 28 patients (64.3%) with additional tests because of the insufficient test quality. Incorrect, inadequate breast screening and false positivity were higher at inexperienced institutes."}, {"id": "pubmed23n0694_7473", "title": "A case of quadruple primary malignancies including breast, tongue, and thyroid cancers and osteosarcoma in a young female without karyotype abnormality.", "score": 0.009708737864077669, "content": "The patient was a 41-year-old, premenopausal woman with a chief complaint of well-circumscribed palpable, right breast mass without nipple discharge. Although she noticed the lump 3 months previously, the size of the tumor (1.1 × 0.9 cm(2)) had been stable. The patient's mother suffered from gastric cancer. Her previous history of the triple different malignancies was as follows: (1) left osteosarcoma [amputation of left lower leg at 15 years old (y/o)]. After the operation, she was treated with various kinds of anticancer drugs including a total of 45 g ifosphamide and 342 g methotrexate; (2) tongue cancer (right radical neck resection; 23 y/o); and (3) thyroid cancer (right lobectomy; 40 y/o). There was no evidence of recurrence of these malignancies at the present consultation. At the time of tongue cancer operation, chromosome abnormality was investigated, but the results were normal. Physical examination showed a well-delimited, elastic-firm, mobile tumor in the central outer right breast. Regional lymph nodes were not palpable. Mammography showed a focal asymmetry in the right upper breast on the mediolateral oblique view. Ultrasonography revealed a hypoechoic mass with irregular margins. Distant metastases could not be detected by whole-body computed tomography scan. The histology of the Mammotome(®) (vacuum-assisted core needle biopsy) specimen revealed that this tumor was low-grade ductal carcinoma in situ (DCIS). She underwent breast-conserving surgery with sentinel lymph node biopsy. On permanent histopathological examination, the diagnosis of the tumor was intracystic papilloma with low-grade DCIS. Surgical margin was negative, and sentinel lymph node metastases could not be observed. Estrogen and progesterone receptor (ER/PR) were strongly positive, but human epidermal growth factor receptor-2 (HER-2) overexpression was not tested because the lesion was DCIS. She has received no adjuvant therapy and is currently disease free 3 months after surgery."}, {"id": "wiki20220301en140_26841", "title": "BI-RADS", "score": 0.009615384615384616, "content": "BI-RADS Assessment Categories are: 0: Incomplete 1: Negative 2: Benign 3: Probably benign 4: Suspicious 5: Highly suggestive of malignancy 6: Known biopsy – proven malignancy An incomplete (BI-RADS 0) classification warrants either an effort to ascertain prior imaging for comparison or to call the patient back for additional views and/or higher quality films. A BI-RADS classification of 4 or 5 warrants biopsy to further evaluate the offending lesion. Some experts believe that the single BI-RADS 4 classification does not adequately communicate the risk of cancer to doctors and recommend a subclassification scheme: 4A: low suspicion for malignancy, about > 2% to ≤ 10% likelihood of malignancy 4B: intermediate suspicion of malignancy, about > 10% to ≤ 50% likelihood of malignancy 4C: moderate concern, but not classic for malignancy, about > 50% to < 95% likelihood of malignancy"}, {"id": "pubmed23n0920_10787", "title": "Breast screening: What can the interval cancer review teach us? Are we perhaps being a bit too hard on ourselves?", "score": 0.009615384615384616, "content": "The aim of this study was to determine the features that make interval cancers apparent on the preceding screening mammogram and determine whether changes in the ways of performing the interval cancer review will affect the true interval cancer rate. This study was approved by the clinical governance committee. Mammograms of women diagnosed with an interval cancer were included in the study if they had been allocated to either the \"suspicious signs\" group or \"subtle signs\" group, during the historic interval cancer review. Three radiologists, individually and blinded to the site of interval cancer, reviewed the mammograms and documented the presence, site, characteristics and classification of any abnormality. Findings were compared with the appearances of the abnormality at the site of subsequent cancer development by a different breast radiologist. The chi-squared test was used in the analysis of the results, seeking associations between recall concordance and cancer mammographic or histological characteristics. 111/590 interval cancers fulfilled the study inclusion criteria. In 17% of the cases none of the readers identified the relevant abnormality on the screening mammogram. 1/3 readers identified the relevant lesion in 22% of the cases, 2/3 readers in 28% of cases and all 3 readers in 33% of cases. The commonest unanimously recalled abnormality was microcalcification and the most challenging mammographic abnormality to detect was asymmetric density. We did not find any statistically significant association between recall concordance and time to interval cancer, position of lesion in the breast, breast density or cancer grade. Even the simple step of performing an independent blinded review of interval cancers reduces the rate of interval cancers classified as missed by up to 39%."}, {"id": "wiki20220301en393_20908", "title": "Triple test score", "score": 0.009523809523809525, "content": "The triple test score (TTS) is a diagnostic tool for examining potentially cancerous breasts. Diagnostic accuracy of the triple test score is nearly 100%. Scoring includes using the procedures of physical examination, mammography and needle biopsy. If the results of a TTS are greater than five, an excisional biopsy is indicated. Scoring To obtain the triple test score, a number from 1 through 3 is assigned to each one of the procedures. A score of 1 is assigned to a benign test result, 2 applies to a suspicious test result, and 3 applies to a malignant result. The sum of the scores of all three procedures is the triple test score. A score of 3 to 4 is most likely benign, whereas a score of greater than 6 is possibly malignant. References Breast cancer"}, {"id": "pubmed23n0509_8108", "title": "[Re-evaluating the role of breast ultrasound in current diagnostics of malignant breast lesions].", "score": 0.009523809523809525, "content": "New evaluation of breast ultrasound based upon review of new literature comparing ultrasound and mammography. Description and discussion of the published trials regarding breast imaging methods. Breast ultrasound is the preferable method in the case of a symptomatic patient (after clinical examination). In the case of a patient without symptoms (screening), breast ultrasound is ascribed a higher sensitivity for detecting breast cancer in women with dense breast tissue, women under the age of 50 and high-risk women. Mammographically occult cancers can be detected by sonography in 10 to 40 % of the cases depending on the patient's breast density and age. The mean size of cancers detected only by ultrasound is not significantly different to that only detected by mammography. The prevalence of breast cancers detected by ultrasound is approximately equal to the one detected by mammography, regarding the total number of examined patients. Breast ultrasound should be the preferred imaging procedure in the case of a palpable lump, leading to a definitive diagnosis itself or with an additional consecutive core needle biopsy. For women without symptoms, breast sonography should be mandatory and complementary to mammography in the case of breast density grade II (BI-RADS) or more. Application of breast ultrasound as a primary method or an alternative to mammography has not yet been evaluated sufficiently. It seems advisable in the case of women with dense breast tissue grade III and IV, women under the age of 50 and high-risk women. The implementation of breast ultrasound in this manner has to be checked by future trials."}, {"id": "Surgery_Schwartz_2203", "title": "Surgery_Schwartz", "score": 0.009458662143225901, "content": "The consequences of a false-positive screening test result also need to be considered. For example, when 1000 screening mammograms are taken, only 2 to 4 new cases of cancer will be identified; this number is slightly higher (6 to 10 prevalent cancers per 1000 mammograms) for initial screen-ing mammograms.106 However, as many as 10% of screening mammograms may be potentially suggestive of an abnormal-ity, which requires further imaging (i.e., a 10% recall rate). Of those women with abnormal mammogram findings, only 5% to 10% will be determined to have a breast cancer. Among women for whom biopsy specimen is recommended, 25% to 40% will have a breast cancer. A false-positive screening result is likely to induce significant emotional distress in patients, leads to unnecessary biopsy specimens, and has cost implications for the health care system.American Cancer Society guidelines for the early detec-tion of cancer are listed in Table 10-9.96 These guidelines are updated periodically to"}, {"id": "pubmed23n0535_1319", "title": "Proliferative high-risk lesions of the breast: contribution and limits of US-guided core biopsy.", "score": 0.009433962264150943, "content": "To retrospectively correlate high-risk proliferative breast lesions (radial scar, atypical lobular hyperplasia, lobular carcinoma in situ and papillary lesions) diagnosed on core biopsy with the definitive histopathological diagnosis obtained after surgical excision or with the follow-up, in order to assess the role of core biopsy in such lesions. To discuss the management of the patient after a core biopsy diagnosis of high-risk proliferative breast lesion. We evaluated 74 out of 1776 core biopsies consecutively performed on 67 patients. The histopathologic findings were as follows: 11 radial scars (RS), 3 atypical lobular hyperplasias (ALH), 3 lobular carcinomas in situ (LCIS), 57 benign papillary lesions. All patients underwent bilateral mammography, whole-breast ultrasound with a linear-array broadband transducer, and core biopsy with a 14 Gauge needle and a mean number of samples of 5 (range 4-7). Sixty-two of 67 patients, for a total of 69/74 lesions, underwent surgical biopsy despite benign histopathologic findings, mostly because of highly suspicious imaging for malignancy (BIRADS 4-5), whereas 5 patients refused surgery and have been followed up for a least 18 months and are still being followed up (2 with RS, 1 with ADH and 2 with papillary lesions). Among the core biopsied lesions with a diagnosis of RS (n = 11) pathology revealed one ductal carcinoma in situ (DCIS) (this case was characterized by granular microcalcifications on mammography and by a mass with irregular margins on ultrasound). Also in the group of ADH (n = 3) pathology revealed one DCIS (lesion not visible on mammography but depicted as a suspicious mass on US). In the group of LCIS (n = 3) pathologists found an invasive lobular carcinoma (ILC). Among the benign papillary lesions (n = 57) histopathologic analysis of the surgical specimen revealed 7 malignant lesions (4 papillary carcinomas and 3 DCIS), whose mammographic and ultrasound findings were indistinguishable from benign lesions. Altogether there were 10 false negative results (underestimation) out of 74 core biopsies with a diagnosis of high-risk proliferative breast lesions. The high rate of histological underestimation after core biopsy (10/74) (13.5%) demands a very careful management of patents with a core biopsy diagnosis of high-risk proliferative breast lesions, especially in the case of RS, lobular neoplasia and papillary lesions. However, the high imaging suspicion for malignancy prompts surgery. It is possible to assume that, when there is a low imaging suspicion for malignancy, when enough tissue has been sampled for pathology and no atypia is found within the lesions, surgery is not mandatory but a very careful follow-up is recommended. We must underline that there is no agreement regarding the quantity of tissue to sample. Vacuum-assisted biopsy may lead to better results, although there is as yet no proof that it can actually replace surgery in this group of lesions, since it seems only to reduce but not abolish the histological underestimation."}, {"id": "pubmed23n0338_3002", "title": "[History of mammography].", "score": 0.009433962264150943, "content": "The History of mammography began in 1913, when a Berliner surgeon, A. Salomon realized a roentgeno-histological study on 3,000 mastectomies. This work is the basis of mammography. Until 1938, few articles were published but were of little help to mammography. From 1947 to 1970, the second period brought the results of roentgenologic and clinical correlation. R. Leborgne was the first accountable for the wide development of this method. Since 1951, many American and European radiologists brought their contribution. Ch. Gros is the best known. He gave this technique an acknowledgment throughout the world for the diagnosis of breast diseases. Since 1970, the third period emphasizes the value of mammography as a technique for detection of breast cancer. Some \"Screening working groups\" are being set up. The problem is mainly economical."}, {"id": "wiki20220301en012_127997", "title": "Mammography", "score": 0.009345794392523364, "content": "Often women are quite distressed to be called back for a diagnostic mammogram. Most of these recalls will be false positive results. Of every 1,000 U.S. women who are screened, about 7% will be called back for a diagnostic session (although some studies estimate the number to be closer to 10% to 15%). About 10 of these individuals will be referred for a biopsy; the remaining 60 cases are found to be of benign cause. Of the 10 referred for biopsy, about 3.5 will have cancer and 6.5 will not. Of the 3.5 who have cancer, about 2 will have an early stage cancer that will be cured after treatment."}, {"id": "wiki20220301en185_5774", "title": "Breast cancer screening", "score": 0.009264223370719051, "content": "If suspicious signs are identified in the image, then the woman is usually recalled for a second mammogram, sometimes after waiting six months to see whether the spot is growing, or a biopsy of the breast. Most of these will prove to be false positives, resulting in sometimes debilitating anxiety over nothing. Most women recalled will undergo additional imaging only, without any further intervention. Recall rates are higher in the U.S. than in the UK. Effectiveness On balance, screening mammography in older women increases medical treatment and saves a small number of lives. Usually, it has no effect on the outcome of any breast cancer that it detects. Screening targeted towards women with above-average risk produces more benefit than screening of women at average or low risk for breast cancer."}, {"id": "pubmed23n1009_24025", "title": "Toddler Sleep Challenges: All in a Day's Work.", "score": 0.009259259259259259, "content": "Leo is a 26-month-old boy who you are seeing for an urgent care visit due to \"sleep difficulty,\" particularly sleep onset. Since age 1, he screams, hits, and kicks his mother every day, starting after she gets home from work at 5 PM (or before the family's dinnertime on her days off) and escalating over the course of the evening until he \"wears himself out\" and falls asleep in a crib in his own room around 9 to 10 PM Once asleep, he sleeps well through the night and wakes easily around 7 AM in a pleasant mood; his mother leaves for work soon after he awakens. He naps after lunch for 2 to 3 hours on weekdays at an in-home child care with 1 to 2 adult caregivers and 5 other children aged 0 to 5 years. He refuses to nap at home.Leo goes to bed easily when his father puts him to bed if his mother is not at home, but his mother feels that evenings are the only time she can spend with Leo, and so, she tries to put him to bed most nights. However, because of Leo's behaviors at bedtime with her, she feels inadequate, depressed, and guilty; when she tries to disengage or allow her husband to help, Leo screams, \"Mommy, mommy!\" and tries to gain access to her and resists his father putting him to bed until his mother returns. Both parents worry that \"he would not grow out of this,\" and his mother now avoids coming home from work for fear of Leo's behavior. Both parents feel that this situation is causing marital strain.Leo was born healthy at full-term and is an only child; pregnancy was complicated by hyperemesis gravidarum. Leo has been healthy and meeting developmental milestones. His parents describe his temperament as \"like his father at that age,\" \"easy, but never able to self-soothe,\" \"intense\" in his emotional reactions, persistent, \"strong-willed and serious,\" and \"shy and observant, withdrawn at first and then getting more pleasant after a while\" in novel situations. Behaviorally, he engaged in noninjurious head-banging at home when upset between 12 and 15 months; bit children a few times at child care between 20 and 24 months; and lately refuses to share or will push other children at child care every few weeks. His parents recently read a book about parenting \"spirited\" children but did not find it helpful. What would you do next?"}, {"id": "pubmed23n0052_18930", "title": "[Errors in mammography. II. False positives].", "score": 0.009259259259259259, "content": "The authors evaluate 261 consecutive mammographic false positives observed from 1985 to 1987. Histological evidence of benign lesion followed in all cases. The comparison with the actual number of cancers and of the whole of mammographic examinations performed in the study period allowed specificity and positive predictive value of mammography to be assessed as 99.5% and 83%, respectively. Specificity and predictivity are lower in younger women, but this is more likely to depend on a different age-related incidence of cancer and benign lesions than on an intrinsic limitation of the method. The reader's diagnostic aggressivity, more than his experience, seems to affect both specificity and predictivity. At review, false positives were mostly due to asymmetric densities (49) or to circumscribed opacities with clear-cut (44) or blurred (62) outlines, whereas irregular star-like opacities or distortions (19) were infrequent. Microcalcifications were, in most cases, apparently benign (39) or dubious (76); strong suspicion was rare (4). Overall, one-fourth to one-third (27.9%) of the cases were reported as strongly suspicious at review. Palpation and cytology were also falsely suspicious--that is, co-responsible for unnecessary biopsies in over 50% of cases. Our results suggest that further improvement in the specificity or positive predictive value of mammography seems unlikely. Moreover, the benign/malignant biopsy ratio (0.2:1) presently achieved in suspicious mammographic cases appears quite satisfactory."}, {"id": "pubmed23n1152_829", "title": "A Single-Center Audit of BI-RADS 3 Assessment Category Utilization in Mammography and Breast Ultrasound.", "score": 0.009174311926605505, "content": "35 mmHg, adjunct glucocorticoid therapy should be used in addition to specific antimicrobials. Overall, treatment should be continued for 21 days and followed by secondary prophylaxis. 2 Prophylaxis for PCP is indicated for any HIV-infected individual who 0 has experienced a prior bout of PCP, any patient with a CD4+ T cell count of <200/μL or a CD4 percentage <15, any patient with unex-"}, {"id": "pubmed23n1069_15658", "title": "In Vitro Activity and In Vivo Efficacy of Cefiderocol against Stenotrophomonas maltophilia.", "score": 0.00980392156862745, "content": "Cefiderocol is a novel siderophore cephalosporin antibiotic with broad coverage against difficult-to-treat Gram-negative bacteria, including those resistant to carbapenems. Its activity against Pneumocystis jirovecii Pneumonia: Current Practice Based on Outdated Evidence.", "score": 0.009615384615384616, "content": "2 weeks, and any patient with a recent history of oropharyngeal candidiasis. The preferred regimen for prophylaxis is TMP/SMX, one double-strength tablet daily. This regimen also provides protection against toxoplasmosis and some bacterial respiratory pathogens. For patients who cannot tolerate TMP/SMX, alternatives for prophylaxis include dapsone plus pyrimethamine plus leucovorin, aerosolized pentamidine administered by the Respirgard II nebulizer, and"}, {"id": "pubmed23n0298_17347", "title": "The disposition of five therapeutically important antimicrobial agents in llamas.", "score": 0.00909090909090909, "content": "The disposition of five therapeutic antimicrobial agents was studied in llamas (Lama glama) following intravenous bolus administration. Six llamas were each given ampicillin, tobramycin, trimethoprim, sulfamethoxazole, enrofloxacin and ceftiofur at a dose of 12 mg/kg, 1 mg/kg, 3 mg/kg, 15 mg/kg, 5 mg/kg, and 2.2 mg/kg of body weight, respectively, with a wash out period of at least 3 days between treatments. Plasma concentrations of these antimicrobial agents over 12 h following i.v. bolus dosing were determined by reverse phase HPLC. Disposition of the five antimicrobial agents was described by a two compartment open model with elimination from the central compartment, and also by non-compartmental methods. From compartmental analysis, the elimination rate constant, half-life, and apparent volume of distribution in the central compartment were determined. Statistical moment theory was used to determine noncompartmental pharmacokinetic parameters of mean residence time, clearance, and volume of distribution at steady state. Based on the disposition parameters determined, and stated assumptions of likely effective minimum inhibitory concentrations (MIC) a dose and dosing interval for each of five antimicrobial agents were suggested as 6 mg/kg every 12 h for ampicillin; 4 mg/kg once a day or 0.75 mg/kg every 8 h for tobramycin; 3.0 mg/kg/15 mg/kg every 12 h for trimethoprim/sulfamethoxazole; 5 mg/kg every 12 h for enrofloxacin; and 2.2 mg/kg every 12 h for ceftiofur sodium for llamas. Steady-state peak and trough plasma concentrations were also predicted for the drugs in this study for llamas."}, {"id": "pubmed23n0643_15158", "title": "Good outcome with trimethoprim 10 mg/kg/day-sulfamethoxazole 50 mg/kg/day for Pneumocystis jirovecii pneumonia in HIV infected patients.", "score": 0.00909090909090909, "content": "Pneumocystis jirovecii pneumonia (PCP) in human immunodeficiency virus (HIV)-infected patients is usually treated with trimethoprim (TMP)-sulfamethoxazole (SMX) 1920 mg 3 times daily (approximately equivalent to TMP 15 mg/kg/day-SMX 75 mg/kg/day) for 21 days. Pharmacokinetic data suggest that lower doses would be equally efficacious and might be associated with a lower incidence of adverse effects. We conducted a retrospective review of case notes for the first episode of laboratory-confirmed PCP in HIV-infected patients treated at Auckland City Hospital, from January 1991 through December 2007. Seventy-three of 84 (87%) patients were treated with TMP-SMX 960 mg 4 times daily or 3 times daily (approximately TMP 10 mg/kg/day-SMX 50 mg/kg/day). The overall mortality was 5/73 (7%). The mortality in patients with severe disease (transcutaneous oxygen saturation on admission < or =84%) was 3/16 (19%) and in patients admitted to the intensive care unit was 5/9 (56%). Fifteen of 73 (21%) patients required a change to an alternative treatment regimen because of adverse effects (rash in 10, rash plus fever in 3, neutropenia in 1, fever plus headache in 1). Treatment of PCP in adult HIV-infected patients with TMP-SMX 960 mg QID or TID appears to have comparable efficacy to treatment with higher doses and to be associated with a lower rate of treatment limiting adverse effects."}, {"id": "wiki20220301en481_31445", "title": "Histoplasma duboisii", "score": 0.009009009009009009, "content": "Isolated lesions may be cleared by surgical removed, although some have been known to heal spontaneously. In contrast, deep lesions and disseminated disease require antifungal drug therapy. To date, no antifungal drug studies have specifically investigated the agent of African histoplasmosis. Hence most treatment approaches are based on the therapeutic strategies used to treat classical histoplasmosis caused by H. capsulatum. Amphotericin B is a mainstay of antifungal treatment, with a recommended dose of 1 mg/kg/day, culminating in a minimum dose of 2 g. Clinical response is typically apparent after 2 weeks of intravenous administration. Ketoconazole is also effective, starting at 600–800 mg/day for 3 months followed by a reduced dose of 400 mg/day for a further 6 months. The organism is also thought to be susceptible to fluconazole in vivo. A multi-month course of Amphotericin B followed by itraconazole has been suggested for complicated infection in immunodeficient individuals."}, {"id": "wiki20220301en193_14898", "title": "Austin Aztex U23", "score": 0.009009009009009009, "content": "History"}, {"id": "InternalMed_Harrison_10928", "title": "InternalMed_Harrison", "score": 0.008942922608665914, "content": "Ampicillin 300 (mg/kg)/d, q6h 12 g/d, q4h Cefepime 150 (mg/kg)/d, q8h 6 g/d, q8h Cefotaxime 225-300 (mg/kg)/d, q6h 12 g/d, q4h Ceftriaxone 100 (mg/kg)/d, q12h 4 g/d, q12h Ceftazidime 150 (mg/kg)/d, q8h 6 g/d, q8h Gentamicin 7.5 (mg/kg)/d, q8hb 7.5 (mg/kg)/d, q8h Meropenem 120 (mg/kg)/d, q8h 6 g/d, q8h Metronidazole 30 (mg/kg)/d, q6h 1500–2000 mg/d, q6h Nafcillin 100–200 (mg/kg)/d, q6h 9–12 g/d, q4h Penicillin G 400,000 (U/kg)/d, q4h 20–24 million U/d, q4h Vancomycin 45-60 (mg/kg)/d, q6h 45-60 (mg/kg)d, q6–12hb aAll antibiotics are administered intravenously; doses indicated assume normal renal and hepatic function. bDoses should be adjusted based on serum peak and trough levels: gentamicin therapeutic level: peak: 5–8 μg/mL; trough: <2 μg/mL; vancomycin therapeutic level: peak: 25–40 μg/mL; trough: 5–15 μg/mL. Meningitis, Encephalitis, Brain Abscess, and Empyema White blood cells 10/μL to 10,000/μL; neutrophils predominate Glucose <2.2 mmol/L (<40 mg/dL) CSF/serum glucose <0.4"}, {"id": "wiki20220301en088_13113", "title": "Pralidoxime", "score": 0.008928571428571428, "content": "Pralidoxime has an important role in reversing paralysis of the respiratory muscles but due to its poor blood–brain barrier penetration, it has little effect on centrally-mediated respiratory depression. Atropine, which is choice of drug to antagonise the muscarinic effects of organophosphates, is administered even before pralidoxime during the treatment of organophosphate poisoning. While the efficacy of atropine has been well-established, clinical experience with pralidoxime has led to widespread doubt about its efficacy in treatment of organophosphorus poisoning. Dosage Adults: 30 mg/kg (typically 1–2 g), administered by intravenous therapy over 15–30 minutes, repeated 60 minutes later. It can also be given as a 500 mg/h continuous IV infusion. Children: 20–50 mg/kg followed by a maintenance infusion at 5–10 mg/kg/h. Intravenous infusions can lead to respiratory or cardiac arrest if given too quickly."}, {"id": "pubmed23n0076_7007", "title": "[The treatment of pulmonary tuberculosis in a special hospital. Its evolution from 1948 to 1986].", "score": 0.008928571428571428, "content": "Nine hundred and sixty six patients diagnosed of pulmonary and/or pleural tuberculosis and admitted to a specialized hospital from 1948 to 1986, have been retrospectively analyzed, investigating their treatment and evolution. Sixty two percent of patients did not fulfil pharmaceutical treatment as far as number and dose of drugs, evolving through the decades (50, 60, 70, and 80s) with a 100%, 82%, 37% and 3% respectively. A 13% of patients did not receive any chemotherapy, 16% underwent surgery, and 53% received a second treatment. Real or hidden monotherapy was given to 38% of patients. Isoniazide has been the most uniformly used drug. Streptomycin has been the most frequently underdosed used drug. Sputum culture turned negative in 42% and 51% of patients during the first 3 and 6 months respectively, with a 42% of positives persisting after one year and a 30% when discharged. A statistically significant difference is observed when comparing all the variables between admitted patients up to 1969 and from 69 to 86 in favor of the second period."}, {"id": "pubmed23n0066_10510", "title": "Therapy for women hospitalized with acute pyelonephritis: a randomized trial of ampicillin versus trimethoprim-sulfamethoxazole for 14 days.", "score": 0.008849557522123894, "content": "The efficacy of the traditionally recommended ampicillin (Amp) plus gentamicin (GM) regimen was compared with that of a trimethoprim-sulfamethoxazole (TMP/SMZ)-plus-GM regimen and the adequacy of 14 days total therapy for acute uncomplicated pyelonephritis (AUPN). Eighty-five women hospitalized for AUPN were randomly assigned to receive either Amp, 1 g intravenously (iv) every 6 h for 3 days, then 500 mg orally four times daily, or TMP/SMZ, 160/800 mg iv every 12 h for 3 days, then 160/800 mg orally twice daily. Initially, all patients also received GM every 8 h iv (mean, 606 doses). Antimicrobial resistance necessitated modifying therapy of 14 (32%) of the Amp recipients but of none of the TMP/SMZ recipients (P less than .001). Both regimens produced a satisfactory bacteriologic and clinical response in all cases. Reinfection occurred in 11% of Amp and in 8% of TMP/SMZ recipients. No patient experienced relapsing infection. The TMP/SMZ regimen was less costly and less likely to require modification due to antimicrobial resistance."}, {"id": "InternalMed_Harrison_11753", "title": "InternalMed_Harrison", "score": 0.008849557522123894, "content": "Resistant to methicillin Clindamycin (300–450 mg/kg tid), Same options as under “Drug of It is important to know the antibiotic susceptibility of TMP-SMX (1 or 2 ds tablets bid), mino-Choice” isolates in the specific geographic region. All draincycline or doxycycline (100 mg q12hb), age should be cultured. linezolid (600 mg bid) or tedizolid (200 mg once daily) aRecommended dosages are for adults with normal renal and hepatic function. bThe dosage must be adjusted for patients with reduced creatinine clearance. cFor the treatment of prosthetic-valve endocarditis, the addition of gentamicin (1 mg/kg q8h) and rifampin (300 mg PO q8h) is recommended, with adjustment of the gentamicin dosage if the creatinine clearance rate is reduced. dDaptomycin cannot be used for the treatment of pneumonia. eVancomycin-resistant S. aureus isolates from clinical infections have been reported."}, {"id": "InternalMed_Harrison_9509", "title": "InternalMed_Harrison", "score": 0.008831417624521073, "content": "Vancomycin, 15 mg/kg q12hb; Ceftriaxone, 2 g q12h; Metronidazole, 500 mg q8h Vancomycin, 15 mg/kg q12hb; Ceftriaxone, 2 g q12h Azithromycin, 500 mg PO × 1, then 250 mg PO qd × 4 days A respiratory fluoroquinolone (moxifloxacin, 400 mg IV/PO qd; gemifloxacin, 320 mg PO qd; or levofloxacin, 750 mg IV/PO qd); A β-lactam (cefotaxime, ceftriaxone, or ampicillinsulbactam) plus azithromycin Azithromycin or a respiratory fluoroquinolone An antipseudomonal β-lactam (cefepime, 1–2 g q8–12 h; ceftazidime, 2 g q8h; imipenem, 1 g q8h; meropenem, 1 g q8h; or piperacillin-tazobactam, 4.5 g q6h); An antipseudomonal fluoroquinolone (levofloxacin or ciprofloxacin, 400 mg q8h) or an aminoglycoside (amikacin, 20 mg/kg q24hc; gentamicin, 7 mg/kg q24he; or tobramycin, 7 mg/kg q24he) Cefoxitin, 2 g q6h; A combination of metronidazole (500 mg q8–12h) plus cefazolin (1–2 g q8h) or cefuroxime (1.5 g q8h) or ceftriaxone (1–2 g q12–24h) or cefotaxime (1–2 g q6–8h)"}]}}}} {"correct_option": 4, "explanations": {"1": {"exist": true, "char_ranges": [[136, 264]], "word_ranges": [[22, 48]], "text": "Fixation in this case is better external to avoid all the material around the affected area, so 4 is correct and 1 and 2 is not."}, "2": {"exist": true, "char_ranges": [[136, 264]], "word_ranges": [[22, 48]], "text": "Fixation in this case is better external to avoid all the material around the affected area, so 4 is correct and 1 and 2 is not."}, "3": {"exist": true, "char_ranges": [[265, 334]], "word_ranges": [[48, 59]], "text": "The 3 would be considered after expiration of the infectious picture."}, "4": {"exist": true, "char_ranges": [[136, 264]], "word_ranges": [[22, 48]], "text": "Fixation in this case is better external to avoid all the material around the affected area, so 4 is correct and 1 and 2 is not."}, "5": {"exist": false, "char_ranges": [], "word_ranges": [], "text": ""}}, "full_answer": "The first thing is to treat the infection and for this we must remove all the osteosynthesis material, debride and give antibiotherapy. Fixation in this case is better external to avoid all the material around the affected area, so 4 is correct and 1 and 2 is not. The 3 would be considered after expiration of the infectious picture. TREATMENT - Suppressive antibiotic treatment: indicated in patients with Ciemy type C, it consists of prolonged oral antibiotic therapy for at least 6 months to \"cool\" the exacerbation of the clinical picture. - Curative treatment: includes a first surgery with aggressive tumor-like debridement of all affected tissues, profuse irrigation, stabilization with external fixator if stability is compromised, and eventual filling of the cavities with antibiotic-releasing substances. After prolonged periods of antibiotic therapy selected according to the antibiograms of the cultures, and once it is certain that the infection has been cured, special techniques for the reconstruction of the bone defect and to achieve adequate coverage of soft tissues should be considered. We are dealing with an infection associated with an implant (intramedullary nail) and pseudoarthrosis of the tibia fracture (absence of union after 11 months). The infection is understood by the clinical manifestations (suppuration, nonunion) and the history of diabetes and open fracture. Management overlaps with the management of chronic osteomyelitis. Isolated antibiotherapy (option 2) is indicated only in patients with severe comorbidity in whom surgical treatment would be more aggressive than continuing the disease. Nail dynamization associated with broad-spectrum antibiotherapy (option 1) is not indicated either, because (a) it has not demonstrated benefit in established pseudarthrosis and (b) we have the same issue as option 1, we do not eliminate biofilm. Option 3, discussed, would be considered only if there was no associated infection.", "full_answer_no_ref": "The first thing is to treat the infection and for this we must remove all the osteosynthesis material, debride and give antibiotherapy. Fixation in this case is better external to avoid all the material around the affected area, so [HIDDEN] and [HIDDEN]. The [HIDDEN] would be considered after expiration of the infectious picture. TREATMENT - Suppressive antibiotic treatment: indicated in patients with Ciemy type C, it consists of prolonged oral antibiotic therapy for at least 6 months to \"cool\" the exacerbation of the clinical picture. - Curative treatment: includes a first surgery with aggressive tumor-like debridement of all affected tissues, profuse irrigation, stabilization with external fixator if stability is compromised, and eventual filling of the cavities with antibiotic-releasing substances. After prolonged periods of antibiotic therapy selected according to the antibiograms of the cultures, and once it is certain that the infection has been cured, special techniques for the reconstruction of the bone defect and to achieve adequate coverage of soft tissues should be considered. We are dealing with an infection associated with an implant (intramedullary nail) and pseudoarthrosis of the tibia fracture (absence of union after 11 months). The infection is understood by the clinical manifestations (suppuration, nonunion) and the history of diabetes and open fracture. Management overlaps with the management of chronic osteomyelitis. Isolated antibiotherapy (option 2) is indicated only in patients with severe comorbidity in whom surgical treatment would be more aggressive than continuing the disease. Nail dynamization associated with broad-spectrum antibiotherapy (option 1) is not indicated either, because (a) it has not demonstrated benefit in established pseudarthrosis and (b) [HIDDEN]. [HIDDEN], discussed, would be considered only if there was no associated infection.", "full_question": "A 70-year-old woman, diabetic and hypertensive, who suffers a fall at home, presenting a 9 cm wound communicating with a fracture site of the right tibia. Radiographically, a short oblique fracture of the mid-distal third of the tibia was observed. An emergency operation was performed by cleaning (Friederich) and placement of an endomedullary steel-plated nail. At 11 months he presents with atrophic pseudarthrosis of the tibia with suppuration in the wound area. What will be his best immediate therapeutic option?", "id": 474, "lang": "en", "options": {"1": "Triple antibiotherapy (gram-positive, gram-negative and anaerobic) and cleaning of the surgical wound, removing the distal locks to promote bone consolidation.", "2": "Expectant attitude and antibiotic treatment with quinolones.", "3": "Autologous graft and growth factors (BMP 2 and 7) to stimulate the bone consolidation process, which is slowing.", "4": "Removal of the nail, debridement, placement of external fixator and antibiotherapy adjusted to culture results.", "5": NaN}, "question_id_specific": 142, "type": "ORTHOPEDIC SURGERY AND TRAUMATOLOGY", "year": 2020, "rag": {"clinical_case_options": {"MedCorp": {"RRF-2": [{"id": "pubmed23n0819_12704", "title": "[Advanced bone graft combined with locking compression plate for the treatment of middle and distal tibia nonunion].", "score": 0.017216117216117217, "content": "To explore methods of treating middle and distal tibia nonunion with the treatment of advanced bone graft combined with locking compression plate. From January 2011 to December 2012, 12 patients with middle and distal tibia nonunion were treated with advanced bone graft combined with locking compression plate. Among patients, there were 8 males and 4 females aged from 20 to 69 with an average of 47 years old. The time from first injuries to bone nonunion was from 9 months to 5 years, avergaed 19 months. Four cases were treated with external fixation, 6 cases were treated with plate fixation, 2 cases of 12 patients occurred broken of plate and nail. Eleven patients were non-infective bone nonunion and 1 patient was infective bone nonunion. Preoperative X-ray and CT showed all patients had sequestration and formation of ossified bone with different degrees. Operative time, blood loss, wound healing were observed, fracture healing time was evaluated by postoperative X-ray. Johner-Wruhs scoring standards was used to evaluate ankle joint function after operation at 10 months. Operative time ranged from 90 to 185 min with an average of (125.00±20.15) min; blood loss ranged from 225 to 750 ml with an average of (415.00±120.00) ml. All patients were followed up from 10 months to 2.5 years with an average of 1.5 years. Postoperative X-ray showed bone union was formed around fracture after operation at 4 months in all patients, 3 cases obtained bone healing within 6 months after operation, 9 cases obtained from 8 to 12 months. No infection, injury of nerve and vessles, and broken of plate and nail were ocurred. According to Johner-Wruhs scoring at 10 months after operation, 10 cases obtained excellent results, 1 good and 1 moderate. Advanced bone graft combined with locking compression plate, which can build fracture multi-point supporting based on full compression of bone nonunion to get effective fixation, is an effective method in treating middle and distal tibia nonunion."}, {"id": "pubmed23n0948_10377", "title": "[Effectiveness of limbs shortening and re-lengthening in treatment of tibial infectious bone defect and chronic osteomyelitis].", "score": 0.015199637023593466, "content": "To evaluate the limbs shortening and re-lengthening in the treatment of tibial infectious bone defect and chronic osteomyelitis. Between January 2011 and April 2016, 19 cases of tibial infectious bone defect and chronic osteomyelitis were treated with the limbs shortening and re-lengthening technique. There were 13 males and 6 females, aged from 22 to 62 years (mean, 44 years). The causes of injury included traffic accident injury in 16 cases, crush injury in 1 case, and falling from height in 2 cases. One patient was infected after plate internal fixation of closed tibial fracture and 18 patients after external fixation of open tibial fractures (Gustilo type IIIB). The mean previous operation times was 3 times (range, 2-5 times). The time from injury to bone transport operation was 3-11 months (mean, 6.5 months). The bone defect length was 2.0-5.5 cm (mean, 4.3 cm) after debridement. After tibial shortening, limb peripheral blood supply should be checked after release of the tourniquet. Seven wounds were closed directly, 5 were repaired with adjacent skin flap, 5 were repaired with sural neurovascular flap, 1 was repaired with medial head of gastrocnemius muscle flap, and 1 underwent skin grafting. Single arm external fixator or ring type external fixator were used, and completely sawed off between 2 sets of external fixation screws at proximal and distal metaphysis of the tibia. Limb lengthening was performed after 1 week with the speed of 1 mm/d. All patients were followed up 10-36 months with an average of 14 months. Two cases delayed healing of the wound after operation, and the other wounds healed primarily. Natural healing of the opposite end of the bone were found in 18 cases, and 1 case had nonunion in the opposite end of the bone because of incomplete removal of lesion bone. There were 5 cases of slow growth of the callus, and healed smoothly by \"accordion\" technology and injecting red bone marrow in 4 cases, and by bone grafting and internal fixation in 1 case. The time of bone lengthening was 1-3 months, the prolongation index was 1.6-2.7 cm/month (2.20 cm/month). The bone healing time was 7-13 months (mean, 11.1 months). According to tibial stem diagnostic criteria Johner-Wruhs score, 9 cases were excellent, 8 cases were good, 2 cases were fair, with an excellent and good rate of 89.5%. Limbs shortening and re-lengthening is an effective method for the treatment of tibial infectious bone defect and chronic osteomyelitis, with the advantages of improving the immediate alignment of the osteotomy ends, significantly shortening the bone healing time of opposite ends of bone."}, {"id": "pubmed23n1004_14096", "title": "Unreamed Intra-Medullary Nail Versus Half Pin External Fixator in Grade III [A & B] Open tibia fractures.", "score": 0.01430976430976431, "content": "Tibia fracture is the most common long bone fracture. The fractures of tibia are commonly open fractures due to subcutaneous position of the tibia. The choice of technique for stabilization of open tibia fractures includes - External fixation, unreamed intra-medullary nails [URTN], Reamed intra-medullary nails, ORIF with Plating. To evaluate & compare the results of Unreamed Intra-Medullary Nail Versus Half Pin External Fixator in Grade III [A & B] Open tibia fractures. This prospective clinical study [Randomized chit box] was done on 50 patients presenting to our institute within 24 h of injury. Only those who were skeletally mature with open tibia fracture Grade IIIA & IIIB [Gustilo-Anderson] were included in this study. After initial management, radiological assessment was done. Following this adequate wound debridement, skeletal stabilization with either primary URTN or external fixator was done. Inspection and debridement were repeated at 48-h intervals until the wound was considered clean. 50 cases [25 each group] were compared in terms of - Final Alignment of the Fracture, Presence of Infection/Non-union/Mal-union, Hardware failure, Time to Bone Union, Number of Operative Procedures after index admission. Mean time to full weight bearing was 20.96 weeks in URTN group versus 24.8 weeks in Ex-fix group. 5 in URTN group required further surgery for non-union versus 11 patients in Ex-fix group. There were 6 significant pin track infection. Removal of nail was required in 1 case of deep infection. This study supports the use of the URTN over External fixator in the treatment of severe open tibia fractures."}, {"id": "pubmed23n1037_14413", "title": "Bone Crushing in Infected Pseudarthrosis - An Extraordinary Way to Treat Osteomyelitis Caused by Resistant Bacteria.", "score": 0.012755102040816327, "content": "Osteomyelitis with multiresistant bacteria in non-union following fracture treated with osteosynthesis requires complete removal of infected sequestrum and dead bone. For consecutive bone defects, it is frequently necessary to bridge with a fixator external. The treatment is not only challenging due to reduced bone stock but also characterized by decreased bioavailability of antibiotics. We report a two-step-surgery approach to preserve the bone stock using autologous cancellous bone in a bacterial infected non-union for subsequently leg length reconstruction. The 24-year-old male patient from Belarus was admitted to our department with persistent wound secretion and subsequent osteomyelitis of the right femur 3 years after initial surgery, several revisions, and several different antibiotic therapies. Biopsy revealed methicillin-resistant and borderline oxacillin-resistant Staphylococcus aureus. Firstly, the Ilizarov ring fixator was removed and a vigorous debridement was performed by refreshing the pseudarthrosis, removing of sequestrum, and dead bone. Finally, an AO fixator external was applied for 10 weeks combined with appropriate antibiotic treatment followed by 5 weeks antibiotic-free window. The bone defect was stabilized by a long gamma trochanteric nail after removal of the AO fixateurexterne. A wide resection of the fragments was performed and the resected bone tissue was crushed and placed adjacent to the nail. Noteworthy, the biopsies of both re-section sides revealed same germs as detected in initial biopsies. Thus, antibiotics were administered for additional 3 months. Frequent radiographic and clinical controls showed a remodeling of the femur during a period of 3 years and no signs of infection. Subsequently, we restored leg length of 4 cm using a fully implantable motorized lengthening nail. In the end, the patient achieved full weight-bearing with unlimited range of motion in hip and knee. No further germ could be revealed in biopsies. In this case report, we used autologous bone from the infected side, crushed, and placed it adjacent to an intramedullary nail. Crushed bone tissue might improve bioavailability of antibiotics when dealing with multiresistant bacteria in non-union healed fracture side. Furthermore, this approach was able to provide new bone formation in a limb resulting in full weight-bearing."}, {"id": "article-30176_18", "title": "Tibia Nonunion -- Treatment / Management -- Operative Treatments", "score": 0.012443972635055436, "content": "Single-stage approach: This involves surgical debridement of all non-viable tissues and bone ends, multiple cultures to be taken from the nonunion site or canal reaming followed by revision open reduction and internal fixation or exchange nailing. Antibiotic administration is tailored to culture results and sensitivities [24] [25] . When open treatment is essential, autologous bone grafting has been recommended as an adjunct [16] . Staged approach: usually indicated for septic tibial nonunion. This involves steps to control infection followed by definitive surgery."}, {"id": "InternalMed_Harrison_10506", "title": "InternalMed_Harrison", "score": 0.010204770097493506, "content": "As mentioned above, correlation between cultures of bone and those of wound swabs or wound punctures is poor. Antibiotic treatment should be based on bone culture. If no bone biopsy is performed, empirical therapy chosen in light of the most common infecting agents and the type of clinical syndrome should be given. Wound debridement combined with a 4to 6-week course of antibiotics has been shown to render amputation unnecessary in about two-thirds of patients. According to the 2012 Infectious Diseases Society of America Clinical Practice Guideline for the Diagnosis and Treatment of Diabetic Foot Infections, the following management strategies should be considered. If a foot ulcer is clinically infected, prompt empirical antimicrobial therapy may prevent progression to osteomyelitis. When the risk of methicillin-resistant S. aureus is considered high, an agent active against these strains (e.g., vancomycin) should be chosen. If the patient has not recently received antibiotics, the"}, {"id": "pubmed23n0894_7468", "title": "Management of Large Bone Defects in Diaphyseal Fractures by Induced Membrane Formation by Masquelet's Technique.", "score": 0.009900990099009901, "content": "Management of the large gap in long bone fractures is a challenging problem after compound injuries. A novel technique called as Masquelet's technique of \"induced membrane formation\", is used to bridge a gap of more than 5 cm using bone cement as a spacer in first stage and autologous cancellous bone graft to fill the gap in second stage. We present two different and difficult cases with bone defects after open injuries associated with long bone fractures in this paper. First case is a 50-year-old lady with grade IIIa open fracture right distal femur with intra-articular extension and bone loss. She underwent wound debriment, stabilization of the fracture with locking compression plate along with antibiotic cement spacer, which is removed latter and underwent bone grafting. Another is a 15-year-old boy with open grade IIIb fracture tibia and fibula (mid-distal third junction) of right leg, wound debridement and ankle spanning triangular external fixation was applied on same day and after two months, external fixation was removed due to florid infection and plaster of Paris was applied. Instead of the tedious and demanding treatment options like Ilizarov, a new technique described by Masquelet is used here. It uses bone cement as a spacer to fill the cavity to form pseudo-membrane around it and in the second stage autologous cancellous bone graft fills the gap of even more than 5 cm, to achieve union. The membrane also secretes vascular and osteo-inductive factors to stimulate bone regeneration and also prevents resorption of the bone graft and achieves early fracture healing avoiding tedious options like bone transport in external fixator. By this two staged technique, union occurred clinically and radiologically in these two cases."}, {"id": "pubmed23n0039_12145", "title": "[Infected tibial shaft pseudarthroses, treatment and results (author's transl)].", "score": 0.009900990099009901, "content": "In a follow up study of 47 patients with infected pseudarthrosis of tibia therapy and results are declared. The most important matter in therapy is the osteosyntesis with compression of the pseudarthrosis. In the majority of cases externe stabilisation is used; seldom compression plate or medullary nailing is indicated. Such cases require much experience in the therapy of bone infection, although if the stabilisation with the fibula is tryed. Flush drainage, suction drainage, antibiotic therapy and cancellous bone grafting are necessary for localisation and decreasing of infection and for induction and acceleration of bone union. In some cases amutation can't be prevented. With correct judgement and consequent treatment in the majority of cases bone union will be obtained."}, {"id": "pubmed23n1163_5845", "title": "Masquelet Technique and Proximal Tibial Autograft Utilizing Avitus® Bone Harvester for Severely Comminuted Open Distal Radius Fracture with Extensive Bone Loss: A Case Report.", "score": 0.00980392156862745, "content": "Distal radius fractures are one of the most common fractures in the United States. Treatment usually involves internal fixation using a volar Henry approach with placement of a volar locking plate. Optimal treatment becomes less apparent when significant bone loss occurs. No case of an open distal radius fracture treated using a staged Masquelet technique involving proximal tibial autograft is available in the literature. Herein, we describe and discuss a case report of a novel technique to treat a large (5 cm) bone defect for an open distal radius fracture. A 59-year-old man suffered an open, comminuted, and intra-articular distal radius fracture with 5 cm of bone loss. He was treated using a staged Masquelet technique with incorporation of ipsilateral proximal tibial autograft with a bone harvester to obtain cancellous autograft and bone marrow graft. The patient initially underwent emergent I and D, acute carpal tunnel release, and internal and external fixation. A 5 cm bone void was filled with antibiotic cement. Four weeks later, the antibiotic cement was removed, cancellous bone graft and marrow were harvested from the proximal tibia, and the graft was placed within the prior bone void. Fracture site healing was confirmed radiographically and with computer-tomography imaging 3 months later. The patient has demonstrated excellent results 1 year post-operative with 60° of wrist flexion, 40° of wrist extension with mild pain, and full finger range of motion with radiographic union. Internal fixation with placement of a volar locking plate remains the mainstay of treatment for distal radial fractures. However, in more comminuted fractures with bone loss, treatment becomes more challenging. We have presented a unique case utilizing a staged Masquelet technique with incorporation of a proximal tibial autograft to educate readers on an alternative option and technique for autograft donor sites in these more complicated fractures."}, {"id": "pubmed23n0260_528", "title": "[Surgical treatment of diaphyseal tibial fractures. Choice of procedure and results of treatment of 187 fractures].", "score": 0.00980392156862745, "content": "From 1988 to 1990 a total of 187 fractures of the tibia (92 compound, 95 closed fractures) were treated with a primary osteosynthesis. In 102 fractures an external fixation was performed, 85 internal fixations were divided into 58 intramedullary nails and 27 plates. In a follow-up study the fracture healing was analysed, 90% of the patients were examined about 18 months after the accident. The mean healing time was between 12 weeks (internal fixation) and 16 weeks (external fixateur). After primary Fixateur externe 54 were treated by secondary internal fixation, intramedullary nailing was the method of choice. Contamination rate of the tibia, taken by an intraoperative wound swab before nailing was 30%. Infection occurred in 3.2%, non union and refractures in 1.5% and 1%. Over all 80% excellent and good results were found after consolidation. 10% fair and 10% bad results including three amputations after III degrees compound fractures and three death after polytrauma."}, {"id": "pubmed23n0916_11638", "title": "A Gustilo Type 3B Open Tibial Fracture Treated with a Proximal Flexor Hallucis Longus Flap: A Case Report.", "score": 0.009708737864077669, "content": "In the treatment of Gustilo Type 3B open tibial fractures, it is important to perform soft tissue reconstruction and bone reconstruction simultaneously. Gastrocnemius muscle and soleus muscle flaps are generally used as rotational flaps for the tibia. The distal third of the tibia can often not be covered with the gastrocnemius muscle and soleus muscle flaps. Treatment distal to the distal third of the tibia is difficult because fewer flap options are available. In the present report, we describe our experience with a Gustilo Type 3B open tibial fracture treated by gastrocnemius muscle and soleus muscle flaps, along with an additional proximally based flexor hallucis longus flap, which is a rare procedure. The participant was a 17-year-old male who injured his left tibia in a motorcycle traffic accident. Physical examination revealed a wound of 13 cm × 7 cm extending from the medial lower leg to the posterior aspect, with extensive skin loss. There was no nerve or vascular injury. The tibia was exposed, with detachment of the periosteum. The radiograph revealed a tibial shaft fracture. The AO/OTA classification was 42-A3.3, and it was classified as a Gustilo-Anderson Type 3B fracture. Gastrocnemius muscle and soleus muscle flaps were lifted in the area of the soft-tissue defect and then, placed over the tibia. Despite this, the distal portion of the tibia remained uncovered. Therefore, a flexor hallucis longus flap was lifted and placed over the distal portion of the tibia. On day 7 after the injury, the external fixation device was removed and the tibial shaft was fixated with two Ender nails (4.5 mm in diameter). The clinical course was satisfactory, and the skin graft and flap were successful. Bone union was achieved without infection, and the resulting range of motion was normal. For the treatment of Gustilo-Anderson Type 3B open tibial fractures, early treatment of the soft-tissue defect is vital. We surgically treated a Gustilo-Anderson Type 3B open tibial fracture with gastrocnemius muscle and soleus muscle flaps, along with an additional proximally based flexor hallucis longus flap, which is a rare procedure. In the event of a soft-tissue defect in the distal third of the tibia, the use of a proximally based flexor hallucis longus flap is an effective surgical approach."}, {"id": "pubmed23n1009_12107", "title": "Prevention of infection in open fractures: Where are the pendulums now?", "score": 0.009708737864077669, "content": "Soft tissue management and fracture fixation including initial external fixation in Gustilo-Anderson type II and type III open fractures are cornerstones in the treatment but details on timing and type of wound closure, irrigation and debridement, systemic and local antibiotics, antimicrobial-coated implants and the use of Bone Morphogenetic Protein-2 remain controversial. This article looks at current clinical evidence of these items for the management of open fractures. Timing of debridement and wound closure remains critical. Early debridement by an experienced team within 24 h seems adequate while gross contamination, a devascularized limb, a multi-injured patient and compartment syndrome require immediate surgical intervention. Wound closure during the first surgery was shown to result in reduced rates for infections and nonunion. If soft-tissue reconstruction is needed, it should be performed within the first 7 days. Regarding types of irrigation fluid, antiseptic and antibacterial solutions did not prove to be superior to saline. High pressure irrigation has not been demonstrated to be beneficial whereas antibiotic administration as soon as possible has been proven to be favorable. Administration of more than 72 h was not superior to shorter systemic antibiotic intervals. For Gustilo-Anderson type I and II, broad spectrum antibiotic therapy is reasonable. Additional aminoglycosides for broader coverage are recommended in Gustilo-Anderson type III fractures. There is newer literature on the beneficial effects of the use of local antibiotics, e.g. by antibiotic beads. Coating of internal fixation devices is a modern approach to improve infection prophylaxis and gentamicin-coated implants have been demonstrated to be safe in clinical application. Vacuum assisted closure (VAC) could not evidence negative pressure wound therapy to reduce infection risk, improve self-rated disability or quality of life in open fractures, however, enhance treatment costs. Recombinant human bone morphogenetic proteins (rhBMP)-2 showed promising data in Gustilo-Anderson type III open tibial shaft fractures with lower rates of invasive secondary procedures. In conclusion, there is evidence for thorough debridement and irrigation with saline, early soft tissue coverage and the use of systemic and local antibiotics. Except for a short-term soft tissue coverage VAC seems not to be beneficial and rhBMP-2 is an additional tool in Gustilo-Anderson type III open fractures."}, {"id": "pubmed23n0735_24439", "title": "Salvage procedures in lower-extremity trauma in a child with hereditary motor and sensory neuropathy type I: a case report.", "score": 0.009615384615384616, "content": "Fractures of the lower extremity are a common type of childhood injury and many can be treated without surgery. Dislocated and open fractures are an indication for fracture stabilization via either intramedullary nailing or, in the case of complicated fractures, external fixation. But if complications are likely because of diseases and disabilities (for example, a neuropathy) that can complicate the post-operative procedure and rehabilitation, what options does one have? We report a nine-year-old Caucasian girl who had hereditary motor and sensory neuropathy type I and who was admitted with a grade I open tibia fracture after a fall from a small height. Plain radiographs showed a dislocated tibia and fibula fracture. An open reduction with internal fixation with a compression plate osteosynthesis was performed, and soft tissue debridement combined with an external fixateur was undertaken. Three months later, she was re-admitted with localized swelling and signs of a local soft tissue infection in the middle of her tibia. Plain radiographs showed a non-union of the tibia fracture, and microbiological analysis confirmed a wound infection with cefuroxime-sensitive Staphylococcus aureus. Because of the non-union, the osteosynthesis was replaced with an Ilizarov external fixateur, and appropriate antibiotic therapy was initiated. Four months after the initial accident, the fracture was consolidated and we removed the external fixateur. If there is a pre-existing neuropathy and if disease makes it difficult for a child to follow all post-operative instructions, salvage procedures should be kept in mind in case of complications. There are multiple therapeutic options, including osteosynthesis, intramedullary nailing systems, cast therapy, or an external fixateur like the Ilizarov or Taylor spatial frame system. The initial use of an external fixateur such as an Ilizarov or Taylor spatial frame in patients with pre-existing neuropathies should be kept in mind as a possible treatment option in complicated fractures, especially in a child with pre-existing neurological or endocrine pathologies."}, {"id": "pubmed23n0728_18192", "title": "Recombinant human BMP-2 for the treatment of open tibial fractures.", "score": 0.009615384615384616, "content": "Recombinant human bone morphogenetic protein-2 (rhBMP-2) improves healing of open tibial fractures treated with intramedullary nail fixation. However, routine use has not occurred. The purposes of the current study were to provide a systematic review of the literature using rhBMP-2 in the treatment of acute open tibial fractures treated with intramedullary nail fixation and to provide a meta-analysis of the randomized, controlled trials. Multiple databases, reference lists of relative articles, and main orthopedic journals were searched. The basic information and major results were compared. Four studies with a total of 609 patients were included.The secondary intervention rate in the standard-of-care (SOC) group was significantly higher than in the rhBMP-2 combined with absorbable collagen sponge (rhBMP-2/ACS) group (27.1% vs 17.5%, respectively; P<.01). The treatment failure rate in the SOC group was significantly higher (34.3% vs. 21.4%, respectively; P<.01). No significant differences were found in infection rate, hardware failure rate, fracture healing rate at 20 weeks, and postoperative pain level. For patients treated with reamed intramedullary nail fixation, only the treatment failure rate in the SOC group was significantly higher (21.5% vs 14.2%, respectively; P=.02); no other significant difference was observed. Adding rhBMP-2 to the treatment of Gustilo-Anderson grade IIIA and B open tibial fractures led to net savings of approximately $6000 per case.Recombinant human bone morphogenetic protein-2 added to intramedullary nail fixation of open tibial fractures could reduce the frequency of secondary interventions and total health care costs. For reamed patients, adding rhBMP-2 reduced treatment failure. This analysis supports the clinical efficacy of rhBMP-2/ACS for the treatment of these severe fractures."}, {"id": "pubmed23n0929_24008", "title": "[Treatment of the postoperative infection of limbs fracture after internal fixation with vacuum sealing drainage (VSD) combined with continual irrigation].", "score": 0.009523809523809525, "content": "To explore the clinical effects of VSD combined with continual irrigation in treating the infection of limbs fracture after internal fixation. From March 2010 to June 2015, 10 patients with infection of limbs fracture after internal fixation were treated with VSD combined with continual irrigation. There were 7 males and 3 females, aged from 11 to 58 years with an average of 34.4 years. Course of disease was from 1 to 8 months with an average of 4.8 months. Postoperative infection occurred in fractures of ulna and radius of 4 cases, tibiofibular fractures of 3 cases, calcaneal fractures of 2 cases, femoral fractures of 1 case. Eight infections were open fracture and 2 infections were close fracture. In additon to above treatment, antibiotics, dressing changing or skingrafting were used in the patients. Informations of wound surface healing, change dressings, original infection focus were observed. All infections got control, the wound healing after change dressings or skingrafting, and no complications such as osteomyelitis were found. The mean treatment time was 38.4 days(ranged, 29 to 45 days) and replacement times was 2.2 times(ranged, 1 to 4 times). All patients were followed up, no recurrent infections were found at 1 year after fracture healing. VSD combined with continual irrigation can effectively decrease the incidence of complications and promote the wound growth, healing and considerably shorten the healing time. It is an effective method for the treatment of infection of limbs fracture after internal fixation."}, {"id": "pubmed23n0986_8054", "title": "Outcome of Locked Compressive Nailing in Aseptic Tibial Diaphyseal Nonunions without Bone Defect.", "score": 0.009433962264150943, "content": "Treatment of tibial diaphyseal nonunions are rather difficult. Plate-screw, intramedullary nailing and external fixation are the methods used for treatment. The aim of this study is to evaluate the treatment results of aseptic diaphyseal nonunions following tibia fractures by intramedullary compressive tibia nailing (IMCN) with or without bone graft. Twenty eight patients who had aseptic tibial nonunion without bone defects operated between 2005 and 2015 were included in the study. The mean age of our patients was 36.4 years (range 20-56 years). There were 22 males and 6 females. Fifteen of the patients exhibited hypertrophic nonunion and thirteen exhibited atrophic nonunion. The average time between fracture occurrence and presentation to our department was 1.6 years (range 1-20 years). All patients underwent fibular osteotomy by removal of a 2 cm bone block from the middle one-third of the fibulas. In all cases, IMCN was applied following the reaming procedure, then maximum bone contacts were achieved manually between proximal and distal bone fragments afterward, and dynamic compressive fixation with 1 mm of compression was performed by a single rotation of the compression screw at the top of the nail. Direct X-ray images were assessed according to the Rust criteria, and functional outcomes were assessed according to the Johner-Wrush criteria. Finite-element analysis was performed for 1 mm of compression. For statistical analysis, Fisher's exact test, Pearson's Chi-square test, and Mann-Whitney U-test were used. Union was achieved in all patients. Radiological union was obtained at an average of 15.5 ± 1.86 weeks. Functional results were found to be good or excellent in 25 (89.2%) patients and average or poor in 3 (10.8%) patients. One patient developed skin necrosis at the wound site, which was treated with rotational flap and skin graft. None of the patients developed implant failure, thromboembolism, deep-vein thrombosis, or infection. The use of compressive intramedullary nailing with or without bone graft is an effective method for the treatment of tibial nonunion."}, {"id": "pubmed23n0024_8786", "title": "[Plate fixation and open wound treatment in infected intramedullar nails].", "score": 0.009433962264150943, "content": "In treating patients who present with an extensive non-union due to infected intramedullary nail, bone healing can be achieved within a few weeks through a policy of active management. Following excision of the infected tissue and thorough irrigation, the fracture site is stabilized by a plate fixation and a packet with cancellous bone. The wound overlying the bone is left open to granulate. No complications were observed during the course of healing in 7 cases (5 tibia, 2 femur) managed this way. The method is based on classical principles of orthopedics and the treatment of infection. Such has been its impressive success, that it would seem to warrant not only close attention but also strong recommendation."}, {"id": "pubmed23n0779_20957", "title": "Treatment of open tibial fracture with bone defect caused by high velocity missiles: a case report.", "score": 0.009345794392523364, "content": "Tibia fracture caused by high velocity missiles is mostly comminuted and followed by bone defect which makes their healing process extremely difficult and prone to numerous complications. A 34-year-old male was wounded at close range by a semi-automatic gun missile. He was wounded in the distal area of the left tibia and suffered a massive defect of the bone and soft tissue. After the primary treatment of the wound, the fracture was stabilized with an external fixator type Mitkovic, with convergent orientation of the pins. The wound in the medial region of the tibia was closed with the secondary stitch, whereas the wound in the lateral area was closed with the skin transplant after Thiersch. Due to massive bone defect in the area of the rifle-missile wound six months after injury, a medical team placed a reconstructive external skeletal fixator type Mitkovic and performed corticotomy in the proximal metaphyseal area of the tibia. By the method of bone transport (distractive osteogenesis), the bone defect of the tibia was replaced. After the fracture healing seven months from the secondary surgery, the fixator was removed and the patient was referred to physical therapy. Surgical treatment of wounds, external fixation, performing necessary debridement, adequate antibiotic treatment and soft and bone tissue reconstruction are essential in achieving good results in patients with the open tibial fracture with bone defect caused by high velocity missiles. Reconstruction of bone defect can be successfully treated by reconstructive external fixator Mitkovic."}, {"id": "pubmed23n1062_15982", "title": "Treatment options for aseptic tibial diaphyseal nonunion: A review of selected studies.", "score": 0.009345794392523364, "content": "In aseptic tibial diaphyseal nonunions after failed conservative treatment, the recommended treatment is a reamed intramedullary (IM) nail.Typically, when an aseptic tibial nonunion previously treated with an IM nail is found, it is advisable to change the previous IM nail for a larger diameter reamed and locked IM nail (the rate of success of renailing is around 90%).A second change after an IM nail failure is also a good option, especially if bone healing has progressed after the first change.Fibular osteotomy is not routinely advised; it is only recommended when it interferes with the nonunion site.In delayed unions before 24 weeks, IM nail dynamization can be performed as a less invasive option before deciding on a nail change.If there is a bone defect, a bone graft must be recommended, with the gold standard being the autologous iliac crest bone graft (AICBG).A reamer-irrigator-aspirator (RIA) system might also obtain a bone autograft that is comparable to AICBG.Although the size of the bone defect suitable to perform bone transport techniques is a controversial issue, we believe that such techniques can be considered in bone defects > 3 cm.Non-invasive therapies and biologic therapies could be applied in isolation for patients with high surgical risk, or could be used as adjuvants to the aforementioned surgical treatments. Cite this article: 40-50 ml/kg/24hrs) is confirmed, and urine osmolality is <800 mOsm/kg, serum sodium level has to be checked. If the serum sodium level is <135 meq, it is diagnostic of PP. If the serum sodium levels are >147, it is diagnostic of diabetes insipidus. If the serum sodium is between 135 and 147, the next step would be the water deprivation test. The traditional test that has been utilized by providers for a long time is the indirect water deprivation test that indirectly measures the activity of AVP. This test is started once hypotonic polyuria is confirmed, and serum sodium is between 135 and 147. Polyuria in primary polydipsia decreases with water deprivation (typically >8hrs), and urine osmolality increases (>800 mOsm/kg typically), this is diagnostic of PP. In diabetes insipidus, polyuria does not get better with water deprivation. If the urine osmolality remains <300 mOsm/kg after water deprivation, it is diagnostic of DI."}, {"id": "pubmed23n0253_8397", "title": "Altered renal handling of electrolytes in a child with central diabetes insipidus (CDI).", "score": 0.013652036156928525, "content": "A 12-year-old female child, with a history of polyuria and polydipsia of about three years duration, was admitted to Ethio-Swedish Paediatric Hospital, in Addis Abeba. Urine output in 24 hours averaged 5-6 litres, with a frequency of 15 times during the day and 7-8 times during the night. Random urine analysis showed an osmolality of 60 mOsm/kg, Na+ 27.1 mmol/L and K+ was 7.6 mmol/L. Basal plasma osmolality was 313 mOsm/kg with Na+ being 156 mmol/L and K+ 4.06 mmol/L. Water deprivation for nine hours failed to produce a concentrated urine, which was only 138 mOsm/kg at the end of the test, with a corresponding plasma osmolality of 336 mOsm/kg. After nine hours of water deprivation, urine Na+ increased from 27.1 to 37.3 mmol/L while K+ increased from 7.1 to 18.7 mmol/L. Lypressin, a vasopressin analogue, at a concentration of 0.3 IU/kg injected intramuscularly, resulted in a marked increase in urine osmolality to 586 mOsm/kg within two hours, associated with relief of symptoms. Urinary excretion of K+ was markedly increased during the vasopressin test while Na+ excretion was little affected. A case of central diabetes insipidus of undefined etiology is presented and the possibility of altered renal handling of electrolytes and an abnormal response to vasopressin in such cases is noted. The problem of management and the currently available treatment options are summarized."}, {"id": "article-25794_31", "title": "Neurohypophysis -- Evaluation -- Diabetic Insipidus", "score": 0.013447816200109779, "content": "Serum and urine sodium, serum and urine osmolality, hourly urine output, 24-hour urine volume, and specific gravity must be obtained. Central DI must be distinguished from nephrogenic DI. A water deprivation test can be used with or without desmopressin injection. In central DI, the vasopressin will correct urine osmolality while in nephrogenic DI, the correction will be suboptimal. [3] [25] [26] Criteria for the diagnosis of central DI: Polyuria in two consecutive hours of > 300 ml/hr or 4 to 5 ml/kg/hr in two consecutive hours Polyuria > 3L/24 hours or > 2ml/kg/hr in 24 hours Serum osmolality > 300 mOsm/kg Urine osmolality < 300 mOsm/kg Urine/plasma osmolality <  1 The specific gravity of less than 1.005"}, {"id": "pubmed23n1110_3988", "title": "Transient Antidiuretic Hormone Insufficiency Caused by Severe Hyperosmolar Hyperglycemic Syndrome Based on Nephrogenic Diabetes Insipidus.", "score": 0.013412109711919959, "content": "The hyperosmolar hyperglycemic state (HHS), an acute complication of diabetes mellitus with plasma hyperosmolarity, promotes the secretion of anti-diuretic hormone (ADH) and reduces the storage of ADH. Magnetic resonance T1-weighted imaging reflects ADH storage in the posterior pituitary lobe, which disappears when the storage is depleted. Whether the HHS induces ADH depletion leading to clinical manifestations has been unclear. A 55-year-old Japanese woman was admitted to our center because of mental disturbance and hypotension. She had received lithium carbonate for bipolar disorder and presented with polydipsia and polyuria from 15 years of age. On admission, she had mental disturbance (Glasgow Coma Scale, E4V1M1), hypotension (systolic blood pressure, 50 mmHg), and tachycardia (pulse rate, 123/min). Plasma glucose was 697 mg/dL osmolality was 476 mOsm/kg•H40 mL/kg Osmolarity <300 mosm/L Basal plasma AVP >1 pg/mL <1 pg/mL Pituitary bright spot Present Absent Anatomy Pathology? GU evaluation Volume <40 mL/kg Osmolarity >300 mosm/L If MRI and/or AVP assays with the requisite sensitivity and specificity are unavailable and a fluid deprivation test is impractical or undesirable, a third way to differentiate between pituitary DI, nephrogenic DI, and primary polydipsia is a trial of desmopressin therapy. Such a trial should be conducted with very close monitoring of serum sodium as well as urine output, preferably in hospital, because desmopressin will produce hyponatremia in 8–24 h if the patient has primary polydipsia."}, {"id": "wiki20220301en075_14909", "title": "Plasma osmolality", "score": 0.011976311560222793, "content": "Osmolality of blood increases with dehydration and decreases with overhydration. In normal people, increased osmolality in the blood will stimulate secretion of antidiuretic hormone (ADH). This will result in increased water reabsorption, more concentrated urine, and less concentrated blood plasma. A low serum osmolality will suppress the release of ADH, resulting in decreased water reabsorption and more concentrated plasma. Syndrome of inappropriate ADH secretion occurs when excessive release of antidiuretic hormone results in inappropriately elevated urine osmolality (>100 mOsmol/L) relative to the blood plasma, leading to hyponatraemia. This ADH secretion may occur in excessive amounts from the posterior pituitary gland, or from ectopic sources such as small-cell carcinoma of the lung. Elevation may be associated with stroke mortality. Calculated osmolarity (CO)"}, {"id": "pubmed23n0777_9374", "title": "Diabetes insipidus.", "score": 0.011754911754911754, "content": "Diabetes insipidus (DI) is characterized by hypotonic polyuria greater than 3 liters/24 hours in adults and persisting even during water deprivation. It is mostly due to a defect in arginin-vasopressin (AVP) synthesis (central DI); other causes are: AVP resistance (nephrogenic DI), abnormal thirst regulation (primary polydipsia) or early destruction of AVP by placental enzymes (gestational DI). A thorough medical history is warranted to investigate nocturnal persistence of polyuria (night waking being a good sign of its organic nature) to specify the onset and duration of the trouble, the medication use and the potential hereditary nature of the disorder. The next step is based on weight and blood pressure measurements and especially the quantification of beverages and diuresis over a 24-hour cycle. Assessment of signs of dehydration, bladder distention, pituitary hormone hyper- or hyposecretion, tumor chiasmatic syndrome, granulomatosis and cancer is required. The diagnosis is based on biological assessment, pituitary magnetic resonance imaging (MRI) and results of a desmopressin test. In severe forms of DI, urine osmolality remains below 250 mOsmol/kg and serum sodium greater than 145 mmol/L. In partial forms of DI (urine osmolality between 250 and 750), the water deprivation test demonstrating the incapacity to obtain a maximal urine concentration is valuable, together with vasopressin or copeptin measurement. The pituitary MRI is done to investigate the lack of spontaneous hyperintensity signal in the posterior pituitary, which marks the absence of AVP and supports the diagnosis of central DI rather than primary polydipsia (although not absolute); it can also recognize lesions of the pituitary gland or pituitary stalk. Acquired central DI of sudden onset should suggest a craniopharyngioma or germinoma if it occurs before the age of 30 years, and metastasis after the age of 50 years. Fifteen to 20% of head trauma lead to hypopituitarism, including DI in 2% of cases. Transient or permanent DI is present in 8-9% of endoscopic transphenoidal surgeries. Current advances in DI concern the etiological work-up, with in particular the identification of IgG4-related hypophysitis or many genetic abnormalities, opening the field of targeted therapies in the years to come."}, {"id": "pubmed23n1060_9822", "title": "ADIPSIC DIABETES INSIPIDUS AFTER SECOND RESECTION OF A HYPOTHAMIC ASTROCYTOMA.", "score": 0.011701839826839828, "content": "We report a case of adipsic diabetes insipidus (ADI) post-astrocytoma resection. Clinical and laboratory data are presented. A 16-year-old female with a history of incompletely resected hypothalamic astrocytoma was admitted with a headache. Head magnetic resonance imaging showed an interval increase in a suprasellar lesion with extension to the third ventricle. Following a second stage resection, she developed an increased urine output with diluted urine resulting in a negative fluid balance; however, she was unable to sense thirst. Blood tests showed a serum sodium of 155 mEq/dL (normal, 136 to 145 mEq/dL), serum osmolality at 321 mOs/kg (normal, 285 to 295 mOs/kg) and a urine osmolality of 128 mOsm/kg (normal, 300 to 1,600 mOsm/kg). Serum creatinine and potassium were normal. Pituitary hormone profiles were found to be normal: growth hormone 0.171 ng/mL (normal, 0.123 to 8.05 ng/mL), luteinizing hormone 3.44 mIU/mL (normal, 7.59 to 89.08 mIU/mL), follicle-stimulating hormone 5.60 mIU/mL (normal, 2.55 to 16.69 mIU/mL), thyroid-stimulating hormone 2.9 mIU/mL (normal, 0.35 to 4.94 mIU/mL), free thyroxine 0.92 ng/dL (normal, 0.7 to 1.48 ng/dL), adrenocorticotropic hormone 19.56 pg/mL (normal, 7.2 to 63.3 pg/mL), and prolactin 7.25 ng/mL (normal, 5.18 to 26.53 ng/mL). The patient was treated with desmopressin acetate 120 μg tablets twice daily with a fixed fluid intake of 1.5 to 2.0 L/day with close monitoring of fluid intake, output, and body weight. The response was good with a gradual reduction of serum sodium level of around 7 to 9 mEq/L/day. ADI is a rare entity of central diabetes insipidus, where the absence of polydipsia can be challenging in diagnosing and managing the condition. Cases of ADI are likely under reported and clinicians need to be aware of this condition."}, {"id": "InternalMed_Harrison_3621", "title": "InternalMed_Harrison", "score": 0.011563724678478778, "content": "Following the correction of hypernatremia and acute renal insufficiency with appropriate hydration (see below), the patient was subjected to a water deprivation test followed by administration of DDAVP. This test helps determine whether an inappropriate water diuresis is caused by CDI or NDI. The patient was water restricted beginning in the early morning, with careful monitoring of vital signs and urine output; overnight water deprivation of patients with diabetes insipidus is unsafe and clinically inappropriate, given the potential for severe hypernatremia. The plasma Na+ concentration, which is more accurate and more immediately available than plasma osmolality, was monitored hourly during water deprivation. A baseline AVP sample was drawn at the beginning of the test, with a second sample drawn once the plasma Na+ reached 148–150 meq/L. At this point, a single 2-μg dose of the V2 AVP receptor agonist DDAVP was administered. An alternative approach would have been to measure AVP"}, {"id": "wiki20220301en086_255", "title": "Hyperosmolar hyperglycemic state", "score": 0.011537163587277481, "content": "Diagnosis Criteria According to the American Diabetes Association, diagnostic features include: Plasma glucose level >30 mmol/L (>600 mg/dL) Serum osmolality >320 mOsm/kg Profound dehydration, up to an average of 9L (and therefore substantial thirst (polydipsia)) Serum pH >7.30 Bicarbonate >15 mEq/L Small ketonuria (~+ on dipstick) and absent-to-low ketonemia (<3 mmol/L) Some alteration in consciousness BUN > 30 mg/dL (increased) Creatinine > 1.5 mg/dL (increased) Imaging Cranial imaging is not used for diagnosis of this condition. However, if MRI is performed, it may show cortical restricted diffusion with unusual characteristics of reversible T2 hypointensity in the subcortical white matter."}, {"id": "article-27735_22", "title": "Primary Polydipsia -- Differential Diagnosis", "score": 0.010912698412698412, "content": "The administration of desmopressin differentiates between central and nephrogenic insipidus. If, after the administration of desmopressin, there is an increase of >50% in the urine osmolality, it is diagnostic of central DI. If there is an increase of <50% in the urine osmolality, it is diagnostic of nephrogenic DI. If the urine osmolality is between 300 mOsm/kg and 800 mOsm/kg after the water deprivation test, this could either be partial central DI or PP. To differentiate partial central DI from PP, desmopressin is administered. If the urine osmolality increases by > 9%, it is diagnostic of PP. If the Urine osmolality increases by <9%, it is diagnostic of partial central diabetes insipidus."}, {"id": "article-20428_60", "title": "Arginine Vasopressin Disorder (Diabetes Insipidus) -- Evaluation -- 2. Diagnosis of the type of polyuria-polydipsia syndrome", "score": 0.010906512116988237, "content": "Hypertonic saline (3% saline, 1027 mOsm/L) infusion coupled with plasma copeptin measurement is an alternative test that is now being recommended by many experts in the field of DI as the preferred test to be used in place of the water deprivation test."}, {"id": "pubmed23n0121_7561", "title": "An assessment of posterior pituitary function in patients with Sheehan's syndrome.", "score": 0.010905253283302064, "content": "Antidiuretic hormone (ADH) function was assessed in a group of 16 patients with Sheehan's syndrome and 17 controls. All patients were on adequate cortisone and thyroxine replacement therapy before testing. During the dehydration test the patients revealed an impairment of ADH function. The maximum urine osmolalities and the urine-plasma osmolality ratios were significantly lower in the patients with Sheehan's syndrome compared to controls (maximum urine osmolalities 633 +/- 38 (SEM) and 873 +/- 29 (SEM) mOsm/kg, respectively, P less than 0.001; urine-plasma osmolality ratios 2.15 +/- 0.14 (SEM) and 3.01 +/- 0.10 (SEM), respectively, P less than 0.001). Plasma osmolalities were significantly higher in the patients (296.1 +/- 1.2 (SEM) and 290 +/- 0.9 (SEM), respectively, P less than 0.001). The patients took a longer period to achieve these maximum urine osmolalities. Three of the patients with Sheehan's syndrome were diagnosed as having diabetes insipidus since their maximum urine osmolalities were below 600 mOsm/kg and following desmopressin all three had an increment in urine osmolality which exceeded 9%. In addition these three patients had a maximum urine-plasma osmolality ratio below 1.9. Thus, it appears the patients with Sheehan's syndrome have an impairment of ADH function which manifests in some as diabetes insipidus."}, {"id": "pubmed23n0059_5294", "title": "[Transient polyuria in pregnancy in diabetes insipidus and gestational diabetes].", "score": 0.010858050847457626, "content": "Two pregnant women developed overt polyuria (up to 11 l/day) and polydipsia during their second and third trimesters of pregnancy. In one patient hydronephrosis was present. Both patients suffered from mild gestational diabetes mellitus. Plasma sodium was 145 and 162 mmol/l. Polyuria and urinary hypo-osmolality responded well to desmopressin acetate. After delivery, polyuria and polydipsia disappeared in one patient and significantly improved in the other. Infusion of hypertonic saline one and two weeks respectively after delivery led to plasma hyper-osmolality (294 mosmol/kg and 305 mosmol/kg) without detectable stimulation of arginine vasopressin (AVP). Anterior pituitary function was normal. No stimulation of AVP occurred following insulin-induced hypoglycemia. AVP plasma disappearance after i.v. pulse injection of 1 microgram AVP as well as AVP plasma concentration after continuous infusion of 10 ng AVP/min was studied two weeks after delivery in one patient. The results suggested markedly elevated degradation of AVP compared to control subjects, probably due to an increased vasopressin activity. Eight months after delivery, hypertonic saline infusion in one patient led to a plasma-osmolality of 312 mosmol/kg without stimulation of AVP. In the second patient, AVP was not detectable (less than 0.2 pg/ml) six months after delivery when plasma osmolality was 290 mosmol/kg. Our studies demonstrate that a subclinical compensated diabetes insipidus was preexistent in both patients. Exacerbation occurred due to an increased AVP-clearance and presumably due to the hemodynamic and hormonal alterations during pregnancy, including a mild gestational diabetes mellitus."}, {"id": "article-20428_59", "title": "Arginine Vasopressin Disorder (Diabetes Insipidus) -- Evaluation -- 2. Diagnosis of the type of polyuria-polydipsia syndrome", "score": 0.010797897754419493, "content": "Copeptin (carboxy-terminal-Pro-vasopressin) is the C-terminal peptide of pro-vasopressin co-secreted with AVP from the posterior pituitary. [8] [53] Unlike plasma AVP measurement, copeptin measurement in the plasma is relatively less cumbersome. It has several advantages: copeptin can remain stable for days after blood sampling and can be measured relatively quickly. [53] Plasma levels of copeptin strongly correlate with plasma AVP levels over a wide range of osmolalities, both in healthy individuals and those with DI or primary polydipsia. [27] [54] Moreover, plasma copeptin demonstrates the same response to plasma osmolality and volume changes as plasma AVP. [9] [27] Several studies have been conducted to validate the utility of plasma copeptin in diagnosing hypotonic polyuric states and to distinguish one form from the other. [8] [9] [11] [38] [27] Hypertonic saline infusion test:"}, {"id": "wiki20220301en090_48172", "title": "Free water clearance", "score": 0.01073231205171345, "content": "For example, for an individual with a urine osmolality of 140 mOsm/L, plasma osmolality of 280 mOsm/L, and a urine production of 4 ml/min, the free water clearance is 2 ml/min, obtained from Interpretation Free water clearance can be used as an indicator of how the body is regulating water. A free water clearance of zero means the kidney is producing urine isosmotic with respect to the plasma. Values greater than zero imply that the kidney is producing dilute urine through the excretion of solute-free water. Values less than zero imply that the kidney is conserving water (likely under the influence of antidiuretic hormone, ADH), resulting in the production of concentrated urine. See also Renal clearance Renal physiology External links Overview at mcg.edu Overview at mmi.mcgill.ca Formula at mmi.mcgill.ca Renal physiology"}, {"id": "article-27735_26", "title": "Primary Polydipsia -- Differential Diagnosis", "score": 0.010549039681303274, "content": "If the level is less than 4.9 pmol/l, it is diagnostic of central DI. 96% of the patients with polyuria were accurately diagnosed with their respective diagnoses by measuring copeptin along with water deprivation/hypertonic saline infusion test. Given the cumbersomeness of the water deprivation test and the long duration involved with it, hypertonic saline infusion gained a reputation. Stimulation with hypertonic saline also requires frequent sodium checks and close monitoring. Keeping this in mind, a study was done with arginine infusion to measure the stimulated levels of copeptin instead of with hypertonic saline, which was promising. [29] More evidence is needed to substantiate this finding, though."}]}}}} {"correct_option": 1, "explanations": {"1": {"exist": true, "char_ranges": [[88, 144]], "word_ranges": [[16, 26]], "text": "The diagnosis would be practically made with the AMA (1)."}, "2": {"exist": true, "char_ranges": [[146, 186]], "word_ranges": [[26, 33]], "text": "With 2 we would rule out hemochromatosis,"}, "3": {"exist": true, "char_ranges": [[188, 229]], "word_ranges": [[33, 41]], "text": "with 3 we would rule out Wilson's disease."}, "4": {"exist": true, "char_ranges": [[231, 325]], "word_ranges": [[41, 57]], "text": "With 4 we could rule out rare diseases such as biliary tract malformations or Caroli's disease"}, "5": {"exist": true, "char_ranges": [[330, 371]], "word_ranges": [[58, 66]], "text": "with 5 we could rule out viral hepatitis."}}, "full_answer": "Book picture of Primary Biliary Cirrhosis, one of those that do not occur in real life. The diagnosis would be practically made with the AMA (1). With 2 we would rule out hemochromatosis, with 3 we would rule out Wilson's disease. With 4 we could rule out rare diseases such as biliary tract malformations or Caroli's disease and with 5 we could rule out viral hepatitis.", "full_answer_no_ref": "Book picture of Primary Biliary Cirrhosis, one of those that do not occur in real life. The diagnosis would be practically made with the AMA (1). With 2 we would rule out hemochromatosis, with 3 we would rule out Wilson's disease. With 4 we could rule out rare diseases such as biliary tract malformations or Caroli's disease and with 5 we could rule out viral hepatitis.", "full_question": "A 52-year-old woman consulted because she had noticed during the previous week a yellowish discoloration of the conjunctivae. She does not refer to risky sexual behaviors or epidemiological history of risk of viral hepatitis. She does not consume alcohol or hepatotoxic drugs. She reports a one-year history of generalized pruritus, asthenia, dry mouth and absence of lacrimation with no known cause. Rest of the anamnesis without pathological data. Physical examination showed scratching lesions, conjunctival jaundice and non-painful hepatomegaly. The patient brings a blood test carried out in his company with the following pathological results: Bilirubin 3 mg/dl, FA 400 UI/ VSG 40mm 1 hour. Indicate which would be the best recommendation to establish the etiological diagnosis of the patient's condition:", "id": 8, "lang": "en", "options": {"1": "Anti-mitochondrial antibodies.", "2": "Study of Fe metabolism.", "3": "Study of copper metabolism.", "4": "Hepatic MRI.", "5": "Serology for B and C viruses."}, "question_id_specific": 232, "type": "DIGESTIVE", "year": 2011, "rag": {"clinical_case_options": {"MedCorp": {"RRF-2": [{"id": "pubmed23n1164_2588", "title": "Epstein-Barr Virus Hepatitis Masquerading as Painless Jaundice.", "score": 0.014586786644565475, "content": "Epstein-Barr virus (EBV) infection typically presents with pharyngeal symptoms and subclinical transaminitis. We present a case of a 27-year-old woman with no known past medical history who presented with painless jaundice and dark-colored urine for three days. Her review of systems was negative for fever, sore throat, nausea, vomiting, pruritus, or rash. Her last sexual contact was six months ago with a male partner, and she only drank alcohol socially. Family and surgical history were non-significant. Physical examination revealed 3+ bilateral conjunctival icterus without abdominal tenderness or organomegaly. She had elevated transaminases: alanine transaminase (ALT) of 1287U/L and aspartate aminotransferase of (AST) 1057U/L but her alkaline phosphatase (ALP) was only slightly above normal at 109U/L (normal range 35-104U/L), with a direct hyperbilirubinemia - total bilirubin 9.5mg/dl, direct bilirubin 6.8mg/dl; the abdominal ultrasound revealed non-dilated bile ducts. Hepatitis A, B, and C serology was negative, but her EBV serology showed an infection. She had incidental thalassemia minor without splenomegaly or asterixis. She was managed conservatively, and her liver enzymes trended down with supportive management. Although EBV is an uncommon cause of painless jaundice, this diagnosis should be considered, especially when other more common causes of jaundice have been ruled out. A high index of suspicion should be maintained to detect EBV hepatitis as it can easily be diagnosed through serological testing."}, {"id": "wiki20220301en023_9732", "title": "Anita Roddick", "score": 0.012958881203921867, "content": "Roddick explained that her hepatitis C was unexpectedly diagnosed in 2004, following a blood test that was part of a medical examination needed for a life insurance policy. The blood test indicated abnormal liver function and subsequent blood tests diagnosed hepatitis C. Roddick explained that she had a large blood transfusion in 1971, after the birth of her younger daughter, and that she was convinced that the transfusion had infected her with hepatitis C. This was about twenty years before blood donors in the United Kingdom were screened for hepatitis C. She reported that she had developed cirrhosis of the liver, and that her main symptoms were itching and poor concentration. She briefly mentioned that medical treatment with interferon did not suit her. Roddick explained that she kept fit and active, and that she attended biannual out-patient hospital appointments in Southampton, as well as being under review by the liver transplant team at the Addenbrooke's Hospital in Cambridge."}, {"id": "wiki20220301en330_13535", "title": "Social history of viruses", "score": 0.01154384328358209, "content": "Hepatitis is a disease of the liver that has been recognised since antiquity. Symptoms include jaundice, a yellowing of the skin, eyes and body fluids. There are numerous causes, including viruses – particularly hepatitis A virus, hepatitis B virus and hepatitis C virus. Throughout history epidemics of jaundice have been reported, mainly affecting soldiers at war. This \"campaign jaundice\" was common in the Middle Ages. It occurred among Napoleon's armies and during most of the major conflicts of the 19th and 20th centuries, including the American Civil War, where over 40,000 cases and around 150 deaths were reported. The viruses that cause epidemic jaundice were not discovered until the middle of the 20th century. The names for epidemic jaundice, hepatitis A, and for blood-borne infectious jaundice, hepatitis B, were first used in 1947, following a publication in 1946 giving evidence that the two diseases were distinct. In the 1960s, the first virus that could cause hepatitis was"}, {"id": "InternalMed_Harrison_23282", "title": "InternalMed_Harrison", "score": 0.010582010582010581, "content": "Approach to the Patient with Liver Disease Suspected Liver Disease Abnormal liver tests Acute < 6 months Chronic > 6 months Diagnostic evaluation 1. IgM Anti-HAV 2. HBsAg 3. IgM Anti-HBc 4. Anti-HCV 5. ANA, SMA 6. Monospot, heterophile 7. Ceruloplasmin 8. Alcohol history 9. Drug history Diagnostic evaluation 1. AMA 2. Drug history 3. Ultrasound/MRI 4. MRCP/ERCP Liver biopsy in acute liver disease: Reserved for patients in whom the diagnosis remains unclear despite medical evaluation Liver biopsy in chronic liver disease: Often valuable for diagnosis as well as staging and grading liver disease Diagnostic evaluation 1. HBsAg 2. Anti-HCV 3. Fe saturation, ferritin 4. Ceruloplasmin 5. ˜1AT 6. ANA, SMA 7. Ultrasound 8. Alcohol history Diagnostic evaluation 1. Drug history 2. AMA 3. P-ANCA 4. Ultrasound 5. MRCP/ERCP Hepatitic: °°ALT Mixed: ˛ALT, ˛AlkP Cholestatic: °°AlkP, °°gGT, ˛ALT Hepatitic: °°ALT Mixed: ˛ALT, ˛AlkP Cholestatic: °°AlkP, °°gGT, ˛ALT"}, {"id": "Pharmacology_Katzung_7019", "title": "Pharmacology_Katzung", "score": 0.010483184333759046, "content": "Her social history is significant for alcohol use (three to four glasses of wine/night). Her vital signs include the following: temperature 99.8°F, blood pressure 132/64 mm Hg, pulse 78 bpm, and respiratory rate 15/min. On physical examination, she had left upper abdominal tenderness with evidence of hepatomegaly and mild scleral icterus. Laboratory data revealed the following: alanine aminotransferase, 527 IU/L (normal 10–35 IU/L); aspartate aminotransferase, 425 IU/L (normal < 35 IU/L); and bilirubin, 2.9 mg/dL (normal 0.1–0.3 mg/dL). What medications do OTC cold and flu preparations typically contain? Which of the OTC medications might have contrib-uted to the patient’s current symptoms? KH, a 55-year-old woman, presents to the emergency department with nausea, vomiting, and complaints of new-onset flu symptoms over the past several days. Her past medical history is significant for allergic rhinitis and chronic lower back pain secondary to a work-related fall 2 years ago. Her"}, {"id": "article-23802_24", "title": "Jaundice -- History and Physical -- History", "score": 0.01034365574954335, "content": "Patients usually present with varying symptoms apart from yellowish discoloration of skin along with pruritus, thus providing clues to narrow down the etiology or can also be asymptomatic. A thorough questioning regarding the use of drugs, alcohol or other toxic substances, risk factors for hepatitis (travel, unsafe sexual practices), HIV status, personal or family history of any inherited disorders or hemolytic disorders is vital. Other important points include the duration of jaundice; and the presence of any coexisting signs and symptoms, like a joint ache, rash, myalgia, changes in urine and stool. [22] A history of arthralgias and myalgias before yellowing indicates hepatitis, either due to drugs or viral infections."}, {"id": "article-22784_30", "title": "Hepatitis -- History and Physical -- Viral Hepatitis", "score": 0.010161576538712991, "content": "Phase 2 (prodromal phase) - Patients in this phase usually present with anorexia, nausea, vomiting, malaise, pruritus, urticaria, arthralgias, and fatigue. Many times these patients are misdiagnosed as having gastroenteritis or viral infection. Phase 3 (icteric phase) - Patients in this phase present with dark-colored urine and pale-colored stool. Some patients develop jaundice and right upper quadrant pain with liver enlargement. Phase 4 (convalescent phase) - Patients typically start noticing the resolution of symptoms, and laboratory studies show liver enzymes returning to normal levels. [32]"}, {"id": "pubmed23n0735_17090", "title": "Fatal hepatitis E viral infection in pregnant women in Ghana: a case series.", "score": 0.009900990099009901, "content": "Viral infections during pregnancy can pose serious threats to mother and fetus from the time of conception to the time of delivery. These lead to congenital defects, spontaneous abortion and even death. The definitive diagnosis and management of pregnancy-related viral infections may be challenging especially in less resourced countries. We present clinical and laboratory responses to the diagnosis and management of three cases of fulminant hepatitis secondary to Hepatitis E viral infection in pregnancy.Case 1 was a 31-year-old Ghanaian woman who presented with a week's history of passing dark urine as well as yellowish discoloration of the eyes. She subsequently developed fulminant hepatitis secondary to Hepatitis E viral infection, spontaneously aborted at 24 weeks of gestation and later died.Case 2 was also a 31-year-old Ghanaian woman who was admitted with a four-day history of jaundice. She had low grade fever, but no history of abdominal pain, haematuria, pale stool or pruritus. She next developed fulminant hepatitis secondary to Hepatitis E viral infection. However, she did not miscarry but died at 28 weeks of gestation.Case 3 was a 17-year-old Ghanaian woman who was referred to the tertiary health facility on account of jaundice and anaemia. She had delivered a live male infant at maturity of 32 weeks but noticed she was jaundiced and had a presentation of active disease 3 days prior to delivery. The baby was icteric at birth and on evaluation, had elevated bilirubin (mixed type) with normal liver enzymes. Hepatitis E virus infection was confirmed in both mother and baby. However, the jaundice and the hepatomegaly resolved in mother and baby after 5 and 12 days respectively. To the best of our knowledge, these are the first documented cases of fatal fulminant hepatic failures resulting from HEV infection in Ghana."}, {"id": "pubmed23n0396_3498", "title": "[Dynamics of serum copper level in patients in the acute phase of hepatitis B and in early convalescence].", "score": 0.009900990099009901, "content": "Analysis of serum copper (Cu) level dynamics during the acute phase of hepatitis acute B and the early convalescence in compliance with gravity of the acute disease course. The study included 39 patients (12 men and 27 women), aged 18 to 76 years. They were hospitalised in the Department of Infectious Diseases of Medical Academy in Lublin because of the hepatitis acute B, without coexisting diseases. The diagnosis was based on the epidemiologic anamnesis, clinical symptoms, biochemical and serological examinations. The studied group was divided in respect to sex and the course of the disease as: light, medium-weighty and weighty. In all examined patients, the serum Cu level was determined according to the following scheme: at the first, tenth, twentieth and the last day of the hospitalisation and additionally one time at four weeks after discharging from the clinic. The serum Cu level was made by atomic absorption spectrometry (AAS) at the wave length of 324.8 nm. The received data were subjected to statistical analysis according to t-Student's test and in cases of significant differences according to the variants of c-Cochran and Cox's tests. According to the SI Unit Conversion Guide, the values 11.22 to 23.58 mumol/l were taken as the normal range. The values derived from the control group of 24 healthy persons (13 men and 11 women) aged 22 to 69 years. The significant increase of serum Cu level in comparison with the control values was found both in the acute phase of hepatitis B and the early convalescence. It could be observed a correlation between serum Cu level and the course of hepatitis viralis acuta B."}, {"id": "pubmed23n0972_7733", "title": "Spontaneous Cure of Acute Hepatitis C.", "score": 0.00980392156862745, "content": "The statistics proved that approximately 25% of the patients with acute HCV present with jaundice, and only 10-20% develop gastrointestinal symptoms. We present the case of a 58 year-old woman, with prior antecedents of arterial hypertension and diabetes mellitus since 25 years old, hypercholesterolemia and hypertriglyceridemia, psoriasis, epilepsy and depressive syndrome. She clinically presents asthenia, anorexia, itching, jaundice and choluria. The objective examination showed an orientated patient, without flapping, hemorrhagic dyscrasia or signs of chronic hepatic disease, with icteric mucosa and skin, abdominal pain, with hepatomegaly and splenomegaly. The laboratory tests have been compatible with acute hepatitis with colestatic pattern: AST/ALT 969/798 UI/ml, FA 796 UI/ml, GGT 2476 UI/ml, BT/BD 7.39/6.10, INR 0.9. The abdominal echography showed: hepatomegaly, regular borders, hepatic steatosis, splenomegaly without ascitic fluid. The viral serological tests revealed protection for hepatitis A ( IgM neg/IgG pos), negative for HVB infection (AgHBs neg, anti-HBc neg), negative for HVE and other viruses (CMV Herpes virus, Epstein Barr, HIV), positive antibodies for HCV and positive RNA VHC (164200 UI/ml), HCV genotype 3a, IL-28B CT, negative autoimmunity. The previous HCV tests were negative, sustaining the recent infection. We assumed an acute hepatitis C. The patient was symptomatically treated with hydroxyzine for the skin itch, with vitamin K for INR correction and she was closely monitored. She had good clinical and laboratorial evolution and she was discharged after one week, maintaining hepatology consultation. She spontaneously cleared HCV infection after 3 months, maintaining negative RNA VHC 6 months after infection. The patient has cured the HCV infection with no need for antiviral treatment."}, {"id": "pubmed23n0244_4563", "title": "[Finding and significance of copper protein complexes in liver cirrhoses (author's transl)].", "score": 0.00980392156862745, "content": "302 liver cirrhoses obtained from the post-mortem material 1970 to 1979 have been examined for the occurrence of the cuprous protein complexes by means of the orcein-staining according to Shikata. The part of the B-posthepatic cirrhoses (89%) could be simultaneously determined with the help of this method. Cuprous protein complexes have been found in bilious (100%), alcoholic (43.9%), etiologically uncertain (34.4%), and B-posthepatic (25,9%) liver cirrhoses. They prove an existing or experienced chronic cholestasis. The demonstration of copper has no principal significance for the etiologic classification of cirrhoses. In the bioptic diagnostics the demonstration of copper has a certain significance for the differentiation of the primary destructive cholangitis from chronically active hepatites of other genesis. The investigations incidentally revealed that HBsAg-containing hepatocytes could be identified by means of orcein in sections performed on archive material after destaining independent of the age of the sections and their primary staining."}, {"id": "wiki20220301en529_26162", "title": "Tenpō Tsūhō", "score": 0.009708737864077669, "content": "History The Tenpō Tsūhō came around a century after the introduction of the Hōei Tsūhō (Kyūjitai: 寳永通寳 ; Shinjitai: 宝永通宝) during the 5th year of the Hōei era (1708), which had a face value of 10 mon (while only containing 3 times as much copper as a 1 mon Kan'ei Tsūhō coin), but was discontinued shortly after it started circulating as it wasn't accepted for its nominal value."}, {"id": "pubmed23n0412_18897", "title": "Cases from the Osler Medical Service at Johns Hopkins University.", "score": 0.009615384615384616, "content": "A 37-year-old woman presented with increasing abdominal pain and jaundice. Six weeks before admission, she developed persistent diarrhea and jaundice of the skin. She also bruised easily, and her gums bled. In the subsequent weeks, her appetite decreased, she was fatigued, and she had nausea, vomiting, and abdominal distension. She had a history of drinking 1 quart of vodka every day for 20 years, with brief periods of abstinence; she stopped consuming alcohol 11 days before admission because it no longer provided symptomatic relief. Her past medical history was also notable for depression, including a suicide attempt 4 years earlier. She did not smoke, use illicit drugs, or have unprotected sexual intercourse. She had received no blood transfusions and had not traveled recently. She took no medications, except for occasional ibuprofen. On physical examination, she was thin and deeply jaundiced, and she trembled and responded slowly to questions. She was afebrile but tachypneic, and she had orthostatic hypotension. Her HEENT examination was notable for scleral and sublingual icterus, as well as crusted blood on her gums and teeth. The jugular veins were flat. The cardiac examination revealed tachycardia (heart rate, 103 beats per minute) without murmurs, rubs, or gallops. The abdomen was nontender and protuberant, with hypoactive bowel sounds; the spleen was not palpable, and there was no fluid wave or caput medusae. The liver percussed to 18 cm, with a smooth edge extending 10 cm below the costal margin. She had cutaneous telangiectases on her chest and bilateral palmar erythema. There was no peripheral edema. The neurologic examination was notable for asterixis. Her stool was guaiac positive. Laboratory studies revealed the following values: hematocrit, 21.2%; white blood cells, 17,310/mm(3); ammonia, 42 micromol/L; serum creatinine, 3.9 mg/dL; serum urea nitrogen, 70 mg/dL; albumin, 2.1 g/dL; total bilirubin, 26.8 mg/dL; alanine aminotransferase, 14 U/L; aspartate aminotransferase, 77 U/L; alkaline phosphatase, 138 U/L; prothrombin time, 103 seconds (international normalized ratio, 10.6); and urinary sodium, <5 mg/dL. Urinalysis revealed an elevated specific gravity and numerous muddy granular casts. Hepatitis A, B, and C serologies were negative. On abdominal ultrasound examination, there was no ascites, and the liver was echogenic. The portal and hepatic veins were patent, and the hepatic arteries were normal. The spleen measured 14 cm. What is the diagnosis?"}, {"id": "article-22788_50", "title": "Hepatitis B -- Differential Diagnosis", "score": 0.009615384615384616, "content": "Wilson disease is a disease of excessive copper accumulation. It is associated with psychiatric disturbances due to copper accumulation in the basal ganglia. Kayser-Fleischer rings are pathognomonic for Wilson disease but are not completely sensitive (requires an expert ophthalmologist to confirm this finding). Laboratory evaluation that favors a diagnosis of Wilson disease includes low serum ceruloplasmin levels and elevated urinary copper, and if abnormal, requires further evaluation by a hepatologist. Alcoholic hepatitis Autoimmune hepatitis Cirrhosis Drug-induced liver injury Hemochromatosis Hepatitis A Hepatitis C Hepatitis D Hepatitis E Hepatocellular carcinoma Human immunodeficiency virus Wilson disease"}, {"id": "pubmed23n0840_2629", "title": "Acute Cholestatic Liver Injury From Hydralazine Intake.", "score": 0.009523809523809525, "content": "Hydralazine is a commonly used oral antihypertensive agent. We report a rare case of hydralazine-induced hepatotoxicity in the form of subacute hepatic necrosis. A 75-year-old African American woman presented with jaundice of 7-day duration. She was started on hydralazine 100 mg 3 times a day 10 weeks before presentation. On physical examination, scleral icterus was noted. Workup revealed elevated liver transaminases, alkaline phosphatase, and conjugated bilirubin. She had no history of liver disease, and liver function tests had been normal before starting hydralazine. Other etiologies, including viruses, common toxins, drugs, autoimmune, and copper-induced hepatitis, were excluded. Abdominal imaging studies did not show any evidence of intrahepatic or extrahepatic biliary ductal dilatation, and no pathologies were seen in the liver and pancreas. The patient's liver biopsy revealed extensive lobular hepatitis, significant necrosis, mixed inflammatory infiltrate, and no significant fibrosis, supporting a diagnosis of drug-induced liver injury. Hydralazine was immediately discontinued. She showed improvement of clinical and laboratory abnormalities within 5 days after discontinuation of hydralazine. To establish the diagnosis of hydralazine-induced liver injury, we used assessment tool outlined by the Council for International Organization of Medical Sciences (CIOMS) scale that led to \"high probable\" relationship. Although rare, clinically significant, and potentially life-threatening liver injury can result from use of hydralazine. Both clinical and histological presentations in our patient suggest acute liver injury. The hydralazine-induced hepatitis seems to be reversible as discontinuation of the drug improves clinical outcomes. We highly recommend monitoring of the liver function during hydralazine treatment. "}, {"id": "pubmed23n0930_24443", "title": "[Clinical and morphological correlations in occult hepatitis B].", "score": 0.009523809523809525, "content": "Occult hepatitis B (ОHB) characterized by the absence of blood HBsAg attracts the attention of specialists of different profiles; however, its clinical morphological aspects have not been practically studied. to estimate the proportion of OHB in the structure of fatal outcomes in chronic viral hepatitis (CVH) and to characterize its clinical course and structural changes on autopsy materials. A total of 455 autopsy cases of CVH were examined for its etiology in the S.P. Botkin Clinical Hospital of Infectious Diseases in 2014-2016. An in-depth prospective clinical analysis was made to investigate 28 cases of OHB in the stage of decompensated liver cirrhosis, which had subsequently culminated in death. The criteria of inclusion were history data and clinical symptoms of CVH in the detection of markers for hepatitis A, C, and D and HIV in serum HBcAb in the absence of HBsAg. HBsAbs were also determined. Along with the traditional morphological examination, immunohistochemistry (IHC) for HBsAg and HBcAg was carried out. There were 108 CVHB cases (23.7% of the total cases of CVH), including 77 OHB cases (71.3% of those of CVHB) while HBsAg was not determined. HBsAb-negative patients were more often observed to have clinical signs of jaundice (p<0.05) and skin itching (p<0.05). Dyspepsia and hemorrhagic manifestations prevailed in patients with HBsAb (more than 10 IU/l) (p<0.05). All the cases were found to have characteristic morphological signs of CVH, including intranuclear inclusions and nuclear polymorphism in 10.7% of deaths. There was an IHC-positive reaction to VHB antigens in 28.6% of the patients and a doubtful reaction in 25.0%. Serum НВсAb may serve as a diagnostic marker for HBV infection. Clinical and morphological correlations enabled the authors to state that CVHB was present in all cases in the absence of serum HBsAg in the patients."}, {"id": "InternalMed_Harrison_23947", "title": "InternalMed_Harrison", "score": 0.009459924320605436, "content": "Diagnosing NAFLD requires demonstration of increased liver fat in the absence of hazardous levels of alcohol consumption. Thresholds for potentially dangerous alcohol ingestion have been set at more than one drink per day in women and two drinks per day in men based on epidemiologic evidence that the prevalence of serum aminotransferase elevations increases when alcohol consumption habitually exceeds these levels. In those studies, one drink was defined as having 10 g of ethanol and, thus, is equivalent to one can of beer, 4 ounces of wine, or 1.5 ounces (one shot) of distilled spirits. Other causes of liver fat accumulation (particularly exposure to certain drugs; Table 364-2) and liver injury (e.g., viral hepatitis, autoimmune liver disease, iron or copper overload, α1 antitrypsin deficiency) must also be excluded. Thus, establishing the diagnosis of NAFLD does not require invasive testing: it can be accomplished by history and physical examination, liver imaging (ultrasound is an"}, {"id": "pubmed23n0960_17714", "title": "Ginseng-Related Drug-Induced Liver Injury.", "score": 0.009433962264150943, "content": "Ginseng is commonly used as a medicinal herb for memory and concentration and general well-being. Drug-induced liver injury (DILI) is one of the most challenging disorders and trending events in the United States which are related to body building and weight loss supplements. Currently, herbal and dietary supplementation is the second most common cause of DILI. Here, we report on a 45-year-old healthy Chinese woman who presented with dull intermittent left upper quadrant abdomen pain for a month. Upon thorough history taking, she had been taking ginseng tea and supplementation for her menopausal symptoms for almost 3 months. Physical examination was unremarkable except mild tenderness in left upper quadrant of the abdomen. Liver function test showed aspartate transaminase (AST) 717 U/L, alanine transaminase (ALT) 343 U/L, total bilirubin 5 mg/dL, direct bilirubin 3.3 mg/dL, alkaline phosphatase 182 U/L, with international normalized ratio (INR) 1.2. Prior liver enzymes (6 months earlier) showed AST 21 U/L, ALT 18 U/L, total bilirubin 0.8 mg/dL, direct bilirubin 0.3 mg/dL, alkaline phosphatase 34 U/L, with INR 0.7. Viral serology for acute hepatitis B, C, E, cytomegalovirus, Epstein-Barr virus, and varicella zoster virus was negative. She was immune to hepatitis A. Her antinuclear antibody was positive. Her anti-Smith antibody, anti-smooth muscle antibody, HFE gene mutation, ceruloplasmin, alpha-1 antitrypsin serologies were within normal references. An abdomen sonogram showed fatty infiltration. Liver biopsy showed moderate to severe portal inflammation and marked lobular disarray. Portal and lobular inflammatory infiltrates consisted of a mixture of histiocytes, lymphocytes, plasma cells, eosinophils, and neutrophils with centrilobular necrosis and focal bridging necrosis, and necro-inflammation. After 6 weeks of follow-up, the patient improved physically, and the abdomen pain resolved. Ginseng has been widely used in the Chinese community as medicinal herb for a variety of conditions for decades. However, proper research has never been done regarding its pharmacokinetics, efficacy, and safety issues. In our case report, the idiosyncratic DILI resulted from ingestion of ginseng as herbal supplementation for premenopausal symptoms. Physicians should be aware of and suspect DILI in any patient with acute liver injury, and patients should be reminded that all medications and supplements have a potential to cause DILI."}, {"id": "article-23802_30", "title": "Jaundice -- Evaluation", "score": 0.009433962264150943, "content": "The results of the bilirubin, enzymes, and liver function tests will direct the diagnosis towards a hepatocellular or cholestatic cause and offer some idea of the duration and severity of the disease. Further evaluation can be conducted based on the initial assessment. Hepatocellular workup: viral serologies, autoimmune antibodies, serum ceruloplasmin, ferritin."}, {"id": "pubmed23n0975_8122", "title": "Cerebral Venous Sinus Thrombosis in Systemic Lupus Erythematosus.", "score": 0.009345794392523364, "content": "A 38-year-old woman presented with general weakness and vaginal bleeding. One month prior, she had been diagnosed with Evans syndrome (haemolytic anemia with positive Coombs test and thrombocytopenia) and was given oral steroid as maintenance therapy. Her serology examination was negative for hepatitis B, hepatitis C, and human immunodeficiency virus (HIV). Her obstetrical history was marked by miscarriage in second pregnancy and preeclampsia in third pregnancy. She used hormonal contraceptives until 5 months prior to admission. On physical examination, she had anemic conjunctiva and no organomegaly. Blood tests were significant for anemia (3.4 g/dl) and thrombocytopenia (28,000/µl). Her vaginal bleeding had ceased, however her platelet continued decreasing to 12,000/µl during first several days of hospitalization despite receiving platelet transfusion. On the tenth hospital day, she suddenly complained of severe headache and blurred vision. She had bilateral edema and erythema of palpebral, chemosis, decreased in visual acuity, and reduced ocular motility. Ear and nose examination were normal. Peripheral blood smear showed no blast. Prothrombine time (PT), INR, APTT tests were normal and D Dimer was slightly increased (3.3 mg/l; NV ≤0.5 mg/l). Urine examination revealed proteinuria with 24 hour urine protein was 1,863 mg (NV <150 mg/day). We assessed her as cavernous sinus thrombosis and treated her empirically with intravenous broad-spectrum antibiotics, morphine drip. Either digital subtraction angiography or anticoagulant was deferred due to low platelet. Further examination revealed positive for ANA, anti-SSA, and diagnosis of SLE was established. Anticardiolipin antibodies of IgG and IgM and anti-beta2 glycoprotein antibodies of IgM and IgG tests were non reactive. Methylprednisolone pulse therapy (1g/day) was given for 3 consecutive days, and then tapered to oral methylprednisolone. She additionally received azathioprine 50 mg tab BID. Meanwhile her clinical symptoms alleviated and platelet count was increased, brain MRI and MR venography finally performed suggesting cerebral venous sinus thrombosis. She got additional oral anticoagulant rivaroxaban 15 mg tab BID and eventually discharged. Cerebral venous sinus thrombosis may be the presenting symptoms or occur concomitantly within the onset of SLE. Our patient had SLE, meeting 4 of the Systemic Lupus International Collaborating Clinic classification criteria (hemolytic anemia, thrombocytopenia, renal involvement, and positive for ANA test). Vasculitis due to endothelial cell injury mediated by immune-complex deposition is proposed to be the pathogenesis of CVST in SLE. Hypercoagulable state could be other etiology factor. Antiphospholipid antibodies were absent in our case as reported in some cases, emphasizing vasculitis as the underlying mechanism. Treatment of CVST in SLE consisting of anticoagulant, steroid, and immunosuppressant. This case elicits intriguing problem: CVST and thrombocytopenia. Anticoagulant treatment is proposed as the cornerstone treatment for CVST, however it was deferred due to risk of bleeding in thrombocytopenia. Steroid plays role in treatment of CVST in SLE, owing to its anti-inflammatory property. As shown in previous cases, the patient had remarkable response to high dose steroid treatment and eventually got anticoagulant after her platelet had increased. In summary, prompt diagnosis and treatment of CVST are important for a favorable prognosis."}, {"id": "pubmed23n0410_9415", "title": "[Rare, but important chronic liver diseases].", "score": 0.009345794392523364, "content": "The presence of steatosis and inflammatory infiltrate in liver biopsies is essential for the diagnosis of non-alcoholic steatohepatitis (NASH). These findings are similar to those with alcoholic liver disease. However, in the NASH-situation alcohol doesn't play an important role. Risk factors for the development of NASH are obesity and diabetes. Most of the patients are clinically asymptomatic. This means, that a diagnosis of NASH is a diagnosis of exclusion: Viral induced, autoimmune, metabolic and toxic liver disease have to be excluded. The disease has a benign clinical course. The risk of cirrhosis is low. So far, there is no established treatment. Preliminary reports suggest a positive effect of weight-loss and ursodeoxycholic acid. Wilson's disease, a copper storage disorder, in which biliary copper excretion is reduced, is inherited as an autosomal recessive trait. Most patients with Wilson disease become symptomatic between the ages of 6 and 15. In about 90% of patients serum ceruloplasmin levels and serum copper concentrations are reduced. Copper excreation is increased. Histologic examination of liver biopsy specimens reveals fatty infiltration, Mallory bodies and ballooned glycogen nuclei, abnormalities which are also found in alcoholic liver disease. The definitive diagnostic parameter is the quantitative determination of liver copper content (> 250 micrograms/g dryweight). Untreated Wilson disease is always fatal. Lifelong treatment with anti-copper drugs are essential, D-penicillamine being the firstline therapy. Hereditary hemochromatosis (HH) is an iron overload disease inherited as an autosomal recessive trait. The frequency of the disease is high. The first symptoms usually can be found at the age of 20-50 years. Arthralgia develops in up to 50% of the patients. Many organs are involved, most often the liver. The organ is usually enlarged, transaminases are always moderately elevated. Laboratory findings disclose a marked elevation in serum ferritin and transferrin saturation. More than 80% of HH-patients are homozygous for the C282Y-mutation in the HFE-gene. The firstline treatment of HH is phlebotomy. Treatment is lifelong. When serum ferritin drops below 50 micrograms/l, the frequency of phlebotomy should be reduced (4-12 per year). If the patient already has cirrhosis, the risk of HCC is very high."}, {"id": "pubmed23n0973_23808", "title": "Paraneoplastic Encephalopathy in a Patient With Metastatic Lung Cancer: A Case Study.", "score": 0.009259259259259259, "content": "300 °gL Reassure, possibly retest later HFE Genotype PhlebotomyNormal Counsel and consider non-HFE hemochromatosis Serum ferritin – 300–1000 °g/L LFT normal Serum ferritin > 1000 °g/L and/or LFT abnormal Serum ferritin <300 °g/L LFT normal Observe retest in 1–2 years C282Y Homozygote C282Y/H63D (Compound Heterozygote) Confirmed iron overload *For convenience both genotype and phenotype (iron tests) can be performed together at a single visit in first-degree relatives. Liver biopsy No iron overload Investigate and treat as appropriate"}, {"id": "pubmed23n0777_87", "title": "Acute cytomegalovirus hepatitis in an immunocompetent host.", "score": 0.009009009009009009, "content": "A 52-year-old woman presented with a 1-week history of recurrent fevers and joint pains accompanied by abdominal and low back discomfort. She has a history of hypoparathyroidism and is on calcium supplements. Physical examination revealed fever and tachycardia. The rest of the examination was normal. Laboratory tests showed newly increased transaminase activity. Serum bilirubin and prothrombin time were normal. She was admitted for evaluation of acute hepatitis. Serology for hepatitis A, B, C and HIV were negative. Her serum acetaminophen and alcohol were undetected. Abdominal imaging was normal. Cultures were sterile. Additional tests for uncommon viral hepatitis included herpes simplex virus, cytomegalovirus and Epstein-Barr virus. Liver biopsy revealed non-specific inflammation. Subsequently, cytomegalovirus serology showed an IgM positive and negative IgG titre. Cytomegalovirus DNA qualitative PCR was also positive. No antiviral medication was given. She continued to have intermittent daily fever but reported no associated symptoms. She was discharged 9 days after admission in stable condition per her request with the advice to follow-up in the clinic in 1 week. Her serum hepatic profile returned to normal and she reported no more episodes of fever. Repeated titres of cytomegalovirus serology showed seroconversion. "}, {"id": "pubmed23n0074_3744", "title": "[Primary biliary cirrhosis--analysis of clinical material].", "score": 0.009009009009009009, "content": "The analysis is presented of 67 patients with primary biliary cirrhosis treated in the years 1971-1988. The group consisted of 60 women (89.5%) and seven men (10.5%). Presenting symptoms were mostly itching (66%) and jaundice (12%). The time between the onset of the first symptoms and the diagnosis was three years, on the average. Autoantibodies to mitochondria were present in 86% of patients. In 27% of cases markers of HB virus infection were found. Cholelithiasis was present in 19 patients (28%). Primary biliary cirrhosis was diagnosed with delay. Third degree of histological changes was observed in 38.8% of cases and in 18% of patients it was already fourth degree. Eighteen patients (27%) died after six years, on the average, from the onset of symptoms. The direct causes of death were, most frequently, liver failure and haemorrhage from oesophageal varices."}, {"id": "pubmed23n0327_11192", "title": "[Manganese superoxide dismutase-inhibiting autoantibodies in cholestatic Epstein-Barr viral hepatitis].", "score": 0.008928571428571428, "content": "A 21-year-old woman reported no serious previous illness. For 3 days before admission she had a fever, headache and joint pains. She had become progressively more jaundiced. Physical examination was normal except for enlarged liver and spleen, swollen lymph nodes and facial oedema. GOT (30 U/l), GPT (33 U/l) and alkaline phosphatase (172 U/l) were slightly elevated. Serum bilirubin was raised to 12.4 mg/dl. The total white blood cell count was normal, but there were 45% atypical lymphocytes (activated T lymphocytes). Abdominal sonography and endoscopic retrograde cholangiopancreatography were unremarkable. Serology for hepatitis A, B and C as well as for antimitochondrial antibodies was negative, but there were specific IgM (1:640) and IgG antibodies (1:80) against Epstein-Barr virus (EBV) capsid antigen in the immunofluorescence test. The EBV infection (infectious mononucleosis) was complicated by cholestatic hepatitis. High concentrations (1832 Göttingen units/ml) of enzyme-inhibiting autoantibodies against the antioxidative enzyme manganese-superoxide dismutase (MSD) were demonstrated. The autoantibodies reduced the antioxidative action of the enzyme by more than 70% and favoured the oxidative cell damage in vitro. After bed-rest for one week without further treatment the symptoms improved and the abnormal laboratory values, including the autoantibodies against MSD, regressed. Autoantibodies against MSD are formed during an acute infection with EBV. Their enzyme-inhibiting action promotes abnormalities of oxidative cell function and may thus be the cause of cholestatic hepatitis in this infection."}, {"id": "pubmed23n0338_15316", "title": "[The characteristics of the course of viral hepatitis C].", "score": 0.008928571428571428, "content": "Time-related course and results of examination were studied in patients with viral hepatitis C, having gotten infected through blood. Based on the comparison of similar indicators in patients with viral hepatitis B particular features were revealed of the clinical course of viral hepatitis C. These are as follows: short-in-duration prejaundice period, in other instances there is no prejaundice period at all, subfebrile states set in quite often; among other features are dyspepsia-like events together with signs of a damage to pancreas. There has been noted a slow dynamics of the icteric period reversibility, with biochemical indicators showing the same tendency."}, {"id": "pubmed23n0620_20686", "title": "Idiopathic autoimmune hemolytic anemia due to lecithin overdose: a case report.", "score": 0.008849557522123894, "content": "Idiopathic Autoimmune Hemolytic Anemia is a potentially fatal condition which requires prompt and potent treatment. Diagnosis of idiopathic autoimmune hemolytic anemia requires both serologic evidence of autoantibody presence and hemolysis. Although most of the times it is considered idiopathic, several underlying causes have been identified, like autoimmune and connective tissue diseases, viral infections, drugs or hyper function of the immune system. To our knowledge, this is the first case in the international literature describing lecithin-induced autoimmune hemolytic anemia. This case report is to highlight a rare but dangerous adverse reaction to overdose of lecithin. A 38 year old white female from Greece, presented to our emergency room with progressive fatigue over a period of ten days and icteric discoloration of her skin and conjunctiva. The patient had been taking lecithin supplements (1200 mg, 3 capsules a day) over a period of ten days for weight loss. She reports that the last 3 days, prior to the examination, she took 5 capsules/day, so that the supplement would take effect more rapidly. Her past medical, social and family history showed no disturbance. Relatives of the patient were requested to submit any blood-tests taken over a period of 20 days prior to the onset of symptoms caused by Lecithin. All tests proved that all functions were within normal scale. Her physical examination revealed pallor and jaundice without palpable hepatosplenomegaly. Blood biochemistry tests showed total bilirubin 7.5 mg/dl, with indirect bilirubin 6.4 mg/dl and complete blood count showed hemoglobin 7.6 g/dl with blood levels 21.4%. In every case of idiopathic autoimmune hemolytic anemia the administration of pharmaceutical substances should always be examined, except for the standard reasons that cause it. In this case the cause of hemolysis was attributed to the excessive intake of lecithin capsules for the loss of body weight. It is important that clinicians and immunologists are aware of this adverse effect."}]}}}} {"correct_option": 4, "explanations": {"1": {"exist": true, "char_ranges": [[645, 813]], "word_ranges": [[102, 126]], "text": "Temporomandibular ankylosis is not considered because, although relatively close to the orbital cavity, it is not part of the orbito-malar complex (option 1 discarded)."}, "2": {"exist": true, "char_ranges": [[814, 965]], "word_ranges": [[126, 149]], "text": "Involvement of the maxilla can lead to dental malocclusion, but usually occurs in fractures located lower than the orbital cavity (option 2 discarded)."}, "3": {"exist": true, "char_ranges": [[966, 1160]], "word_ranges": [[149, 179]], "text": "Naso-ethmoidal fractures are included in midface fractures, but the bones of the nose are located more anteriorly to the medial orbital rim, and therefore outside the orbit (option 3 discarded)."}, "4": {"exist": true, "char_ranges": [[130, 644]], "word_ranges": [[23, 102]], "text": "Assuming therefore that we are asked which is one of the most frequent complications of the floor of the orbit, two complications should always be highlighted that may indicate surgical treatment, even urgent: diplopia, due to dislocation of the inferior rectus muscle to the underlying maxillary sinus (and even its entrapment); and enophthalmos, which may cause other associated complications in the medium and long term, such as superior palpebral pseudo-ptosis due to loss of orbital volume (option 4 correct)."}, "5": {"exist": false, "char_ranges": [], "word_ranges": [], "text": ""}}, "full_answer": "Fractures of the midface in the orbito-malar region may involve the floor and, to a lesser extent, the lateral wall of the orbit. Assuming therefore that we are asked which is one of the most frequent complications of the floor of the orbit, two complications should always be highlighted that may indicate surgical treatment, even urgent: diplopia, due to dislocation of the inferior rectus muscle to the underlying maxillary sinus (and even its entrapment); and enophthalmos, which may cause other associated complications in the medium and long term, such as superior palpebral pseudo-ptosis due to loss of orbital volume (option 4 correct). Temporomandibular ankylosis is not considered because, although relatively close to the orbital cavity, it is not part of the orbito-malar complex (option 1 discarded). Involvement of the maxilla can lead to dental malocclusion, but usually occurs in fractures located lower than the orbital cavity (option 2 discarded). Naso-ethmoidal fractures are included in midface fractures, but the bones of the nose are located more anteriorly to the medial orbital rim, and therefore outside the orbit (option 3 discarded).", "full_answer_no_ref": "Fractures of the midface in the orbito-malar region may involve the floor and, to a lesser extent, the lateral wall of the orbit. Assuming therefore that we are asked which is one of the most frequent complications of the floor of the orbit, two complications should always be highlighted that may indicate surgical treatment, even urgent: diplopia, due to dislocation of the inferior rectus muscle to the underlying maxillary sinus (and even its entrapment); and enophthalmos, which may cause other associated complications in the medium and long term, such as superior palpebral pseudo-ptosis due to loss of orbital volume ([HIDDEN]). Temporomandibular ankylosis is not considered because, although relatively close to the orbital cavity, it is not part of the orbito-malar complex ([HIDDEN]). Involvement of the maxilla can lead to dental malocclusion, but usually occurs in fractures located lower than the orbital cavity ([HIDDEN]). Naso-ethmoidal fractures are included in midface fractures, but the bones of the nose are located more anteriorly to the medial orbital rim, and therefore outside the orbit ([HIDDEN]).", "full_question": "20-year-old patient who comes to the emergency department after suffering a bicycle accident with facial trauma. A cranial CT scan was performed showing a fracture of the middle third of the face involving the orbito-malar region. One of the most frequent complications of this type of fracture is:", "id": 621, "lang": "en", "options": {"1": "Temporomandibular ankylosis.", "2": "Dental malocclusion.", "3": "Naso-ethmoidal pseudoarthrosis.", "4": "Enophthalmos.", "5": NaN}, "question_id_specific": 60, "type": "OPHTHALMOLOGY (ECTOPIC)", "year": 2022, "rag": {"clinical_case_options": {"MedCorp": {"RRF-2": [{"id": "pubmed23n0584_7976", "title": "Panfacial fractures: analysis of 33 cases treated late.", "score": 0.01951265943270512, "content": "The aim of this retrospective study was to analyze the characteristics of delayed panfacial fractures and evaluate treatment results. Thirty-three patients with delayed panfacial fractures were treated in the Maxillofacial Trauma Center of Peking University, School and Hospital of Stomatology between 1998 and 2004. Each patient was examined by computed tomography (CT) scans before operation. For those who had no severe opening restriction, dental impressions were taken to fabricate dental casts. For those with severely comminuted fractures, 3-dimensional (3D) models of the facial skeleton were used. Re-establishing the continuity of the mandible was the first step and then used as a platform to reconstruct the maxillary fractures via maxillomandibular fixation after Le Fort I osteotomy. The third step was to restore the mid- and upper-facial width and projection by coronal approach to expose the zygomatic complex and frontal bone/sinus and/or naso-orbito-ethmoid (NOE) fractures. There were 3 types of mandibular fractures that affected the treatment plan: 1) type I, mandibular body/symphysis fracture(s) (17/33, 51.52%); 2) type II, mandibular angle and/or condylar fracture(s) (6/33, 18.18%); and 3) type III, both mandibular body/symphysis and angle/condylar fractures (10/33, 30.30%). Fourteen cases were associated with NOE fractures (42.42%) and 3 cases had frontal sinus fractures (9.1%). Twelve cases had enophthalmos (36.36%) and 3 lost 1 eyeball. The order of treatment was dependent on the mandibular fracture type. For type I fractures, reconstructing the mandibular arch was the first step. For type II fractures, repairing the angle, ascending rami, and condylar areas was the first step. For type III fractures, when both mandibular height and arch were disrupted, freeing the malunited angle or condyle was the first step before restoring the mandibular arch form. Reconstruction of the mandibular height and projection was then carried out. For all 3 types, the second step was to restore the mid- and upper facial width and projection by reducing the zygomatic complex and frontal bone/sinus or NOE fractures. Maxillary fixation across the Le Fort I level was the last step. Le Fort I osteotomy was used for all 33 cases. Bone grafts and soft tissue suspension also were used. Twenty-one cases (63.64%) had good results, 7 (21.21%) cases were acceptable, and 5 (15.15%) were not good. There were 7 cases (21.21%) that still had soft tissue problems that needed secondary operations. Reconstruction of the mandible first with Le Fort I osteotomy is a good way to treat delayed panfacial fractures. Computed tomography and 3D CT, model surgery, and occasionally 3D models are necessary aids for diagnosis and treatment. Soft tissue problems, including lacerations and asymmetries, were often the factors that caused an unfavorable outcome."}, {"id": "pubmed23n0794_2527", "title": "Etiology, incidence and patterns of mid-face fractures and associated ocular injuries.", "score": 0.019419306184012067, "content": "A prospective study on mid-face fractures was carried out in the Department of Oral and Maxillofacial Surgery at College of Dentistry, Indore, from August 2007 to September 2009 to analyze etiology, incidence and patterns of midface fractures and associated ocular injuries. Two hundred patients were included in this study, amongst those who reported to the Department of OMFS, College of Dentistry, Indore. After confirmed diagnosis of mid face fracture all the patients were stratified according to age, sex, cause of the accident, influence of alcohol, location, type of fractures and associated ocular injuries. The study included 200 patients with a mean age of 29.6 years. The most frequently injured patients belonged to the 21-30 year-old age group. The male predilection was 76 %. Road traffic accident was the most common causative factor (64 %), followed by assault (21 %), cases of fall (9.5 %) and other causes (5.5 %). The most common fracture in this study was found to be zygomatic complex fractures (62.5 %) (more in the age group of 21-30 years). This was followed by Lefort II fractures (23 %), multiple fractures (10 %) and Lefort I fractures (6 %), Lefort III fractures (4.5 %) and Naso-ethmoidal fractures (4 %) in descending order. 84.5 % subjects were having ocular involvement. Subconjunctival hemorrhage was present mostly in 83.5 % followed by remaining as corneal injury 15 %, reduced acuity 11.5 %, diplopia 10.5 %, enophthalmos 8.5 %, telecanthus 5 %, hyphema 3.5 %, blindness 3 % and proptosis 0.5 %. Zygomatic complex fractures were the most frequent type of injury that was complicated by blindness or a serious eye injury (61 %). Collection of data regarding the epidemiology of maxillofacial fractures is important because it may assist healthcare providers to provide necessary information for the development and evaluation of preventive measures. Ocular injuries should have an early ophthalmological examination at the time of trauma to detect any kind of ocular dysfunction. "}, {"id": "pubmed23n1042_9268", "title": "Maxillofacial Trauma in Geriatric Population.", "score": 0.0186932215234102, "content": "The worldwide population is increasingly aging. Maxillofacial fractures of the geriatric population have been increased. Evaluation of the demographic variables, causes and the patterns of maxillofacial traumas in the elderly population is the main aim of this study. Seven hundred thirteen maxillofacial tomography images which were scanned between 2010 and 2019 were evaluated. Data from 50 patients aged 65 years old and/or older, who were treated for maxillofacial fracture at the Department of Otorhinolaryngology, Gaziantep University, were retrospectively analyzed. Two groups were created according to the facial fracture pattern. Facial fractures were reclassified into 2 groups; mandibula, orbital, zygomaticomaxillary complex group fractures and the other group of frontal, naso-orbito-ethmoid fractures and were used as a comparison. The mean age of the patients was 72.5 (min 65- max 93). The gender distribution was 17 females (34%) and 33 males (66%). The most common fractured bone was the nasal bone and the least one is the frontal bone. Approximately one-quarter of 50 fractures were seen in 70 to 79 years old. Falling is more common in females and men are more prone to work-related accidents than home-related accidents. Facial fractures in the elderly often seen in midface location. Falling is the common etiology of facial fracture in all genders at elderly. However, male dominance is seen in other etiological factors. Additional diseases in the elderly seem to increase the severity of facial fracture."}, {"id": "pubmed23n1152_16109", "title": "Ophthalmic Complications in Maxillofacial Trauma: A Prospective Study.", "score": 0.016693108919086708, "content": "To determine the incidence and types of ophthalmic complications associated with maxillofacial trauma over a period of 24 months. An institutional prospective study was conducted on 62 patients presenting with maxillofacial trauma to study the correlation between facial trauma and ophthalmic complications. Road traffic accidents were reported to be the primary etiologic factor for most trauma cases studied. Zygomaticomaxillary complex (ZMC) fracture was associated with more ophthalmic complications while fractures involving the orbital rims and walls were associated with severe complications. Maxillofacial trauma, particularly those associated with midface, including ZMC fracture, Le Fort II, Le Fort III, and naso-orbito-ethmoidal fractures, can commonly cause ophthalmic complications and blindness in rare cases. Hence, every patient with maxillofacial trauma should undergo an ophthalmic examination and should be placed under close observation for necessary treatment when required."}, {"id": "pubmed23n0369_18125", "title": "Late sequelae after high midface trauma.", "score": 0.016025641025641024, "content": "The upper midface area comprises mainly the naso-orbito-ethmoidal (NOE) region which plays a paramount role in facial expression. Fractures of this area often result in neglected bony defects in the fragile periorbital region with major secondary impairments such as traumatic telecanthus, orbital dystopia, and/or enophthalmos. Permanent cranial nerve deficits also can occur as the result of post-traumatic/post-operative sequelae. Seventy-one patients (age range 7-78 years) with severe high midface trauma, treated from January 1989 to December 1996, were reviewed with a minimum follow-up of 2 years. The patient population has been distributed according to the fracture type in three groups: Group 1 (n = 35): Isolated NOE with/without associated central midface injury; Group 2 (n = 22): NOE associated with craniofacial injury and Group 3 (n = 14): NOE associated with orbital displacement. The estimated post-surgical parameters included qualitative and quantitative data from the long-term clinical evaluation. Persistent headache and/or concentration difficulties were mainly noted in Group 1. Smell reduction or anosmia was reported mainly in Group 2. Deficits of the trigeminal and/or the facial nerve were found in Group 3. Enophthalmos and/or telecanthus were predominantly seen with injuries associated with orbital displacement."}, {"id": "pubmed23n1091_5003", "title": "Nasoorbitoethmoid fractures in a tertiary care hospital of eastern India: A prospective study.", "score": 0.015685328185328185, "content": "The purpose of this study was to report on the pattern of occurrence of nasoorbitoethmoid (NOE) fractures in Odisha and the various factors that influence their distribution. The study period was from January 1, 2016 to December 15, 2017. After approval from the Institutional Ethics Committee, all patients diagnosed with naso-orbito-ethmoid fractures reporting to the department of OMFS and Level-1 trauma centers were included in the study. Sociodemographic data along with the etiology and type of fracture were mentioned. Associated injuries to other body parts were noted. Open reduction was possible only in five cases of NOE fractures. The treatment plan including the operative approach and postoperative results was evaluated. A total of 1192 patients with facial fracture were seen, of which 52 (4.36%) patients had NOE fractures. Males far outnumbered females in a ratio of 9:1. Thirty-three patients (63.46%) had unilateral NOE fracture, while the rest 19 (36.54%) had bilateral NOE fracture. Sixteen (30.76%) cases were classified as Type I, 35 (67.30%) as Type II, and 1 (1.92%) as Type III. Road traffic accidents were the most common cause of NOE fractures (69%), followed by fall (17%) and assault (10%). The most common neurological injury to be associated with NOE fractures was pneumocephalus (29%), followed by diffuse axonal injury (8%). Telecanthus (100%) was found to be the primary clinical feature in patients of NOE fracture, followed by a depressed nasal bridge (92%). Fracture of the nasal bone was invariably associated with NOE fracture. Complications observed due to untreated NOE fractures included a shortened and retruded nose, shortened palpebral fissures, telecanthus, and enophthalmos. Contemporary management of NOE complex fractures demands precise diagnosis and immediate surgical management with anatomic reduction and rigid fixation of the involved bone segments. With an improvement in socioeconomic status and increased awareness among maxillofacial surgeons, hopefully, a greater number of NOE fracture patients will avail the benefits of open reduction in future."}, {"id": "pubmed23n0873_4203", "title": "Fractures in the Maxillofacial Region: A Four Year Retrospective Study.", "score": 0.014916269269158258, "content": "The incidence of maxillofacial injuries is on the rise due to motor vehicle accidents and increased incidence of violence in recent times. The aim of this retrospective study was to determine the incidence, aetiology, the pattern of fractures, their management with open reduction and internal fixation (ORIF) and complications, if any. A retrospective analysis of 621 fractures in 361 patients managed by ORIF over a four year period was carried out. The average age of patients was 24.3 years with a male to female ratio of 21.2:1. Panfacial fractures comprised 4.7%, frontal bone fractures 8.9%, orbital fractures 0.7%, naso-orbito-ethmoid complex (NOE) fractures 0.7%, zygomatic complex fractures 23.5%, fracture maxilla 11.5% and mandibular fractures 52.2% of all facial fractures. All the cases were successfully managed by ORIF under general anaesthesia (GA). Complications were noticed in 6.8% of cases in the form of reactive implants in 3.6%, deranged occlusion in 1% and infection at operated site in 1% cases which were managed satisfactorily. The findings of this study reveal sharp annual increase in the number of cases of maxillofacial trauma. Road traffic accidents (RTA) were the commonest cause and the age group most affected was between 20-25 years. ORIF of these fractures was chosen for its obvious advantages of direct anatomical reduction, early return to function and minimal complications."}, {"id": "pubmed23n0930_2873", "title": "Facial and Orbital Fractures: A Fifteen Years Retrospective Evaluation of North East Sicily Treated Patients.", "score": 0.013696831787152109, "content": "Orbital fractures are classified as diseases usually related to common midface trauma. It represents the most challenging treatment due to the complex anatomy, physiology, and aesthetic role. A midface trauma involves also the zygomatic complex and the nose, however the orbit fracture seems to be a more frequent disease due to its anatomical features. The purpose of this work is to retrospectively evaluate and record the frequency of the midfacial traumas and orbital fractures observed in the North Eastern Sicily. The results of the present data may be useful for the clinicians in order to recognize the kind of fracture just from the first general visit having a quick diagnosis and management. In the years between 2001 and 2016, about 1200 patients with midfacial trauma and about 100 patients involving the orbital floor have been evaluated. All those patients underwent the surgical fracture reduction and a CT scan follow up control at one month, three months, six months and one year. Data showed high percentage of orbital floor, nose and mandibular body and ramus fractures; moreover the most frequent causes of fractures seem to be related to motor vehicle accident, followed by assaults, work and fall. The results have highlighted the changing trends in the causes of facial injuries, particularly the increasing incidence of assaults and the falling incidence of motor vehicle accidents in developed countries. The quick diagnosis and management proved fundamental for the successful treatment. Clinicians should be able to recognize the first symptoms in order to avoid possible complications."}, {"id": "pubmed23n0830_3757", "title": "Analysis of symptoms according to areas of orbital floor in orbital inferior wall fractures.", "score": 0.009932958690722665, "content": "A considerable number of patients experiencing facial trauma are diagnosed with blowout fracture. Preoperative computed tomographic scan is often different from the actual surgical area. This study is restricted to orbital floor fracture. This study is expected to help speculating fracture site and making surgical plans according to symptoms of periorbital trauma. From March 2005 to September 2013, a total of 150 cases of orbital floor fracture surgeries have been analyzed. This study analyzed the preoperative symptoms at the certain fractured area of orbital floor, at the aspects of sagittal view of computed tomography, which is sectioned into anterior one-third, middle one-third, posterior one-third, and mixed types. Symptoms for analysis are diplopia, extraocular movement limitation, enophthalmos and other combined facial bone fractures, and the like. Fracture areas of orbital floor are 21 cases (14%) of anterior one-third, 47 cases (31%) of middle one-third, 7 cases (5%) of posterior one-third, and 75 cases (50%) of the mixed. Frequency of diplopia was 0 case, 24 cases (42.1%), 4 cases (7.0%), and 29 cases (50.9%), respectively. In the case of extraocular movement limitation, 0 case, 15 cases (39.5%), 2 cases (5.3%), and 21 cases (55.2%) were found, respectively. In the case of enophthalmos, 0 case, 5 cases (16.7%), 7 cases (23.3%), and 18 cases (60.0%) were found, respectively. The most commonly associated other facial bone fractures were nasal bone fractures. In the case of blowout fracture, diplopia, extraocular movement limitation, enophthalmos, and other symptoms are checked through physical examination. This study would help speculating fracture site and making surgical plans according to symptoms of periorbital trauma."}, {"id": "pubmed23n0614_19472", "title": "[Secondary reconstruction of posttraumatic orbital deformities with canthus dislocation].", "score": 0.009900990099009901, "content": "To explore the secondary surgical reconstruction for orbital bone deformities accompanied with canthus dislocation after trauma. From June 1998 to July 2007, 37 patients with secondary orbital bone fracture deformity accompanied with medial or lateral canthal ligament dislocation posttraumatically were treated, among whom there were 22 males and 15 females, aged 13-46 years old (21 on average). There were 29 cases of traffic accident, 6 of boxing injury and 2 of beating injury by sticks. The latest reconstruction was performed on these 37 cases during 3 months to 8 years after injuries. There were 11 cases of orbital maxillary zygoma (OMZ) fracture, 15 of naso-orbito-ethmoid (NOE) fracture, 8 of OMZ and NOE fracture and 3 of frontal fracture. There were 31 patients who were reconstructed for the first time and 6 for the second time. Typical bicoronal and subciliary incisions and intra-oral approach were employed to expose all the fractured sites. According to the fractured position and the degree of deformity and dislocation, the orbito-zygomatic fracture was repositioned after osteotomy and rigid fixation, or the healed fragments were trimmed with a burr and the depressed fragments were filled with autogenous bone such as ilium, cranial outer table or Medpor in order to reconstruct orbital wall framework; the orbital walls were repaired to correct the enophthalmos with autogenous bone or Medpor after the herniated orbital contents were released. The medial canthal ligament was anchored superior-posteriorly to the lacrimal fossa with transnasal wires fixation or fixed with titanium miniplates and nails. The 36 patients' incisions obtained healing by first intention after the operation, and 1 case failed because of wound infection from maxillary sinusitis. There were 24 patients who were cured successfully with facial appearance and function improved significantly. During the follow-up for 3-6 months, no complication was found such as dislocation of the implant, rejection and infection. Two patients still showed slight enophthalmos while 3 patients with canthus dislocation regained improved appearances but not satisfactory. At 6 months after operation, the CT scan conducted in 3 patients with autogenous bone and Medpor grafting showed all fractures were fixed rigidly. Surgical reduction combined with bone grafting is a satisfactory method for the correction of secondary orbital bone deformity, and the repair of canthus dislocation and correction of enophthalmos should be considered at the same time. An ideal result could be achieved only through all-round consideration and comprehensive treatment."}, {"id": "pubmed23n0573_8784", "title": "Occurrence of mandibulofacial injuries presenting to the otorhinolaryngology and head & neck surgery department.", "score": 0.009900990099009901, "content": "Trauma is the fourth major cause of mortality in the Western countries, of which approximately one half involve maxillofacial injury. Statistics reported by emergency room officials show motor vehicles cause many of the injuries and deaths that occur in Iran. Having completed a retrospective descriptive study of 200 patients who experienced maxillofacial trauma, the authors report its occurrence with respect to age, sex, trauma type, and site of injury so as to evaluate the operational functionality of the Department of Otolaryngology and Head & Neck Surgery of Hazrat-e Rasoul Akram Hospital from 2000 to 2004. Mandibular fractures (36.2%) occurred in the subsequently listed sites and at the specified frequencies: mandibular angle (9.7%), mandible body (6.9%), parasymphysis, ramus and subcondyle at 5.6% each, and symphysis at 2.8%. No condylar fractures were reported. Frontal bone fracture was observed in 9.7% of the patients with eye globe injury occurring simultaneously in 8.3% of corresponding cases. Orbital fracture (63.9%) also occurred in various cases as follows: orbital floor 39%, lateral rim 24%, inferior rim 22%, medial wall 11%, and the superior rim and orbital roof at 2% each. Motor vehicle accidents were the most common causes of trauma (42%). The most common fracture was in the zygoma (43%) with 8.3% of them being orbital injury. Fractures of mandibular bones (36.2%) and the maxilla (33%) were the most commonly seen in trauma occurring to the maxillomandibular region."}, {"id": "pubmed23n0083_4231", "title": "[Clinical study of mandibular condyle injury].", "score": 0.009708737864077669, "content": "Mandibular condyle fractures develop frequently and show the variable type of injury and complication. New opinions have emerged from recent investigation into condylar fractures. The author investigated 246 patients with condylar fractures who visited SNUDH from January 1980 to August, 1988, 8. with regard to clinical and treatment aspects, area and displacement of fractures, associated teeth injury and other body injury, complications. At last I have got the following results. 1. The incidence to condylar fractures in a series of 765 mandibular fractures may be as high as 32.2%. 2. The male patients are 3 times more than female patients. The highest frequency was recorded in the group 21-30 years of age. (34.1%). 3. Falls caused the greatest number of condylar fractures (45.2%) and next was in assult (25.6%), traffic accidents (22.4%). 4. Unilateral condylar fractures were present in 74.8%, giving a left: right ratio of 1.2:1. In cases of unilateral fracture, subcondylar fractures were by far the commonest (32.9%) but in cases of bilateral fracture, condylar neck fractures were by far the commonest. In children under 15 years of age, condylar neck fractures were more common but in patients over 16 years of age, subcondylar fractures were common. 5. Anteromedial fracture dislocations were by far the commonest (20.3%). In children under 15 years of age, fracture deviations were common but in patients over 16 years of age, fracture displacements were common. 6. 44.7% of patients with condylar fractures sustained the teeth injuries. Teeth fractures were by far the commonest. 7. Single condylar fractures showed a frequency of 30.5%. Of the concomitant fractures elsewhere in the mandible, symphysis fractures were by far the commonest (54.1%). 8. Associated other body injuries showed a frequency of 28.0%. Of them, head injuries were by far the commonest. 9. The mean interval from injury to treatment was 14.3 days. Of the treatment of condylar fractures, open reduction was by far the commonest (70.3%). Closed reduction comprised 19.9% and functional therapy comprised 8.5%. 10. In 67 patients with possible follow up period, the following complications were developed, two ankylosis, anterior open bite, mouth opening limitation, mouth opening deviation."}, {"id": "pubmed23n1003_17152", "title": "Patterns of facial fractures in children.", "score": 0.009523809523809525, "content": "Morbidity and mortality among children is usually the result of trauma. Because a child's face is retruded relative to the protecting skull, has a thicker layer of adipose tissue, more elastic bones, flexible sutures lines, the presence of tooth buds within the jaws, and the lack of pneumatisation of the sinuses, the facial bones fracture less commonly than in adults. Our aim was to assess the patterns of such fractures in children who presented to the department of Oral and Maxillofacial Surgery, King Edward Medical University/Mayo Hospital Lahore, Pakistan. All 535 eligible children between the ages of 1-16 years who presented during the two years December 2009 - December 2011 were included in the study. Facial fractures were diagnosed by clinical examination, plain radiographs, and computed tomography, and the pattern of fractures of the facial bones including the frontal bone, orbital bones, maxilla, zygoma, naso-orbito-ethmoidal complex, mandible, and dentoalveolar region was documented. The male:female ratio was 2:1 with 369 male (70%) and 166 female (31%) patients. Fall was the cause in 212 (39%), and in 167 (31%) it was road traffic accidents, while sports were the cause in 135 (25%). The naso-orboto-ethmoid complex was fractured in 37 cases (7%) while 104 children (19%) presented with isolated fractures of the zygomatic bone. The maxilla was fractured in 195 cases (36%), the mandible in 380 (71%), and dentoalveolar trauma was the cause in 256 (50%). The mandible was the bone that was most often fractured (mostly in boys and usually as a result of falls during summer vacations), with the peak occurring in those aged 8-12 years."}, {"id": "pubmed23n0748_7976", "title": "Treatment of complex facial fractures: clinical experience of different timing and order.", "score": 0.009523809523809525, "content": "Given the variability of the timing and order of surgeries, it is difficult to choose the best treatment for patients with complex facial fractures. Based on the clinical experiences, the authors have reviewed their experience with the timing and order of operations depending on the sites of complex facial fractures and their concurrent injuries. The current study was based on a total of 105 patients with complex facial fractures from the year 2002 to 2011. After assessing the patients' clinical records, radiological data, and clinical photographs, the following data were analyzed: patients' age and sex, causes of injury, concurrent injuries, sites of fractures, the interval between trauma and the operations, the presence of additional surgeries, and the aesthetic and functional outcomes.For most of the patients, early operation was performed (within 2 weeks in 95.2%). Additional surgeries within 1 month after injuries were performed in 22 patients. Usually, a top-to-bottom direction repair was applied when head injuries were involved, and bottom-to-top direction repair was applied when occlusal problems were involved. Of 105 patients whom we were able to follow up, 49 patients showed complications or were dissatisfied with the outcomes. However, except them, most of the patients were satisfied with the outcomes of surgical treatments. There were 14 cases of cheek asymmetry, 9 enophthalmos, 30 paresthesia, 4 malocclusion, and a single case of persistent trismus.In the current study, satisfactory results could be achievable under the following principles: a repair should be done in the early stage after the onset of the injury; supportive surgeries should be done, if necessary, within 2 weeks (no later than 4 weeks); and the order of surgical treatment should be determined by the severity of bone fracture and the systemic status."}, {"id": "pubmed23n1072_12427", "title": "Prevalence of Ocular Complications in Patients with Zygomatic Bone Fractures in an Iranian Population.", "score": 0.009433962264150943, "content": "Damages to the middle third of the facial bone generally involve the orbital skeleton and can lead to eye impairment. In this study, it is attempted to determine the incidence of ophthalmic injuries in maxillofacial trauma with zygomatic bone fractures. One hundred and fifteen cases with ophthalmic (ocular) involvement after maxillofacial trauma were referred to the Shariati Hospital, Tehran, Iran, and were visited at the Ophthalmology Department between 2016 and 2018. Zygomatic fractures and resulting ocular complications were evaluated in 87 males and 28 females with the mean ages of 26 and 32 years, respectively. Subconjunctival ecchymosis was detected in 23.07% of men and 21.05% of women. Displacement of the palpebral fissure was detected in 26.5% of men and 27.6% of women. Furthermore, the unequal pupillary level was observed in 18.37% of men and 15.78% of women. Diplopia was detected in 8.9% of men and 10.5% of women. Additionally, enophthalmos was observed in 23.1% of men and 25% of women. The most common ocular presentations in midfacial trauma are diplopia and reduced visual acuity. Even after the operation, a significant number of patients experience poor vision and diplopia. Ophthalmology consultation is essential for these patients."}, {"id": "article-25562_29", "title": "Naso-Orbito-Ethmoid Fractures -- Complications", "score": 0.00937242975459536, "content": "Mental health issues, as patients with facial injuries, are at greater risk of developing post-traumatic stress disorder or anxiety-related disorders, particularly those who were victims of assault. [31]"}, {"id": "pubmed23n0913_4425", "title": "A study of sports-related orbital fractures in Singapore.", "score": 0.009345794392523364, "content": "With an increased popularity of sport and active living worldwide, our study aims to explore the incidence and features of sports-related orbital fractures in Singapore. 1421 computer tomography (CT) imaging scans of the face and orbits done at the National University Hospital over a 24-month period from January 2013 and December 2014 were reviewed retrospectively for orbital fractures. We identified 483 orbital fractures of which sports injury was the fourth most common etiology (n = 65; 13.5%) after road traffic accident (n = 131; 27.1%), geriatric fall (n = 81; 16.8%) and workplace injury (n = 67; 13.9%). The three most common sport in orbital fractures were soccer (n = 20; 30.8%), bicycling (n = 11; 16.9%) and jogging (n = 8; 12.3%). The three most common fracture patterns were zygomatico-maxillary complex fractures (n = 24; 36.9%), isolated one wall blowout fractures (n = 19; 29.2%) and naso-orbito-ethmoid fractures (n = 7; 10.8%). Sports-related orbital fractures were associated with a low mean age of patients (45.9 years, range, 14-79 years), a higher proportion of males (n = 58; 89.2%) than that from geriatric falls (n = 37, 45.6%) (P < 0.01), a higher likelihood of unilaterality (n = 62; 95.4%) than that from traffic accidents (n = 99; 75.6%) (P < 0.01) and a lower likelihood of pan-facial involvement (n = 4; 6.15%) than that from traffic accident (n = 60; 45.8%) (P < 0.01). Sports-related orbital fractures are the fourth most common cause of orbital fractures. Though commonly seen in young male adults, in view of the aging population and people exercising more regularly, education of safety measures among sports users is paramount to preventing sports-related orbital fractures."}, {"id": "pubmed23n0401_12660", "title": "The causes and consequences of maxillofacial injuries in elderly people.", "score": 0.009345794392523364, "content": "The occurrence of trauma in older people is well-documented; however the incidence of maxillofacial trauma is scarcely reported. Therefore, the objective of this study is to determine the causes and consequences of maxillofacial trauma in older people. A five-year (March 95 - March 2000) retrospective study was carried out of all patients over the age of 65 years with facial trauma presenting to Accident and Emergency Department (A&E). The information was collected using the medical notes and discharge summaries. The Departments of A&E and Maxillofacial Surgery. A total of 42 patients' records were examined for study related data. A total of 42 patients were seen during the study period. Thirty-six gave a history of a fall, of which 15 had tripped, 5 had slipped, 3 resulted from a Transient Ischaemic Attack (TIA), 1 as a result of alcohol abuse, in 1 a prosthetic knee gave way and 11 gave no cause for the fall. Of the remaining 6 patients, 5 were assaulted and 1 had a wardrobe fall on top of him. The majority of the falls occurred during the winter months. Maxillofacial injuries were noted in 27 of the 42 patients. Sixteen patients had cheekbone fractures, 8 mandibular fractures, 2 midface and 1 orbital complex fracture. Twenty-five percent of cheekbone fractures and 50% of mandibular fractures were treated surgically. Medical history was noted in 27 patients. This study clearly demonstrates the majority of the facial trauma in the older people can be treated conservatively unless the patients complain of functional problems."}, {"id": "article-25562_23", "title": "Naso-Orbito-Ethmoid Fractures -- Differential Diagnosis", "score": 0.00925965527565117, "content": "Pan-facial fractures - these are fractures involving the upper, middle, and lower regions of the face."}, {"id": "wiki20220301en064_21413", "title": "Basilar skull fracture", "score": 0.009259259259259259, "content": "A basilar skull fracture is a break of a bone in the base of the skull. Symptoms may include bruising behind the ears, bruising around the eyes, or blood behind the ear drum. A cerebrospinal fluid (CSF) leak occurs in about 20% of cases and may result in fluid leaking from the nose or ear. Meningitis occurs in about 14% of cases. Other complications include injuries to the cranial nerves or blood vessels. A basilar skull fracture typically requires a significant degree of trauma to occur. It is defined as a fracture of one or more of the temporal, occipital, sphenoid, frontal or ethmoid bone. Basilar skull fractures are divided into anterior fossa, middle fossa and posterior fossa fractures. Facial fractures often also occur. Diagnosis is typically by CT scan."}, {"id": "wiki20220301en480_4380", "title": "Zygomaticomaxillary complex fracture", "score": 0.009217469795861685, "content": "There is an association of ZMC fractures with naso-orbito-ethmoidal fractures (NOE) on the same side as the injury. Concomitant NOE fractures predict a higher incidence of post operative deformity. Treatment Non-displaced or minimally displaced fractures may be treated conservatively. Open reduction and internal fixation is reserved for cases that are severely angulated or comminuted. The purpose of fixation is to restore the normal appearance of the face. Specific attention is given to the position of the malar eminence and reduction of orbital volume by realigning the zygoma and sphenoid. Failure to correct can result in rotational deformity and increase the volume of the orbit, causing the eye to sink inwards."}, {"id": "pubmed23n0879_2053", "title": "Do Radiologists and Surgeons Speak the Same Language? A Retrospective Review of Facial Trauma.", "score": 0.009174311926605505, "content": "The objective of the present study is to examine the concordance of facial fracture classifications in patients with trauma who underwent surgery and to assess the epidemiologic findings associated with facial trauma. Patients with trauma who underwent facial CT examination and inpatient operative intervention during a 1-year period were retrospectively analyzed. Patient demographic characteristics, the mechanism of injury, the radiology report, the surgical diagnosis, and clinical indications were reviewed. Fractures were documented according to bone type and were classified into the following subtypes: LeFort 1, LeFort 2, LeFort 3, naso-orbital-ethmoidal, zygomaticomaxillary complex (ZMC), orbital, and mandibular. Concordance between the radiology and surgery reports was assessed. A total of 115,000 visits to the emergency department resulted in 9000 trauma activations and 3326 facial CT examinations. One hundred fifty-six patients (4.7%) underwent facial surgical intervention, and 133 cases met criteria for inclusion in the study. The mean injury severity score was 10.2 (range, 1-75). The three most frequently noted injury mechanisms were as follows: assault (77 cases [57.9%]), a traffic accident (21 cases [15.8%]), and a fall (20 cases [15%]). The three most frequently noted facial bone fractures were as follows: mandible (100 cases [75.2%]), maxilla (53 cases [39.8%]), and orbit (53 cases [39.8%]). The five descriptors most frequently found in the radiology and surgery reports were the mandibular angle (25 cases), the orbital floor (25 cases), the mandibular parasymphysis (22 cases), the mandibular body (21 cases), and ZMC fractures (19 cases). A classification was not specified in 31 of the radiologic impressions (22.5%), with 28 of 31 radiologists expecting the surgeon to read the full report. The descriptors used in the radiology and surgery reports matched in 73 cases (54.9%) and differed in 51 cases (38.3%). No classifications were used by one or both specialties in nine cases (6.8%). For 38.3% of patients needing facial surgery, descriptors used in the radiologic and surgery reports differed. Speaking a common language can potentially improve communication between the radiology and surgery services and can help expedite management of cases requiring surgery."}, {"id": "pubmed23n0629_14904", "title": "Epistaxis as the only initial symptom in pediatric naso-orbital-ethmoid fracture complicated with meningitis.", "score": 0.00909090909090909, "content": "Epistaxis is a frequent finding in patients with facial trauma. Herein, we report an unusual presentation of pediatric naso-orbital-ethmoid (NOE) fracture with epistaxis as the only initial symptom. The course of the patient's condition was later complicated by meningitis, related in part to the delay in diagnosis. A 3-year-old girl with preexisting upper respiratory symptoms was involved in a traffic accident, sustaining blunt trauma to the right side of her face. During the initial examination, only right-sided epistaxis was noted. Five days later, she developed febrile convulsion and was admitted to the intensive care unit with other signs of meningitis such as mental status change and neck stiffness. Her craniofacial computed tomographic scan showed a right-sided NOE fracture with minimal displacement and without dura tear. The cerebrospinal fluid culture grew Streptococcus pneumoniae, which may be due to ascending infection as a result of cribriform plate fracture. Intravenous antibiotic therapy was initiated with good response, and she was discharged from the hospital after 2 weeks. The presence of epistaxis and periorbital bruise, together with other symptoms and signs, helps in the identification of NOE and cribriform plate fracture. A high index of suspicion with repetitive computed tomographic scans is necessary to achieve correct early diagnosis. Parental antibiotic therapy is indicated if ascending cerebrospinal fluid infection develops."}, {"id": "pubmed23n0071_4796", "title": "[Clinical studies on treatment of fractures of the zygomatic bone].", "score": 0.00909090909090909, "content": "The author has made clinical studies on treatment of fractures of the zygomatic bone in terms of frequency of fractures according to sex, age, fracture type, main manifestations and treatment methods from 106 patients with zygomatic bone fractures among 969 patients with maxillofacial bone fractures. The results obtained were as follow: 1. The frequency of malar bone fracture was 4 times more in male than that in female. 2. The most prevalent age of malar bone fracture was 21-30 years of age, and the nexts were followed 11-20, 31-40, 0-10, 41-50, 51-60, 61-70, and over 71 in the orders. 3. Among maxillofacial bone fractures, mandibular fracture was most prevalent as 76.3%, and the nexts were followed by the maxilla (10.8%), the molar bone (9.7%) and the nasal bone (3.3%). 4. Among 106 fractures of the malar bone, zygomatic bone fracture only was occupied 48.1%, but the rests were accomplished by another maxillofacial bone fractures. 5. In classification of molar bone fractures according to Knight and North's, group 3 fractures were most prevalent, and followed by group 2, 1, 4 and 6 in the orders. 6. Main manifestations were upper cheek flattening, lower eyelid ptosis, subconjunctival ecchymosis, epistaxis, difficulty of mouth opening, pain during mouth opening and others in the orders. 7. 93.4% of malar bone fracture has been treated surgically, but the rests treated conservatively, and [symbol: see text] shaped elastic stapler wire has been effectively used to get fixation at zygomatico-maxillary fractures."}, {"id": "pubmed23n0894_8066", "title": "Dental trauma and bicycle safety: a report in Italian children and adolescents.", "score": 0.009009009009009009, "content": "This retrospective study aims to analyze the pattern of oro-facial trauma from bicycle accidents in Italian children and adolescents, focusing on the safety devices used. The medical records of 1405 patients of the Dental Clinic of the University of Brescia, between the age of 0 to 18, who experienced a dento-facial trauma from the use of a bicycle, were analyzed. Data regarding age, gender, weight, height, dominant hand, type of bicycle, use of safety devices, location and type of dental trauma, teeth involved, bone fractures and soft tissue lesions were recorded. Statistical analysis was performed. The majority of the traumatic events occurred in children within the 8-10 years of age-range; 1085 teeth were injured, of which 975 permanent teeth (89.9%) and 110 primary teeth (10.1%). The most common dental lesions were the coronal fractures (complicated and not complicated) while the most frequently involved teeth were the upper central incisors; 11% of patients were also treated for maxillo-facial fractures. A protective helmet was worn only in 3% of the cases; not one patient wore a mouth-guard. The use of helmets was more frequent in children and adolescents riding racing-bikes competitively, compared to those who were mountain bikers (p < 0.05). Bicycle accidents can have serious oro-facial consequences. Therefore, national and regional efforts should be made in Italy to promote head and mouth protection in cycling."}, {"id": "pubmed23n0069_5048", "title": "The incidence and management of middle third facial fractures at the University College Hospital, Ibadan.", "score": 0.009009009009009009, "content": "A study of 59 patients with middle third maxillofacial fractures over a 5 year period was undertaken. The incidence of, pattern of fracture, type of accompanying injuries, and assessment of the outcome of given treatment were studied. The young adult male featured prominently in the study giving a male/female ratio of 14:1. The zygomatico-maxillary complex was fractured in 42.4% of the cases, while the Le Fort 1 fracture was the most common of the Le Fort Fracture types. Road traffic accidents accounted for 81.4% of the aetiological factors, while armed robbery attacks was the next common source of trauma at 6.8%. Treatment by the Gillies temporal approach and immobilization within the tissues techniques produced very satisfactory results in 96% of the patients who received treatment. Loss of vision in one eye was the most common residual complication."}, {"id": "wiki20220301en162_12653", "title": "Le Fort fracture of skull", "score": 0.008989898989898989, "content": "Signs and symptoms Le Fort I — Slight swelling of the upper lip, ecchymosis is present in the buccal sulcus beneath each zygomatic arch, malocclusion, mobility of teeth. Impacted type of fractures may be almost immobile and it is only by grasping the maxillary teeth and applying a little firm pressure that a characteristic grate can be felt which is diagnostic of the fracture. Percussion of upper teeth results in cracked pot sound. Guérin's sign is present characterised by ecchymosis in the region of greater palatine vessels. Le Fort II and Le Fort III (common) — Gross edema of soft tissue over the middle third of the face, bilateral circumorbital ecchymosis, bilateral subconjunctival hemorrhage, epistaxis, CSF rhinorrhoea, dish face deformity, diplopia, enophthalmos, cracked pot sound. Le Fort II — Step deformity at infraorbital margin, mobile mid face, anesthesia or paresthesia of cheek."}, {"id": "pubmed23n0090_20814", "title": "[Facial bone fracture--statistical analysis and clinical aspects].", "score": 0.008928571428571428, "content": "We experienced 102 cases of facial bone fracture during 16 months of 1986 to 1987. These cases were analyzed statistically concerning causes, age and locations of the fracture. These fractures have increased rapidly in number. The causes were classified into three types; occurrence during sport, traffic accident and fighting, which were equal in number. There were 85% males and 15% females in the patient cohort, which were concentrated at the ages of 10-20 years. A large part of the fractures was mostly consisted of maxillo-facial components (95%). These trends were similar to the previous report of our clinic (1972-1979). On the other hand, not only severe dysfunctioning cases but also complicated cases increased in number, so that the several clinical aspects were reported. Case 1: 17-year-old male presented with retraction of left cheek caused by Rugby foot ball, whose malar bone was dislocated backward and anticlockwise, was treated with oroantral reduction and with the intermaxillary packing of silicon blocks. Case 2: 10-year-old boy with complaint of double vision occurred by head blow to right eye. Pure type blowout fracture of the orbital floor was presented, which was reconstructed by silicon plate from the incision of the lower eyelid. Case 3: 59-year-old male presented with 6 month history of diplopia and retraction of left eye ball, had been under the conservative care by an eye doctor. X-ray examination showed the intraorbital soft tissue was blown out into the ethmoidal sinus. However the transethmoidal reduction was performed, the result was not satisfactory.(ABSTRACT TRUNCATED AT 250 WORDS)"}, {"id": "pubmed23n1055_4827", "title": "Craniofacial Trauma Due to Stone-pelting - Patterns of Injury and Management.", "score": 0.008849557522123894, "content": "The aims of the study were to elucidate the pattern of stone-pelting induced cranio-facial injuries and to document soft and hard tissue injuries, their management, and complications. A retrospective descriptive study was conducted using a sample of patients reporting to our department in the years 2015 to 2020. Cranio-facial injuries were assessed for soft and hard tissue injuries, including tissue loss, and corresponding management. Follow up ranged from 18 ± 6 months. & A standardized surgical regime was followed for patient management, which included primary survey, debridement of wounds, and routine primary repair of soft tissue. Bony defect reconstruction was performed by open reduction and internal fixation. Cranial bone was used as split calvarial graft in postcraniectomy cranioplasty procedures, which were performed after 6 months. Local flaps were used for the reconstruction of soft tissue defects. Being a military hospital, majority of cases fell in the 20 to 30 age group with a male preponderance. The etiology in all cases was stone-pelting. Among cranio-facial injuries, cranial vault injuries and mid-face injuries (71%) were most prevalent, mandibular fractures (24%) and remaining were soft tissue injuries (5%). & Frontal & parietal bone injuries were seen in 23.6% cases (n = 9) and orbito-zygomatic complex injuries were seen in (36.8%) cases (n = 14). Isolated blow-out fractures were seen in 4 patients of our series. 52.6% of patients of our series suffered associated soft tissue injuries to the head, face, and neck region. The most common cause of injury was due to the direct impact of stone hitting the mid-face/cranial vault and the most common pattern of injury was gross comminution of the skeleton. & 2 patients suffered ocular injuries that required management and 6 patients of our series who suffered head injuries to the cranium required a secondary cranioplasty procedure (n = 4) &The most commonly used technique for treatment was open reduction internal fixation, which was used in 89% of patients. Soft-tissue injuries overall occurred most frequently on the forehead, nose, lips, and chin which was managed by primary suturing. Cranial vault injuries & orbito-zygomatic complex fractures are most commonly seen in patients with stone-pelting injuries. Early management of such injuries improves outcomes in terms of function and restitution of preinjury skeleton structure. The most common patterns seen is gross comminution to the cranio-facial skeleton that can be treated with immediate primary wound repair after meticulous wound debridement and open reduction and internal fixation. Importance of stone-pelting as a cause of craniofacial injuries is highlighted as it leads to significant disruption of craniofacial skeleton."}, {"id": "pubmed23n0818_15083", "title": "Maxillofacial fracture experiences: a review of 152 cases.", "score": 0.008849557522123894, "content": "The fractures of facial structures lead to great morbidity. Cross-sectional studies are needed to evaluate the current state of maxillofacial traumas. Thus, this study aims to evaluate these experiences and to compare these results with the current literature. The medical records of the maxillofacial fracture cases hospitalized between January 2004 and November 2011 were examined. The age, sex, etiology, fracture localization and treatment method for each case were documented. The affected facial bones were grouped as mandible, maxilla, zygoma, naso-orbitoethmoid complex (NOEC) and blow-out. Nasal fractures were excluded. The cases were assigned to 3 groups with respect to age (below 16, above 65 and between 17 and 64). The chi Square test was used to assess the significance of the difference in mandibular fracture rates in the pediatric population compared to others. The total number of cases was 152. The total number of fractures was 185. Of the 152 cases, 117 were male and 35 were female. The average age was 31.4 (±18.3), ranging between 2 and 81. Thirty-one cases were 16 years old or less. Nine cases were 65 years old or more. Mandibular and zygomatic fractures were the most prevalent fractures in the adult group. Mandibular fractures were significantly more common in the pediatric age group compared to rest of the population (X(2), p<0.05). Traffic accidents were the most common etiological factor, with a 55.3% ratio. Open reduction and internal fixation was the most frequently conducted treatment modality in all age groups. Retrospective studies are important for the projection of future prospects. In summary, our results indicate that pediatric fractures are mostly in the lower face and usually affect the condylar region, which is consistent with the literature."}, {"id": "pubmed23n0743_11302", "title": "Zygomaticomaxillary complex fractures and their association with naso-orbito-ethmoid fractures: a 5-year review.", "score": 0.008771929824561403, "content": "Zygomaticomaxillary complex fractures associated with ipsilateral naso-orbito-ethmoidal fractures are more complex injuries than isolated zygomaticomaxillary complex fractures. This injury pattern can have significant long-term morbidity if not recognized and treated appropriately during the initial operation. The purpose of this study is to compare mechanisms of injury, treatment, and outcome between patients with zygomaticomaxillary complex fractures and those with zygomaticomaxillary complex and ipsilateral naso-orbito-ethmoidal fractures. A 5-year retrospective review of all patients treated with zygomaticomaxillary complex fractures at a level I trauma center was performed. Computed tomographic scans were reviewed to divide patients into those with zygomaticomaxillary complex fractures alone and those with zygomaticomaxillary complex and ipsilateral naso-orbito-ethmoidal fractures. Demographics, treatment protocols, outcomes, complications, reoperations, and length of follow-up were identified for both groups and compared to determine differences between these populations. A total of 245 patients were identified by the Current Procedural Terminology codes for zygomaticomaxillary complex fractures. One hundred eighty-five patients had zygomaticomaxillary complex fractures and 60 patients had zygomaticomaxillary complex/naso-orbito-ethmoidal injuries. The demographics for both populations were similar. There are differences between the groups with regard to mechanism of injury, operative findings, and techniques. The patients with zygomaticomaxillary complex/naso-orbito-ethmoidal fractures had higher rates of postoperative complications and deformities. Patients who sustain a zygomaticomaxillary complex fracture associated with an ipsilateral naso-orbito-ethmoidal fracture have a higher incidence of postoperative complications and deformities. It is important to recognize this fracture pattern early to help minimize postoperative morbidity. Risk, II."}, {"id": "pubmed23n0410_21117", "title": "An assessment of maxillofacial fractures: a 5-year study of 237 patients.", "score": 0.008771929824561403, "content": "This descriptive analytical study assesses the cause, type, incidence, demographic, and treatment data of maxillofacial fractures managed at our medical center during a 5-year period and compares them with the existing body of literature on the subject. A 5-year retrospective clinical and epidemiologic study evaluated 237 patients treated for maxillofacial fractures from 1996 to 2001 at one medical center. There were 211 male patients (89%) and 26 (11%) female patients. The patients ranged in age from 3 to 73 years, with 59.0% (140 patients) in the 20- to 29-year age group. A number of parameters, including age, gender, cause of injury, site of injury, type of injury, treatment modalities, and complications, were evaluated. All maxillofacial injuries were assessed and treated by a single oral and maxillofacial surgeon. Other concomitant bodily injuries were treated by appropriate consultant specialists. There were 173 (72.9%) mandibular, 33 (13.9%) maxillary, 32 (13.5%) zygomatic, 57 (24.0%) zygomatico-orbital, 5 (2.1%) cranial, 5 (2.1%) nasal, and 4 (1.6%) frontal injuries. Car accidents caused 73 (30.8%), motorcycle accidents caused 55 (23.2%), altercations 23 (9.7%), sports 15 (6.3%), and warfare caused 23 (9.7%) of the maxillofacial injuries. Regarding distribution of mandibular fractures, 32% were seen in the condylar region, 29.3% in the symphyseal-parasymphyseal region, 20% in the angle region, 12.5% in the body, 3.1% in the ramus, 1.9% in the dentoalveolar, and 1.2% in the coronoid region. The distribution of maxillary fractures was Le Fort II in 18 (54.6%), Le Fort I in 8 (24.2%), Le Fort III in 4 (12.1%), and alveolar in 3 (9.1%). Of the 173 mandibular fractures, 56.9% were treated by closed reduction, 39.8% by open reduction, and 3.5% by observation only. Of 33 maxillary fractures, 54.6% were treated using closed reduction, 40.9% using open reduction, and 4.5% with observation only. Approximately 52.1% of the patients were treated under general anesthesia, and 47.9% were treated under local anesthesia and sedation. Postsurgical complications were recorded in 5% of patients. These complications included infection, asymmetry, and malocclusion. Overall mortality in this series was 0.84% (2 patients); mortality was caused by pulmonary infection. The findings of this study, compared with similar studies reported in the literature, support the view that the causes and incidence of maxillofacial injuries vary from 1 country to another."}]}}}} {"correct_option": 1, "explanations": {"1": {"exist": true, "char_ranges": [[0, 236]], "word_ranges": [[0, 39]], "text": "We are being described an acute pain crisis in a patient with gonarthrosis. In this situation, the first thing to do is to resolve the pain crisis and to propose an appropriate conservative treatment for this osteoarthritis (1 correct)."}, "2": {"exist": true, "char_ranges": [[237, 304]], "word_ranges": [[39, 52]], "text": "A knee arthroplasty is not considered at the outset, so 2 is false."}, "3": {"exist": true, "char_ranges": [[305, 433]], "word_ranges": [[52, 75]], "text": "They are not telling us an infectious clinic to suspect an arthritis that would justify a debridement and washing so 3 is false."}, "4": {"exist": true, "char_ranges": [[540, 844]], "word_ranges": [[92, 144]], "text": "Baker's cyst would only interest us in a severe pain crisis in the differential diagnosis with a deep thrombosis and it is evaluated with an echo-Doppler, in the picture of gonarthrosis it has no value to detect a Baker's cyst. Tendinitis is diagnosed by examination, not with MRI. Therefore, 4 is false."}, "5": {"exist": false, "char_ranges": [], "word_ranges": [], "text": ""}}, "full_answer": "We are being described an acute pain crisis in a patient with gonarthrosis. In this situation, the first thing to do is to resolve the pain crisis and to propose an appropriate conservative treatment for this osteoarthritis (1 correct). A knee arthroplasty is not considered at the outset, so 2 is false. They are not telling us an infectious clinic to suspect an arthritis that would justify a debridement and washing so 3 is false. In a patient with gonarthrosis there will always be meniscopathy, it is part of the degenerative changes. Baker's cyst would only interest us in a severe pain crisis in the differential diagnosis with a deep thrombosis and it is evaluated with an echo-Doppler, in the picture of gonarthrosis it has no value to detect a Baker's cyst. Tendinitis is diagnosed by examination, not with MRI. Therefore, 4 is false.", "full_answer_no_ref": "We are being described an acute pain crisis in a patient with gonarthrosis. In this situation, the first thing to do is to resolve the pain crisis and to propose an appropriate conservative treatment for this osteoarthritis ([HIDDEN]). A knee arthroplasty is not considered at the outset, so [HIDDEN]. They are not telling us an infectious clinic to suspect an arthritis that would justify a debridement and washing so [HIDDEN]. In a patient with gonarthrosis there will always be meniscopathy, it is part of the degenerative changes. Baker's cyst would only interest us in a severe pain crisis in the differential diagnosis with a deep thrombosis and it is evaluated with an echo-Doppler, in the picture of gonarthrosis it has no value to detect a Baker's cyst. Tendinitis is diagnosed by examination, not with MRI. Therefore, [HIDDEN].", "full_question": "A 73-year-old woman with a history of obesity, type 2 diabetes mellitus, hypertension and dyslipidemia. She consults for unbearable pain in the right knee of 5 days of evolution, without previous trauma. Examination: globular knee, moderate varus, extension and flexion limited by pain, diffuse medial pain. X-ray shows osteophytes and mild impingement of the medial interlining. What would be his initial management?", "id": 470, "lang": "en", "options": {"1": "Explanation of the diagnosis, relative rest, paracetamol 1g/8h plus metamizol 500 mg/ 8 h rescue naproxen.", "2": "Preferential referral to Traumatology outpatients for evaluation of total cemented prosthesis.", "3": "Preferential referral to Traumatology outpatient clinic for arthroscopic debridement.", "4": "Preferred MRI request for evaluation of meniscopathy, Baker's cyst and/or tendinitis.", "5": NaN}, "question_id_specific": 138, "type": "ORTHOPEDIC SURGERY AND TRAUMATOLOGY", "year": 2020, "rag": {"clinical_case_options": {"MedCorp": {"RRF-2": [{"id": "wiki20220301en012_37209", "title": "Tendinopathy", "score": 0.013476378693769999, "content": "Tendinopathy, a type of tendon disorder that results in pain, swelling, and impaired function. The pain is typically worse with movement. It most commonly occurs around the shoulder (rotator cuff tendinitis, biceps tendinitis), elbow (tennis elbow, golfer's elbow), wrist, hip, knee (jumper's knee, popliteus tendinopathy), or ankle (Achilles tendinitis). Causes may include an injury or repetitive activities. Groups at risk include people who do manual labor, musicians, and athletes. Less common causes include infection, arthritis, gout, thyroid disease, and diabetes. Diagnosis is typically based on symptoms, examination, and occasionally medical imaging. A few weeks following an injury little inflammation remains, with the underlying problem related to weak or disrupted tendon fibrils."}, {"id": "InternalMed_Harrison_26086", "title": "InternalMed_Harrison", "score": 0.012804713096275813, "content": "X-rays are indicated to evaluate chronic hand pain and hip pain thought to be due to OA, as the diagnosis is often unclear without confirming radiographs. For knee pain, x-rays should be obtained if symptoms or signs are not typical of OA or if knee pain persists after inauguration of effective treatment. In OA, radiographic findings (Fig. 394-7) correlate poorly with the presence and severity of pain. Further, radiographs may be normal in early disease as they are insensitive to cartilage loss and other early findings. Although MRI may reveal the extent of pathology in an osteoarthritic joint, it is not indicated as part of the diagnostic workup. Findings such as meniscal tears andcartilage and bone lesions occurin most patients with OA in the knee, but almost never warrant a change in therapy."}, {"id": "pubmed23n0965_8967", "title": "Knee Pain in Adults and Adolescents: The Initial Evaluation.", "score": 0.012089999269486448, "content": "Knee pain affects approximately 25% of adults, and its prevalence has increased almost 65% over the past 20 years, accounting for nearly 4 million primary care visits annually. Initial evaluation should emphasize excluding urgent causes while considering the need for referral. Key aspects of the patient history include age; location, onset, duration, and quality of pain; associated mechanical or systemic symptoms; history of swelling; description of precipitating trauma; and pertinent medical or surgical history. Patients requiring urgent referral generally have severe pain, swelling, and instability or inability to bear weight in association with acute trauma or have signs of joint infection such as fever, swelling, erythema, and limited range of motion. A systematic approach to examination of the knee includes inspection, palpation, evaluation of range of motion and strength, neurovascular testing, and special (provocative) tests. Radiographic imaging should be reserved for chronic knee pain (more than six weeks) or acute traumatic pain in patients who meet specific evidence-based criteria. Musculoskeletal ultrasonography allows for detailed evaluation of effusions, cysts (e.g., Baker cyst), and superficial structures. Magnetic resonance imaging is rarely used for patients with emergent cases and should generally be an option only when surgery is considered or when a patient experiences persistent pain despite adequate conservative treatment. When the initial history and physical examination suggest but do not confirm a specific diagnosis, laboratory tests can be used as a confirmatory or diagnostic tool."}, {"id": "pubmed23n0772_19934", "title": "An Intra-tendonous ganglion cyst causing impingement between the anterior cruciate ligament and anterior root of the medial meniscus: a case report.", "score": 0.009900990099009901, "content": "There are several reports of symptomatic ganglion cysts near the anterior cruciate ligament (ACL), posterior cruciate ligament (PCL), and lateral and medial meniscus, but symptomatic ganglia arising from the anterior horn of the medial meniscus to the ACL have not been reported. Here we report the arthroscopic resection of a ganglion cyst arising from the anterior horn of the medial meniscus with a meniscal tear to the ACL. A 43-year-old female presented with a 10-year history of continuous aching pain in the right knee, but without any history of trauma. Clinical examination revealed right-sided knee pain in the medial joint line, exacerbated by end range flexion and extension, a -10°-100° active range of movement, and a -5°-110° passive range of movement。McMurray's, patellar compression, and compression rotation tests were positive. Magnetic resonance imaging (MRI) and arthroscopic examination revealed a cyst related to the ACL and medial meniscus. Histological examination confirmed the cyst to be a ganglion cyst. We present a new type of ganglion cyst, this is the first reported case of an ganglion cyst impinged between the ACL and the medial meniscus. It is hoped that this study will provide a better understanding of the condition and lead to better diagnosis and treatment."}, {"id": "pubmed23n0773_16081", "title": "[Knee pain: choosing the right imaging].", "score": 0.009900990099009901, "content": "Gonalgia is a frequent reason for consultation of a primary care physician. The road leading to diagnosis is mainly clinical. A detailed medical history and physical examination are capital for establishing diagnostic hypotheses and choosing the most appropriate imaging test. Initially, a simple X-ray of the knee joint is the most common exam, even though it is not always needed, especially after a minor trauma. MRI and CT-scan allow a more detailed examination of the structures; however, they should only be ordered to answer a specific question. Most of the time, echography is reserved to extra-articular pathologies and for guiding an articular tap."}, {"id": "InternalMed_Harrison_26032", "title": "InternalMed_Harrison", "score": 0.009890563700185834, "content": "Additional imaging techniques may possess greater diagnostic sensitivity and facilitate early diagnosis in a limited number of articular disorders and in selected circumstances and are indicated when conventional radiography is inadequate or nondiagnostic (Table 393-5). Ultrasonography is useful in the detection of soft tissue abnormalities, such as tendinitis, tenosynovitis, enthesitis, bursitis, and entrapment neuropathies. Wider use, lower cost, better technology, and enhanced site-specific transducers now allow for routine use in outpatient care. Owing to low cost, portability, and wider use, ultrasound use has grown and is the preferred method for the evaluation of synovial (Baker’s) cysts, rotator cuff tears, tendinitis and tendon injury, and Approach to Articular and Musculoskeletal Disorders Strongly consider synovial fluid aspiration and analysis if there is Trauma with joint effusion Monarthritis in a patient with chronic polyarthritis"}, {"id": "article-26373_12", "title": "Osteoarthritis -- Evaluation", "score": 0.009870724251583926, "content": "A thorough history and physical exam (with a focused musculoskeletal exam) should be performed on all patients, with some findings summarized above. OA is a clinical diagnosis and can be diagnosed with confidence if the following are present: 1) pain worse with activity and better with rest, 2) age more than 45 years, 3) morning stiffness lasting less than 30 minutes, 4) bony joint enlargement, and 5) limitation in range of motion. A differential diagnosis should include rheumatoid arthritis, psoriatic arthritis, crystalline arthritis, hemochromatosis, bursitis, avascular necrosis, tendinitis, radiculopathy, among other soft tissue abnormalities. [9] [10]"}, {"id": "pubmed23n0728_10812", "title": "Arthroscopic management of calcific tendonitis of the medial collateral ligament.", "score": 0.00980392156862745, "content": "Calcific tendinitis most commonly occurs to the shoulder, but may also involve other structures of the locomotor system. It is a rare cause of knee pain. We report a 46-year-old woman with severe medial knee pain and limitation of movement in her right knee. There was a marked tenderness site at the proximal insertion of the medial collateral ligament (MCL). Flexion was able to provoke the painful symptoms in the medial knee. The involvement of differentiated diagnoses were excluded by history, laboratory and radiograph examinations, while X-ray, CT and MRI suggested calcific tendonitis of the MCL. Due to the failure of conservative treatments, we offered her arthroscopic excision of calcific deposit which was sent for biopsy. Histopathological evaluation confirmed the diagnosis of calcific tendinitis. This patient recovered shortly afterwards with immediate resolution of symptoms following excision. Thus far, calcifications involving the MCL have been documented thrice. Calcific tendonitis of the MCL diagnosed and treated by arthroscopy has not previously been reported, which can be challenging to diagnose and treat because of its rarity. Although conservative treatment appears to be frequently satisfactory, arthroscopic excision may be a better option for the refractory or severe cases."}, {"id": "wiki20220301en014_26817", "title": "Patella", "score": 0.00980392156862745, "content": "Etymology The word patella originated in the late 17th century from the diminutive form of Latin or or paten, meaning shallow dish. See also Patellar reflex Knee pain Osteoarthritis Lateral retinaculum Lateral release References External links Knee Sesamoid bones Bones of the lower limb"}, {"id": "pubmed23n0774_13662", "title": "[The \"Oxford\" unicondylar knee prostesis (UCP): 21 reviewed cases].", "score": 0.009708737864077669, "content": "The unicompartimental knee prosthesis known as \"Oxford\" is a non constraint prosthesis, entrusting the whole of its stability to an intact ligamentary apparatus. Where the support surfaces of most prostheses remain limited, even punctiform, the originality of the Goodfellow prosthesis lies in the fact that the prosthetic condyle, whatever the flexion angle is, leans against a mobile prosthetic meniscus with spheric superior concavity of the same radius as the condylian radius, which increases considerably the prosthetic leaning surfaces and therefore lessens the pressure constraints. The superior surface, concave, of this prosthetic meniscus takes charge of the rolling, where the inferior plane surface realizes the gliding on the metallic tibial plate. The total conformity of the components minimizes the forces of friction.Between July 1988 and March 1993, 24 patients underwent the placing of UCP. Three patients died and 2 were lost of sight. 19 patients could be seen again or checked, corresponding of 21 operated knees. Two knees benefited from the start from UCP (medial and lateral) and 2 knees had a UCP in the first instance and then a second UCP in the compartment left safe primarily. For the 21 UCP, there are 16 medial and 3 lateral. Our mean drawback is of 3 years and 3 months, all the drawbacks being superior to 1 year and 4 months. The mean age is of 64 years. There were 17 female and 2 male patients. The mean weight is of nearly 80 kg (79,8) and nearly 52% of the operated patients have an important overweight (Body Mass Index superior to 30). Preoperative clinical analysis. It is based on a retrospective study of files using the quotation described by AUBRIOT for the «GUEPAR» group. This one establishes a gradation of four levels for each of the three criteria retained (Pain, Mobility, Instability), thus determining a global result imposed by the lowest level retained.For walking, other factors than just the state of the operated knee may intervene, this being the reason why it doesn't show in this chart. The GUEPAR group quantifies it with letters A, B, C, D.Concerning pain, all 21 knees were quoted as \"Bad\" in preoperative. Pain constitutes the decisive argument for the operative indication. In our series, only one knee had an average amplitude, all the others had a mobility superior to 89°. In 5 cases there was a flessum between 11 and 20° (penalizing of a level). Concerning walking and stability, they were taken into account, thanks to a precise questionnaire about the daily life acts. Concerning the walking perimeter, it was found as unlimited (A) in 1 case, superior to 500 m (B) in 2 cases, inferior to 500 m (C) in 17 cases and limited to home (D) in 1 case. The early after effects. At the end of the intervention, the knee is placed into a splint with limited flexion. As soon as the second day the patient is sat on the border of his bed. The first partial support at the third of the body weight is authorized between the fourth and the fifth day, when at the same time flexion exercises on electrical splint are started, as soon as the Redon draining is removed. The average hospitalization length was of a fortnight. Among secondary late complications and retakes, let us stop on meniscal luxations which constitute a specific complication of the Oxford arthroplasty. They concern 3 times the medial compartment and 4 lateral compartment. They happened in 1 case early, at D 22, in 3 cases within the 6 first months and in 3 cases after 2 years. They were treated : 3 times by reduction under general anæsthetic, no more ; 3 times changing the meniscusus for a meniscusus of superior size and once by placing a total prosthesis at the place of the UCP. The deteriorations of the opposed compartment not prosthesized occured in three cases. They were treated by unicompartmental additional arthroplasty in two cases and by total prosthesis in the third case. The clinical results on pain are very satisfactory as from the early check up onwards we have 17 successes (no pain 11 cases and occasional pains 6 cases) and as after 3 years and 5 months in average, we have 19 successes (no pain : 10 cases - occasional pain : 9 cases). At the maximal drawback, the mobility is quoted very good in 7 cases and good in 13 cases, mean in 1 case. At the latest check up, we note an excellent stability in 17 cases and good in 3 cases, that is to say 20 successes and 1 case of stability quoted as mean. At the latest check up we note 17 successes (A and B) and 4 relative failures (C) concerning the quality of walking.At the question «are you pleased with the intervention and would you advise it to a friend?» and with the nuance «very pleased» and «simply satisfied», we get 10 cases «very pleased», 8 cases «pleased» and 3 cases «moderately satisfied»; only those 3 cases advise against the intervention. The radiological results are less satisfying as they show frequent imperfections : • for the 16 medial UCP : only 9 cases hypocorrected or normo axed, but 1 case strongly hypocorrected (residual varus of 7°) and 6 hypercorrected cases. • for the 5 lateral UCP : 3 normo-axed cases, 1 case strongly hypocorrected (residual valgus of 6°) and 1 case strongly hypercorrected (10° varus). • the failures due to rapid deterioration of the non prosthetized compartment occurred on hypercorrected knees. • on 21 knees, 14 borders of tibial plate were noticed, out of which 9 had no plate displacement and 5 had a slight displacement, at the origin of a small angular loss. • accumulations of cement on the tibial side, towards the back or in medial were noticed in 8 cases, which explains a slope of the tibial plate to the back inferior to 5° in 11 cases (should be of 7°). • 4 femoral components seem to be too posterior and one shows curved.In total, only 7 cases out of 21 were estimated with no peculiarities on the radiological point of view. It seems difficult to place a UCP well. The meniscal luxations are favored by an alignment rotational defect of the tibial plate, specially for the lateral UCP, the meniscus coming to hit the lip of the tibial plate during the lifting from a sitting position. For 5 of these luxations, we must recognize the existence of a ligamentary collateral laxity which should have altered the surgical indication either to an osteotomy, or to a total arthroplasty. Conclusions. Under the condition of respecting the absolute counter indications, of thoroughly evaluating the relative counter indications and of reducing at the best the defects linked to the surgical technique, the unicompartmental arthroplasty, including that of Oxford, gives good functional results after more than three years. In our series, the result on pain is constant if we exclude the cases with risk with ligamentary laxity and that of centered gonarthrosis at obese subject, that is to say 15 successes on 15 knees thus selected retrospectively. The gain on mobility is weak, of 5° in average. The result on stability is, as for pain, excellent, if we exclude the cases with risk, as we get then also 15 successes on 15 knees. Concerning the global result according to the quotation of Aubriot-Guepar, we note 14 successes and 1 relative failure. 4 knees were bad indications and should have benefited from a total arthroplasty or from an osteotomy. "}, {"id": "pubmed23n0852_20928", "title": "Acute leg pain with suspected beginning leg compartment syndrome and deep vein thrombosis as differential diagnoses in an unusual presentation of Brodie's abscess: a case report.", "score": 0.009615384615384616, "content": "Brodie's abscess is an uncommon form of subacute osteomyelitis where the main presenting symptom is mild to moderate pain of insidious onset for several months' duration. We report a case of a patient presenting with acute leg pain resembling that of a deep vein thrombosis, and a beginning leg compartment syndrome following a suspected ruptured Baker's cyst. Our case is unusual because of the acute presentation of the Brodie's abscess with acute leg pain and acute swelling without any preceding trauma; to the best of our knowledge, this presentation has not been reported before. A 17-year-old white boy presented to our out-patient clinic with a 6-month history of pain in his left knee joint of insidious onset. There was no history of trauma to the extremity. After performing physical and radiological (X-ray) examinations, we initially diagnosed medial meniscus damage. One week later he presented to our emergency department with acute sudden increase in the pain and swelling of his left knee, and pain and swelling of his left leg, without any trauma. Deep vein thrombosis and beginning leg compartment syndrome from ruptured Baker's cyst were initially diagnosed. Magnetic resonance imaging was performed and Brodie's abscess was the most probable diagnosis. We performed open surgical debridement and curettage with drainage of the abscess and administered postoperative antibiotics. He presented to our out-patient clinic 3 months postoperatively, where he was pain-free with no residual local tenderness. In cases of sudden acute increase in joint or extremity pain or swelling that has been insidiously present for months, Brodie's abscess should be considered as one of the differential diagnoses, as it may present acutely in cases with accompanying fasciitis and myositis and be clinically mistaken for deep vein thrombosis or limb compartment. Magnetic resonance imaging remains the gold standard imaging study, and surgical treatment followed by postoperative antibiotics remains the standard treatment."}, {"id": "pubmed23n1146_16199", "title": "Popliteus Tendon Injuries.", "score": 0.009615384615384616, "content": "Popliteus tendinopathies are rare injuries that can occur from overuse, trauma, or secondary causes, such as sesamoid bones or calcifications. They present with nonspecific symptoms and should be considered in any patient with posterolateral knee pain, instability, popliteus tenderness, and a positive Garrick test. Diagnosis can be made with magnetic resonance imaging, but arthroscopy remains the criterion standard. For minor popliteus tendinopathies, initial management involves conservative treatment, including rest, activity modification, physical therapy, and quadriceps strengthening. For more severe or refractory disease, corticosteroid injections and arthroscopy should be considered. [